You are on page 1of 511

Allergy & Immunology

Anaphylaxis
Angioedema
Urticaria
Allergic Rhinitis
Primary Immunodeficiency
Disorders

Allergy & Immunology


Niket Sonpal, MD
Chief Resident
Lenox Hill Hospital NSLIJ
Assistant Clinical Professor Touro College of Medicine

Anaphylaxis

Anaphylaxis

Worst form of acute allergic reaction

Sensitization

Re-exposure Anaphylaxis

DEATH

Synonymous with immediate hypersensitivity


Pathogenesis
Initial sensitization to antigen with subsequent reexposure

Initial
sensitization
to antigen

Upon re-exposure, IgE binds to mast cells leading to


release of their granules (e.g., histamine,
prostaglandins, and leukotrienes)

HISTAMINE

Results in abnormalities that essentially define


anaphylaxis

MTB S2CK

p. 41

No presensitization
to antigen

Non-IgE related

p. 41

p. 41

Hives
Itching
Constriction of
airways
Swollen tongue
Wheezing
Dyspnea
Tachycardia
Nausea, vomiting,
or diarrhea
Dizziness or
fainting
Hypotension

Anaphylaxis/Etiology
Anaphylactoid Reactions

MTB S2CK

LEUKOTRIENES

PROSTAGLANDINS

MTB S2CK

Anaphylaxis

Upon re-exposure,
IgE binds to mast
cells leading to
release of their
granules

Identical Tx

Causes of anaphylaxis = causes of any


allergic event
Insect bites and stings
Medications: penicillin, phenytoin, lamotrigine,
quinidine, rifampin, and sulfa
Foods
Latex is a very important cause of anaphylaxis in
healthcare workers

MTB S2CK

p. 41

Anaphylaxis/Presentation

Anaphylaxis/Treatment

Characterized by
Hypotension
Tachycardia
Respiratory: stridor
Rash

The best initial treatment is


A Airway Protection
Intubation
Cricothyroidodotomy

B Breathing
g
C - Circulation

Source: Fashad Bagheri. MD

MTB S2CK

p. 41 42

Epinephrine
Antihistamines H1 and H2
Glucocorticoids
MTB S2CK

p. 42

Angioedema

Angioedema/Presentation

Angioedema is sudden swelling of

Hereditary
angioedema is a
genetic disorder

Face
Tongue
Eyes
Airway

Look for recent start of ACE inhibitors


preceding symptoms

This can be from deficiency of C1 esterase inhibitor


Characteristic association of onset with minor
physical trauma
Angioedema often idiopathic

Glucocorticoids
dont work

Source: commons.wikimedia.org

MTB S2CK

p. 42

Angioedema/Tests

MTB S2CK

p. 43

Angioedema/Treatment

The best initial test...


C2 and C4 levels in complement pathway

Airway

Acute Tx

Long Term Tx

Will be decreased

C1 esterase inhibitor
Also decreased

3 types of hereditary angioedema


Type I - decreased levels of C1INH (85%)
Type II - normal levels but decreased function of
C1INH (15%)
Type III - no detectable abnormality in C1INH

MTB S2CK

p. 43

Ensure
airway
protection
first

MTB S2CK

Fresh frozen
plasma
Ecallantide
E ll tid

Androgens
Danazole
And
Stanazole

p. 43

Urticaria

Urticaria/Treatment

Form of allergic
reaction that causes
sudden swelling of
superficial skin layers
Can be caused by

1. Antihistamines

Insects
Medications
Pressure
Cold
Vibration

MTB S2CK

Hydroxyzine,
diphenhydramine,
fexofenadine,
loratidine, cetirizine,
or ranitidine

2 L
2.
Leukotriene
k ti
receptor
t
antagonists
Source: commons.wikimedia.org

p. 43

Montelukast or
zafirlukast
Source: commons.wikimedia.org

MTB S2CK

p. 44

Allergic Rhinitis

Allergic Rhinitis/Diagnostic Tests

IgE-dependent triggering of mast cells


Seasonal allergies such as hay fever are
common
Presents with recurrent episodes of
Watery
y eyes,
y , sneezing,
g, itchyy nose,, and
itchy eyes
Inflamed, boggy nasal mucosa
Pale or violaceous turbinates
Nasal polyps

Diagnosed
clinically
Skin testing and
blood testing
IgE levels may be
elevated
Nasal smear with
eosinophils

MTB S2CK

p. 44

Source: commons.wikimedia.org

MTB S2CK

p. 44

Allergic Rhinitis/Treatment

Common Variable Immunodeficiency (CVID)

1. Prevention & avoidance


2. Intranasal corticosteroid sprays
3. Antihistamines H1 blockers
4. Intranasal anticholinergic medications
5. Desensitization to allergens

B cells are present but decreased Igs


Decrease in all Ig subtypes
IgG, IgM, and IgA

LOW B cells output, normal T cells

MTB S2CK

p. 44

MTB S2CK

p. 45

Common Variable Immunodeficiency (CVID)

Common Variable Immunodeficiency (CVID)

Recurrent sinopulmonary infections


Frequent episodes of
Bronchitis
Pneumonia
Sinusitis
And
Otitis media

Other manifestations are


Giardiasis
Sprue-like intestinal malabsorption

MTB S2CK

p. 45

Clue to CVID is a decrease in the


output of B lymphocytes with a normal
number of B cells as well as normal
amounts of lymphoid tissue (e.g., nodes,
adenoids, and tonsils)

MTB S2CK

p. 45

Common Variable Immunodeficiency (CVID)

Common Variable Immunodeficiency (CVID)

Immunoglobulin levels

Treatment
Antibiotics are used for each infection
as it develops
Chronic maintenance

Decreased

Antigen Stimulation
Decreased response
Normal Number of B Cells

MTB S2CK

p. 45

Regular
Reg lar inf
infusions
sions of IVIG

MTB S2CK

p. 45

X linked (Bruton) Agammaglobulinemia

Severe Combined Immunodeficiency

Low B, normal T in males


Male children with increased sinopulmonary
infections
B cells and lymphoid tissues are diminished
T cells are normal
Treatment: Abx for infections as they arise
Long-term regular administration of IVIG keeps
children healthier

LOW B AND LOW T


The word combined in severe combined
immunodeficiency (SCID) means that there is
deficiency in both B and T cells
Results in infections related to both deficiencies

LOW B cells and LOW T cells. Analogous to HIV


LOW B cells, normal T cells in young male children
MTB S2CK

p. 45

MTB S2CK

p. 46

Severe Combined Immunodeficiency

IgA Deficiency

B cells
Decreased Ig production

These patients present with recurrent


sinopulmonary infections
The difference with this syndrome is:
Atopic diseases
Anaphylaxis to blood transfusion when blood donor
has normal levels of IgA
Sprue-like condition with fat malabsorption
Increase in risk of vitiligo, thyroiditis, and
rheumatoid arthritis

T cells
Markedly decreased numbers of T cells
Long-Term Treatment
Bone marrow transplant
MTB S2CK

p. 46

MTB S2CK

p. 46

IgA Deficiency

Hyper IgE Syndrome

Treat infections as they arise


Washed blood transfusions otherwise
anaphylaxis
IVIG injections will not work
The
Th ttrace amounts
t off IIgA
A iin IVIG may
provoke anaphylaxis

Presents with
recurrent skin
infections due to
Staphylococcus
Treat infections as
they arise
Consider prophylactic
antibiotics (e.g.,
dicloxacillin or
cephalexin)
Source: wikimedia commons

MTB S2CK

p. 46

MTB S2CK

p. 46

Wiskott Aldrich Syndrome

Chronic Granulomatous Disease (CGD)

Normal T cells normal B cells decline with age


Immunodeficiency combined with
thrombocytopenia and eczema
T lymphocytes markedly deficient in blood and
lymph nodes
Bone marrow transplantation is only definitive
treatment

CGD is genetic disease resulting in


extensive inflammatory reactions
Leads to lymph nodes with purulent
material leaking out

Normal T cells normal B cells decline with age

MTB S2CK

p. 46

MTB S2CK

p. 46

Chronic Granulomatous Disease (CGD)

Chronic Granulomatous Disease (CGD)

Aphthous ulcers

Granulomas may become obstructive in


the GI or urinary tract
Look for infections with odd combination
of:

And

Staphylococcus
Burkholderia
Nocardia
Aspergillus

Inflammation of
nares is
common

Source:commons.wikimedia.org

MTB S2CK

p. 46

Chronic Granulomatous Disease (CGD)

Abnormal nitroblue tetrazolium testing


This is a decrease in NADPH oxidase,
which generates superoxide
CGD test is negative

MTB S2CK

p. 46 47

Primary Immunodeficiency Disorders


CVID

X-linked (Bruton)
Agammaglobulinemia

SCID

IgA
Deficiency

LOW B
cell
output
Normal
T cells

LOW B cells
normal T
cells in young
male children

LOW B & T
cells
Analogous
to HIV

Atopic
Anaphylaxis

Hyper IgE
Syndrome
Skin infections
Staphylococcus

MTB S2CK

p. 47

MTB S2CK

p. 45 47

Wiskott-Aldrich
Syndrome

Normal T cells
Normal B cells
Low Platelets
Eczema

CGD
Lymph nodes with
purulent material
Infections, combined with
Staphylococcus
Burkholderia
Nocardia
Aspergillus

Coronary Artery
Disease Part 1

The Cardiovascular System

Definition
D
fi iti
Risk Factors
Clinical Presentation

Conrad Fischer, MD
Associate Professor of Medicine
Touro College of Medicine
New York City

Coronary Artery Disease (CAD)/Definition

Can be used interchangeably with:


Atherosclerotic heart disease
Ischemic heart disease

All imply insufficient perfusion of


coronary arteries
Abnormal narrowing of vessels
Insufficient oxygen delivery to
myocardial tissue
MTB S2CK

p. 49

48-year-old woman in office with chest pain for


several weeks. Pain isn't reliably related to exertion.
No pain now. The pain is retrosternal and sometimes
associated with nausea. No SOB and no radiation
beyond chest. She has no past medical history.
What is the most likely diagnosis?
a. Gastroesophageal reflux disease (GERD)
b. Unstable angina Acute, severe pain in ED
c. Pericarditis Pain worse with lying down, better when sitting up
d. Pneumothorax Sharp, pleuritic pain, tracheal deviation
e. Prinzmetal angina Nonexertional chest pain,
MTB S2CK

p. 49

early in morning, ST
segment elevation

Risk Factors for CAD


Menstruating women virtually never have MIs

Which of the following is most likely to benefit a


patients risk of coronary disease?
Estrogen simply does NOT help CAD

Risk factors are most important with


equivocal or uncertain histories

a. Administration of estrogen replacement at time of


menopause
b Stopping tamoxifen Tamoxifen unrelated to CAD
b.
c. Stopping aromatase inhibitors Zero relationship to CAD
d. Regular exercise
e. Relaxation methods (e.g., meditation) Good, but
immeasurable,
no evidence of
proven benefit

Women > men eventually die of heart disease


MTB S2CK

p. 50

MTB S2CK

p. 50

Risk Factors for CAD

Risk Factors for CAD

Clear ones:

Diabetes (worst risk)


HTN
Defined as > 140/90
More common than diabetes
20% of total population (60 million
people) with HTN
50% unaware theyre hypertensive

Diabetes mellitus
Tobacco smoking
HTN
Hyperlipidemia
Family history of premature CAD
Age > 45 men; > 55 women

MTB S2CK

p. 50

MTB S2CK

p. 50

Risk Factors for CAD

Family history not a risk if


CAD developed in elderly relatives
Relatives were grandparents, cousins, or
aunts and uncles
Family
y history
y is a risk ONLY if
First-degree relatives (siblings/parents)
Premature disease:
Defined as:

Which of the following is the most dangerous to a


patient in terms of risk for CAD?
Triglycerides not as dangerous as elevated LDL

a. Elevated triglycerides
b. Elevated total cholesterol Its not the TOTAL cholesterol
or low
l
HDL
HDL, it
its LDL!
c. Decreased
D
d HDL
d. Elevated LDL
e. Obesity The danger of obesity is from its association
with high LDL, DM, and HTN

Male relative < 55


Female relative < 65
MTB S2CK

p. 50 51

MTB S2CK

p. 51

Risk Factors for CAD

Less clear
Physical inactivity
Excess alcohol
Insufficient fruits & vegetables
Emotional stress
CT scan calcium scores
Positron emission tomography (PET)
scanning
MTB S2CK

p. 51

Postmenopausal woman develops chest pain immediately on


hearing news of her sons death. She develops chest pain, dyspnea,
and ST segment elevation in leads V2 to V4. Troponin levels rise
with MI. Coronary angiography is normal including an absence of
vasospasm.
Echocardiography: Apical LV ballooning
Whats the presumed mechanism of this disorder?
a. Absence of estrogen
g
Its not absent estrogen
g only
y causing
g MI
b. Massive catecholamine discharge
c. Plaque rupture There were no plaques on angiography
d. Platelet activation Platelets cause MI from CAD, not emotions
e. Emboli to coronary arteries
It wouldve been seen

MTB S2CK

p. 52

Risk Factors for CAD

Risk Factors for CAD

Unreliable (Unproven) Risk Factors for CAD


Homocysteine
Chlamydia infection
C-reactive protein
No clear benefit in therapeutic intervention on
these factors
Theyre the wrong answers

The Most Common Wrong Answer


If risk factor question involves:
Family history
Mistaking CAD in elderly relatives as a
risk for patient

Frequently used wrong


answers are just as important
to learn as right answers
MTB S2CK

p. 52

MTB S2CK

p. 52

Chest Pain/Presentation
Correcting which of the following risk factors for CAD
results in the most immediate benefit?
a.
b.
c
c.
d.
e.

Diabetes mellitus
Tobacco smoking
Hypertension
Hyperlipidemia
Weight loss

MTB S2CK

What Is the Most Likely Diagnosis?


Ischemic pain
Dull /Sore
Squeezing (Pressure-like)

Fixing all of them is good.


But
Not as fast as smoking cessation!

p. 53

Like any muscle thats starved for oxygen


Produces sore-muscle type pain when ischemic
Qualities that go against ischemia
Sharp (knifelike) or pointlike
Lasts for few seconds

MTB S2CK

p. 53

Chest Pain/Presentation

Chest Pain/Presentation

Three features help tell whether or not the


pain is ischemic in nature:

< 10% with chest pain in ED end up having


an MI
50% have no cardiac disease at all

1. Changes with respiration (pleuritic)


2. Changes with position of body
3. Changes with touch of chest wall
(tenderness)

MCC of chest pain that isnt


isn t
ischemic in nature is GI related

Each (pleuritic, positional, tender) excludes


ischemia with 95% negative predictive value
(NPV)

MTB S2CK

p. 53

MTB S2CK

p. 53

Causes of Chest Pain

Characteristics of Ischemic Pain

Duration
Stable angina: > 2 - < 10 min
ACS: > 10 - 30 min

Provoking factors
Physical activity,
activity cold
cold,
emotional stress

NOT tender
NOT positional
NOT pleuritic

Quality
Squeezing, tightness,
heaviness, pressure,
burning, aching
NOT: sharp, pins,
stabbing, or knifelike

Associated
symptoms
SOB, nausea,
diaphoresis,
dizziness,
lightheadedness,
fatigue

Location
Substernal

Alleviating factors
Rest

Radiation
Neck, lower jaw and teeth,
arms, shoulders

If the case
describes.

Answer as most
likely diagnosis

Answer as most
accurate test

Chest wall tenderness

Costochondritis

Physical examination

Radiation to back,
unequal blood
pressure between
arms

Aortic dissection

Chest X ray
with widened
mediastinum, chest
CTA, MRA, or TEE
confirms disease

Pain worse with lying


flat, better when
sitting up

Pericarditis

EKG with ST elevation


everywhere, PR
depression

MTB S2CK

p. 54

Causes of Chest Pain

Causes of Chest Pain

If the case
describes.

Answer as most
likely diagnosis

Answer as most
accurate test

If the case
describes.

Answer as most
likely diagnosis

Answer as most
accurate test

Epigastric discomfort,
pain related to eating

Duodenal ulcer
disease

Endoscopy

Sudden onset, SOB,


tachycardia, hypoxia

Pulmonary embolus

Spiral CT, V/Q scan

Chest X ray

p g
Gastroesophageal
reflux

Response
p
to PPIs;; or
liquid antacids

Sharp, pleuritic pain,


tracheal deviation

Pneumothorax

g ,
Bad taste,, cough,
hoarseness

Cough, sputum,
hemoptysis

Pneumonia

Chest X ray

MTB S2CK

p. 54

MTB S2CK

p. 54

Chest Pain/Presentation

Features that DONT help answer the


diagnosis question
Nausea
Fever
SOB (d
(dyspnea))
Sweating (diaphoresis)
Anxiety

MTB S2CK

Coronary Artery
Disease Part 2
Diagnosis

p. 54

Diagnostic Tests/Electrocardiogram

Diagnostic Tests/Enzymes (CK MB/Troponin)

The best initial test for all forms of


chest pain is certainly an
electrocardiogram (EKG)
In office-based setting:

Cardiac enzymes are not the answer in


Office/ambulatory case
Chronic or stable chest pain

Expect normal EKG

But
You cant do other testing until you
know the EKG
MTB S2CK

p. 55

If the patient has acute chest pain then the


answer is:
Transfer

MTB S2CK

to the emergency department

p. 55

Diagnostic Tests/Enzymes (CK MB/Troponin)

Diagnostic Tests/Stress Testing

Enzymes are the answer when:


Acute cases of chest pain
Emergency department

Exercise tolerance testing (ETT) is the answer when:


Etiology is unclear and
EKG is not diagnostic
Answer Stress test
ETT without nuclear isotopes: when: Etiology
1. Read EKG
2 Exercise
2.

uncertain &
EKG not diagnostic

Exercise means heart rate > 80% of maximum


Maximum heart rate = 220 patient age
MTB S2CK

p. 55

Diagnostic Tests/Stress Testing


Thallium (Nuclear) Stress
Normal myocardium picks up thallium like
potassium via Na/K-ATPase
Myocardium alive & perfused?
Nuclear isotopes will be picked up
Abnormalities/Ischemia or Infarction?
Decreased thallium or nuclear uptake
Stress or Dobutamine Echo
Normal myocardium moves on echo
Abnormalities decrease wall motion
Dyskinesis, Akinesis, or Hypokinesis
MTB SCK

p. 56

MTB SCK

p. 55

Diagnostic Tests/Stress Testing

Ischemia versus infarction:


Ischemia = Reversible decreased perfusion
Reversal of decrease in thallium uptake or
wall motion
It returns to normal after a period of rest
Infarction = Irreversible
Ischemia is reversible wall motion or
thallium uptake between rest and exercise.
Infarction is irreversible or fixed.
MTB SCK

p. 56

Diagnostic Tests/Stress Testing

Use of Exercise Tolerance Testing

Cant exercise?

Test

Indication

Persantine (dipyridamole) or adenosine with nuclear


isotopes (e.g., thallium or sestamibi)
Or

Exercise tolerance

Determine presence ST segment depression


of ischemia

Exercise thallium

Cant read EKG

Decreased uptake of
p
nuclear isotope

Exercise echo

Cant read EKG

Decreased wall motion

Dobutamine in combination with echocardiography


Dobutamine increases myocardial oxygen consumption
Dobutamine provokes ischemia
Ischemia decreases wall motion on echocardiogram

Ischemia detected

Dipyridamole thallium Cant exercise

Dipyridamole may provoke bronchospasm.


Avoid in asthmatics.
MTB SCK

p. 56

Cant exercise

Dobutamine echo
MTB S2CK

Diagnostic Tests/Stress Testing

Decreased uptake of
nuclear isotope
Decreased wall motion

p. 56

Chest pain (likelihood of CHD is high)


Resting EKG abnormalities?

Nuclear and Echo:

No

Sensitivity = Specificity

Able to exercise?

Yes
No

Yes

Exercise Thallium = Exercise Echo


Dipyridamole Thallium = Dobutamine Echo

Exercise stress
test

Chemical stress test


(dipyridamole thallium or
dobutamine echo)

p. 56

ANGIOGRAPHY
1 or 2 vessel disease

MTB SCK

Stress
echocardiogram or
nuclear stress test

3 vessel disease, left main or


2 vessel disease in diabetics

Stent placement

CABG

Diagnostic Tests/Coronary Angiography

Diagnostic Tests/Coronary Angiography

Detects anatomic location of disease


Determines surgery, angioplasty, or
other methods of revascularization
Sometimes used if noninvasive tests
are equivocal

Stenosis (narrowing) < 50% of diameter


is insignificant
Surgery or angioplasty is done for at
least 70% stenosis

MTB SCK

p. 56 57

MTB SCK

p. 57

Diagnostic Tests/Holter Monitoring

Continuous ambulatory EKG monitor


Records rhythm
Usually for 24-hour period
May be for 48 to 72 hours
Holter monitor detects rhythm disorders:
A-fib, atrial flutter
Ectopy (e.g., premature beats)
V-tach
Holter monitor does not detect ischemia
Not accurate for evaluating the ST segment
MTB SCK

48-year-old woman comes to office with chest pain occurring over


several weeks. Pain isnt reliably related to exertion. Shes
comfortable now. The pain is retrosternal. Past medical history and
the EKG is normal.
What is the most appropriate next step in management?

a. CK-MB Cardiac enzymes evaluate ACS


b. Troponin
Evaluate valve function,
function wall motion
motion, EF
c. Echocardiogram
E h
di
d. Exercise tolerance testing
Detect the anatomic location of stenosis;
e. Angiography
f. CT angiography determines method of revascularization
CT Angiogram and MRI not accurate enough
g. Cardiac MRI
h. Holter monitor
Used for rhythm evaluation

p. 57

CAD/Treatment
USMLE Step 2 CK is most concerned that you know:

Coronary Artery
Disease Part 3
Treatment

Medications that lower


mortality
Chronic angina (not an acute coronary syndrome)
Aspirin
Beta blockers
Nitroglycerin

Step 2 will not test dosing


Route of administration is tested
MTB S2CK

CAD/Treatment

Nitroglycerin:
Chronic Stable Angina
Oral
Transdermal patch
Acute Coronary Syndrome
Sublingual
Paste
Intravenous

MTB S2CK

p. 58

p. 57

CAD/Treatment

Nonspecific beta blockers (e.g.. propranolol)


are not used routinely in cardiology
Clopidogrel
Acute MI (all forms)
Aspirin intolerance (e.g., allergy)
Recent angioplasty with stenting
Adverse effect
Rare thrombotic thrombocytopenic purpura
MTB S2CK

p. 58

Treatment/Prasugrel

Treatment/ACE Inhibitors

Thienopyridine - same class as


clopidogrel and ticlopidine
Prasugrel: Antiplatelet medication for
use in:
Angioplasty & stenting
All acute MI
Intolerant of aspirin
75 increased hemorrhagic stroke

Low EF/systolic dysfunction (best


mortality benefit)
Regurgitant valvular disease

MTB S2CK

p. 58

Mostt common adverse


M
d
effect:
ff t
Cough
7% of patients
Switch to ARBs
MTB S2CK

p. 58

Treatment/Lipid Management
64-year-old man starts lisinopril for CAD with EF of 24%, and
symptoms of breathlessness. He sometimes has rales, but is
asymptomatic today. Physical reveals minimal edema of lower
extremities. Potassium level is elevated and its present on a
repeat measurement. EKG is unchanged.
How would you best manage the patient?

Statins (HMG-CoA reductase inhibitors)

CAD with LDL > 100 mg/dL

Remove K from the body;You


y;
should eliminate the cause

a.
b.
c.
d.
e.

Add kayexalate (potassium-binding resin)


Insulin and glucose Drive K into the cell, given in acute situation
He should get an alternative drug
Stop lisinopril
Switch lisinopril to candesartan ARBs lead to hyperkalemia
Switch lisinopril to hydralazine and nitrates

MTB S2CK

p. 58 59

Treatment/Lipid Management

LDL > 70: Treat when patient has


Coronary disease AND diabetes
Youre tested on national guidelines from
nonbiased federal organizations
g
Not private organizations (e.g., ACC)
Everyone will agree:
With CAD, goal of LDL at least < 100 mg/dL

MTB S2CK

p. 59

MTB S2CK

p. 59

CAD Equivalents

CAD equivalents
Statins to bring LDL down if > 100:
Peripheral artery disease (PAD)
Carotid disease (not stroke)
Aortic disease (aortic artery, not valve)
Diabetes mellitus

MTB S2CK

p. 59

Other Lipid Lowering Therapies


Which is the most common adverse effect of statin
medications?
a. Rhabdomyolysis
b. Liver dysfunction
c Renal failure
c.
d. Encephalopathy
e. Hyperkalemia

MTB S2CK

The most common wrong answer

They arent adverse effects of statins

p. 60

Niacin, gemfibrozil, cholestyramine, and ezetimibe


all have beneficial effects on lipid profiles
None is Best initial therapy
None has clear mortality benefit statins provide
Niacin and fibric acid derivatives such as
gemfibrozil have some mortality benefit
benefit, but not as
much as statins
Statins:
Antioxidant effect on endothelial lining of
coronary arteries
Gives benefit transcending lowering LDL number
MTB S2CK

p. 60

Niacin

Gemfibrozil

Associated with:
Glucose intolerance
Elevation of uric acid
Uncomfortable itchiness from histamine
Niacin is excellent to add to statins if full lipid
control not achieved with statins
Although statins, exercise, and cessation of
tobacco use will all raise HDL level, niacin
will raise HDL somewhat more

Fibric acid derivatives:


Lower triglyceride levels > statins
Benefit of lowering triglycerides alone not
proven as useful as straightforward mortality
benefit of statins
Use caution combining fibrates with statins

MTB S2CK

p. 60

Fibrates + Statins= Increased myositis

MTB S2CK

p. 60

Cholestyramine

Ezetimibe

Bile acid sequestrant


Significant interactions with
medications in gut
Potentially blocking their absorption
Cholestyramine
Ch l t
i associated
i t d with
ith
uncomfortable GI complaints
Constipation & flatus

Definitely lowers LDL

MTB S2CK

p. 60

No clear benefit to patient


LDL levels are an imperfect marker of benefit
with cholesterol-lowering therapies
Ezetimibe: No better than placebo
No Change in MI, stroke, or death

MTB S2CK

p. 60

Lipid Lowering Therapy

Lipid Lowering Meds and Adverse Effects

Lipid-lowering therapy: What is clear?


Statins lower mortality the most
Questions about adverse effects
Besides the benefit of statins in CAD with
LDL levels > 100 mg/dL, the only truly clear
aspectt off other
th therapies
th
i iis th
their
i adverse
d
effects

Agent

Adverse effect

Statins

Elevations of transaminases (LFTs),


myositis

Niacin

Elevation in glucose and uric acid level,


Pruritus

Fibric acid
derivatives

Increased risk of myositis when combined


with statins

Cholestyramine

Flatus and abdominal cramping

Ezetimibe

Well tolerated and nearly useless

Check AST & ALT


with statins
MTB S2CK

p. 61

MTB S2CK

p. 61

Calcium Channel Blockers (CCBs)

Calcium Channel Blockers (CCBs)

Dihydropyridine CCBs:

CCBs are:
Negative inotropes
Should decrease myocardial oxygen
consumption
Increased heart rate in the aggregate
gg g
will
increase myocardial oxygen consumption
Bottom line:

Nifedipine
Nitrendipine
Nicardipine
Nimodipine
Ni di i

May increase mortality with CAD


because of raising heart rate

Do not routinely use CCBs in CAD


None of the calcium-channel blockers have
been shown to lower morality in CAD

MTB S2CK

p. 61

MTB S2CK

p. 61

Calcium Channel Blockers (CCBs)

Calcium Channel Blockers (CCBs)

Verapamil and diltiazem, which do not


increase heart rate, are used in those
who cannot tolerate beta blockers
because of severe asthma
70% with reactive airway diseases (e
(e.g.,
g
asthma) still tolerate beta-1 specific BBs

Use CCBs (verapamil/diltiazem) in CAD only


with:
Severe asthma precluding the use of beta
blockers
Prinzmetal variant angina
Cocaine-induced chest pain
Adverse Effects of CCBs
Edema
You must know adverse effects
Constipation
Heart block (rare)

MTB S2CK

p. 61

MTB S2CK

p. 61 62

10

Revascularization

Coronary Artery Bypass Grafting (CABG)

Angiography:
Evaluates possibility of
revascularization
Either coronary bypass surgery or
angioplasty
Symptoms alone cannot tell the
number of vessels involved

Lowers mortality only with:


Three vessels with > 70% stenosis in
each
Left main occlusion
Two-vessel
T
l di
disease with
ith di
diabetes
b t
Persistent symptoms despite maximal
medical therapy

MTB S2CK

p. 62

MTB S2CK

p. 62

Coronary Artery Bypass Grafting (CABG)

Percutaneous Coronary Intervention (PCI)

Benefit greatest with left ventricular


dysfunction
Internal mammary artery grafts last 10
years
Saphenous vein grafts last 5 years

Percutaneous coronary intervention (PCI)


(angioplasty)
Intervention best therapy in acute
coronary syndromes
Particularly those with ST segment
elevation
Mortality benefit of PCI
Much harder to demonstrate in chronic stable
angina

MTB S2CK

p. 62

MTB S2CK

p. 62

Percutaneous Coronary Intervention (PCI)

Max medical therapy with aspirin, beta


blockers, ACEi/ ARBs, and statins has
proven benefit thats PCI in stable CAD
PCI decreases dependence on medication
Decreasing frequency of angina episodes
PCI: Best in acute coronary syndromes,
particularly with ST segment elevation.
PCI doesnt provide clear mortality
benefit for stable patients.
MTB S2CK

Acute Coronary
Syndromes Part 1
Definition
Presentation
Diagnosis

p. 62

11

Acute Coronary Syndromes (ACS)

Definition
Impossible to determine precise
etiology of (ACS) from history & physical
alone
Risk factors (e
(e.g.,
g hypertension
hypertension, diabetes
mellitus, tobacco) same for CAD

Acute Coronary Syndrome


EKG
ST elevation

No ST elevation
Cardiac Biomarkers

+
STEMI
MTB S2CK

p. 62

NSTEMI (Ischemia)

MTB S2CK

NSTEMI

UA

p. 62

STEMI

S4 Gallop
70-year-old woman in ED with crushing substernal
chest pain for last hour. Pain radiates to left arm and
is associated with anxiety, diaphoresis, and nausea.
Pain is sore and dull and clenches fist in front of
chest. History of hypertension.
Pulsus paradoxus,
Which is most likely in this patient? tamponade
p
a. >10 mmHg decrease in BP on inhalation
b. Increase in jugular venous pressure on inhalation
Kussmaul sign: constrictive
friction rub: pericarditis
c. Triphasic scratchy sound Pericardial
pericarditis
d. Continuous machinery murmur PDA (patent ductus)
e. S4 gallop
LVH/dilated cardiomyopathy
f. Point of maximal impulse displaced to axilla
MTB S2CK

p. 63

12

Acute Coronary Syndromes


Increased jugulovenous pressure on inhalation
Kussmaul sign
Constrictive pericarditis
Triphasic scratchy sound:
Pericardial friction rub
Dressler syndrome:
Not until several days after MI
Much rarer
Everyone is on aspirin already!
MTB S2CK

p. 63

Benign compared to LBBB

p. 63 64

Continuous machinery murmur


Patent ductus arteriosus
Displaced point of maximal impulse (PMI)
LVH as well as dilated cardiomyopathy
Displaced
p
PMI is an anatomic abnormality
y
Could not occur with ACS
Dont walk into Step 2
without knowing when
you will expect each
cardiac physical findings
MTB S2CK

70-year-old woman in ED with crushing substernal


chest pain for the last hour.
Which EKG finding gives the worst prognosis?
a. ST elevation in leads II, III, aVF IWMI better prognosis
than AWMI
b. PR interval >200 milliseconds First degree AV block
c ST elevation in leads V2-V4
c.
V2 V4
d. Frequent premature ventricular complexes (PVCs)
e. ST depression in leads V1 and V2 Posterior MI
f. Right bundle branch block (RBBB)

MTB S2CK

Acute Coronary Syndromes

PVCs should not be treated,


even with acute infarction

No specific physical finding


allows you to answer a most
likely diagnosis question in
terms of ST elevation or
depression without an EKG

p. 63

70-year-old woman comes to ED with crushing substernal chest pain for


the last hour. EKG shows ST segment elevation in V2 to V4.
What is the next step in management?
a. CK-MB level Neither of them would be elevated
b. Oxygen
All of them should be given,
c. Nitroglycerin sublingual
but neither lowers mortality
d. Aspirin
e. Thrombolytics All will be done, but first aspirin
f. Metoprolol
Sh ld b
Should
be given,
i
titime d
doesntt matter
tt
g. Atorvostatin Not in acute settings
h. Angioplasty
Never the right answer
i. Consult cardiology
j. Transfer patient to intensive care unit
Start treatment first
k. Troponin level
l. Morphine
m. Angiography
MTB S2CK

p. 64 65

Acute Coronary Syndromes

Learn the order in which to do things.


You must know what is first.

MTB S2CK

p. 65

70-year-old woman comes to ED with crushing substernal chest


pain for last hour. EKG shows ST segment elevation in V2 to V4.
Aspirin has been given to chew.
What is next step in this patient?
a. CK-MB level Neither of them would be elevated
b. Oxygen
All of them should be given,
c. Nitroglycerin sublingual but none lowers mortality
d. Morphine
Angioplasty is superior to thrombolytics
e. Thrombolytics
BBs and statins should be given, but they are
f. Metoprolol
not critically dependent upon time
g. Atorvostatin
h. Angioplasty
i. Troponin level
j. Lisinopril
MTB S2CK

p. 65

13

Acute Coronary Syndromes

ACS/Diagnostic Tests

On USMLE Step 2 CK, consultation is


almost never the correct choice.
Do everything yourself.
In ACS:
Does the treatment lower

mortality?
first?

Which is most important to do

MTB S2CK

p. 65

Test

Time to being
abnormal

Duration of abnormality

EKG

Immediately at
onset of pain

ST elevation progresses
to Q waves over days
to a week

Myoglobin

1 4 hours

1 2 days

CK MB

4 6 hours

1 2 days

Troponin

4 6 hours

10 14 days

MTB S2CK

p. 66

ACS/Diagnostic Tests

Diagnostic Tests/Reinfarction

Troponin
Cant distinguish reinfarction several
days after the first event
Renal insufficiency gives false positive
tests
Troponin: excreted through the kidney

New episode of pain within a few days of the


first?
EKG detects new ST segment abnormalities
Check CK-MB levels
After 2 days,
y , the CK-MB level from the initial
infarction returns to normal
CK-MB elevated several days after initial MI
indicates a new ischemic event

MTB S2CK

p. 66

MTB S2CK

p. 66

Intensive Care Unit Monitoring

After initial management patient should be in


ICU
Continuous rhythm monitoring is essential
Monitoring and rapid cardioversion improves
survival
Most common cause of death first several
days after myocardial infarction is ventricular
arrhythmia (ventricular tachycardia,
ventricular fibrillation)
Rapid electrical cardioversion or defibrillation
MTB S2CK

Acute Coronary
Syndromes Part 2
Treatment

p. 66

14

Treatment/STEMI

Treatment/STEMI

Angioplasty versus Thrombolytics


Angioplasty (PCI) superior to thrombolytics

Complications of PCI
Rupture of coronary artery
Restenosis
Hematoma at entry site into artery

Better survival and mortality


Less bleeding
Complications of MI decreased
Less arrhythmia, less CHF, fewer ruptures of
septum, free wall [tamponade] and papillary muscles
[valve rupture]

Standard of care:
PCI within 90 minutes of arriving in emergency
department with chest pain
Door to balloon time:
under 90 minutes
MTB S2CK

p. 67

Only 20% of U.S. hospitals can do primary


angioplasty
Important to have ability to perform
emergency cardiac surgery to repair the
vessel
MTB S2CK

p. 67

Treatment/STEMI
Which is most important in decreasing the risk of
restenosis of the coronary artery after PCI?
Doesnt change risk with each vessel done

a. Multistage procedure (i.e., doing 1 vessel at a time,


with multiple procedures)
Used only at procedure
b Use of heparin for 36
b.
3 6 months after the procedure
c. Warfarin use after the procedure For DVT and PE not
coronary disease
d. Placement of bare metal stent
e. Placement of drug-eluting stent (paclitaxel,
sirolimus)

Restenosis within 6 Months of PCI


No stent: 30% 40%
Bare metal stent:15% 30%
Drug-eluting stent:10%
Cant use thrombolytics?
Transfer to a facility performing PCI

Has much more risk for restenosis than drug-eluting stent

MTB S2CK

p. 67

MTB S2CK

p. 67

Treatment/STEMI

Absolute Contraindications to
Thrombolytics
Major bleeding:
Bowel (melena), brain

Recent surgery (within the last 2 weeks)


Severe hypertension (>180/110)
Nonhemorrhagic stroke within the last 6
months
Heme-positive brown stool is
not an absolute contraindication
to the use of thrombolytics
MTB S2CK

p. 67 68

Patient comes to small rural hospital without


catheterization lab with chest pain and ST segment
elevation.
What is the next step in management?
a. Transfer for angioplasty Should be done within 90 minutes
b. Administer thrombolytics now
c. Consult cardiology Never the right answer
Time is muscle. Delay = Death
Door to needle time: under 30 minutes
MTB S2CK

p. 68

15

Treatment Indications and Benefits

Treatment Indications and Benefits

Therapy

Therapy

In what cases is effect greatest?

In what cases is effect greatest?

Aspirin

Everyone, best initial therapy

Statins

Everyone, best with LDL > 100 mg/dL

Clopidogrel

All MIs, undergoing angioplasty and


stenting

Oxygen, nitrates

No clear mortality benefit

Heparin
Beta blockers

Everyone, effect not dependent on


time; started any time during
admission

After thrombolytics/PCI to prevent


restenosis, initial therapy with NSTEMI
and unstable angina

ACEI/ARBs

Everyone, benefit best with ejection


fraction < 40%

Calcium channel
blockers

Cant use beta blockers, cocaine


induced pain, Prinzmetal angina

MTB S2CK

p. 68

MTB S2CK

p. 68

Treatment/ST Segment Depression ACS


A man comes to the ED with chest pain for the last hour that is
crushing in quality and does not change with respiration or
position of his body.
EKG: ST segment depression, V2 to V4.
Aspirin has been given.
What is the next step in this patient?
a. Low molecular-weight heparin
b. Thrombolytics Benefit only with STEMI
c. Glycoprotein IIb/IIIa inhibitor (abciximab) PCI with stenting
d.
e.
f.
g.

Nitroglycerin No clear benefit


Morphine
Angioplasty Heparin first
Metoprolol Should be given, but not urgent

MTB S2CK

p. 69

Glycoprotein IIb/IIIa Inhibitors (Abciximab,


Tirofiban, Eptifibitide)
Used in acute coronary syndromes
Those who undergo angioplasty and stenting
Not beneficial in acute ST elevation infarctions
Inhibits
I hibi platelet
l l aggregation
i
Reduction in mortality with ST depression,
particularly in patients whose troponin or CK-MB
levels rise and then develop an MI requiring PCI
with stenting
MTB S2CK

Summary of Treatment Differences between


Cardiac Events
Stable angina

UA/NSTEMI

STEMI

Aspirin

Yes

Yes

Yes

Beta blockers

Yes

Yes

Yes

Nitrates

Yes

Yes

Yes

Heparin

No

Yes

Yes, only after


thrombolytics

GPIIb/IIIa meds No
MTB S2CK

p. 70

Yes

p. 69

Summary of Treatment Differences between


Cardiac Events
Stable angina

UA/NSTEMI

STEMI

Thrombolytics No

No

Yes, but not as


good as PCI

CCBs

No

No

No

Warfarin

No

No

No

No
MTB S2CK

p. 70

16

Acute Coronary Syndromes/Treatment

Treatment/ST Segment Depression ACS

Bottom line:
1. tPA (thrombolytics) are beneficial only with STEMI
2. Heparin is best for NSTEMI
3. GPIIb/IIIa inhibitors are best for NSTEMI and those
undergoing PCI and stenting

Calcium-channel blockers & warfarin:


No mortality benefit in ACS

In non-ST elevation ACS, when all medications have been


given and the patient is not better, urgent angiography and
possibly angioplasty (PCI) should be done.
Not better means:
Persistent pain
S3 gallop or CHF developing
Worse EKG changes
Rising troponin levels

LMW heparin superior to unfractionated heparin for


mortality benefit
MTB S2CK

p. 70

MTB S2CK

Complications of Acute MI

Acute Coronary Syndromes


STEMI

Non-STEMI/UA
Aspirin/Clopidogrel
Beta Blockers
Statin
ACE
Morphine
p
Nitrates

PCI
If available <90 min
after patient arrives
1.
2.

Emergency CABG
Failed PCI
Ischemia refractory ALL
therapy

MTB S2CK

p. 70

Heparin
Early PCI

Thrombolytics
If PCI not available.
Use within 12 hrs from
start of chest pain

p. 70

Absolute contraindications to
thrombolytics
Major bleeding (bowel/brain)
Recent surgery (within last 2
weeks)
Severe hypertension (>180/110)
Nonhemorrhagic stroke last 6
months

Excellent source of:


What is the most likely diagnosis? questions,
(most common question on Step 2)
All MI complications can result in low BP
Hypotension: Not help determine diagnosis

MTB S2CK

p. 71

Complications of Acute MI/Bradycardia

Complications of Acute MI/Bradycardia

Heart rate:
Key to establishing diagnosis

Third-degree (complete) AV block:

Sinus bradycardia:
Very common with MI
From ischemia of sinoatrial (SA) node

MTB S2CK

p. 71

Cannon a waves
Distinguishes 3rd degree block from sinus
bradycardia before EKG
From atrial systole against closed tricuspid
Tricuspid valve closed because essence of
third-degree block is atria and ventricles
contracting separately
Atria/ventricles out of coordination with each
other
MTB S2CK

p. 71

17

Complications of Acute MI/Bradycardia

Complications of Acute MI/Tachycardia

Cannon is jugulovenous wave


bouncing up into the neck
Look for: right ventricular infarction and
third-degree AV block
All symptomatic bradycardias:
1. Treated first with atropine
2. Then place pacemaker if the atropine
is not effective
3. Pace all permanent 3rd degree blocks

Right Ventricular Infarction


Look for:
New inferior wall MI & clear lungs on
auscultation
Cant g
get blood into lungs
g if blood cant enter
heart
Flip EKG leads from left side to right side of
chest
Most specific finding:
ST elevation in right lead 4 (RV4)

MTB S2CK

p. 71

MTB S2CK

p. 71

Complications of Acute MI/Tachycardia

Complications of Acute MI/Tachycardia

Right coronary supplies:

Tamponade/Free Wall Rupture


Usually takes several days after infarction for
wall to scar & weaken enough for rupture
Look for sudden loss of pulse
Lungs: Clear
Cause of Pulseless Electrical Activity (PEA)
Test: Echocardiography
Treat: Pericardiocentesis is done on way into
operating room for repair

Right ventricle (RV)


AV node
Inferior wall of the heart
Up to 40% with inferior wall myocardial
infarctions (IWMI) have right ventricular
infarction
Treat RV infarct with high-volume fluid
Avoid nitroglycerin (markedly worsens filling)
MTB S2CK

p. 71

MTB S2CK

p. 72

Complications of Acute MI/Tachycardia

Complications of Acute MI/Tachycardia

Ventricular Tachycardia/Ventricular
Fibrillation
Both can cause sudden death
No way to distinguish without EKG if no
pulse
Cardiovert/ Defibrillate

Valve or Septal Rupture


Both present with new onset murmur and
pulmonary congestion
Mitral regurgitation best heard at apex with
radiation to axilla
Ventricular septal rupture best heard at lower left
sternal border

These complications are the reason patients


with acute MI are monitored in an ICU for the
first several days after the infarction
MTB S2CK

p. 72

Look for a step-up in oxygen saturation as you


go from the right atrium to the right ventricle to
hand you the diagnosis of septal rupture
MTB S2CK

p. 72

18

Complications of Acute MI/Tachycardia

Most accurate test: Echocardiogram for both


You cant always depend on buzzwords like
step-up for oxygenation
Often, only numbers are presented:
42% oxygen saturation found on blood from
right atrium and 85% saturation found in right
ventricular sample

MTB S2CK

p. 72

Intraaortic Balloon Pump

Intraaortic balloon pump (IABP):


Answer for pump failure from anatomic
problem that can be fixed in operating room
IABP contracts & relaxes in sync with natural
heartbeat
Helps give a push forward to blood
IABP is never a permanent device (bridge to
surgery)
MTB S2CK

p. 72

Extension of the Infarction/Reinfarction

Aneurysm/Mural Thrombus

Second event infarct


Look for:
Recurrence of pain
New rales
Bump up in CK-MBs
Sudden onset pulmonary edema
Actions:
Repeat EKG
Re-treat with angioplasty or thrombolytics

Detected with echocardiography


Most aneurysms dont need specific
therapy
Mural thrombi are treated with heparin
followed by warfarin

Continue: aspirin, metoprolol, nitrates, ACE, statins


MTB S2CK

p. 72 73

MTB S2CK

What is the Most Likely Diagnosis?

p. 73

What is the Most Likely Diagnosis?

Diagnosis

Key feature

Diagnosis

Key feature

Third degree AV
Block

Bradycardia, cannon a
Waves

Valve rupture

New murmur,
rales/congestion

Sinus bradycardia

No cannon a waves

Septal rupture

Tamponade/wall
Rupture

Sudden loss of pulse, JVD

New murmur, increase in


oxygen saturation on
entering the right ventricle

RV infarction

IWMI in history, clear lungs,


hypotension

Ventricular
fibrillation

Loss of pulse, need EKG to


answer question

MTB S2CK

p. 73

MTB S2CK

p. 73

19

Preparation for Discharge from Hospital

Preparation for Discharge from Hospital

Detection of Persistent Ischemia


Stress test prior to discharge
Stress test determines if
angiography needed
Angiography
A i
h d
determines
t
i
need
d ffor
revascularization (angioplasty or bypass
surgery)

Everyone should go home on:


Aspirin
Clopidogrel
Beta blockers (metoprolol)
Statins
ACE inhibitors
Best for anterior wall infarctions because
of high likelihood of developing systolic
dysfunction

Dipyridamole is never the right


choice for coronary artery disease

MTB S2CK

p. 73

MTB S2CK

p. 74

Preparation for Discharge from Hospital

Sexual Issues Postinfarction

Clopidogrel: All MIs, or intolerant of


aspirin, or post-stenting
ARBs: those with cough to ACE inhibitor
Ticlopidine: for rare person intolerant of both
aspirin and clopidogrel

The most commonly tested facts are:


Dont combine nitrates with
sildenafil; hypotension can result
because theyre both vasodilators
Erectile dysfunction postinfarction is
most commonly from anxiety
Most common medication is beta
blocker

Prophylactic antiarrhythmic medications:


Do not use amiodarone, flecainide, or any rhythmcontrolling medication to prevent the development of
ventricular tachycardia or fibrillation. Do not be fooled
by the question describing frequent PVCs and
ectopy. Prophylactic antiarrhythmics increase
mortality.
MTB S2CK

p. 74

MTB S2CK

p. 74

Sexual Issues Postinfarction

Wait after MI for sexual activity


2-6 weeks

If post-MI stress test is normal, any form


of exercise program can be started
including sex

MTB S2CK

Congestive Heart Failure


Definition
Presentation
Diagnosis
Treatment

p. 74

20

Congestive Heart Failure (CHF)/Definition

CHF/Causes of Systolic Dysfunction

Shortness of breath (dyspnea) - essential


feature of congestive heart failure (CHF)
Dysfunction of heart as a pump of blood
Insufficient oxygen delivery and fluid in lungs
Either from:
S t li d
Systolic
dysfunction:
f
ti
Low Ejection Fraction (EF) and dilation of heart
Diastolic dysfunction:
EF is preserved
Heart cant relax and receive blood

Most Common:
Hypertension resulting in
cardiomyopathy
Initially theres preservation of EF
Over time
time, the heart dilates resulting in
systolic dysfunction and low EF
Valvular heart disease of all types
results in CHF

MTB S2CK

p. 74 75

CHF/Causes of Systolic Dysfunction

MTB S2CK

p. 75

CHF/Causes of Systolic Dysfunction

Myocardial infarction (MI) is a very common cause


of dilated cardiomyopathy and decreased EF
Dead or infarcted heart wont pump
CHF most common cause of hospital admission
in USA
MI death rate down from:

Infarction
Dilation

Thrombolytics
Beta blockers
Angioplasty
Aspirin, clopidogrel

Regurgitation

Many are normal, are living with CHF


MTB S2CK

p. 75

CHF
MTB S2CK

p. 75

CHF/Causes of Systolic Dysfunction

CHF/Presentation

Less common causes are:


Alcohol
Postviral (idiopathic) myocarditis
Radiation
Adriamycin (doxorubicin) use
Chagas
Ch
di
disease
and
d other
th iinfections
f ti
Hemochromatosis (also causes restrictive
cardiomyopathy)
Thyroid disease
Peripartum cardiomyopathy
Thiamine deficiency

In addition to dyspnea on exertion look for:


Orthopnea (worse when lying flat, relieved when
sitting up or standing)
Peripheral edema
Rales on lung exam
Jugulovenous distention (JVD)
Paroxysmal nocturnal dyspnea (PND)(sudden
worsening at night, during sleep)
S3 gallop rhythm
(Be prepared to identify the sound on Step 2. It
may be played.)

MTB S2CK

p. 75

MTB S2CK

p. 76 77

21

Heart Sounds Timing in the Cardiac Cycle

CHF/Presentation

The most frequently asked USMLE Step


2 CK question is:
What is the most likely diagnosis?

MTB S2CK

p. 76

MTB S2CK

What is the Most Likely Diagnosis?


for Dyspnea

p. 76

What is the Most Likely Diagnosis?


for Dyspnea

Key feature

Most likely diagnosis is

Key feature

Most likely diagnosis is

Sudden onset, clear lungs

Pulmonary embolus

Circumoral numbness, caffeine


use, history of anxiety

Panic attack

Sudden onset, wheezing,


increased expiratory phase

Asthma
Pallor, gradual over days to
weeks
k

Anemia

Slower, fever, sputum, unilateral


rales/rhonchi

Pneumonia
Pulsus paradoxus, decreased
heart sounds, JVD

Tamponade

Decreased breath sounds


unilaterally, tracheal deviation

Pneumothorax
Palpitations, syncope

Arrhythmia
(of almost any kind)

MTB S2CK

p. 77

MTB S2CK

p. 77

What is the Most Likely Diagnosis?


for Dyspnea

What is the Most Likely Diagnosis?


for Dyspnea

Key feature

Most Likely diagnosis is

Dullness to percussion at base

Pleural effusion

Long smoking history, barrel chest

COPD

All of these will lack:


Orthopnea/PND
S3 gallop

Recent anesthetic use,, brown blood


not improved with oxygen, clear
lungs on auscultation, cyanosis

g
Methemoglobinemia

Burning building or car, wood


burning stove in winter, suicide
attempt

Carbon monoxide poisoning

MTB S2CK

p. 77

MTB S2CK

p. 77

22

CHF/Diagnostic Tests

CHF/Ejection Fraction

Echocardiography
Most important test of CHF
There is no OTHER way to distinguish

What is the best initial test?


Transthoracic echo

Systolic vs. Diastolic dysfunction


Will NOT distinguish by:
History
Physical
Tests (e.g., EKG, chest X-ray, or BNP)
MTB S2CK

p. 77

Whats the most accurate test?


Multiple
Multiple-gated
gated acquisition scan (MUGA) or
nuclear ventriculography
Transesophageal echocardiography (TEE):
More accurate for valves
MTB S2CK

p. 78

CHF/Diagnostic Tests

CHF/Diagnostic Tests

When should you answer nuclear


ventriculography?
Rarely needed
Person receiving chemotherapy with
doxorubicin
Trying to give max dose to cure
lymphoma
But not cause cardiomyopathy

When should you answer BNP?


Acute SOB
Etiology unclear
You cant wait for echo
Normal BNP excludes CHF

MTB S2CK

p. 78

MTB S2CK

Tests Used to Determine Etiology of CHF

Test

Etiology of CHF

EKG

MI, heart block

Chest X ray

Dilated cardiomyopathy

Holter monitor

Paroxysmal arrhythmias

Cardiac
catheterization

Precise valve diameters,


septal defects

CBC

Anemia

MTB S2CK

p. 78

p. 78

Tests Used to Determine Etiology of CHF


Test

Etiology of CHF

T4/TSH

Both high and low thyroid levels


cause CHF

Myocardial biopsy

Excludes sarcoid, amyloid


Rarely needed
Can biopsy other sites

Swan Ganz right heart


catheterization

Distinguishes CHF from ARDS; not


routine

MTB S2CK

p. 78

23

CHF/Treatment

CHF/Treatment

Systolic Dysfunction (Low EF)


ACE inhibitors or angiotensin receptor
blockers (ARBs)
Beta blockers
Spironolactone,
S i
l t
E l
Eplerenone
Diuretics
Digoxin

ACE/ARBs
All patients with systolic dysfunction
All stages of disease
Beneficial effects: any drug in class
When are ARBs Next best step?
ACE induced cough

MTB S2CK

p. 79

MTB S2CK

p. 79

CHF/Treatment

CHF/Treatment

Beta Blockers
Not clearly any drug in class
Evidence only for:

Beta Blockers are:


Anti-ischemic
Decrease heart rate
Decrease O2 consumption
Antiarrhythmic

Metoprolol
Bisoprolol
Carvedilol

MTB S2CK

p. 79

MTB S2CK

p. 79

CHF/Treatment
Which of the following is the MCC of death from
CHF?
a.
b.
c
c.
d.
e.

Pulmonary edema We can fix this almost all the time


Myocardial infarction CHF doesnt cause MI
Arrhythmia/sudden death
Emboli Clots rare in CHF unless AFib
Myocardial rupture This from MI, not CHF

MTB S2CK

p. 79

Spironolactone
Benefit from inhibition of aldosterone
Only proven for more advanced CHF
(class III and IV) with dyspnea on
minimal exertion or at rest
What is the most common adverse
effect?
Hyperkalemia
Gynecomastia
MTB S2CK

p. 79 80

24

CHF/Treatment

When is eplerenone the answer?


When spironolactone leads to
antiandrogenic effects (e.g.,
gynecomastia)

CHF/Treatment

Diuretics
ED: Acute pulmonary edema
Office: Combination with ACEi or ARB
Furosemide, torsemide, or bumetanide equal
Spironolactone, although a diuretic, is not
used at doses where it has a diuretic effect
Diuretics control symptoms of CHF.
They do not lower mortality.

MTB S2CK

p. 80

CHF/Treatment

Digoxin
Digoxin does NOT lower mortality in CHF
This is often the single most important
question concerning CHF on USMLE
Digoxin will:
Control symptoms
Decrease frequency of hospitalizations
No positive inotropic agent (digoxin,
milrinone, amrinone, dobutamine) has
been proven to lower mortality
MTB S2CK

p. 80

MTB S2CK

p. 80

CHF/Treatment
Devices with mortality benefit

1. Implantable defibrillator: Ischemic CM


& EF < 35%. Remember: Arrhythmia and
sudden death are MCC of death in CHF
2. Biventricular pacemaker: EF < 35% &
wide QRS > 120 ms with persistent
symptoms

MTB S2CK

p. 80 81

CHF/Treatment

CHF/Treatment

Biventricular pacemaker is NOT a dualchamber pacemaker


Dual-chamber pacer has wire in right
ATRIUM and right VENTRICLE
Biventricular pacemaker resynchronizes the
heart when theres a conduction defect
Defers/delays need for cardiac
transplantation
Symptoms markedly improved

Transplantation
Symptoms despite maximal medical
therapy (ACE, BB, spironolactone,
diuretics, digoxin) and possibly
biventricular pacemaker
Warfarin
Always wrong in absence of clot in
heart

MTB S2CK

p. 81

MTB S2CK

p. 81

25

CHF/Treatment

Mortality Benefit in Systolic Dysfunction


ACEi/ARBs
Beta blockers
Spironolactone
Hydralazine/nitrates
Implantable defibrillator
Calcium-channel blockers (CCBs)
can actually raise mortality
MTB S2CK

p. 81

CHF/Treatment

Diastolic Dysfunction
(CHF with preserved EF)
Less clear
Beta blockers have clear benefit
No mortality benefit in diastolic
dysfunction
Digoxin clearly has no benefit

MTB S2CK

p. 81

CHF/Treatment

Diuretics control symptoms of fluid


overload
Do not confuse diastolic dysfunction
from hypertrophic CM with hypertrophic
obstructive cardiomyopathy (HOCM)
Diuretics are contraindicated in HOCM
because they increase obstruction

MTB S2CK

Acute Pulmonary Edema


Definition
Presentation
Diagnosis
Treatment

p. 81

Acute Pulmonary Edema

Acute Pulmonary Edema/Presentation

Definition
Worst (most severe) form of CHF
Rapid fluid accumulation in lungs

How do I answer What is the most likely diagnosis?


Acute shortness of breath with:
Rales
JVD
S3 gallop
Edema
Ed
Orthopnea
MAY be:
Ascites & enlargement of liver/spleen from chronic
passive congestion of right side of heart

MTB S2CK

p. 82

MTB S2CK

p. 82

26

S3 Gallop Rhythm

JVD

Pitting Edema

Wet Crackles

What is the best initial test?

MTB S2CK

p. 82 82

Pulmonary edema
with cephalization
of flow and
engorged
pulmonary veins
Source: Saba Ansari, MD.

Brain Natriuretic Peptide (BNP) is the answer if:


Diagnosis not clear
Normal BNP excludes CHF
Chest X-ray shows:
Vascular congestion
Filling
Filli off bl
blood
d vessels
l ttoward
dh
head
d ((cephalization
h li ti
of flow)
Flow mostly at base because of gravity
Enlargement of heart
Effusion

Pulmonary Edema

MTB S2CK

p. 83

27

Acute Pulmonary Edema/Diagnostic Tests

Oximetry/Arterial Blood Gas (ABG)


Hypoxia expected
Respiratory alkalosis from
hyperventilation
CO2 leaves
l
more easily
il than
th oxygen
enters

MTB S2CK

p. 83

Acute Pulmonary Edema/


Which Test is most likely to alter acute management?

Answer!!!
EKG
A-fib, Atrial flutter, or V-tach
What to do first?
Synchronized cardioversion
Restore atrial systole = Return atrial
contribution to cardiac output
MTB S2CK

p. 83

Acute Pulmonary Edema/Diagnostic Tests

Acute Pulmonary Edema/Diagnostic Tests

EKG
Diseased hearts need atrial systole more than
normal hearts
Up from 10% to 30 - 40% of cardiac output
Diseased hearts means:

Echocardiography
MUST be done on all patients
ONLY WAY to determine systolic or
diastolic dysfunction
No difference in initial therapy
Huge difference in chronic therapy

Dil
Dilated
t d cardiomyopathy
di
th
Valvular heart disease

If acute pulmonary edema is from arrhythmia


fix it fast with..
Cardioversion!!!!!
MTB S2CK

p. 83

MTB S2CK

p. 83

Acute Pulmonary Edema/Treatment


74-woman comes to ED with the acute onset of SOB,
RR of 38/minute, rales to apices, S3 gallop, and JVD.
What is the best initial step in management?
Will be done, but first you have to treat
a. Oximeter
Should be done, but not urgently
b. Echocardiography
c. Intravenous furosemide
d. Ramipril
Although they are used in CHF, they are not
e. Metoprolol
Part of the treatment of acute setting
f. Nesiritide
No mortality benefit. Not better than nitrates

MTB S2CK

p. 84

Preload Reduction
Initial therapy:
Oxygen
Loop diuretics (e.g., furosemide or
b
bumetinide)
ti id )
Morphine
Nitrates

MTB S2CK

p. 84

28

Acute Pulmonary Edema/Treatment

Acute Pulmonary Edema/Treatment

The majority in acute pulmonary edema


can be managed with preload reduction
Removing 1 to 2 liters of fluid is best
Nesiritide does NOT work better than
other agents

What do you do if the questions say:


Preload reduction hasnt been effective?
Positive Inotropic Agents
Dobutamine in ICU
Amrinone and milrinone
Phosphodiesterase inhibitors that perform
the same role
Increase contractility

Decrease afterload

MTB S2CK

p. 84

MTB S2CK

Acute Pulmonary Edema/Treatment

p. 84

Acute Pulmonary Edema/Treatment

Positive Inotropic Agents


Digoxin too slow!
Increases contractility, but
Needs several weeks to work
No benefit of digoxin in acute setting

Afterload Reduction
ACEi and ARBs:
Used on discharge
Long-term use with systolic dysfunction (low
EF)
Nitroprusside in ICU
Hydralazine alternate for ACE/ARB
Heparin is always wrong for acute
pulmonary edema, use for clots

MTB S2CK

p. 85

MTB S2CK

p. 85

Valvular Heart Disease/Definition

Valvular Heart Disease


Definition
Presentation
Diagnosis
Treatment

All can be congenital in nature


Rheumatic fever can lead to any form
and mitral stenosis is most common
Aging = Aortic stenosis
Regurgitant
R
it t via
i HTN & iischemia
h i
Infarction regurgitation
Regurgitation
dilation

MTB S2CK

p. 85

29

Valvular Heart Disease/Presentation

Valvular Heart Disease/Diagnostic Tests

All forms associated with SOB and CHF


Only murmurs are specific for presentation
Right-sided heart lesions (tricuspid and
pulmonic valve) increase with inhalation
Inhalation venous return to right side
Left-sided lesions (mitral and aortic valve)
with exhalation
Exhalation squeezes blood out of lungs into
left side

Best initial test: Echocardiogram


Transesophageal: more sensitive &
specific than transthoracic echo
Most accurate test: Catheterization
C th t i ti
Catheterization:
Most precise valvular diameter
Most exact pressure gradient across
valve

MTB S2CK

p. 85

MTB S2CK

p. 85

Valvular Heart Disease/Diagnostic Tests

Valvular Heart Disease/Treatment

EKG:
Not specific with valvular heart disease
Shows hypertrophy of chambers
Cannot confirm diagnosis from EKG
Chest X
X-ray:
ray:
Shows enlargement chambers
Precise anatomic correlation poor
X-ray is neither the most accurate
test nor the best initial test

All forms associated with fluid overload


All benefit from diuretics
Meds alone cant improve stenotic
lesions
Mitral
Mit l stenosis:
t
i Dilated
Dil t d with
ith balloon
b ll
Aortic stenosis: Surgical replacement

MTB S2CK

p. 85

MTB S2CK

p. 86

Valvular Heart Disease/Treatment

Valvular Heart Disease/Treatment

Regurgitant lesions
Respond best to vasodilators
ACEi/ARBs, nifedipine, or hydralazine
Surgical replacement must be done before
heart dilates too much
If heart dilates excessively valve replacement
will not be able to correct decrease in
systolic function
If myocardium stretches too much it wont
return to normal size and shape

Assess ventricular size based on:


End-systolic diameter
Ejection fraction
Expansion of end-systolic diameter (must
replace
p
the valve))

MTB S2CK

p. 86

Endocarditis prophylaxis is not


indicated for any of these valve lesions.

MTB S2CK

p. 86

30

Mitral Stenosis (MS)

Mitral Stenosis

Definition/Etiology
Most often from rheumatic fever
Extremely uncommon in US
Low incidence of acute rheumatic fever
Treatment if symptomatic
Do not treat asymptomatic MS

MTB S2CK

p. 86

Look for: Pregnancy and Immigrant


Pregnancy:
50% increase in plasma volume
Contraction of uterus squeezes
squeezes 500 mL
extra into central circulation
Most with MS are immigrants to the US
Come from places where acute rheumatic
fever is still common
MTB S2CK

p. 86

Mitral Stenosis

Mitral Stenosis/Presentation

Presentation
SOB and CHF
AND!
Unique features of presentation:

Enlarged left atrium in


mitral stenosis compresses
the esophagus causing
dysphagia

young adult patients

MTB S2CK

p. 86 87

Mitral Stenosis

MTB S2CK

p. 87

Source: Andrew Peredo

Dysphagia: Left atrium (LA) presses on


esophagus
Hoarseness: LA presses on laryngeal nerve
Atrial fibrillation & stroke from enormous
LA
MS: Look for
Hemoptysis

Mitral Stenosis headphones required

Physical findings
Diastolic murmur after opening snap
Squatting & leg raising increase it!
Increased venous return increases
murmur

MTB S2CK

p. 87

MTB S2CK

p. 87

31

Mitral Stenosis

Diagnostic tests
Echo
TTE: Best initial test
TEE more accurate than TTE
Catheterization: Most accurate test
EKG
Atrial rhythm disturbance, particularly atrial
fibrillation, very common
LA hypertrophy: Biphasic P wave: V1 and V2
MTB S2CK

p. 87

Mitral Stenosis/Treatment

Diagnostic tests
Chest X-ray: Left
Atrial Hypertrophy
Straightening of left
heart border
Elevation of left main
mainstem bronchus
Second bubble
behind heart

Diuretics & sodium restriction:


When fluid overload present
Balloon valvuloplasty
With a catheter
Valve replacement
Only
O l when
h a catheter
h
procedure
d
cannot be
b done
d
or fails
Warfarin
A-fib to INR 2 to 3
Rate control: Digoxin, beta blockers, or diltiazem/
verapamil

MTB S2CK

p. 87 88

Source : Andrew Peredo

Mitral Stenosis/Diagnostic Tests

MTB S2CK

p. 88

Aortic Stenosis

Aortic Stenosis

Definition/Etiology
Congenital bicuspid valve
Increasing calcification with age
Presentation
Angina (most common)
Syncope
CHF:

Murmur
Systolic, crescendo-decrescendo
Peaks in diamond-shape mid-systole
Heard best at 2nd right intercostal
space
Radiates to carotids

Poorest prognosis
2-year average survival
Source: Shwan Christian

MTB S2CK

p. 88

MTB S2CK

p. 88

32

Aortic Stenosis

Aortic Stenosis headphones required

Murmur
Valsalva & standing
Decrease intensity of murmur
Less venous return = Less Murmur
Handgrip
Softens murmur
Less blood ejected = Less murmur

MTB S2CK

p. 88

MTB S2CK

p. 88

Aortic Stenosis/Diagnostic Tests

Aortic Stenosis/Treatment

TTE, then TEE,


then catheterization
Chest X-ray

Replacement:
Only truly effective therapy for AS
Diuretics CHF but dont tolerate volume
depletion well
Balloon valvuloplasty
p
y
Not routine for AS
AS calcification doesnt improve well with
balloon valvuloplasty
Only if surgery isnt an option
Unstable/fragile patients

Left ventricular
hypertrophy

EKG
LV hypertrophy (LVH)
Source: Nihar Shah, MD.

S V1 + R V5 > 35 mm

MTB S2CK

p. 89

Cardiac enlargement is defined as a heart greater in


diameter than 50% of the total transthoracic diameter

MTB S2CK

p. 89

Mitral Regurgitation

Mitral Regurgitation

Definition/Etiology
MR is abnormal backward flow of
blood through mitral valve that doesnt
fit together
Hypertension
Endocarditis
Myocardial infarction
Papillary muscle rupture
Any heart dilation leads to MR

Presentation
Signs/Symptoms of CHF
Unique finding:
Pansystolic (holosystolic) murmur
Obscures S1 and S2
Radiates to axilla
Handgrip worsens murmur of MR
Handgrip increases afterload
Pushes blood backwards
Handgrip worsens AR and MR

MTB S2CK

p. 89

MTB S2CK

p. 90

33

Mitral Regurgitation/Presentation

Mitral Regurgitation headphones required

Squatting & leg raising worsen MR


Increase venous return to heart
All left-sided murmurs except mitral
valve prolapse (MVP) and hypertrophic
obstructive cardiomyopathy will
increase with expiration
MR diagnosed with echo

MTB S2CK

p. 90

MTB S2CK

p. 90

Mitral Regurgitation

Mitral Regurgitation/Treatment

Treatment
Vasodilators:
ACE or ARBs are best
Decrease rate of progression
Digoxin & diuretics for symptomatic
CHF

Valve replacement:
Indicated when heart dilates
Dont wait for left ventricular end systolic
diameter (LVESD) to become large
Replace
R l
when:
h
LVESD > 40 mm or EF
< 60%
Valve repair:
Placing a clip or sutures across valve to
tighten

MTB S2CK

p. 90

MTB S2CK

p. 90

Aortic Regurgitation

Aortic Regurgitation

Definition/Etiology
AR caused by:
Anything that makes the heart or aorta dilate:
MI
HTN
Endocarditis
Marfan syndrome or cystic medial necrosis
Inflammatory disorders (e.g., Ankylosing
spondylitis, Reiter syndrome)
Syphilis

Presentation
Besides CHF unique physical findings are:
Wide pulse pressure
Water-hammer (wide, bounding) pulse
Quincke pulse (pulsations in nail bed)
Hill sign (BP in legs as much as 40 mmHg
above arm BP)
Head bobbing (de Musset sign)

MTB S2CK

p. 90 91

MTB S2CK

p. 91

34

Aortic Regurgitation

Aortic Regurgitation headphones required

Murmur
Diastolic, decrescendo murmur
Heard best: Lower left sternal border
Valsalva & Standing: Softer
Handgrip (increases afterload): Worse

MTB S2CK

p. 91

Aortic Regurgitation

Diagnostic tests
EKG & Chest X-ray: LVH

MTB S2CK

p. 91

MTB S2CK

p. 91

Aortic Regurgitation

Treatment
ACEi/ARBs or nifedipine:
Vasodilators
Increase forward flow of blood
Delay progression
Digoxin & Diuretics: Little benefit
Surgical valve replacement:
Acute valve rupture (MI)
Replace valve before LV dilates excessively
EF < 55%
LVESD > 55 mm
MTB S2CK

p. 91

Mitral Valve Prolapse (MVP)

Mitral Valve Prolapse

Presentation
Most often asymptomatic
When symptoms do occur:
Symptoms of CHF usually absent
Most common is:

Common
Considered normal anatomic variant
2% to 5% of population
Particularly in women
Marfan or Ehlers-Danlos syndrome

Atypical chest pain


Palpitations
Panic attack
MTB S2CK

p. 91

MTB S2CK

p. 91

35

Mitral Valve Prolapse

Mitral Valve Prolapse headphones required

Murmur
Presents with:
Midsystolic click
When severe associated with murmur
Mitral regurgitation
Valsalva & Standing worsen MVP
Squatting & Handgrip improve
(diminish) MVP
MTB S2CK

p. 92

MTB S2CK

p. 92

Mitral Valve Prolapse

Mitral Valve Prolapse

Redundant mitral
valve leaflet doesnt
seal allowing
regurgitation

Diagnostic tests
Echocardiography: Best choice
Catheterization: Rarely, if ever, done
Valve replacement: Rarely needed
Source: Andrew Peredo

MTB S2CK

p. 92

MTB S2CK

p. 92

Mitral Valve Prolapse

Treatment
Beta blockers:
When symptomatic
Valve repair
With catheter
Place clip to tighten valve
Stitches valve to tighten leaflets
Surgical repair rarely necessary
Endocarditis prophylaxis not recommended
MTB S2CK

Cardiomyopathy &
Pericardial Disease
Dilated Cardiomyopathy
Hypertrophic
h Cardiomyopathy
d
h
Restrictive Cardiomyopathy
Acute Pericarditis
Pericardial Tamponade
Constrictive Pericarditis

p. 92

36

Cardiomyopathy/Definition

Cardiomyopathy/Etiology

Abnormal function of heart muscle


Frequent valve or auscultory
abnormalities
But!
It originates
i i t from
f
an abnormally
b
ll
contracting or relaxing myocardium

Can be dilated, hypertrophic, or restrictive


The terms dilated cardiomyopathy, systolic
dysfunction, and low EF are often used
interchangeably
Hypertrophic cardiomyopathy
interchangeable with diastolic dysfunction
Or!!
Cardiac failure with preserved EF

MTB S2CK

p. 92

MTB S2CK

p. 93

Cardiomyopathy/Presentation

Cardiomyopathy/Diagnostic Tests

All forms give:


SOB, particularly worsened by exertion
Edema
Rales
JVD

Echocardiography:
Best initial test
Often Most accurate test used
EKG & Chest X-ray:
Should be performed
Nothing specific on them confirm the
diagnosis
Murmurs not increasing
with expiration:
HOCM
MVP

MTB S2CK

p. 93

MTB S2CK

p. 93

Cardiomyopathy/Treatment

Dilated Cardiomyopathy

All treated with diuretics


Other treatments based on type of
cardiomyopathy
The only real functional difference in
management and answers to questions
is: TREATMENT

In addition to previous MI and ischemia,


dilated cardiomyopathy can be from:

MTB S2CK

p. 93

Alcohol
Postviral myocarditis
Radiation
Toxins (e.g., doxorubicin)
Chagas disease

MTB S2CK

p. 93

37

Dilated Cardiomyopathy

Dilated Cardiomyopathy/Treatment

Dyspnea, gallop, edema, testing same


as section on CHF

MTB S2CK

Multiple meds mortality


ACEi (or ARBs)
Beta blockers (metoprolol, carvedilol)
Spironolactone (or eplerenone)
Symptom Control ONLY:
Diuretics & Digoxin

p. 93

MTB S2CK

p. 93

Dilated Cardiomyopathy/Treatment

Hypertrophic Cardiomyopathy

QRS wide (> 120 ms)


Biventricular pacemaker
Improve both symptoms and survival
Automated implantable
cardioverter/defibrillator
di
t /d fib ill t has
h mortality
t lit
benefit

HTN - MCC
MUST distinguish between hypertrophic
cardiomyopathy (HCM) and HOCM
HCM: Reaction to BP
Heart hypertrophies
yp
p
to carry
y load
Develops difficulty relaxing in diastole
Cant relax = Cant receive blood
Patient becomes short of breath

MTB S2CK

p. 93 94

MTB S2CK

Hypertrophic Obstructive Cardiomyopathy

MTB S2CK

p. 94 95

Hypertrophic Cardiomyopathy

Source: Andrew Peredo

Genetic disorder
Abnormal shape of
septum
Asymmetrically
hypertrophied septum
and valve leaflet blocks
blood leaving the heart

p. 94

S4 gallop
Fewer signs of right heart failure
Less ascites
Less enlargement of liver and spleen

MTB S2CK

p. 94

38

HOCM/Diagnostic Tests

HOCM/Presentation

Dyspnea
Chest pain
Syncope & lightheadedness
Sudden death, particularly in healthy athletes
Worsened by heart rate
(e.g., exercise, dehydration, and diuretics)

Worsened by
size

Systolic anterior motion


(SAM) of mitral valve is
classic for HOCM. It
contributes
t ib t tto obstruction.
b t ti

Catheterization is most
accurate test to
determine precise
gradients of pressure
across the chamber.

in left ventricular chamber

(e.g., ACEi, ARBs, digoxin, hydralazine, valsalva,


and standing suddenly)
MTB S2CK

Echo (best initial test)


Septum 1.5x thickness of posterior wall

p. 94

EKG: Nonspecific ST and


T wave changes are
common. LVH is
common. EKG can be
normal in a quarter
MTB S2CK

Septal Q waves in the


inferior and lateral
leads are common in
HOCM.

p. 95

Hypertrophic Cardiomyopathy/Treatment

HOCM/Specific Therapy

Beta blockers:
Best initial therapy both HOCM &
HCM
Diuretics help HCM

Implantable defibrillator:
HOCM with syncope
Ablation of septum:
Catheter placing absolute alcohol in muscle
Causes small infarctions
If symptoms
t
persist:
i t Surgical
S i l myomectomy
t

Diuretics harm HOCM


Digoxin and spironolactone are
definitely always wrong in
hypertrophic cardiomyopathy.
MTB S2CK

p. 95

Surgical myomectomy
is the therapy only if all
medical and catheter
procedures fail.
MTB S2CK

In HOCM, ACEi and


diuretics definitely dont
help. This is the major
difference between
HOCM and HCM

p. 95

Restrictive Cardiomyopathy
Hypertrophic versus Dilated Cardiomyopathy

Hypertrophic

Dilated

Beta Blockers

Yes

Yes

Diuretics

Yes

Yes

ACEi/ARB

Unclear benefit

Yes

Spironolactone

No

Yes

Digoxin

No

Yes

MTB S2CK

p. 95

Combines worst aspects of dilated and


hypertrophic cardiomyopathy
Heart neither contracts nor relaxes
Infiltrated with substances creating
immobility

MTB S2CK

p. 96

39

Restrictive Cardiomyopathy

Restrictive Cardiomyopathy

Causes are:
Sarcoidosis
Amyloid
Hemochromatosis
Endomyocardial fibrosis
Scleroderma

Presentation
Dyspnea: Most common
Right heart failure
Ascites, edema, JVD
Enlargement
g
of liver & spleen
p
Pulmonary hypertension: Common
Kussmaul sign:
jugulovenous pressure on inhalation

MTB S2CK

p. 96

MTB S2CK

Restrictive Cardiomyopathy

Restrictive Cardiomyopathy

Diagnosis
Echocardiography: Initial test
EF normal or elevated
EKG: Low voltage
Amyloid: Speckling of septum on echo or
cardiac MRI
Most accurate test:
Endomyocardial biopsy
Rarely done
Diagnosis made from biopsies elsewhere
MTB S2CK

p. 96

Squatting/Leg raising

Standing/Valsalva

Increases both

Decreases both

Mitral and aortic


regurgitation

Increases both

Decreases both

Mitral valve
prolapse

Decrease

Increase

HOCM

Decrease

Increase

p. 96

p. 96

Murmurs and the Effects of Maneuvers

Mitral and aortic


stenosis

MTB S2CK

Treat underlying cause


Diuretics relieve some pulmonary HTN
and signs of right heart failure
No other clear therapy

MTB S2CK

Murmurs and the Effects of Maneuvers

Lesion

p. 96

More blood increases all


murmurs except MVP and
HOCM.

Less blood decreases all


murmurs except MVP and
HOCM.
MTB S2CK

p. 96

40

Effects of Maneuvers

Standing from a squatting position:


Opens vessels of legs
Decreases venous return to heart
Valsalva:
Exhalation against a closed glottis
Increases intrathoracic pressure
Decreases venous return to heart
MTB S2CK

p. 97

Effects of Maneuvers

MVP & HOCM: Worsen with diuretics


Diuretics decrease LV size
Diuretics worsen MVP and obstruction
of HOCM
Standing
St di and
d valsalva
l l worsens them
th

MTB S2CK

p. 97

Effects of Maneuvers
Amyl nitrate:
Direct arteriolar vasodilator
Decreases afterload
Simulates ACE inhibitors or ARBs on heart
Valvular disease treated with ACEi/ARB will
improve with amyl nitrate
Improves AR and MR
Handgrip = Fuller left ventricle
Amyl nitrate = ACEi = Emptier left ventricle
MTB S2CK

p. 97

Effects of Maneuvers

Standing or Valsalva = Diuretic use


Stenotic & regurgitant murmurs all
treated with diuretics and salt restriction
Standing
St di & Valsalva
V l l will
ill improve
i
th
them

MTB S2CK

p. 97

Effects of Maneuvers

Handgrip
Handgrip increases afterload
Contraction of arm muscles compresses
arteries
Decreases emptying of heart
Opposite of ACE inhibitor
Worsens AR and MR

MTB S2CK

p. 97

Effect of Maneuvers on
Intensity (loudness) of Murmurs
Lesion

Handgrip

Amyl nitrate

Aortic stenosis

Decreases

Increases

Mitral stenosis

No effect

No effect

A i regurgitation
Aortic
i i

I
Increases

D
Decreases

Mitral regurgitation

Increases

Decreases

Mitral valve prolapse

Decreases

Increases

HOCM

Decreases

Increases

MTB S2CK

p. 97

41

Effect of Maneuvers on
Intensity (loudness) of Murmurs

Handgrip and amyl nitrate have no


meaningful effect on mitral
stenosis in the same way ACEi
stenosis,
has no meaningful effect on MS

MTB S2CK

p. 97

Pericardial Disease

Causes of pericarditis, pericardial tamponade


& constrictive pericarditis overlap
If the etiology of pericarditis extravasates
fluid, then tamponade can occur
If the cause of pericarditis is chronic, fibrosis
and calcification of pericardium lead to
constrictive pericarditis

MTB S2CK

p. 98

Pericarditis/Etiology

Pericarditis/Etiology

Any
Infection
Inflammatory disorder
Connective tissue disorder
Chest trauma or cancer near the heart
can cause pericarditis

Systemic lupus erythematosus:


Most common connective tissue disorder
But
Wegener granulomatosis
Goodpasture syndrome
Rheumatoid arthritis
Polyarteritis nodosa and others
can cause pericarditis

Most common infection is viral


MTB S2CK

p. 98

MTB S2CK

p. 98

Pericarditis/Presentation

Pericarditis/Diagnostic Tests

What Is the Most Likely Diagnosis?


Sharp chest pain
Changes with respiration
Changes with position of body
Worsened by lying flat
Improved by sitting up

EKG shows ST segment elevation in all leads, but


the most specific finding is PR segment depression

MTB S2CK

p. 98

MTB S2CK

p. 98 99

42

Pericarditis/Treatment

Pericardial Tamponade/Etiology

Treat cause
Majority: Idiopathic

Any cause of pericarditis can extravasate


enough fluid to cause tamponade
Compression heart starts on right side
Walls are thinner
50 mL acutely
y cause tamponade
p
Over weeks to months, pericardium stretches
for 2 L of fluid
Can also be from trauma
Bleed requires emergent thoracotomy

Idiopathic presumed viral


Coxsackie B virus

T
Treated
t d with
ith NSAIDs
NSAID (e.g.,
(
ibuprofen,
ib
f
naproxen)
Colchisine - recurrences

MTB S2CK

p. 99

Pericardial Tamponade/Presentation

MTB S2CK

p. 99

Pericardial Tamponade

What Is the Most Likely Diagnosis?


Hypotension
Tachycardia
Distended neck veins
Clear lungs
Which of the following physical findings is
most likely to be associated with this patient?
Pulsus paradoxus
Decrease of more than 10 mmHg in
systolic blood pressure on inhalation
MTB S2CK

p. 99

Pericardial Tamponade

EKG:
Electrical alternans (different heights of QRS
complexes between beats)

MTB S2CK

p. 100

Pericardial Tamponade

Chest X-ray:
Enlarged cardiac shadow expanding in both
directions (globular heart)
Echocardiogram:
Right atrial and ventricular diastolic collapse
Right heart catheterization:
Equalization of pressures in diastole

MTB S2CK

p. 100

MTB S2CK

p. 100 101

43

Pericardial Tamponade
78-year-old man with lung cancer experiences several days of
increasing SOB. Hes lightheaded today. BP 106/70; pulse
112; JVD present; lungs: clear, BP drops to 92/58 on
inhalation.
Which is most appropriate to confirm the diagnosis?

a.
b
b.
c.
d.
e.

EKG Low voltage non-specific. Rare alternans


y; globular
g
heart later
Chest X-ray
X ray Normal acutely;
Echocardiogram
Most accurate;
Right heart catheterization should do echo first
Cardiac MRI Not clearly right for

MTB S2CK

Treatment
Pericardiocentesis
Needle rapidly reexpands the heart
IV fluids
A hole or window recurrent cases
Diuretics will decrease intracardiac
filling pressure and may markedly
worsen collapse of right side of heart

ANYTHING at this time

p. 100

MTB S2CK

p. 101

Constrictive Disease

Constrictive Disease

Any cause of pericarditis with


calcification and fibrosis
Prevents filling of right side of heart
Tuberculosis

What Is the Most Likely Diagnosis?


Signs of right heart failure such as:
Edema
Ascites
Enlargement of liver and spleen
JVD

Constrictive pericarditis is a combination of


physical findings described above with
calcification on chest X-ray
MTB S2CK

p. 101

Constrictive Disease

MTB S2CK

p. 101

Pericardial Knock headphones required

Which physical findings is most likely


associated with this patient?
Kussmaul sign:
Increase in JVD on inhalation
Normally neck veins go down on inhalation
Knock:
Extra heart sound in diastole
From ventricular filling
Heart fills to its maximum, it hits the stiff, rigid
pericardium with a knock
MTB S2CK

p. 101

44

Constrictive Disease

Constrictive Disease

Best initial test


Chest X-ray: Calcification & fibrosis

CT scan or MRI more accurate, not 1st


Echocardiogram: Excludes RV
hypertrophy or cardiomyopathy as
cause
Myocardium moves normally with
constrictive pericarditis

MTB S2CK

p. 101

Commons.wikimedia.org.
Used with permission

MTB S2CK

p. 101

Constrictive Disease

Treatment
Diuretics:
Decompress filling of heart
Relieves edema and organomegaly
Surgical removal of pericardium

MTB S2CK

Peripheral Artery Disease, Aortic


Disease, & Heart Disease
in Pregnancy
Definition
Etiology
Presentation
Diagnosis
Treatment

p. 101

Peripheral Artery Disease (PAD)/Etiology

Peripheral Artery Disease/Presentation

Stenosis of peripheral arteries from:


Diabetes mellitus
Hyperlipidemia
Hypertension
Tobacco smoking

What Is the Most Likely Diagnosis?


Pain in calves on exertion
Relieved by rest
Walking up or down hills
Severe disease causes loss of:

Same causes as Coronary Disease!!

Hair
H i ffollicles
lli l
Sweat glands
Sebaceous glands

The skin becomes smooth and shiny


Spinal stenosis pain is worse when walking
down hills because of leaning back
MTB S2CK

p. 102

MTB S2CK

p. 102

45

Peripheral Artery Disease


Diagnostic tests
Best initial test = Ankle-brachial index (ABI)
ABI: Ratio of BP in ankles to brachial arteries
Normally equal or slightly greater in ankles
because of gravity
If difference between them is > 10% (ABI < 0
0.9),
9)
disease is present
Most accurate test = angiogram
Angiogram not necessary unless revascularization
will be done

Peripheral Artery Disease

Treatment
The best initial therapy is:
Aspirin
Smoking cessation
Cilostazol

Single most effective medication: Cilostazol


Surgery or angioplasty if medical therapy not
effective
In all major vascular disease
control the following:
BP
LDL < 100
Diabetes

There is no routine screening for PAD since


there is no mortality benefit to be obtained
MTB S2CK

p. 102

MTB S2CK

p. 102 103

Aortic Dissection
67-year-old man in ED with sudden onset chest pain
is also felt between his scapulae. He has a history of
HTN and tobacco smoking. BP 169/108.
What is the best initial test?
a.
b.
c.
d.
e.
f.
g.

Chest X-ray
Chest CT Dont show specific changes
MRA, TEE, and CTA
MRA
have same accuracy;
Transesophageal echocardiogram
not most accurate,
Transthoracic echocardiogram
neither best initial
CT angiogram
Angiography Most accurate, but not best initial test

MTB S2CK

Aortic Disease

Most frequently tested points in aortic disease


Diagnosis & treatment of acute dissection
Screening recommendations
Key points for presence of aortic dissection
Pain in between the scapulae
Difference in blood pressure between the arms

MTB S2CK

Author: J. Heuser; commons.wikimedia.org


Used with permission

p. 103

p. 103

Aortic Disease/Treatment
For dissection, Most important step is: Control BP
This can be done with:
Beta blockers
Nitroprusside

Beta blockade decreases shearing forces that


worsens dissection
Beta blockers must start before nitroprusside to
protect against reflex tachycardia of nitroprusside,
which worsens shearing forces

MTB S2CK

p. 104

46

Abdominal Aortic Aneurysm


Which of the following is the most appropriate
screening for aortic aneurysm?
a.
b.
c.
d
d.
e.

Everyone > 50 with CT angiography


Men who ever smoked > 65 with ultrasound
Everyone > 50 with ultrasound
Everyone > 65 with ultrasound
Men > 65 with ultrasound
Incidence of AAA is less in both nonsmokers
and in women, so there is no recommendation
for screening in those groups

MTB S2CK

p. 104

Peripartum Cardiomyopathy
Which of the following is most dangerous to a
pregnant woman?
a.
b.
c.
d.
e.

Mitral stenosis
Peripartum cardiomyopathy
Choose this if p
peripartum
p
Ei
Eisenmenger
phenomenon
h
cardiomyopathy is not one of
the choices
Mitral valve prolapse
Atrial septal defect

Not as dangerous as choices b and c

MTB S2CK

p. 104

Unknown why antibodies are made


against the myocardium in some
pregnant women
LV dysfunction often reversible and
short term
If LV dysfunction does not improve, then
must undergo cardiac transplantation

MTB S2CK

p. 105

Peripartum Cardiomyopathy

Eisenmenger Syndrome

Therapy the same as for dilated cardiomyopathy of


any cause:

From right-to-left shunt from pulmonary


HTN
Develops in person with ventricular septal
defect with left-to-right shunt that eventually
develops pulmonary HTN
When pulmonary HTN becomes very severe
shunt reverses
Right-to-left shunting develops

ACEi/ARB
Beta blockers
Spironolactone
Diuretics
Digoxin

Repeat pregnancy with peripartum cardiomyopathy


will kill 50-70% of women!!
Peripartum cardiomyopathy develops
after delivery in most cases;
ACEi/ARBs must be used
MTB S2CK

p. 105

MTB S2CK

p. 105

47

Eisenmenger Syndrome

Eisenmenger Syndrome

Large left-to-right shunt


(congenital heart defect)

If peripartum cardiomyopathy is not one of


the choices in asking, What is the worst
cardiac disease in pregnant women? then
Look for Eisenmenger in the choices

P l
Pulmonary
HTN

Pregnancy increases plasma volume by 50%.


Mitral stenosis will worsen in pregnancy,
But not as much as peripartum cardiomyopathy
or Eisenmenger

MTB S2CK

p. 105

Reverse of the shunt:


right-to-left shunt
MTB S2CK

p. 105

48

Dermatology Part 1
Dermatology Part 1

Cutaneous Malignancies
Atopic Dermatitis (Eczema)
Psoriasis
Pityriasis Rosea
Seborrheic Dermatitis (Dandruff)
Blistering Diseases

Conrad Fischer, MD
Associate Professor of Medicine
Touro College of Medicine
New York City

Cutaneous malignancies

Malignant Melanoma

More frequent in sun-exposed areas


But!
Not exclusive to those areas
Benign vs. Malignant
Melanoma clinically ABCDE:
A: Asymmetry
B: Border irregularity
C: Color irregularities
D: Diameter > 6 mm
E: Evolution (changing over time)

Greater with pale skin


More sun-exposed areas
Diagnosis: Biopsy
Treatment: Surgical removal
No truly effective chemotherapy
Skin Cancer
More sun, more cancer
Biopsy
Remove
MTB S2CK

p. 363

MTB S2CK

p. 363

Malignant Melanoma

Distinction Between Benign and Malignant

Diagnosis?
Suspicious?

Benign

Malignant

Round

Asymmetric

Even borders

Borders uneven

Color evenly spread

Color uneven

Diameter constant

Diameter increases

Biopsy!
Include entire thickness of lesion if
possible
ibl
Worst prognosis!!

Growing lesions
MTB S2CK

p. 363

MTB S2CK

p. 364

Malignant Melanoma

Benign Lesion

Diagnostic Test
Full thickness biopsy: indispensible
Dont perform shave biopsy
Treatment/Prognosis
Surgical removal
Must include normal skin surrounding lesion
Interferon injection helpful in widespread
disease
Melanoma has strong tendency to metastasize
to brain
MTB S2CK

p. 364

Source: Andrew Peredo

Squamous Cell Cancer

MTB S2CK

p. 364

Squamous Cell Cancer

Sunlight

Organ transplant!!!!

Immunosuppressive drugs!
Squamous starts looking like ulcer
Doesnt heal, grows
Biopsy and remove

Source: James Heilman MD

MTB S2CK

p. 364

Basal Cell Carcinoma

MTB S2CK

p. 364

Basal Cell Carcinoma

Most common skin cancer


Question describes:
Waxy lesion

Shiny like a pearl


U
Unlike
lik melanoma,
l
wide
id margins
i nott
necessary
Shave biopsy fine! (diagnostic)
Recurrence < 5%
Basal cell: use Mohs microsurgery
MTB S2CK

p. 364

MTB S2CK

p. 364

Basal Cell Carcinoma/


Mohs Micrographic Surgery

Remove skin cancer under dissecting


microscope
Very thin slices of skin
Immediate frozen section
VERY precise way to treat skin cancer
Mohs:

Basal Cell Carcinoma/


Mohs Micrographic Surgery

Stop resecting when margin is cancerfree


No removal of wide margins
Mohs Best:
Delicate areas
Eyelid & Ear

Remove cancer
Keep normal
MTB S2CK

p. 365

Kaposi Sarcoma (KS)

MTB S2CK

p. 365

Kaposi Sarcoma (KS)

In past, KS seen only in older men of


Mediterranean origin
Now: AIDS
From HHV-8, which is oncogenic
Reddish/purplish
R ddi h/
li h
More vascular than others

MTB S2CK

p. 365

MTB S2CK

p. 365

Kaposi Sarcoma (KS)

Kaposi Sarcoma/Treatment

Also in GI tract and lung


Only from AIDS via sexual contact
associated with KS
IDU AIDS rarely associated with KS

Unlike other skin cancers, KS not


routinely removed surgically
1. Treat AIDS with antiretrovirals
2. Majority disappear as CD4 improves
3 Intralesional
3.
I t l i
l iinjections
j ti
off vincristine
i i ti or
interferon
4. If these fail, use chemotherapy with
liposomal doxorubicin

MTB S2CK

p. 365

MTB S2CK

p. 366

Actinic Keratoses

Actinic Keratoses

Premalignant
From high-intensity sun exposure in fairskinned people
Very small risk of SCC for each
individual lesion

Richard Usatine, M.D.


Used with permission.

MTB S2CK

p. 366

MTB S2CK

Actinic Keratoses

Actinic Keratoses

Many occur in a single person


Risk is cumulative
Like cervical dysplasia and cervical cancer
Slow to progress
Remove with:
Curettage
Cryotherapy
Laser
Topical 5-fluorouracil
Imiquimod

Richard Usatine, M.D.


Used with permission.

MTB S2CK

p. 366

p. 366

MTB S2CK

p. 366

Actinic Keratoses

Seborrheic Keratoses

Imiquimod
Local immunostimulant
Also for molluscum contagiosum
Condyloma acuminatum

MTB S2CK

p. 366

Extremely common in elderly


Hyperpigmented lesions
Commonly referred to as liver spots
Stuck on appearance
May look like melanoma to some people
Seborrheic keratoses = Zero malignant
potential

MTB S2CK

p. 366

Seborrheic Keratoses

Seborrheic Keratoses

Removed with:
Cryotherapy
Surgery
Laser
Removal (cosmetic reasons)

Source: James Heilman MD

MTB S2CK

p. 366

MTB S2CK

p. 366

Atopic Dermatitis (Eczema)

Atopic Dermatitis (Eczema)/Presentation

Common skin disorder


Associated with overactivity of mast cells &
immune system
Look for history of:
Asthma
Allergic rhinitis
Family history of atopic disorders
Onset < age 5, very rare to start > 30

From premature and idiosyncratic release


of transmitters (e.g., histamine)
Pruritus & scratching is most common
presentation
Scratching leads to scaly rough areas of
thickened skin
On face, neck & skin folds
Popliteal area behind knee

MTB S2CK

p. 366

MTB S2CK

Atopic Dermatitis (Eczema)/Presentation

MTB S2CK

p. 366

p. 366

Atopic Dermatitis (Eczema)/Presentation

Richard Usatine, M.D.


Used with permission.

MTB S2CK

p. 366

Richard Usatine, M.D.


Used with permission.

Atopic Dermatitis (Eczema)/Presentation

Atopic Dermatitis (Eczema)/Skin Care

Itching
scratching
Scratching
more itching
Superficial skin infections from Staphylococcus are
common
Microorganisms driven under epidermis by
scratching
This, in turn,
more itching

1. Stay moisturized: dry skin is more itchy


2. Use a humidifier, especially in winter
3. Avoid bathing, soap, and washcloths (skin
hyperirritable)
4. Avoid brushes,, washcloths,, hot water,, and
anything that rubs on skin
5. Cotton less irritating than wool

Skin thickened because of


scratching and drying =
lichenified
MTB S2CK

Food allergies dont


exacerbate atopic
dermatitis

p. 367

IgE levels: Elevated in


atopic dermatitis
MTB S2CK

p. 367

Atopic Dermatitis (Eczema)/Medical Therapy

Atopic Dermatitis (Eczema)/Medical Therapy

1. Topical corticosteroids: used in flares of


disease (oral steroids only in most severe
acute flares)
2. Tacrolimus and pimecrolimus

3. Antihistamines:

T cellinhibiting agents
L
Longer-term
t
control
t l
Help get patient off steroids
Used systemically in organ transplant
recipients
Prevent organ rejection
Used topically for atopic dermatitis
MTB S2CK

p. 367

Psoriasis/Definition/Presentation

Mild disease: nonsedating drugs


(cetirizine, fexofenadine, loratadine)
Severe disease: hydroxyzine,
diphenhydramine, doxepine

4. Antibiotics (e.g., cephalexin, mupirocin,


retapamulin) when impetigo occurs
5. UV light (phototherapy) for severe
recalcitrant disease
MTB S2CK

p. 367

Psoriasis/Definition/Presentation

Incredibly common
2 million in US
Characterized by silvery, scaly plaques
Not itchy most of the time
Arthritis < 10%
Extensive disease associated with
depression

MTB S2CK

p. 367

MTB S2CK

p. 368

Psoriasis/Treatment
Local Disease
1. Topical high-potency steroids: fluocinonide,
amcinolone, betamethasone, clobetasol
2. Vitamin A & Vitamin D ointment
Helps get patient off steroids
Vitamin D agent is calcipotriene
Steroids cause skin atrophy
3. Coal tar preparation
4. Pimecrolimus and tacrolimus
Used in delicate areas (e.g., face & penis)
Alternative to steroids
Less deforming
MTB S2CK

p. 368

Psoriasis/Treatment

Steroids cause atrophy


Inhibit collagen formation and growth
Convert AAs
glucose
Gluconeogenesis

MTB S2CK

p. 368

Psoriasis/Treatment

Extensive Disease
1.UV light
2.Antitumor necrosis factor (TNF) inhibitors
Etanercept
Adalimumab
Infliximab
Miraculous for severe disease
3.Methotrexate:
Last because of effects on liver & lung
First for psoriatic arthritis
MTB S2CK

p. 368

Pityriasis Rosea

TNF inhibitors reactivate TB


Screen with PPD prior to use

MTB S2CK

p. 368

Pityriasis Rosea

Idiopathic, transient dermatitis


Starts with a single lesion (herald patch)
Then disseminates
Can look like secondary syphilis
But spares palms & soles
Transient
If symptomatic: treat with steroids or UV
light

MTB S2CK

p. 368

MTB S2CK

p. 369

Source: Andrew Peredo

Seborrheic Dermatitis (Dandruff)

Seborrheic Dermatitis (Dandruff)

Increased in:

Hypersensitivity reaction
Dermal infection
Noninvasive dermatophyte organisms
Both topical steroids and antifungal
agents
t (e.g.,
(
kketoconazole)
t
l ) are useful
f l

AIDS
Parkinsons disease

Seborrheic = Benign

MTB S2CK

p. 369

MTB S2CK

p. 369

Pemphigus Vulgaris

Pemphigus Vulgaris

Both idiopathic/autoimmune form and


Drug induced form
Associated with:
ACE inhibitors
Penicillamine
Phenobarbital
Penicillin

Autoantibodies split epidermis, resulting


in:
Bullae that easily rupture
Thin-walled
Involvement
I
l
t off mouth
th
Fluid loss & infection
Widespread (acts like burn)

MTB S2CK

p. 369

MTB S2CK

Pemphigus Vulgaris

p. 369

Pemphigus Vulgaris

Source: commons.wikimedia.org
Source: phil.cdc.gov

Pemphigus Vulgaris

Pemphigus Vulgaris

Characteristic finding: Nikolsky sign


Loss or denuding of skin from mild
pressure

Most accurate test = biopsy


Biopsy shows: autoantibodies on
immunofluorescent (IF) studies

Nikolsky
y sign:
g
Removal of superficial layer of skin
Single sheet while pulling on it
Fingers worth of pressure
MTB S2CK - p. 370

Without treatment,
pemphigus is fatal

MTB S2CK

p. 370

Pemphigus Vulgaris/Treatment

Bullous Pemphigoid

1. Systemic steroids (prednisone)


2. Azathioprine or mycophenolate to wean
patient off steroids
3. Rituximab (anti-CD20 antibodies) or IVIG
in refractory cases

Much milder than pemphigus:


Bullae stay intact
Less loss of fluid
Less infection
Mouth involvement uncommon

MTB S2CK

p. 370

MTB S2CK

Bullous Pemphigoid

p. 370

Bullous Pemphigoid

Biopsy with IF stains is Most accurate test


Best initial therapy: Prednisone
To get patients off steroids, use azathioprine,
cyclophosphamide, or mycophenolate
Mild bullous p
pemphigoid
p g
responds
p
to
erythromycin, dapsone, and nicotinamide
(not niacin)
Nikolsky sign absent in
bullous pemphigoid
Richard Usatine, M.D.
Used with permission.

MTB S2CK

p. 370

Porphyria Cutanea Tarda

Blistering skin disease


Sun-exposed areas
With history of:
Liver disease
Hepatitis C
Alcoholism
Estrogen use
Iron overload (hemochromatosis)
MTB S2CK

p. 370

Porphyria Cutanea Tarda

Hepatitis C:
Most frequently tested association with
PCT
Look for involvement of:
backs of hands & face

MTB S2CK

p. 370

Porphyria Cutanea Tarda/


Diagnostic Tests/Treatment

Most accurate test


Increased uroporphyrins on 24-hr urine
Deficiency of uroporphyrin decarboxylase
activity
Treatment
Correct underlying cause
Stop alcohol
Stop estrogens
Remove iron with phlebotomy
MTB S2CK

Dermatology Part 2
Skin Infections
Drug Reactions
Staphylococcal Scalded Skin Syndrome and
Toxic Shock Syndrome
Acne

p. 370

Impetigo

Impetigo/Treatment

Most superficial bacterial skin infection


Staphylococcus and Streptococcus
invade epidermis
Results in weeping, crusting, oozing, &
draining of skin

Mild disease with topical agents:


Mupirocin
Bacitracin
Retapamulin
Severe disease with oral agents:
Dicloxacillin or cephalexin
Community-acquired MRSA with:
Doxycycline
Clindamycin
Trimethoprim/sulfamethoxazole (TMP/SMZ)

MTB S2CK

p. 371

MTB S2CK

p. 371

10

Erysipelas

Erysipelas/Presentation

Much more severe than impetigo


Occurs at deeper level in skin
Streptococcus > Staphylococcus
Invades dermal lymphatics
Causes bacteremia,
bacteremia leukocytosis
leukocytosis, fever
fever, and
chills
Fatal, if untreated

Bright, red, hot swollen


Lesion on face
Leukocytosis
Often systemic disease

Skin infections can cause glomerulonephritis,


but not rheumatic fever.
MTB S2CK

p. 371

MTB S2CK

Erysipelas/Presentation

p. 371

Erysipelas/Treatment
Although erysipelas is more often from
Streptococci, you must treat for Staphylococcus
as well unless you have a definitive diagnostic test
such as blood cultures.
Treatment of all skin infections is similar
Same answers as for:

Source: Thomas F Sellers

Cellulitis
Folliculitis
Furuncles
Carbuncles

MTB S2CK

p. 371

Erysipelas/Treatment

Erysipelas/Treatment

Mild disease - oral medications:


Dicloxacillin, cephalexin, cefadroxyl
Penicillin allergic: erythromycin,
clarithromycin, or clindamycin

Severe disease (e.g., fever):


IV medications:
Oxacillin, nafcillin, cefazolin
Penicillin allergic: Clindamycin, vancomycin
MRSA:
Vancomycin, linezolid, daptomycin,
tigecycline, ceftaroline

MRSA:
Doxycycline, clindamycin, or TMP/SMX

Cross reaction
Between penicillin and cephalosporins
unusual (< 5%)
MTB S2CK

p. 372

MTB S2CK

p. 372

11

Erysipelas/Treatment

Step 2 CK tests:
Route of administration
(O l vs Intravenous)
(Oral
I t
)

MTB S2CK

p. 372

Cellulitis

Cellulitis

Soft tissue infection of skin


Extends from dermis into subcutaneous
tissue
Skin: Warm, red, swollen, and tender

MTB S2CK

p. 372

Cellulitis
Involves legs > arms
Doesnt have collections of walled-off infection,
which is an abscess
Cellulitis isnt only at hair follicle; that's folliculitis,
furuncles, and carbuncles

Skin infection: Caused by S. aureus

NOT S. epidermidis
S. epidermidis lives on skin as normal flora
MTB S2CK

p. 372

MTB S2CK

p. 372

Cellulitis/Diagnostic Tests

Cellulitis/Treatment

No diagnostic testing needed


Most accurate test

Same as for erysipelas


Topical antibiotics will not cover
cellulitis
Below dermal/epidermal junction
Topical
T i l antibiotics
tibi ti wontt reach
h

Inject sterile saline into skin & aspirate


for culture
Yield only 20%

Staphylococcus > Streptococcus

MTB S2CK

p. 372

MTB S2CK

p. 372

12

Folliculitis, Furuncles, Carbuncles

Originate around hair follicles


Different terms dont have precise
definitions
Indistinguishable, no cutoff in size

Folliculitis, Furuncles, Carbuncles/


Size of the Infection

Folliculitis: Earliest & mildest


Furuncle: Small abscess or collection of
infected material
Carbuncle: Collection of furuncles

Folliculitis < Furuncle < Carbuncle

Severe disease =
fever, chills, bacteremia

Treat: Ox/Clox/Diclox/Naf
MTB S2CK

p. 372

MTB S2CK

Folliculitis, Furuncles, Carbuncles

p. 372

Folliculitis, Furuncles, Carbuncles

Source: cdc.gov

Folliculitis, Furuncles, Carbuncles

Source: commons.wikimedia.org

Folliculitis, Furuncles, Carbuncles/


Penicillin Allergy

Reaction - Rash
Use cephalosporins
Reaction - Anaphylaxis
Mild infection: Macrolides, clindamycin,
doxycycline, or TMP/SMZ
Severe infection: Vancomycin, linezolid,
daptomycin, or tigecycline
MTB S2CK

p. 373

MTB S2CK

p. 373

13

Folliculitis, Furuncles, Carbuncles/


Other Antistaphylococcal Medications

Fungal Infections

Beta-lactam/beta-lactamase combinations
Amoxicillin/clavulanate
Ticarcillin/clavulanate
Ampicillin/sulbactam
Piperacillin/tazobactam

Carbapenems (imipenem, meropenem)

MTB S2CK

p. 373 374

Dermatophyte = superficial fungal


infection = tinea
For example:
Tinea corporis
p
= body
y
Tinea manum = hand
Tinea pedis = foot
Tinea cruris = groin (jock itch)
MTB S2CK

Fungal Infections

p. 374

Fungal Infections/Diagnostic Tests/Treatment

Best initial test


KOH (potassium hydroxide)
Dissolves epidermal skin cells
Leaves fungi intact
Most accurate test
Fungal culture

Richard Usatine, M.D.


Used with permission.

MTB S2CK

p. 374

Fungal Infections/Diagnostic Tests/Treatment

Fungal Infections/Treatment

Best initial therapy


Topical antifungal agent if no hair or
nails involved
For hair (tinea capitis) and nail (tinea
unguium)

Topical antifungal agents:


Clotrimazole
Ketoconazole
Oral ketoconazole
Econazole
causes gynecomastia
Miconazole
Mi
l
Nystatin
Ciclopirox

Terbinafine
Itraconazole

MTB S2CK

p. 374

MTB S2CK

p. 375

14

Fungal Infections/Oral and Vaginal Candidiasis

Drug Reactions

Hypersensitivity reactions to medications vary in


severity
When severity of reaction changes, the name of
reaction changes
Drugs causing hypersensitivity reactions:
Penicillins
Sulfa drugs (including thiazides, furosemide, and
sulfonylureas)
Allopurinol
Phenytoin
Lamotrigine
NSAIDs

Same answers for both


KOH: Best initial test
Fungal culture: Most accurate test
With clear presentation: Treat
Topical antifungal from previous list

MTB S2CK

p. 375

MTB S2CK

Drug Reactions

Morbilliform Rash

Drugs that cause hypersensitivity


reactions of the skin are the same
that cause:
Hemolysis
Interstitial
I t titi l nephritis
h iti
Thrombocytopenia

MTB S2CK

p. 375

p. 375

Mildest reaction
Skin stays intact
No mucous membrane involvement
No specific therapy

MTB S2CK

Morbilliform Rash

p. 375

Erythema Multiforme

Widespread
Target lesions
Mostly on trunk
Mucous membrane uninvolved
From herpes or mycoplasma
Prednisone may benefit

Source: commons.wikimedia.org

MTB S2CK

p. 376

MTB S2CK

p. 375

15

Erythema Multiforme

Stevens Johnson Syndrome

Erythema multiforme
is characterized by
multiple small
target-shaped
l i
lesions
that
th t can b
be
confluent

MTB S2CK

p. 375

Source: Andrew Peredo

Toxic Epidermal Necrolysis

Mucous membrane involvement


Nikolsky sign
Steroids definitely dont help
Treat with IVIG

MTB S2CK

p. 375

Very severe
Involves mucous membranes
Sloughs off respiratory epithelium
May lead to respiratory failure
Steroids not beneficial
Use intravenous immunoglobulins
(IVIG)

MTB S2CK

p. 375

Toxic Epidermal Necrolysis

The skin comes


off in a sheet,
simulating a
burn.

MTB S2CK

p. 376

Source: Conrad Fischer, MD

Staphylococcal Scalded Skin Syndrome (SSSS)


and Toxic Shock Syndrome (TSS)

Staphylococcal Scalded Skin Syndrome and


Toxic Shock Syndrome

Different severities of same event


Reaction to toxin in surface of Staphylococcus
SSSS looks similar to TEN, including Nikolsky
sign
life
TSS has skin involvement as well as lifethreatening multi-organ involvement:

Both treated with:


Antistaphylococcal medications
Oxacillin or nafcillin are most effective
Cefazolin is essentially equal
Antibiotics dont reverse disease
Kills Staphylococcus that produces toxin

Hypotension
Renal dysfunction ( BUN and creatinine)
Liver dysfunction
CNS involvement (delirium)

MTB S2CK

p. 377

MTB S2CK

p. 377

16

Acne/Treatment

Acne/Treatment

Mild acne
Topical antibacterials: Benzoyl peroxide
If ineffective add topical antibiotics
(e.g., clindamycin or erythromycin)

Moderate acne
Add topical vitamin A derivatives:
tretinoin, adapalene, or tazarotene to
topical antibiotics
If no response to topical vitamin A
derivatives and antibiotics, use oral
antibiotics (e.g., minocycline or
doxycycline)

MTB S2CK

p. 377

MTB S2CK

p. 377

Acne/Treatment

Severe acne
Add oral vitamin A, isotretinoin to oral
antibiotics
Isotretinoin causes hyperlipidemia

Vitamin A derivatives
extremely teratogenic

MTB S2CK

p. 377

17

Toxicology, Poisoning, &


Overdose

EMERGENCY MEDICINE
Niket Sonpal, MD
Chief Resident
Lenox Hill Hospital NSLIJ
Assistant Clinical Professor Touro College of Medicine

Treatment of Overdose
32-year-old woman with a history of depression comes to ED
30 minutes after taking a bottle of pills in a suicide attempt. BP
118/70, pulse 90, and respirations normal. She refuses to tell
you what she took.
What is the next step?

Gastric Lavage
OVERDOSE
TREATMENTS

It could be caustic and injure on

a. Induce emesis with ipecac


p
th way up
the
b. Gastric lavage
c. Psychiatric consultation Consultations are 99% wrong on USMLE
Takes too long
d. Serum chemistry
e. Urine toxicology screen Urine wont show up yet
Pills will not make that transit time
f. Cathartics/laxatives
g. Whole bowel irrigation Doesnt change outcomes
We dont know if its opiates
h. Naloxone
We dont know if its benzos
i. Flumazenil
MTB S2CK

p. 533

Initial Management of Poisoning

Bowel
Irrigation

MTB S2CK

p. 534

Ipecac
Fluids and Diuresis

MTB S2CK

p. 534

Initial Management of Poisoning

Gastrointestinal Emptying
Gastric lavage
Gastric emptying of any kind is
always wrong with
Caustics (acids and alkali)
Altered mental status

Cathartics

Gastric lavage is rarely done.


Removes 50% of pills at 1 hour
Removes 15% of pills at 2 hours

Ipecac is always a wrong answer in ED

MTB S2CK

p. 534

Initial Management of Poisoning

Initial Management of Poisoning

Ipecac
No inpatient benefit
15-20 minutes onset
Hinders antidotes

Cathartics
Cathartic agents WRONG answer
Prokinetics WRONG answer

MTB S2CK

p. 534

MTB S2CK

p. 534

Initial Management of Poisoning

Initial Management of Poisoning

Forced Diuresis
Fluids and diuretics is always a
wrong answer
Risk of PE > benefit

Whole Bowel Irrigation

NGT with polyethylene glycol-electrolyte


solution is almost always wrong
Only for:
Iron ingestion
Lithium
Drug-filled packets

MTB S2CK

p. 534

MTB S2CK

p. 534

Initial Management of Poisoning

When answer is unclear and cause of


overdose is asked say:
Acetaminophen 1st Most Common
Aspirin 2nd Most Common

What to do is often unclear. What is useless


or dangerous (ipecac, forced diuresis,
cathartics) is very clear.
MTB S2CK

p. 535

Woman comes to ED one hour after taking a bottle of


pills. BP 118/70, pulse 90/min, and respirations
14/min. She is confused, disoriented, and lethargic.
What is the next step in the management?
Seizure risk too high 2 line
a. Flumazenil
Dangerous in AMS
b. Gastric lavage
c. Psychiatric consultation Never get a consult youre an MD
d. Naloxone and dextrose
Not the first step
e. Intubation
nd

MTB S2CK

p. 535

Initial Management of Poisoning

Initial Management of Poisoning

Opiate overdose is fatal:


Give naloxone immediately

Psychiatric consultation is
indicated for suicide attempt, but
is a wrong answer on USMLE S2
CK when specific antidotes and
diagnostic tests are needed.

MTB S2CK

p. 535

Benzodiazepine overdose by
itself is not fatal and acute
withdrawal causes seizures
Dont give flumazenil

MTB S2CK

p. 535

Initial Management of Poisoning

Acetaminophen

Charcoal
Charcoal is benign
Charcoal is not dangerous
Charcoal is superior to lavage and
ipecac

Legal drugs kill more people in the United


States than illegal drugs because theyre
less expensive and more available
Toxicity of acetaminophen with ingestion
> 8 to 10 g
Fatality with ingestions > 12 to 15 g

When you dont know what to do


in toxicology, give charcoal
MTB S2CK

p. 535 536

MTB S2CK

p. 536

Acetaminophen

Acetaminophen

Four Most Common Acetaminophen Overdose


Questions:

3. Amount of ingestion unclear?

1. If a clearly toxic amount of acetaminophen has


been ingested (> 810 g)
Ans er N-acetylcysteine
Answer:
N acet lc steine
2. If the overdose was > 24 hours ago
No therapy

MTB S2CK

p. 536

Drug level
4. Charcoal does not make N-acetylcysteine
ineffective
Charcoal isnt contraindicated with Nacetylcysteine

MTB S2CK

p. 536

Aspirin Overdose

Aspirin Overdose

What is the most likely diagnosis?

Aspirin causes diffuse, multisystem toxicity

Renal Toxicity

Tinnitus

Altered
Mental
Status

Aspirin Overdose

Increased Anion
Gap

Hyperventilation
MTB S2CK

Increased PT

Respiratory
Alkalosis
Metabolic
Acidosis

p. 536

ARDS
Aspirin Overdose

Respiratory
Failure

Increased
PTT
Lactic Acidosis

MTB S2CK

p. 536

Aspirin Overdose

Treatment is alkalinizing urine, which


increases rate of aspirin excretion
Know blood gas in aspirin overdose
Tinnitus, respiratory alkalosis,
and metabolic acidosis are
the key to diagnosing aspirin
overdose.

MTB S2CK

p. 536

Depressed patient presents with altered mental status


from ingesting multiple toxic substances. You know
for certain that he took some lorazepam only today,
for the first time. There is no response to naloxone
or dextrose. The patient is given flumazenil and
immediately seizes.
What is the most likely cause of the seizure?
a. Cocaine withdrawal Cocaine toxicity causes seizures, not withdrawal
Doesnt cause seizures
b. Opiate withdrawal
c. Tricyclic antidepressants
d. SSRIs
SSRIs toxicity causes serotonin syndrome (SS)
Causes tinnitus and hyperventilation, not seizures
e. Aspirin
MTB S2CK

p. 537

Which of the following is most likely to be found in


aspirin overdose?
(Normal values: pH 7.40, pCO2 40, HCO3 24)

a. pH 7.55 pCO2 50 HCO3 24


b. p
pH 7.25 p
pCO2 62 HCO3 38
c. pH 7.46 pCO2 22 HCO3 16
d. pH 7.35 pCO2 32 HCO3 20

MTB S2CK

This is a distracter
p
overdose never g
gives
Aspirin
respiratory acidosis
Metabolic acidosis with
respiratory compensation

p. 536 537

What is the best initial test for the patient


previously described?
a. Urine toxicology
b. Electroencephalogram
c. EKG
Wont show the cause of the seizure
d. Head CT
in this patient
e. Potassium level

MTB S2CK

p. 537

Tricyclic Antidepressant Toxicity

Tricyclic Antidepressants
Seizures

Widened
QRS
Complex

Arrhythmia

TCA Toxicity

Urinary
retention

Dry mouth
Constipation

MTB S2CK

p. 537

MTB S2CK

p. 538

Tricyclic Antidepressants

Caustics

The best initial treatment of TCA


overdose is with sodium bicarbonate

Caustic ingestion of acids and alkalis


(e.g., drain cleaner) causes

Bicarbonate will protect the heart


against arrhythmia
Bicarbonate does not increase urinary
y
excretion of TCAs as it does for aspirin

MTB S2CK

p. 538

Mechanical damage to oropharynx


Esophageal perforation
Stomach perforation

Dont give alkali to reverse acids

MTB S2CK

p. 538

Caustics

Carbon Monoxide Poisoning

Flush out caustics


Use water in high volumes
Endoscopy is performed to assess
degree of damage

Carbon monoxide (CO) poisoning is the


MCC of death in fires

Thermal injury is always


the wrong answer

Steroids dont prevent


injury from caustics

MTB S2CK

p. 538

MTB S2CK

p. 538

Carbon Monoxide Poisoning

Carbon Monoxide Poisoning

CO binds oxygen to Hb so tightly that


carboxyhemoglobin will not release oxygen
to tissues
Carboxyhemoglobin acts functionally like
anemia

Myocardial
infarction

Functional Anemia

Carbon
Monoxide

Confusion
Seizures

Dyspnea

Lightheadedness

Source: cdc.gov

MTB S2CK

p. 538

MTB S2CK

p. 538

Carbon Monoxide Poisoning

No functional difference between absence of


blood and carboxyhemoglobin; 60%
carboxyhemoglobin = loss of 60% of blood
The left ventricle cant
can t
distinguish between anemia,
carboxyhemoglobin, and
stenosis of coronary arteries.

MTB S2CK

p. 538

Carbon Monoxide Poisoning

Which of the following blood gas results would you


find in carbon monoxide poisoning?
a.
b.
c.
d.

pH 7.55 pCO 50 HCO 24


pH 7.25 pCO 62 HCO 38
pH 7.46 pCO 22 HCO 16
pH 7.35 pCO 26 HCO 18

MTB S2CK

Distracter
Respiratory acidosis with metabolic
alkalosis
Respiratory alkalosis with metabolic
acidosis

p. 538

Carbon Monoxide Poisoning

Diagnostic tests
Carbon monoxide poisoning
gives normal pO2 because
oxygen doesnt detach
from hemoglobin

Routine oximetry will be falsely normal


Most accurate test is a level of
carboxyhemoglobin

Treatment
Remove patient from exposure
Give 100% oxygen

MTB S2CK

p. 538

MTB S2CK

p. 539

Carbon Monoxide Poisoning

Methemoglobinemia

Treatment
Severe disease is treated with hyperbaric
oxygen
Severe symptoms are defined as:

CNS symptoms
Cardiac symptoms
Metabolic acidosis

MTB S2CK

Wikimedia

p. 539

Methemoglobin is oxidized Hb
Ferric
3+
Ferric = Fe
Ferrous = Fe2+
Oxidized hemoglobin is brown and
will
ill nott carry oxygen
Chocolate-brown blood

MTB S2CK

p. 539

Methemoglobinemia

Methemoglobinemia/Presentation

Methemoglobinemia from idiosyncratic


reaction of hemoglobin to drugs such as:
Benzocaine and anesthetics
Nitrites and nitroglycerin
Dapsone
p

The effects of methemoglobinemia are similar to


carboxyhemoglobin
Oxygen isnt delivered to tissues
Functional Anemia

Metabolic
Acidosis
Methemoglobinemia

Ferrous
Hemoglobin

MTB S2CK

Benzocaine
Nitrites
Dapsone

Dyspnea
Cyanosis

Methemoglobinemia
Ferric Hemoglobin

p. 539

Confusion
Seizures
Headache
MTB S2CK

Methemoglobinemia/Presentation

Carbon monoxide:
blood abnormally red
Methemoglobinemia:
blood abnormally brown

Lightheadness

p. 539

Methemoglobinemia/Diagnostic Tests/
Treatment

Both methemoglobinemia and


carboxyhemoglobin give normal pO2 on blood
gas
Most accurate test is methemoglobin level
Best initial therapy is 100% oxygen
Most effective therapy is methylene blue,
which half-life of methemoglobin

Cyanosis + normal pO2 =


methemoglobinemia
MTB S2CK

p. 539

MTB S2CK

p. 539 540

Organophosphate (Insecticide) Poisoning and


Nerve Gas

Organophosphates and nerve gas


identical in their effects
Nerve gas faster and more severe
Massive increase in acetylcholine by
i hibiti metabolism
inhibiting
t b li

Organophosphate (Insecticide) Poisoning and


Nerve Gas
Bronchospasm
Bronchorrhea
Organophosphate
g
p
p
Poisoning
Salivation

MTB S2CK

p. 540

Organophosphate (Insecticide) Poisoning and


Nerve Gas

Acetylcholine constricts bronchi


and increases bronchial secretions

MTB S2CK

p. 540

Organophosphate (Insecticide) Poisoning and


Nerve Gas

Nerve gas and


organophosphates
g
p
p
are
absorbed through the skin

MTB S2CK

p. 540

Polyuria
y

Lacrimation

p. 540

56-year-old military commander attacked with nerve


gas. Presents with salivation, lacrimation, urination,
defecation, and SOB. Pupils are constricted.
What is the first step in management?
a. Atropine
p
b. Decontaminate (wash) the patient Stabilize first
c. Remove his clothing
Stabilize first
d. Pralidoxime
Takes too long
e. No therapy is effective Never the right answer

MTB S2CK

p. 540

Digoxin Toxicity/Etiology

Hypokalemia predisposes to digoxin


toxicity because potassium and digoxin
compete for same site on the Na+/K+ATPase
Less potassium is bound, more
digoxin is bound
K+

MTB S2CK

Respiratory arrest

MTB S2CK

Increased
Digoxin Binding

p. 540

Digoxin Toxicity/Presentation

Digoxin Toxicity/Presentation

Yellow Halo Vision

Confusion

Hypokalemia
Digoxin
g
Toxicity

Hyperkalemia

MTB S2CK

Arrhythmias
y

Digoxin toxicity

Digoxin toxicity
Hyperkalemia

Nausea
Vomiting
Abdominal Pain

p. 540 541

Digoxin Toxicity/Diagnostic Tests

MTB S2CK

p. 541

Digoxin Toxicity/Diagnostic Tests

Most accurate test: digoxin level


Best initial tests: potassium level and EKG
EKG: downsloping of ST segment in all
leads
Atrial tachycardia with variable AV block is
the most common digoxin toxic arrhythmia

Digoxin can produce


any arrhythmia
MTB S2CK

p. 541

Digoxin Toxicity/Treatment

Control potassium and give digoxinspecific antibodies


Digoxin-binding antibodies rapidly
remove digoxin from circulation
Strongest indication for
digoxin-binding antibodies:
CNS and cardiac involvement.

MTB S2CK

p. 541

Lead Poisoning
Renal Tubule
Toxicity (ATN)

Abdominal pain
lead colic

Lead
Toxicity
Sideroblastic
Anemia

MTB S2CK

Memory
y loss
confusion
Peripheral
Neuropathy
wrist drop

p. 541

Lead Poisoning

Lead Poisoning/Treatment

Best initial test: level of free


erythrocyte protoporphyrin
Most accurate test: lead level

Chelating agents remove lead from


body

Most accurate test for sideroblastic


anemia: Prussian blue stain

MTB S2CK

p. 541

MTB S2CK

Mercury Poisoning
Neurological
problems
Nervous, jittery,
twitchy, and
sometimes
hallucinatory

Theres no therapy to reverse pulmonary


toxicity
Chelating agents such as dimercaprol and
succimer are effective in removing mercury
from body and decreasing neurological
t i it
toxicity

Lung
toxicity & interstitial
fibrosis

Inhaled

p. 541

Toxic Alcohols/Methanol and Ethylene Glycol


Methanol
Ethylene glycol

1. Intoxication
1
2. Metabolic Acidosis
3. Increased Anion
Gap
4. Osmolar Gap

MTB S2CK

p. 541

Mercury Poisoning

Oral ingested

MTB S2CK

Succimer: only oral


Ethylenediaminetetraacetic acid
(EDTA) and dimercaprol (BAL) are
parenteral

p. 542

Treated w/
1. Fomepizole
2. Dialysis

Differences between Methanol and


Ethylene Glycol
Methanol

Ethylene glycol

Source

Wood alcohol, cleaning


solutions, paint thinner

Antifreeze

Toxic
metabolite

Formic acid/formaldehyde Oxalic acid/oxalate

Presentation

Ocular toxicity

Initial diagnostic Retinal inflammation


abnormality
MTB S2CK

p. 542

Renal toxicity
Hypocalcemia,
envelope shaped
oxalate crystals in
urine

10

Toxic Alcohols/Methanol and Ethylene Glycol


Osmolar Gap
Osmolar gap =
measured serum osmolality - calculated osmolality
Serum osmolality =
2(Na+) + BUN/2.8 + Glucose/18
Ex: Measurement 350 - Calculated 300 = gap of 50 osmoles

Toxic Alcohols/Methanol and Ethylene Glycol

Treatment
Best initial therapy: fomepizole,
which inhibits alcohol dehydrogenase
and prevents production of toxic
metabolite
Only dialysis removes methanol and
ethylene glycol

Ordinary alcohol (ethanol) also


osmolar gap
MTB S2CK

p. 542

MTB S2CK

p. 542

Snake Bites

Snake Bites/Treatment

Most common injury from snake bites is


local wound
Death from snake bites:

Ineffective or dangerous
treatment

Beneficial therapy

Tourniquets blocking
arterial flow

Pressure

Ice

Immobilization decreases
movement of venom

Incision and suction,


especially by mouth

Antivenin

Hemolytic toxin: hemolysis, DIC, and


damage to endothelial lining of tissues
Neurotoxin: can result in respiratory
paralysis, ptosis, dysphagia, and diplopia

MTB S2CK

p. 542 543

Spider Bites/Presentation

All spider bites present with a sudden,


sharp pain that patient may describe as:
1.I stepped on a nail
2. A piece of glass was in my shoe.

MTB S2CK

p. 543

MTB S2CK

p. 543

Differences between Types of Spider Bites


Black widow

Brown recluse

Presentation

Abdominal
pain,
muscle pain

Local skin
necrosis

Lab test
abnormalities

Hypocalcemia

None

Treatment

Calcium,
antivenin

Debridement,
steroids,
dapsone

MTB S2CK

p. 543

11

Dog, Cat, and Human Bites

Dog, Cat, and Human Bites

Management of dog, cat, and human bites is


essentially identical
Theyre managed with:

Rabies vaccine only if:


Animal has altered mental
status/bizarre behavior
Attack was unprovoked, by a
stray dog that cannot be
observed or diagnosed

Amoxicillin/clavulanate
Tetanus vaccination booster if > 5 years since
last injection

Dog and Cats: Pasteurella multocida


Humans: Eikenella corrodens
Human bites are more damaging
than dog and cat bites
MTB S2CK

p. 543

MTB S2CK

p. 544

Rabies and Bats

Head Trauma
If a bat was noted to be in the
room and the patient was
asleep VACCINATE!

MTB S2CK

p. 544

Head Trauma

Any head trauma resulting in altered mental


status or loss of consciousness (LOC) is
managed first with a head CT
Head CT without contrast is best initial test to
detect blood
Contrast detects mass lesions such as cancer
and abscess, not blood

Head Trauma

LOC = CT
Concussion:
No focal neurological abnormalities
Normal CT scan

Contusion:
Occasionally (rarely) has focal findings
Ecchymoses found on CT (blood mixed in
with brain parenchyma)

MTB S2CK

p. 544

MTB S2CK

p. 544

12

Contusion

Head trauma

Subdural and epidural hematomas: usually


associated with more severe trauma than a
concussion
Impossible to distinguish without head CT,
epidural hematoma more frequently with
skull
k ll ffracture
t

MTB S2CK

p. 544

MTB S2CK

Epidural Hematoma

MTB S2CK

p. 544

Subdural Hematoma

p. 545

Source: Brendan T. Doherty

Lucid Interval

Treatment

Lucid interval is a second loss of


consciousness occurring several minutes to
several hours after initial loss of
consciousness

Concussion: no specific therapy

Patient wakes up after initial LOC, but loses


consciousness a second time due to
accumulation of blood

Wait at least 24 hours before returning


to sports

Contusion: vast majority need no


specific treatment
Rarely need surgical debridement

Time between first and second episodes


of LOC is lucid interval

Those with concussion safe to go


home. Hospitalization isnt
necessary. Observe at home for
altered mental status.

Both epidural and subdural hematomas


are associated with a lucid interval.
MTB S2CK

p. 545

MTB S2CK

p. 545

13

Treatment

Definition of a Large Intracranial hemorrhage

Subdural and epidural hematoma:


Treatment is based on size and signs
of compression of brain
Small ones are left alone
Large
L
h
hematomas
t
are managed
d with:
ith

Compression of ventricles or sulci


Herniation with abnormal breathing and
unilateral dilation of pupil
Worsening mental status or focal
findings

1. Intubation and hyperventilation


2. Mannitol
3. Drainage
MTB S2CK

p. 545

MTB S2CK

Summary of Severe Head Trauma

p. 546

Summary of Severe Head Trauma

Concussion

Contusion

Subdural

Epidural

Concussion

Contusion

Subdural

Epidural

No focal
findings

Rarely focal

+/ focal
findings

+/ focal
findings

Normal CT

Ecchymoses

Venous,
crescent

No lucid
Interval

No lucid
Interval

+/ lucid
Interval

+/ lucid
interval

Arterial,
biconvex or
lens shaped
hematoma

No specific
treatment;
observe at
home

No specific
treatment;
observe in
hospital

Drain large
ones

Drain large
ones

MTB S2CK

p. 546

MTB S2CK

p. 546

Head Trauma
25-year-old man sustains head trauma in MVA. A
large epidural hematoma is found. Immediately after
intubation and mannitol, surgical evacuation is
successfully performed.
Which of the following will benefit the patient?

Steroids dont benefit intracranial


bleeding. They decrease edema
around mass lesions.

a. Repeated doses of mannitol Doesnt reduce mortality


Doesnt always work
b. Continued hyperventilation
c. Proton pump inhibitor (PPI)
For SAH only
d. Nimodipine
e. Dexamethasone Doesnt change outcomes
MTB S2CK

p. 546

MTB S2CK

p. 546

14

Burns

Burns & Hypothermia

Best initial therapy for those caught in


a fire is 100% oxygen to treat smoke
inhalation and CO poisoning
Airway burn is 2nd MCC of death from
burns only if theres been airway
injury

MTB S2CK

Burns

Burns

Burns inside mouth

Stridor

Burn Victim
Indications for
Intubation

Hoarseness

MTB S2CK

p. 547

Burns inside
nasopharynx

If airway burn is not present, the 2nd


MCC of death is volume loss
Fluid replacement is based on
percentage of body surface area (BSA)
burned

Wheezing

p. 547

MTB S2CK

p. 547

Volume of Fluid Replacement

Volume of Fluid Replacement

Replace with Ringer lactate


Give one-half in first 8 hours, a quarter in
second 8 hours, and a quarter in the third 8
hours
Give 4 mL for each percentage of BSA
burned (including 2nd and 3rd degree burns)
for each kilogram of body weight

Patchy burns that arent continuous make


the percentage of BSA burned hard to
assess. Use the width of patients hand to
make an estimate
Each hand width is 1% of BSA
The short answer is: give the largest amount
of Ringer lactate or normal saline listed as a
choice; its probably the right answer

Head: 9% BSA
Arms: 9% BSA each
Legs: 18% BSA each
Chest or back: 18% BSA each
MTB S2CK

p. 547

Fluid replacement:
(4 mL) (% BSA burned) weight (kg)
MTB S2CK

p. 547

15

Heat Disorders
What is the MCC of death several days to weeks after
a burn?
a. Infection
Rhabdomyolysis causes renal failure
b. Renal failure
c. Cardiomyopathy Not affected so quickly
Most common immediate cause of death
d Lung injury
d.
Fluid loss doesnt mean malnutrition
e. Malnutrition

MTB S2CK

p. 547

Malignant
hyperthermia

Risk

Antipsychotic
medications

Anesthetics
administered
systemically

Body temp

Elevated

Elevated

CPK and K+ level

Elevated

Elevated

Treatment

Dantrolene or
dopamine agonists:
bromocriptine, cabergoline

Dantrolene

Hypothermia

Risk

Exertion; high
outside temp

Exertion; high
outside temp

Body temp

Normal

Elevated

CPK and K+
level

Normal

Elevated

Treatment

Oral fluids and


electrolytes

IV fluids;
evaporation

p. 548

Hypothermia
Neuroleptic
malignant syndrome

p. 548

Heatstroke

MTB S2CK

Heat Disorders

MTB S2CK

Heat cramps/
exhaustion

Look for intoxicated person with


hypothermia
MCC of death: cardiac arrhythmia
Best initial step: EKG

MTB S2CK

p. 548

Drowning

Manage with airway and administer positive


pressure ventilation
Steroids and antibiotics are not beneficial
Salt water drowning: acts like CHF with wet,
heavy lungs

Wrong answers for drowning include:


Steroids
Antibiotics
MTB S2CK

p. 548

MTB S2CK

p. 549

16

Drowning

Fresh water drowning: causes hemolysis


from absorption of hypotonic fluid into
vasculature

RBC
RBC

Cardiac Rhythm Disorders

RBC

Fresh H20

Hemolysis
MTB S2CK

p. 549

Initial Management of Cardiac Arrest

Initial Management of Cardiac Arrest

First step :

After patient has been shown to be


unresponsive, and EMS activated, the
next step is:

Make sure patient is truly unresponsive


Call for help: call 911/activate Emergency
Medical Services (EMS)

Truly unresponsive
Chest compressions
Rescue breaths

MTB S2CK

p. 549

Initial Management of Cardiac Arrest

When is a precordial thump the


answer?
Very recent onset of arrest (<10
minutes) with no defibrillator available
You know its recent because you saw it
happen (witnessed)

MTB S2CK

p. 549

1. Open airway: head tilt, chin lift, jaw thrust


p
and start chest compressions
p
if
2. Check pulse
pulseless
3. Give rescue breaths if not breathing
CPR doesnt restart the
heart; CPR keeps patient alive until
cardioversion can be performed.
MTB S2CK

p. 549

Pulselessness

Sudden loss of a pulse can be caused


by:
Asystole
Ventricular fibrillation (VF)
Ventricular tachycardia (VT)
Pulseless electrical activity (PEA)

Best initial management of all forms of


pulselessness is CPR
MTB S2CK

p. 549 550

17

Pulselessness

Pulselessness

Asystole
Besides CPR, therapy for asystole is
with epinephrine
Vasopressin is alternative to
epinephrine
They both constrict blood vessels in
tissues (e.g., skin)
Shunts blood into critical central areas
(e.g., heart and brain)

Ventricular Fibrillation
Best initial therapy for VF is an immediate,
unsynchronized cardioversion followed
by CPR
Unsynchronized = defibrillation
All electrical cardioversions synchronized
except VF and pulseless VT

MTB S2CK

p. 550

MTB S2CK

Pulselessness

p. 550

Ventricular fibrillation

Only VF and VT without a


pulse
l gett unsynchronized
h i d
cardioversion.

MTB S2CK

p. 550

MTB S2CK

p. 550

Pulselessness

Pulselessness

After defibrillation, then is epinephrine


or vasopressin followed by another
electrical shock
Amiodarone or lidocaine
Magnesium

Ventricular Tachycardia (VT)


Wide complex tachycardia with regular rate
Management entirely based on
hemodynamic status

Bretylium is always a wrong answer


MTB S2CK

p. 550 551

Pulseless VT: manage exactly same way as


VF
Hemodynamically stable VT: Amiodarone,
then lidocaine, then procainamide. If all
medical therapy fails, then cardiovert patient

MTB S2CK

p. 551

18

Pulselessness

Ventricular Tachycardia

Ventricular Tachycardia
Hemodynamically unstable VT:
Perform electrical cardioversion several
times followed by medications (e.g.,
amiodarone or lidocaine)
VT is managed with shock, drugs,
and CPR at all times in between
the shocks.
MTB S2CK

p. 551

Pulselessness

Hemodynamic instability is defined as:


Chest pain
Dyspnea/CHF
Hypotension
Confusion

MTB S2CK

p. 551

Pulselessness

Pulseless Electrical Activity (PEA)


PEA = electrically normal, but no
motor contraction
PEA = no cardiac output

These qualities of instability are the same for


all rhythm disturbances
Direct intracardiac medication administration
is always a wrong answer
MTB S2CK

p. 551

Pulselessness

MTB S2CK

p. 551

Pulselessness/Treatment

PEA treated by correcting underlying cause


We synchronize delivery of
electricity in cardioversion of VT
to prevent worsening of
arrhythmia into ventricular
fibrillation or asystole.
asystole

PEA: look for patient with a normal EKG and


no pulse
MTB S2CK

p. 551 552

Tamponade

Tension Pneumothorax

Causes of PEA

Massive PE

MTB S2CK

p. 552

HYPOvolemia
HYPOglycemia
HYPO
l
i
HYPOxia
HYPOthermia

Metabolic acidosis
HYPERkalemia
HYPOkalemia

19

Atrial Arrhythmias

Atrial rhythm disturbances rarely


associated with hemodynamic
compromise
Look for :
Palpitations,
p
, dizziness,, or lightheadedness
g
Exercise intolerance or dyspnea
Embolic stroke

MTB S2CK

p. 552

Atrial Arrhythmias

Atrial Arrhythmias

Irregularly irregular rhythm suggests Afib as the most likely diagnosis even
before EKG is done
A-fib:
A fib: most common arrhythmia in the
United States

MTB S2CK

p. 552

Atrial Fibrillation

Atrial Fibrillation and Atrial Flutter


Two disorders with nearly identical
management
Major points of difference are:
Flutter
Fl tt is
i a regular
l rhythm
h th vs.
fibrillation is irregular
Flutter changes to sinus rhythm or
deteriorates into fibrillation

MTB S2CK

p. 552

Atrial Flutter

MTB S2CK

p. 553

Source: Abhay Vakil, MD.

Atrial Arrhythmias/Treatment

Hemodynamically unstable atrial


arrhythmias = synchronized
cardioversion
Synchronization prevents electricity
from being delivered during refractory
period (ST-T wave)
Synchronization prevents change
into VT or VF

MTB S2CK

p. 553

MTB S2CK

p. 553

20

Atrial Arrhythmias/Treatment

Atrial Arrhythmias/Treatment

Chronic Atrial Fibrillation


A-fib > 2 days
> 7 days risk of clot formation
Routine cardioversion is not
i di t d
indicated
Chronic is usually secondary atrial or
valvular anatomic change

Chronic Atrial Fibrillation


Shocking doesnt correct a dilated left atrium
causing A-fib
Over 90% will revert to fibrillation
Rate control and warfarin is standard of care
f A-fib
for
A fib

MTB S2CK

p. 553 554

Chronic A-fib should be anticoagulated


before cardioversion. Unstable, acute
disease doesnt need anticoagulation.

MTB S2CK

p. 553 554

Atrial Arrhythmias/Treatment

Atrial Arrhythmias/Treatment

Best initial therapy for fibrillation and


flutter is to control the rate
1. Goal HR < 100/minute
2. INR between 2-3

Calcium-channel blockers used to


control HR with atrial arrhythmias are
diltiazem and verapamil

1.
1 Slow rate
2. Anticoagulate

Reliably block AV node


Other calcium-channel blockers control
BP
No matter how much you might think it
better to shock every patient into sinus, it
just doesnt work in long run.

Rate control drugs do not


convert patient into sinus
rhythm.
MTB S2CK

p. 554

MTB S2CK

p. 554

Atrial Arrhythmias/Treatment

Atrial Arrhythmias/Treatment

Warfarin
Without anticoagulation 6% a year
stroke risk
INR 2-3, rate: 2-3% stroke risk
Dabigatran and Rivaroxaban
Alternatives to warfarin
For non-valvular A-Fib
No INR monitoring

A-fib is caused by anatomic cardiac


defects dilating atrium
Thats why vast majority revert
Acute disease normalizes spontaneously;
dont force it
Chronic disease reverts into arrhythmia;
dont force it either

MTB S2CK

p. 554

MTB S2CK

p. 554

21

Atrial Arrhythmias/Treatment

Atrial Arrhythmias/Treatment

Atrial rhythm problems


can cause acute PE from
loss of atrial kick
kick in those
with cardiomyopathy.

MTB S2CK

p. 554

MTB S2CK

Atrial Arrhythmias/Treatment

p. 554

Atrial Arrhythmias/Treatment

Major bleeding from warfarin is


defined as:
Intracranial
I t
i lh
hemorrhage
h
Requires transfusion

MTB S2CK

Lone Atrial Fibrillation


Patients with low risk of stroke can use
ASA
2-3% per year vs. 1% per year bleeding
Scoring
S
i is
i called
ll d CHADS score
CHADS 2 = warfarin, dabigatran,
rivaroxiban

p. 554

Definition/Criteria for Low Risk of Stroke from A-fib


No cardiomyopathy/CHF/atherosclerosis
No HTN
Age 75
No diabetes
No stroke in past
The answer for management of lone atrial fibrillation is:
Rate control
Aspirin
No warfarin or dabigatran

MTB S2CK

Atrial Arrhythmias/Treatment
Supraventricular Tachycardia

p. 555

Atrial Arrhythmias/Treatment

SVT
Vagal Maneuvers

Carotid Massage
Valsalva
Dive Reflex
Ice immersion

Adenosine

Beta Blockers
CCBs
Digoxin
MTB S2CK

p. 555

Source: Abhay Vakil, MD.

MTB S2CK

p. 555

22

Atrial Arrhythmias/Treatment

Atrial Arrhythmias/Treatment

Wolff-Parkinson-White Syndrome (WPW)


WPW is an anatomic abnormality in cardiac
conduction pathway
Answer most likely diagnosis question by
looking for:
SVT alternating with ventricular tachycardia
SVT gets worse after diltiazem or digoxin
Observing delta wave on EKG
Vagal maneuvers slow and convert SVT.
They dont convert atrial fibrillation.
MTB S2CK

p. 556

Most accurate test for WPW is cardiac


electrophysiology (EP) studies
MTB S2CK

p. 556

Atrial Arrhythmias/Treatment

Atrial Arrhythmias

Acute therapy: Procainamide or


amiodarone
Chronic therapy: Radiofrequency catheter
ablation is curative for WPW
EP studies tell you where the anatomic
defect is
Digoxin and calcium-channel blockers are
dangerous in WPW

Multifocal Atrial Tachycardia


Multifocal atrial tachycardia (MAT) is
associated with chronic lung disease
such as COPD
Treat underlying lung disease
Treat MAT as you would A-fib, but
avoid beta blockers because of lung
disease

MTB S2CK

p. 556

MTB S2CK

p. 556

Atrial Arrhythmias
Woman comes to office for routine evaluation. Shes
found to have a pulse of 40 and an otherwise
completely normal history and physical examination.
What is the most appropriate next step in the
management of this patient?
a. Atropine
You dont know the rhythm
y
yet
y
b. Pacemaker
Too invasive
c. EKG
Too invasive
d. Electrophysiology studies
Can result in ischemia
e. Epinephrine
f. Isoproterenol
Old and no longer used; always wrong
g. Nothing; reassurance Without EKG cannot say
MTB S2CK

p. 557

MTB S2CK

p. 557

23

Bradycardia and AV block

Bradycardia and AV block

Sinus Bradycardia
Isoproterenol is never the right answer
to anything

Sinus
Bradycardia
Asymptomatic

No Treatment

Symptomatic

Atropine 1st
Pacemaker Long Term

MTB S2CK

p. 557

MTB S2CK

p. 558

Bradycardia and AV block

Bradycardia and AV block

First-Degree AV block
Use same management as sinus
bradycardia

Second-Degree AV block
Mobitz I or Wenckebach block: progressively
lengthening PR interval results in a dropped
beat
Mobitz I is most often a sign of normal aging of
conduction system.
y
If there are no symptoms,
y p
, its
managed the same way as sinus bradycardia
Dont treat if asymptomatic

Atropine
p
and p
pacemaker are used
for sinus bradycardia only if
symptomatic.

MTB S2CK

p. 558

Bradycardia and AV block

MTB S2CK

p. 558

Bradycardia and AV block

Second-Degree AV block
Mobitz II block: far more pathologic than Mobitz I
Mobitz II just drops a beat without progressive
lengthening of PR interval. Mobitz II progresses or
deteriorates into third-degree AV block. Treat it like
third-degree AV block. Everyone with Mobitz II
block gets a pacemaker even if they are
asymptomatic

MTB S2CK

p. 558

MTB S2CK

p. 558

24

58-year-old woman is admitted to hospital with an


acute MI. On the second hospital day she develops
sustained VT even though she is on aspirin, heparin,
lisinopril, and metoprolol.
What is the most appropriate next step in
management?
a. Increase the dose of metoprolol Wont treat ischemia
b. Add diltiazem Will not affect rhythm
c. Angiography for angioplasty or bypass
d. Implantable defibrillator Underlying cause can be fixed
e. EP studies Not ectopy, but rather from ischemia
MTB S2CK

p. 559

73-year-old man has his third syncopal episode in last


6 months. An EKG done in the field shows VT. His
stress test is normal.
What is the most appropriate next step in the
management of this patient?
a. Metoprolol
Not enough
g to p
prevent death
b. Diltiazem
Have no affect in VT
c. Angiography Normal stress therefore no need for angio
d. Implantable defibrillator
e. EP studies
EKG shows cause so need for EP

MTB S2CK

p. 559 560

Which of the following tests would you do for this


patient to determine a risk of recurrence?
For unexplained syncope
a. EP studies
b. Echocardiography
c. MUGA scan (nuclear ventriculography) For perfusion
d. Ventilation/perfusion
p
scan
For PE study
e. Tilt-table testing For syncope

MTB S2CK

p. 559

46-year-old man has intermittent episodes of


palpitations, lightheadedness, and near-syncope. His
EKG is normal. The echo shows an EF of 42%. Holter
monitor shows several runs of wide complex
tachycardia lasting 5-10 seconds.
Which of the following is most likely to benefit
this patient?
a. Pacemaker placement Dont know underlying cause yet
b. Digoxin
Does nothing for vent arrhythmias
c. Warfarin
Low risk for clot
d. EP studies
e. Swan-Ganz catheter
Swan if for diagnosing SHOCK
MTB S2CK

p. 560

25

Pituitary Disorders
Panhypopituitarism
Diabetes Insipidus
p
Acromegaly
Hyperprolactinemia

Endocrinology
Dr. Conrad Fischer, MD
Associate Professor of Medicine
Touro College of Medicine
New York City

Panhypopituitarism/Etiology

The Pituitary Gland

Compression or damage of the pituitary


gland
Tumors, cancer, adenomas, cysts,
meningiomas, craniopharyngiomas, or
lymphoma
Trauma and radiation are damaging

MTB S2CK

p. 107

MTB S2CK

p. 107

Panhypopituitarism/Etiology

Panhypopituitarism/Presentation

Conditions such as:


Hemochromatosis
Sarcoidosis
Histiocytosis X
Infection with fungi
fungi, TB
TB, or parasites
Autoimmune and lymphocytic infiltration
damages gland

Prolactin deficiency
Men

MTB S2CK

p. 107

No symptoms

Women:
Prolactin = In favor of or pro
p
lactation
If deficient, the patient cannot lactate normally
after childbirth

MTB S2CK

p. 107

Panhypopituitarism/Presentation

Panhypopituitarism/Presentation

Luteinizing hormone (LH) and follicle-stimulating


hormone (FSH) deficiency
Both genders will have decreased libido and
decreased axillary, pubic, and body hair
Men

Growth hormone (GH) deficiency


Adults

Unable to produce testosterone or sperm


Erectile dysfunction and decreased muscle mass

Women
Unable to ovulate or menstruate normally and
become amenorrheic

MTB S2CK

p. 107

Few symptoms

Children
Dwarfism

Catecholamines, glucagon, and cortisol


act as stress hormones

MTB S2CK

p. 108

Panhypopituitarism/Presentation

Panhypopituitarism/Diagnostic Tests

Kallman Syndrome
Decreased FSH and LH
Decreased GnRH
Anosmia

Hyponatremia from:
Hypothyroidism
Glucocorticoid underproduction
Potassium levels remain normal

MTB S2CK

p. 108

Aldosterone is unaffected and aldosterone


excretes potassium

MTB S2CK

p. 108

Panhypopituitarism/Diagnostic Tests

Panhypopituitarism/Diagnostic Tests

MRI detects compressing mass lesions on pituitary

Growth Hormone (GH)


IGF level
Arginine Stimulation: increases GH
GHRH stimulation: increases GH
ACTH and Cortisol levels
High
Hi h or normall levels
l
l excludes
l d panhypopituitarism
h
it it i
Sex Hormones
LH, FSH level
Testosterone level
Thyroid
TSH

MTB S2CK

p. 108

Source: James G. Smirniotopoulos, MD

MTB S2CK

p. 108

Panhypopituitarism/Diagnostic Tests

Panhypopituitarism/Treatment

Older, Less Useful Tests

Replace deficient hormones with...

Metyrapone
Inhibits 11-beta hydroxylase and decreases cortisol
Normal: ACTH and 11deoxycortisol levels rise

I
Insulin
li stimulation
ti
l ti
Normal: decreased glucose levels raise GH
Failure of GH to rise in response to insulin
indicates pituitary insufficiency

MTB S2CK

p. 109

Thyroxine
Cortisone
Testosterone and estrogen
Recombinant human growth hormone
Antidiuretic hormone (ADH) and oxytocin (Posterior Pituitary)

Note: No deficiency disease described for oxytocin


Oxytocin helps uterine contraction during delivery, but delivery
still occurs even if its absent
ADH deficiency also known as central diabetes insipidus

MTB S2CK

p. 109

Diabetes Insipidus/Etiology

Diabetes Insipidus/Etiology

Decrease in amount of ADH from pituitary (central DI)


or its effect on kidney (nephrogenic DI)

Nephrogenic DI (NDI):
Chronic pyelonephritis
Amyloidosis
Myeloma
Sickle cell disease
Lithium
Hypercalcemia or hypokalemia inhibits
ADH effect

Central Diabetes Insipidus (CDI)


Damage to brain:

Stroke
Tumor
Trauma
Hypoxia
Infiltration (sarcoidosis, hemochromatosis)
Infection

MTB S2CK

p. 109

MTB S2CK

p. 109

Diabetes Insipidus/Presentation

Diabetes Insipidus/Diagnostic Tests

Excessive thirst
Extremely high-volume urine
Volume depletion

Urine osmolality: Low

Severe Hypernatremia
Neurological symptoms
Confusion,
C f i
disorientation,
di i t ti
llethargy,
th
and
d eventually
t ll
seizures and coma
Only when volume losses are unmatched by fluid
intake

MTB S2CK

p. 109

Urine sodium: Low


Serum osmolality: High
Urine volume: Enormous

MTB S2CK

p. 110

Diabetes Insipidus/Diagnostic Tests

Diabetes Insipidus/Treatment

Difference between central and nephrogenic DI is:

Central DI:
Long-term vasopressin (desmopressin)

Response to vasopressin
Central DI:
Urine volume decrease & urine osmolality increase
Nephrogenic DI:
No effect of vasopressin use on urine volume or
osmolality
MTB S2CK

p. 110

Nephrogenic DI:
1. Correct the cause (hypokalemia or
hypercalcemia)
2. Hydrochlorothiazide, NSAIDs,
amiloride
MTB S2CK

Diabetes Insipidus/Evaluation
High-volume urine, plus
excessive thirst

Serum:
Osmolality: Elevated
Sodium: Elevated

p. 110

Diabetes Insipidus/Evaluation
Vasopressin
(Desmopressin)
stimulation test

Volume depletion, plus


hypernatremia

Urine:
Volume: HIGH
Osmolality: Decreased
Sodium: Decreased

Effect

Urine:
Volume decrease +
osmolality increase

Diagnosis

Proceed with
vasopressin
(desmopressin)
stimulation test
MTB S2CK

p. 109

Treatment

MTB S2CK

Central
diabetes insipidus
Vasopressin

Urine:
No effect

Nephrogenic diabetes
insipidus

Treat underlying cause,


hydrochlorothiazide,
amiloride, NSAIDs

p. 110

Acromegaly

Acromegaly/Etiology

Soft tissue overgrowth


throughout the body

Pituitary adenoma
Part of Multiple Endocrine Neoplasias
(MEN)
Combined with parathyroid and
pancreatic disorders (e
(e.g.,
g gastrinoma
or insulinoma)
Rarely caused by ectopic GH or GHRH
production

MTB S2CK

p. 110

Copyright Richard Usatine, M.D.


Used with permission.

MTB S2CK

p. 110

Acromegaly/Presentation
Increased hat, ring, and
shoe size
Carpal tunnel syndrome
Obstructive sleep apnea
from soft tissues enlarging
Body odor from sweat
gland hypertrophy
Teeth widening from jaw
growth
Deep voice and
macroglossia (big tongue)

MTB S2CK

Acromegaly/Diagnostic Tests
Colonic polyps
Arthralgias from joints
growing out of
alignment
Hypertension for
unclear reasons in 50%
Cardiomegaly, CHF,
and erectile
dysfunction from
increased prolactin
cosecreted with
pituitary adenoma

p. 110 111

Hyperglycemia
Glucose intolerance
Hyperlipidemia
Best initial test...
Insulin-like growth factor (IGF)
M t accurate
Most
t ttest...
t
Glucose suppression test
Normal: Glucose should suppress growth hormone
MRI?
Only after the laboratory identification of
acromegaly
MTB S2CK

p. 111

Acromegaly/Treatment

Hyperprolactinemia/Etiology

1. Surgery

Many causes not associated with pituitary


adenoma

Transphenoidal resection of pituitary


Cures 70%

2. Medications
Cabergoline: Dopamine agonist inhibits GH release
Octreotide or lanreotide: Somatostatin inhibits GH
release
Pegvisomant: GH receptor antagonist

Prolactin increases via:


Cosecretion with GH in acromegaly
g y
Hypothyroidism with pathologically high TRH
levels

3. Radiotherapy
Only when not responsive to surgery or medications
MTB S2CK

p. 111

MTB S2CK

p. 111

Hyperprolactinemia/Etiology

Hyperprolactinemia/Presentation

Women
Galactorrhea
Amenorrhea
Infertility
Men
Erectile dysfunction
Decreased libido
Galactorrhea (very rare in men)

Pregnancy
Chest wall stimulation
Cutting pituitary stalk
Antipsychotic medications, tricyclic
antidepressants,
p
, and SSRIs
Methyldopa
Metoclopromide
Opioids

MTB S2CK

p. 111

MTB S2CK

p. 111

Hyperprolactinemia/Diagnostic Tests

Hyperprolactinemia/Treatment

After the prolactin level is found to be high, perform:


Thyroid function tests
Pregnancy test
BUN/creatinine (kidney disease elevates prolactin)
Liver function tests (cirrhosis elevates prolactin)

1. Dopamine agonists

MRI is done after...


High prolactin level is confirmed AND
Secondary causes like medications are excluded
AND
Patient is not pregnant
MTB S2CK

p. 112

Cabergoline is better tolerated than


bromocriptine

2 Transphenoidal surgery when NOT


2.
responding to medications
3. Radiation is rarely needed
MTB S2CK

p. 112

What to look for...

Thyroid Disorders
Hypothyroidism
Hyperthyroidism

Hypothyroidism

Hyperthyroidism

Bradycardia

Tachycardia, palpitations,
arrhythmia (A fib)
Diarrhea (hyperdefecation)
Weight loss
Anxiety, nervousness,
restlessness
Hyperreflexia
Heat intolerance
Fever

Constipation
Weight gain
Fatigue, lethargy, coma

Thyroid Nodules
Decreased reflexes
Cold intolerance
Hypothermia (hair loss,
edema)
MTB S2CK

p. 113

Hypothyroidism

Hypothyroidism

Etiology
Most from failure of thyroid gland from burnt-out
Hashimoto thyroiditis

Diagnostic tests
Best initial test for all thyroid disorders is...

Acute phase is rarely perceived

Dietary deficiency of iodine


Amiodarone
What to look for...
Hypothyroidism: Everything is SLOW!
Except menstrual flow, which is increased
MTB S2CK

p. 112

TSH
TSH levels are markedly elevated if gland has failed

If the TSH level is suppressed, measure...


T4

Treatment
Replace thyroxine (synthroid)

MTB S2CK

p. 113

Hyperthyroidism/Findings

Diagnosis

Myxedema

Unique feature

Graves disease

Eye proptosis (20 40%)


Skin findings (5%)
Subacute thyroiditis
Tender thyroid
Painless silent
silent thyroiditis Nontender,
Nontender normal P/E
Involuted gland isnt
Exogenous thyroid
palpable
hormone use
High TSH level
Pituitary adenoma

MTB S2CK

p. 113

Multinodular Goiter

Thyroid Bruit

Hyperthyroidism/Diagnostic Tests

Hyperthyroidism/Treatment

T4 (thyroxine) level
Elevated in all forms of hyperthyroidism
TSH level
Pituitary release of TSH is inhibited in all forms
EXCEPT...
Pituitary adenomas, will have high TSH level
Graves disease (unique features):
Eye and skin abnormalities
Elevated radioactive iodine uptake
TSH receptor antibodies
MTB S2CK

p. 113 114

Graves Ophthalmopathy
Best initial therapy...
Steroids

For those unresponsive


to steroids...
Radiation

What if unresponsive to
other therapy...

Source: Jonathan Trobe, MD

Decompressive
surgery
MTB S2CK

p. 114

Hyperthyroidism/Treatment

Acute Hyperthyroidism/Thyroid Storm

Diagnosis

Treatment

Treatment:

Graves disease

Radioactive iodine

Subacute thyroiditis

Aspirin

y
Painless silent thyroiditis

None

Propranolol
Blocks target organ effect
Inhibits peripheral conversion of T4 T3

Exogenous thyroid
hormone use

Stop use

Pituitary adenoma

Surgery

MTB S2CK

p. 114

Thiourea drugs
Methimazole and propylthiouracil
Block hormone production

MTB S2CK

p. 114

Acute Hyperthyroidism/Thyroid Storm

Thyroid Nodules

Treatment:

5% women
1% men
95% benign

Iodinated contrast material


Blocks peripheral conversion of T4 to T3
Blocks release of existing hormone
Steroids (hydrocortisone)
Role in treating hyperthyroidism?

Adenoma, colloid nodule, cyst

Thyroid nodules rarely hyperfunctioning

Radioactive iodine
Ablates gland for permanent cure
MTB S2CK

p. 114

MTB S2CK

p. 114

Thyroid Nodules/Diagnostic Tests


46-year-old woman with a small mass on palpation of
thyroid. No tenderness and otherwise asymptomatic.
What is the most appropriate next step?
a. Fine-needle aspiration Done if TFTs are normal
b Radionuclide iodine uptake scan Determines etiology
b.
of hyperfunctionality
c. T4 and TSH levels
Guides biopsy
d. Thyroid ultrasound
e. Surgical removal (excisional biopsy)

Biopsy with a fine-needle aspirate if theres


normal thyroid function (T4/TSH)
Ultrasound or radionuclide scanning not
required (tests cannot exclude cancer)

Follicular adenoma
when you cant be sure
MTB S2CK

p. 115

MTB S2CK

p. 115

46-year-old woman with thyroid nodule and normal


thyroid function testing has a biopsy showing
indeterminate for follicular adenoma.
What is the most appropriate next step?

Calcium Disorders
Hypercalcemia

g
, determines extent
a. Neck CT Doesnt make a diagnosis,
a
b. Surgical removal (excisional biopsy)
c. Ultrasound Cant excluded cancer, still need biopsy
d. Calcitonin levels Suggests extent of medullary

Hyperparathyroidism
Hypocalcemia

carcinoma only

MTB S2CK

p. 115

Hypercalcemia/Etiology

Hypercalcemia/Etiology

Vitamin D intoxication
Sarcoidosis and other granulomatous
diseases
Thiazide diuretics
Hyperthyroidism
yp
y
Metastases to bone and multiple myeloma

Most common cause is...

MTB S2CK

p. 116

Primary hyperparathyroidism (PTH)


Most cases are asymptomatic
Severe, acute symptomatic hypercalcemia
a high prevalence of cancer

MTB S2CK

p. 116

Hypercalcemia/Presentation

Hypercalcemia/Treatment

Acute, symptomatic hypercalcemia

Acute hypercalcemia, treat with...


Saline hydration at high volume
Bisphosphonates: pamidronate, zoledronic
acid
Calcitonin

Confusion, stupor, lethargy


Constipation
Bone lesions: Osteoporosis
Renal: Nephrolithiasis,
Nephrolithiasis DI,
DI renal insufficiency
Cardiovascular: Short QT syndrome

MTB S2CK

p. 116

Prednisone:
ONLY for sarcoidosis and granulomatous
disease
MTB S2CK

p. 116

Hyperparathyroidism
75-year-old man with history of malignancy admitted with
lethargy, confusion, and abdominal pain. Found to have a
markedly elevated calcium level. After 3L normal saline
and pamidronate, his calcium level is still markedly
elevated the following day.
What is the most appropriate next step in management?
a Calcitonin
a.
Doesnt add to pamidronate
b. Zolendronic acid
c. Plicamycin Less efficacy than pamidronate. Always wrong
d. Gallium Less efficacy than pamidronate. Always wrong
e. Dialysis Not needed. Renal failure has low Ca++

Primary hyperparathyroidism:
Solitary adenoma (80%85%)
Hyperplasia of all 4 glands (15%20%)
Parathyroid malignancy (1%)

f. Cinacalcet Inhibits PTH


MTB S2CK

p. 116 117

MTB S2CK

p. 117

Hyperparathyroidism

Hyperparathyroidism

Elevation in calcium levels often asymptomatic


When symptomatic:
Osteoporosis
Nephrolithiasis and renal insufficiency
Muscle weakness
Anorexia, nausea, vomiting, and abdominal
pain
Peptic ulcer disease (calcium stimulates
gastrin)

Besides high calcium and PTH levels,


you also find:
Low phosphate level
Short QT on EKG
Sometimes an elevated BUN and creatinine
Alkaline phosphatase elevated from effect of
PTH on bone

MTB S2CK

p. 117

MTB S2CK

p. 117

Hyperparathyroidism/Management

Hyperparathyroidism/Treatment

Bone X-ray is NOT a good test for bone


effects of PTH

Surgical removal of involved


parathyroid glands

DEXA densitometry is better


Preoperative imaging of neck with
sonography or nuclear scanning may be
helpful in determining the surgical approach

MTB S2CK

p. 117

MTB S2CK

p. 117 118

10

Hypocalcemia

Hypocalcemia/Presentation

Low calcium =
twitchy, hyperexcitable

Neural hyperexcitability:
Chvostek sign (facial
nerve hyperexcitability)
Carpopedal spasm
Perioral numbness
Mental irritability
Seizures
Trousseau sign (tetany)

High calcium =
lethargic, slow

Souce: nih.gov

MTB S2CK

p. 118

MTB S2CK

p. 118

Hypocalcemia/Diagnosis & Treatment


EKG:
Prolonged QT
May cause arrhythmia

Adrenal Disorders

Ventricular tachycardia

Hypercortisolism

Treatment
Replace calcium and vitamin D

Hyperparathyroidism
Hypocalcemia

MTB S2CK

p. 118

Pituitary Adrenal Axis

Hypercortisolism
Cushing disease
Pituitary overproduction of ACTH
Cushings syndrome
Due to ectopic production of ACTH
C
Carcinoid
i id ((mostt common iis smallll cellll carcinoma
i
off
the lung)
Overproduction autonomously in adrenal gland

Prednisone (glucocorticoid use) causes same


manifestations of hypercortisolism
Source: nih.gov

MTB S2CK

p. 119

11

Hypercortisolism/Etiology

Hypercortisolism/Presentation

Cause of Hypercortisolism

Frequency

Pituitary ACTH
(Cushing disease)

70%

Adrenals

15%

EEctopic
i ACTH
(carcinoid)

10%

Unknown source of ACTH

5%

MTB S2CK

Fat redistribution

HTN

Moon face, truncal


obesity, buffalo hump,
thin extremities,
increased abdominal fat

Skin

MTB S2CK

p. 119

Hypercortisolism/Diagnostic Evaluation

Always confirm the source of


hypercortisolism with biochemical tests
before you perform imaging studies

10% of population has an abnormality of pituitary


on MRI
If you start with a scan, you may remove the
pituitary when the source is in the adrenals

CC: I feel weak and tired, and I notice hair growth


on my face and strange marks on my stomach

24-hour urine
cortisol

Low-dose (1mg)
dexamethasone
suppression test

Increased

High?
ACTH-dependent
Cushings syndrome

p. 120

MTB S2CK

Hypercortisolism/Diagnostic Evaluation
ACTH-dependent
Cushings syndrome

ACTH-independent
Cushings syndrome

Pituitary vs. Ectopic


ACTH production?

Adrenal
Mass?

Supression
of cortisol?
Pituitary adenoma
(Cushing disease)
MTB S2CK

p. 120

From hyperglycemia and


increased free water
clearance

Osteoporosis

p. 119

High-dose
dexamethasone
suppression
i ttestt

Menstrual disorders
Erectile dysfunction
Polyuria

Striae, easy bruising,


decreased wound
healing, thinning of skin

Hypercortisolism/Diagnostic Tests

MTB S2CK

From increased sodium


reabsorption in kidney
and increased vascular
reactivity

Late-night salivary
cortisol

Decreased =
Disease
Excluded

Serum
ACTH

Increased

Low?
ACTH-independent
Cushings syndrome

p. 119 120

Confirmatory Laboratory Findings in


Adrenal Disorders
Adrenal

Pituitary

Ectopic

ACTH level

Low

High

High

Petrosal sinus
ACTH

Not done

High
g ACTH

Low ACTH

Suppresses
Cortisol

No suppression

CT Adrenals
Increased
ACTH &
cortisol?

Ectopic ACTHsecreting tumor

Chest
CT

High dose
No suppression
dexamethasone

Pituitary
Mass?
Pituitary
MRI

No mass
seen?

Petrosal sinus
sampling for
ACTH

MTB S2CK

p. 121

12

Hypercortisolism/Other Findings

Hypercortisolism/Treatment

Cortisol - anti-insulin stress hormone


Hyperglycemia
Hyperlipidemia

Surgically remove source of


hypercortisolism
Transsphenoidal surgery for pituitary sources
Laparoscopic removal for adrenal sources

Theres some aldosterone-like effect of cortisol


Hypokalemia
H
k l i
Metabolic alkalosis
Leukocytosis from demargination of WBCs

MTB S2CK

p. 120

MTB S2CK

p. 121

Evaluation of Adrenal Incidentaloma

Hypoadrenalism (Addisons)/Etiology

How far should you go in the evaluation of an


unexpected, asymptomatic adrenal lesion found on
CT?
1. Metanephrines of blood or urine to exclude
pheochromocytoma
2 Renin and aldosterone levels to exclude
2.
hyperaldosteronism
3. 1 mg overnight dexamethasone suppression test

Addisons disease
Chronic
hypoadrenalism
Etiology

MTB S2CK

p. 121

Autoimmune
destruction ((80%))
Infection (TB)
Adrenoleukodystrophy
Metastatic cancer to
adrenal gland

MTB S2CK

Adrenal crisis
Acute adrenal
insufficiency
Etiology
Hemorrhage, surgery,
hypotension,
yp
, trauma
Suddenly stopping
chronic high-dose
prednisone

p. 121

Hypoadrenalism/Presentation

Hypoadrenalism/Diagnostic Tests

Weakness, fatigue
Acute adrenal crisis
can also present with
Altered mental status
profound hypotension,
Nausea, vomiting,
fever, confusion, and
anorexia, hypotension
coma
Hyperpigmentation from
chronic adrenal
insufficiency

Pituitary failure:
Hypoglycemia
ACTH is low
Hyperkalemia
Metabolic acidosis
Adrenal failure:
Hyponatremia
ACTH is high
High BUN

Eosinophilia is
common in
hypoadrenalism
MTB S2CK

p. 121 122

Copyright Richard Usatine, MD


Used with permission.

MTB S2CK

p. 122

13

Hypoadrenalism

Hypoadrenalism/Treatment

Signs & Symptoms


Weakness
Hypotension
Weight Loss
Hyperpigmentation

Cosyntropin Stimulation Test


Plasma cortisol before & after 250 ug cosyntropin IM or IV

Cortisol fails to rise


High ACTH
Aldosterone Low too

Primary adrenal
insufficiency
MTB S2CK

p. 121 122

Low ACTH
Aldosterone an increase

Secondary adrenal insufficiency


Adrenal atrophy from pituitary
insufficiency

Treatment is more important than testing in


acute adrenal crisis
Replace
p
steroids with hydrocortisone
y
Fludrocortisone
For postural instability
A steroid hormone particularly high in
mineralocorticoid or aldosterone-like effect
MTB S2CK

p. 122

Primary Hyperaldosteronism
Patient brought to ED after sustaining severe abdominal
trauma in MVA. On second hospital day, he becomes
markedly hypotensive without evidence of bleeding. Theres
fever, high eosinophil count, hyperkalemia, hyponatremia, and
hypoglycemia.
What is the next step?
a.
b.
c.
d.
e.

Norepinephrine will constrict


CT scan of the adrenals
vessels more effectively
Draw cortisol level and administer hydrocortisone
Treating severe hypotension
Cosyntropin stimulation testing
more important than finding
ACTH level
etiology
Dexamethasone suppression testing

MTB S2CK

p. 122 123

The autonomous overproduction of


aldosterone despite a high pressure
with a low renin activity

Solitary adenoma: 80%


Bilateral hyperplasia:15-20%
Malignant: 1%

MTB S2CK

p. 123

Primary Hyperaldosteronism

Primary Hyperaldosteronism/Diagnostic Tests

All forms of secondary HTN are more likely in those


whose onset is...

Best initial test...


Plasma aldosterone to plasma renin ratio
A low plasma renin with high aldosterone =
Primary hyperaldosteronism

< 30 or > 60 years


Uncontrolled by 2 antihypertensive medications

Has a characteristic finding on the history, physical,


or labs
Primary hyperaldosteronism =
High BP + Low K+

MTB S2CK

p. 123

Most accurate test...


Adrenal venous blood sampling

high aldosterone!

CT Scan?
Only after laboratory testing reveals...
Low potassium, low plasma renin, and
high aldosterone despite a high-salt diet
MTB S2CK

p. 123

14

Primary Hyperaldosteronism/Treatment

Pheochromocytoma

Unilateral adenoma
Resected by laparoscopy

Nonmalignant lesion of adrenal medulla


Autonomous overproduction of catecholamines
despite high BP

Bilateral hyperplasia
Eplerenone or spironolactone

Pheochromocytoma is the answer when theres:


Episodic HTN
Headache
Sweating
Palpitations and tremor

Spironolactone causes...
Gynecomastia
Decreased libido
Anti-androgenic
MTB S2CK

p. 124

MTB S2CK

p. 124

Pheochromocytoma/Diagnostic Tests

Pheochromocytoma/Diagnostic Tests

Best initial test...

Imaging of adrenal glands (CT or MRI)


Done only after biochemical testing

Plasma catecholamines
Confirmed with...
24-hour urine metanephrines and catecholamines

MIBG scanning
Nuclear isotope scan
Detects
D t t location
l
ti off pheochromocytoma
h
h
t
that
th t
originates outside adrenal gland

More accurate than VMA level

MTB S2CK

p. 124

MTB S2CK

Pheochromocytoma/Diagnostic Tests

p. 124

Pheochromocytoma/Treatment

MIBG scan showing unilateral pheochromocytoma

Source: Laura N Modzelewski

1.
2.
3.
4.

Phenoxybenzamine: (IV alpha blocker)


Propranolol
Calcium-channel blocker (possible)
Laparoscopic removal

MTB S2CK

p. 124

15

Diabetes Mellitus (DM)

Diabetes Mellitus
Presentation
Diagnosis
Treatment
Diabetic Ketoacidosis
Health Maintenance
Complications

Persistently fasting blood glucose levels > 125


on at least 2 separate occasions
Type 1 DM

Type 2 DM

Onset in childhood
Insulin dependent from
early age
Not related to obesity
Insulin deficiency

MTB S2CK

Onset in adulthood
Directly related to
obesity
Insulin resistance

p. 124 125

Diabetes Mellitus/Presentation

Diabetes Mellitus/Diagnostic Tests

Polyuria, polyphagia, and polydipsia

2 FBG measurements > 125 mg/dL

Type 1 DM: thinner than Type 2 diabetics

One glucose level about 200 mg/dL with


symptoms

Type 2 DM: more resistant to diabetic


ketoacidosis (DKA)

Abnormal oral glucose tolerance testing

Both types:

Hemoglobin A1c > 6.5%

MTB S2CK

wound healing

p. 125

MTB S2CK

p. 125

Diabetes Mellitus/Treatment

Diabetes Mellitus/Treatment

Best initial treatment...

Oral Hypoglycemic Medications


Best initial drug: metformin
Metformin blocks gluconeogenesis
Sulfonylureas insulin release from pancreas
weight gain

Diet, exercise, and weight loss


Weight loss controls as much as 25% of
cases of type
yp 2 DM without medications,,
Exercising muscle doesnt need insulin

MTB S2CK

p. 125

Dipeptidyl Peptidase Inhibitors


Increase insulin
Decrease glucagon
Sitagliptin, Saxagliptin, Linagliptin
MTB S2CK

p. 125

16

Diabetes Mellitus/Treatment

Diabetes Mellitus/Treatment

Thiazoladinediones (glitazones)
Rosiglitazone, Pioglitazone
Relatively contraindicated in CHF
Increase fluid overload

Incretins (exenatide, liraglutide)


Raise insulin
Decrease glucagon levels
Decrease gastric motility
Helps in weight loss

Nateglinide and repaglinide


Stimulates insulin release
Similar to sulfonylureas
No additional therapeutic benefit to
sulfonylureas
MTB S2CK

p. 125 126

MTB S2CK

p. 126

Diabetes Mellitus/Treatment

Diabetes Mellitus/Treatment

Alpha-glucosidase inhibitors (acarbose, miglitol)


Block glucose absorption in bowel
Cause flatus, diarrhea, and abdominal pain

Insulin
Added if patient isnt controlled with oral
hypoglycemic agents
Insulin glargine gives steady state of insulin for
entire day
Dosing isnt
isn t tested
Glargine provides much more steady blood levels
than NPH insulin
Combined with short-acting insulin (e.g., lispro,
aspart, or glulisine)

Pramlintide
Analog
A l off protein
t i called
ll d amylin
li thats
th t secreted
t d
normally with insulin
Amylin
decreases gastric emptying
decreases glucagon levels
decreases appetite
MTB S2CK

p. 125 126

Diabetes Mellitus

MTB S2CK

p. 126

Diabetes Mellitus/Treatment
Pharmacokinetics of insulin formulations

MTB S2CK

p. 126

Drug

Type

Onset (hr)

Peak (hr)

Duration (hr)

Aspart
Glulisine
Lispro

Rapid-acting

0.20.5

0.52

34

Regular

Short-acting

0.51

23

68

NPH

Intermediate

1.5

410

1624

Lente

Intermediate

1.53

715

1624

Ultralente

Long-acting

34

915

2228

Glargine

Long-acting

No peak

2436

MTB S2CK

p. 126

17

Diabetic Ketoacidosis (DKA)

Diabetic Ketoacidosis/Treatment

More common with type 1 diabetes


Can definitely present in type 2 diabetes
Presents with...
Hyperventilation
Altered mental status
Metabolic
M t b li acidosis
id i with
ith increased
i
d anion
i gap
Hyperkalemia in blood, but decreased total body
potassium because of urinary spillage
Increased anion gap on blood testing
Serum is positive for ketones

Treat with...
1. Large-volume saline and insulin replacement
2. Replace potassium when potassium level
approaches normal
3. Correct the underlying
y g cause

MTB S2CK

p. 126

Noncompliance with medications


Infection
Any serious illness

MTB S2CK

p. 127

Diabetes Mellitus/Health Maintenance


57-year-old man admitted to ICU with altered mental
status, hyperventilation, and markedly elevated
glucose level.
Which of the following is the most accurate measure
of the severity of his condition?
a. Glucose level Can be elevated without DKA
If very low, theres risk of death
b. Serum bicarbonate
Mean
very
little
c. Urine ketones
Not
all
blood
ketones are detected
d. Blood ketones
e. pH level on blood gas Need to know whats

All patients with DM should receive...


Pneumococcal vaccine
Yearly eye exam to check for proliferative
retinopathy, which needs laser therapy
Statins to g
get LDL < 100 mg/dL
g
ACEi or ARBs to get BP < 130/80 mmHg
ACEi or ARB if urine tests positive for
microalbuminuria
Foot exam for neuropathy and ulcers

responsible for pH level


MTB S2CK

p. 127

MTB S2CK

p. 127

Complications of Diabetes

Complications of Diabetes

Gastroparesis
Immobility of bowels
Bloating, constipation
Early satiety, vomiting
Abdominal discomfort
Treat with metoclopramide or
erythromycin

Non-proliferative retinopathy
Tighter control of glucose
Aspirin doesnt help retinopathy

MTB S2CK

p. 128

Proliferative retinopathy
Neovascularization and vitreous
hemorrhages
Treated with laser photocoagulation

MTB S2CK

p. 128

18

Complications of Diabetes

Neuropathy
Decreased sensation in feet
Main cause of skin ulcers
Leads to osteomyelitis
Treatment pain with
Pregabalin
Gabapentin
Tricyclic antidepressants

MTB S2CK

p. 128

19

Autonomy
Advance Directives
Minors
Brain
i Death
h

Ethics
Dr. Conrad Fischer, MD
Associate Professor of Medicine
Touro College of Medicine
New York City

Ethics

Ethics

Every human being of adult years and sound mind


has the right to determine what shall be done with his
own body; and a surgeon who performs an operation
without his patients consent commits an assault, for
which he is liable in damages except in cases of
emergency where the patient is unconscious and
where
h
it iis necessary tto operate
t before
b f
consentt can
be obtained.

This states the premise underlying half


the ethics questions on S2 CK of USMLE:
1. Autonomy
2. Adult
3 Capacity
3.
C
it tto understand
d t d

Justice Benjamin Cardozo, Schloendorff v. Society of New York Hospital,


211 NY 125, 105 NE 92 (1914)

MTB S2CK

MTB S2CK

p. 561

p. 561

Autonomy

Patients have sole right to determine what


treatment they shall and shall not accept
Autonomy beats Beneficence
Beneficence: trying to do good for others
Trying
y g to help
p not as important
p
as following
g
wishes
Patients have the right
to refuse treatments
that are good for them if
they dont want them.
MTB S2CK

p. 561

A man has an ugly house you offer to paint free in his favorite
color. Everyone in neighborhood agrees the house is ugly
& what you offer is clearly superior. He understands everything
you are offering, including the clear benefit to him. The man
Cost and benefit and the
still refuses.
common good arent as
What do you do?
important as the autonomy
have to just do
a. Honor the mans
man s wishes: no paint job individuals
what
h t they
th wantt with
ith th
their
i
own property.
b. Paint his house against his will
c. Ask the neighborhood council to consent to the paint job
d. Get a psychiatric evaluation on the man
e. Get a court order to allow the paint job
f. Ask his family for consent to the paint job
g. Wait until he is out of town, then paint his house
MTB S2CK

p. 561 562

Advance Directives
Man comes to ED after MVA that causes a ruptured spleen. Hes
fully conscious. He understands that hell die without splenectomy,
and that hell live if he has the splenectomy. He refuses the repair
and blood transfusion. Entire family including brother who is
healthcare proxy and document completed only a few weeks ago
clearly state, Everything possible should be done, including
surgery.
What do y
you do?
a. Honor his current wishes, no surgery
b. Wait until he loses consciousness, then perform the surgery
You must follow the last known wishes of the
c. Psychiatric consult
patient, even if they are verbal, and even if they
contradict the written proxy. You cannot wait until
d. Ethics committee
his consciousness is lost, then go against his
e. Emergency court order wishes.
f. Follow what is written in the documented health-care proxy
g. See if there is consensus from the family
MTB S2CK

p. 562

Tell caregivers parameters of care the


patient wanted
Agent = person designated by patient to
carry out patients wishes
Term Agent sometimes used
interchangeably with healthcare proxy
Healthcare proxy is written document
outlining parameters of care
Major problem with proxy is details of care
often unclear
MTB S2CK

p. 562

Advance Directives

Advance Directives

Not helpful to just say, No heroic measures


To be useful, document must specifically
state, No intubation, no CPR, no
chemotherapy, no dialysis
Can also specifically state wishes about fluid
and nutrition
If proxy says, No NG tube, no artificial
feeding, then its useful
Proxy takes effect only when patient has lost
capacity to make decisions

Order of Decision Making

MTB S2CK

p. 561 562

1. Patient with capacity supersedes all


else
2 Healthcare
2.
H lth
proxy: an agentt ((person))
to carry out wishes

MTB S2CK

p. 563

Advance Directives

Advance Directives

Order of Decision Making


3. Living will

Order of Decision Making


4. Persons clearly familiar with the
patients wishes

Document outlining patients wishes


Stating, I never want dialysis is more valid than a
family member or friend saying, From what I know
about
b t hi
him, h
he wouldnt
ld t wantt dialysis,
di l i or He
H ttold
ld me
he never wants dialysis.
Documentation!
Written living will with concrete statements I never
want blood transfusion or chemotherapy is valid

MTB S2CK

p. 563

Problem with documentation


Difficult for friends to document they knew the
patients wishes
If case clearly states friend knows and can
prove that she knew the patients wishes,
then this is the plan of care thats followed

MTB S2CK

p. 563

Advance Directives

Advance Directives

Order of Decision Making


5. Family

Ethics Committee
The answer when:

General order of decision making


1.
2
2.
3.
4.

Spouse
Adult children
Parents
Siblings

Unlike life, USMLE S2 CK must be clear


If family is split, then answer is:
Ethics committee or court order
MTB S2CK

p. 563

1. Patient has lost capacity to make decisions


2 Advance directive is missing or unclear
2.
Critically important for medical futility
E.g. when the patient or proxy asking for
tests and treatments that may have no
benefit
MTB S2CK

p. 563

Advance Directives

Advance Directives

Court order is right answer when:


Theres no advance directive and
1. Patient has no capacity
2. Family in disagreement

Psychiatric Evaluation of Patient


The answer when...
It is not clear if patient has capacity

Like a house being


g left equally
q
y to four children
who cannot agree what to do with it

Caregivers want to withdraw care and ethics


committee cannot reach a conclusion
MTB S2CK

p. 563

Question clearly states patient has capacity?


Not necessary!
Clearly delirious or psychotic?
Not necessary!
MTB S2CK

p. 563

Minor

Minor

Minors do not have decision-making capacity


Cannot consent to or refuse medical treatments
Only parents or legal guardian can consent and
refuse
Exceptions are:
Contraception
Prenatal care
Substance abuse treatment
Sexually transmitted diseases (STDs) including
HIV/AIDS

Abortion
States are split on parental notification
laws
Some require it, some dont
Your
Y
answer is:
i
Tell the minor patient to notify her
parents.

MTB S2CK

p. 564

MTB S2CK

p. 564

Brain Death

Brain death = death in our legal system


If brain dead, you dont need consent to stop
therapy such as mechanical ventilation or
antibiotics
Court order and ethics committee arent
correct answers

Consent
Do Not Resuscitate Orders
Physician Assisted Suicide
Euthanasia
Terminal Sedation and
Law of Double Effect
Futile Care
Organ and Tissue Donation

USMLE S2 CK will want you to


discuss, educate, explain, and
confer before everything else.
MTB S2CK

p. 564

Consent

Consent

Patient signs consent for ovarian biopsy on left


side. At surgery you find cancer of right side.
What do you do?
Wake patient up & obtain consent to remove
ovary on right side.

Only adults can consent to procedures


Each procedure needs individual consent
Consent implied in emergency
Person doing procedure must obtain consent
Adverse effects of procedure must be explained to
make consent valid
Consequences of refusing procedure must be
explained to make consent valid
Pregnant women can refuse procedures and
treatments for their unborn children
Telephone consent is valid
MTB S2CK

p. 564

MTB S2CK

p. 564

Consent

Do Not Resuscitate Orders

Patient needs colonoscopy. Gastroenterologist asks


you to obtain consent for procedure.
What do you do?

DNR orders refer only to withholding


CPR
They dont refer to withholding any other
form of therapy

The gastroenterologist who will perform the


procedure needs to obtain consent.
Do you know all complications of the procedure and
alternatives?
If you dont explain the possibility of perforation
because you are unfamiliar with it, the consent isnt
valid
If patients colon perforates and you didnt explain
alternate procedures, the consent isnt valid
MTB S2CK

p. 564

MTB S2CK

p. 565

Do Not Resuscitate Orders

Patient with capacity consents to DNR before


losing consciousness. She needs a surgical
procedure, but the surgeon refuses because
the patient is DNR.
What do you do?
Perform the surgery
DNR doesnt mean withholding antibiotics,
chemotherapy, or surgery
DNR means only that, if the patient dies, you
wont attempt resuscitation
MTB S2CK

p. 565

Physician-administered treatment
intended to end or shorten patients life

Always
y wrong
g

p. 565

Terminal Sedation and


Law of Double Effect

The question is one of intent:


If the meds are given with intent to
relieve pain, and as an adverse effect
they shorten life, its ethical
If the primary intent is to shorten life, its
unethical

MTB S2CK

p. 565

Always a wrong answer!


This includes states in which its legal
Ethical requirements for physicians
supersede legality
Physician
Physician-assisted
assisted suicide is administered by
the patient, but this is still unethical for
physicians
Physician ethics come before legal
requirements. You cannot do something
unethical even if its legal at the moment.
MTB S2CK

Euthanasia

MTB S2CK

Physician Assisted Suicide

p. 565

Terminal Sedation and


Law of Double Effect

Is it acceptable to administer pain medication


even if theres the possibility treatment
shortens life?
For example, its acceptable to give pain
medications to a person with COPD who has
metastatic cancer even if the only way to
relieve pain is to give enough opiates that
breathing may be impaired, causing the
patient to die earlier
MTB S2CK

p. 565

Futile Care

Physician not obligated to render care


thats futile even if the family or patient
wants it
If brain dead & family insists on
continued mechanical ventilation
ventilation, you
are under no obligation to do so
You are under no obligation to perform
tests and treatments you consider
worthless
MTB S2CK

p. 565

Organ and Tissue Donation

Consent for Organ Donation

Payment for ORGANS is unacceptable

Only organ donor network should ask for


consent for organs
Ethical conflict of interest for physician to ask
for consent for organ donation
Organ donor network has fewer refusals than
physician
Organ donor cards give an indication of
patients wishes, but family can refuse organ
donation even if patient has organ donor
card

Payment for RENEWABLE tissues


(sperm & eggs) is acceptable

MTB S2CK

p. 565

MTB S2CK

p. 566

Confidentiality

Physician
Responsibilities
Confidentiality
Doctor/Patient Relationship
Gifts from Industry
Abuse
Impaired Drivers
Execution of Prisoners
Torture

Confidentiality

Right to confidentiality cannot be broken


for:
Employers
Coworkers
Government agencies
Family
Confidentiality is
Friends
important, but not as
important as protecting
others from harm.
MTB S2CK

p. 566

Patients right to confidentiality can be


broken when theres danger to others
STDs
HIV/AIDS
Airborne
Ai b
communicable
i bl diseases
di
((e.g.,
tuberculosis)
Court order demanding information

MTB S2CK

p. 566

Patient with HIV/AIDS has repeatedly refused to


disclose his HIV status to his sexual partner. The
partner accompanies the patient to the office visits
and is in the waiting room. The patient insists you not
tell the partner.
What do you do?
a.
b.
c.
d.

The confidentiality of the patient

Honor the patients wishes is not as important as protecting


the health of the partner.
Obtain a court order
Consult the ethics committee
Either the physician or the department of health
can notify the partner

MTB S2CK

p. 566

Confidentiality
Woman comes to your office with valid identification
from a law enforcement/government agency. She
requests a copy of your patients medical records.
What do you do?
Provide health-related protected records to
government agencies, including those from law
enforcement,
f
t only
l if:
if

Confidentiality

HIV-positive healthcare workers do


not have to disclose their status to
their patients or their employers.

Valid warrant or subpoena from courts


Otherwise violates the constitutional protection against
illegal search and seizure of property
This violates HIPAA, which protects health information

MTB S2CK

p. 566

Doctor/Patient Relationship

Physicians arent obligated to accept


everyone coming to him or her as a patient
You have the right to end the doctor/patient
relationship but must give the patient
sufficient time to obtain another caregiver
Small
S ll gifts
ift ffrom patients
ti t are acceptable
t bl as
long as they arent tied to a specific
treatment request
Romantic or sexual contact between
patients and their current physicians is never
acceptable
MTB S2CK

p. 566 567

MTB S2CK

p. 566

Gifts from Industry

Never acceptable!
Even pens, penlights, pads, and cups
are unacceptable
Meals in direct association with
educational activities arent considered
gifts

MTB S2CK

p. 567

Elder Abuse

Domestic Violence and Spousal Abuse

You can report elder abuse against


the consent of patient
Abused older adults may be too weak,
fragile, or vulnerable to protect
themselves
Elder abuse is treated ethically like
child abuse

Unlike child abuse, domestic abuse


cannot be reported against the
patients wishes
You can report and intervene only with
patients
patient
s consent

MTB S2CK

p. 567

MTB S2CK

p. 567

Impaired Drivers
(Seizure Disorders and Driving)

Least clear area nationally


No uniformity of laws between states
Answer suggest that the patient find another
means of transportation
Wrong answers would be:
a. Confiscating car keys and reporting to law
enforcement
b. Hospitalizing patient
c. Refusing to let the patient get in car
MTB S2CK

p. 567

Torture

Execution of Prisoners

Never ethical for a physician to


participate in executions at any level
You cannot ethically formulate a lethal
injection or even do so much as
pronounce a prisoner dead
Even if state law makes execution
legal, physicians should never
participate at any level

MTB S2CK

Torture

Physicians are never to participate in torture


of prisoners or detainees
Even if the question states that youre in the
military, your ethical obligation as a
physician supersedes your obligation to the
military
ilit
This would include:

Torture is the ethical


equivalent of child abuse.
Your p
participation
p
is never
acceptable; youre obligated
only to report it.

a. Refusing orders from military superiors to


participate in torture
b. Keeping the torture safe so that its not fatal
or damaging
MTB S2CK

p. 567

p. 567 568

MTB S2CK

p. 568

Esophagus
Achalasia
Esophageal Cancer
Esophageal Spasm
Esophagitis
Rings and Webs
Zenkers Diverticulum
Scleroderma
Mallory Weiss Tear
Boerhaaves Syndrome

Gastroenterology
Niket Sonpal, MD
Chief Resident
Lenox Hill Hospital NSLIJ
Assistant Clinical Professor Touro College of Medicine

Esophageal Disorders/Definitions

Esophageal Disorders/Presentation

Dysphagia
Difficulty swallowing
Odynophagia
Pain while swallowing

Both can lead to weight loss


Hence, weight loss cannot be used to answer
What is the most likely diagnosis?

MTB S2CK

p. 237

With the symptoms


Weight loss
Anemia
Heme-positive stool

MTB S2CK

Endoscopy!

p. 237

Achalasia/Diagnosis

Achalasia/Diagnosis

Inability of lower esophageal sphincter (LES) to


relax due to a loss of nerve plexus within
lower esophagus
Etiology unclear
Aperistalsis of esophageal body

Look for...
Young patient (< 50)
Progressive worsening dysphagia to both
solids and liquids at the same time
No association with alcohol and tobacco
use
Complain of...
Regurgitation and halitosis

Chagas Disease = recent travel to


South America and new onset dysphagia
MTB S2CK

p. 237

MTB S2CK

p. 237

Achalasia/Diagnostic Tests

Marked Dilation of the Esophagus

Barium esophagram
Will show a birds beak

Source: commons.wikimedia.com

MTB S2CK

Source: Niket Sonpal, MD

p. 238

Achalasia/Diagnostic Tests

Achalasia/Diagnostic Tests

Most accurate test is


Manometry

Chest X-ray
May show abnormal widening of esophagus,
but is neither very sensitive nor very specific

Chest X-ray is never the right


answer to diagnose achalasia

Source: Niket Sonpal, MD

MTB S2CK

p. 238

MTB S2CK

p. 238

Achalasia/Diagnostic Tests

Achalasia/Treatment

Upper endoscopy
Shows normal
mucosa in achalasia

Cannot exactly be cured


All treatment is based on simple
mechanical dilation of esophagus

Source: Niket Sonpal, MD

MTB S2CK

p. 238

MTB S2CK

p. 238

Achalasia/Treatment

Achalasia/Treatment

Pneumatic dilation

Botulinum toxin injection


Will relax LES, but effects wear off in about
3-6 months, requiring reinjection

Effective > 80% to


85% of patients
Repeat sessions
are necessaryy

Site of
Botulinum
Injection

Source: commons.wikimedia.com

Freedictionary.com

MTB S2CK

p. 238

MTB S2CK

p. 238

Achalasia/Treatment

Esophageal Cancer/Diagnosis

Surgical sectioning
or myotomy can help
to alleviate
symptoms
Known as Heller
M t
Myotomy

Look for:
Age 50 or older
Dysphagia with solids first then progresses to
liquids
Association with p
prolonged
g alcohol and
tobacco use
> 5 years of GERD symptoms

Niket Sonpal MD

MTB S2CK

p. 238

MTB S2CK

Esophageal Cancer/Diagnostic Tests

p. 239

Esophageal Cancer/Diagnostic Tests

Endoscopy is indispensible, since only a


biopsy can diagnose cancer
NO TISSUE, NO ISSUE

CT and MRI scans


Only shows extent of tissue
PET scan
Gives information about other anatomic
lesions

Barium cannot
diagnose cancer

Source: commons.wikimedia.com

MTB S2CK

p. 239

MTB S2CK

p. 239

Esophageal Cancer/Treatment

Esophageal Cancer/Treatment

Surgical resection is always the thing to try


No resection (removal) = no cure

Stent placement
Purely palliative

Chemotherapy and radiation

Source: Niket Sonpal, MD

MTB S2CK

p. 239

MTB S2CK

p. 239

Esophageal Spasm/Background

Esophageal Spasm/Diagnosis

Diffuse esophageal spasm (DES) and nutcracker esophagus are clinically


indistinguishable
Both present

EKG and stress

Sudden onset of chest pain


Not related to exertion

MTB S2CK

p. 239

Esophageal Spasm/Treatment

Normal

Esophagram best initial test


Normal

Manometry
y most accurate test
Abnormal contraction in various section of the
esophagus

MTB S2CK

p. 239

Esophageal Spasm/Barium Study

Treated with:
Nitrates
Relax smooth muscle

Calcium-channel blockers
No Ca+2= no smooth muscle contraction

MTB S2CK

p. 240

MTB S2CK

p. 240

Infectious Esophagitis/Management
43-year-old man recently diagnosed with AIDS comes to
ED with pain on swallowing thats become progressively
worse over the last several weeks. Theres no pain when
not swallowing. His CD4 count is 43 mm3. The patient
isnt currently taking any medications.
What is the most appropriate next step in management?
Doesntt diagnose candida
a Esophagram Doesn
a.
b. Upper endoscopy Too invasive
c. Oral nystatin swish and swallow Only for oral candida
d. Intravenous amphotericin Too big gun
e. Oral fluconazole

MTB S2CK

Dysphagia with
HIV CD4 < 100

Yes
Empirically start
fluconazole

Yes

Improvement

No
Perform upper
endoscopy with
biopsy

Continue therapy
and HAART

CMV large
ulcerations. Tx:
ganciclovir or
foscarnet

HSV small
ulcerations. Tx:
acyclovir

p. 240

Infectious Esophagitis

Rings and Webs


Schatzki ring
Schatzki rings and Plummer-Vinson syndrome both
cause dysphagia

Source: Aaron Cho

MTB S2CK

p. 241

Source: commons.wikimedia.com

Rings and Webs

Rings and Webs

Schatzki ring
Associated with
intermittent dysphagia

Plummer-Vinson syndrome
Triad of:
1.
Dysphagia due to Esophageal Webs
2.
Iron Deficiency Anemia
3.
Glossitis
More
M
proximal
i l than
th Schatzki
S h t ki rings
i
Diagnosis
Barium esophagram

Treatment
Iron replacement
MTB S2CK

p. 241

Source: Azmeena Laila, MD

MTB S2CK

p. 241

Zenkers Diverticulum

Zenkers Diverticulum

Outpocketing of
posterior pharyngeal
constrictor muscles

Best test for diagnosis is


Esophogram

No medical treatment and


surgical intervention is
best

Stanford Hospital Media

Source: commons.wikimedia.com

MTB S2CK

p. 241

MTB S2CK

p. 241

Scleroderma

Mallory Weiss Syndrome/Presentation

Presentation

Upper GI bleed
secondary to repetitive
retching
Self-limited
Dx: endoscopy

Symptoms of reflux
Scleroderma or progressive systemic sclerosis

Diagnosis
Manometry

Treatment
PPIs
Screening for Barretts esophagus
Source: commons.wikimedia.com

MTB S2CK

p. 242

MTB S2CK

p. 242

Mallory Weiss Syndrome/Treatment

Boerhaaves Syndrome

No specific therapy and will resolve


spontaneously
Severe cases with persistent bleeding are
managed with injection of epinephrine or
electrocautery to stop bleeding

Esophageal rupture
due to prolonged
retching
A full thickness tear

Physical exam
Hammens sign crepitus
Subcutaneous air =
snap crackle and
pop

Source: commons.wikimedia.com

EMERGENCY!
MTB S2CK

p. 242

Stomach
Epigastric Pain
Gastroesophageal Reflux Disease
Barretts Esophagus
Gastritis
Peptic Ulcer Disease
Non ulcer Dyspepsia
Gastrinoma
Diabetic Gastroparesis

44-year-old woman comes to see you because of


epigastric pain for several months. She denies
nausea, vomiting, weight loss, or blood in her stool.
On physical examination you find no abnormalities.
What is the most likely diagnosis?
a.
b.
c.
d.
e.
f.

Duodenal ulcer disease No anemia or heme (+) stools


Gastric ulcer disease Heme (+) stools
Gastritis Need EGD to diagnose
Pancreatitis Acute not chronic
Non-ulcer dyspepsia
Pancreatic cancer No painless jaundice

MTB S2CK

Abdominal Pain/Causes of Pain by Location

p. 242 243

Epigastric Pain/Most Likely Diagnosis


If this is in the history:

Right Upper
Quadrant

Left Upper
Quadrant

Cholecystitis
Biliary colic
Cholangitis
Perforated
duodenal ulcer
Right Lower
Quadrant

Splenic rupture
IBS Splenic
flexure syndrome

Mid-Epigastrium
Pancreatitis
Aortic dissection
Peptic ulcer disease

Appendicitis
Ovarian torsion
Ectopic pregnancy
Cecal diverticulitis

Left Lower
Quadrant
Sigmoid volvulus
Sigmoid diverticulitis
Ovarian torsion
Ectopic pregnancy

Pain worse with food


Pain better with food
Weight loss
Tenderness
Bad taste, cough, hoarse
Diabetes, bloating
Nothing

MTB S2CK

Epigastric Pain/Treatment

Endoscopy
Only way to truly
understand the etiology
of epigastric pain from
ulcer disease
Only
y way
y to g
give a
precise diagnosis

Proton pump inhibitors


(PPIs)
First line therapy
Empiric
Minimum 4 weeks

MTB S2CK

p. 243

H2 blockers
Ranitidine, nizatidine,
cimetidine, famotidine
Not as effective, but will
work in about 70% of
patients
MTB S2CK

Gastric ulcer
Duodenal ulcer
Cancer, gastric ulcer
Pancreatitis
Gatroesophageal reflux
Gastroparesis
Non ulcer dyspepsia

p. 243

Epigastric Pain/Diagnostic Tests

Wikimedia

The most likely diagnosis is:

Liquid antacids
Roughly the same
efficacy as H2 blockers
Misoprostol
Artificial prostaglandin
analogue
Used to treat NSAIDinduced gastric damage
When PPIs arrived,
misoprostol became
wrong answer

p. 243

Gastroesophageal Reflux Disease (GERD)


Inappropriate relaxation of LES
Results in acid contents of stomach coming up into
esophagus
Patient complains of...
Heartburn
Metallic taste
Cough

MTB S2CK

p. 244

42-year-old man comes to office with epigastric pain


radiating up under his chest, which becomes worse
after lying flat for an hour. He also has a brackish
taste in his mouth.
What is the most appropriate next step in the
management of this patient?
a. Ranitidine
Not as effective
b. Liquid antacid Not as effective
c. Lansoprazole
Not the first step must fail 1st
d. Endoscopy
e. Barium swallow Not an anatomic disease
f. 24-hour pH monitoring Too invasive
MTB S2CK

p. 244

GERD/Diagnostic Tests

GERD/Treatment

Most often diagnosed using patient history


When not clear, the most accurate test is...

All patients should try lifestyle changes

24-hour pH monitoring

Endoscopy when:
Dysphagia or odynophagia
Weight loss
Anemia or heme-positive stools
> 5 years of symptoms to exclude Barretts
esophagus
MTB S2CK

p. 244 245

Lose weight, avoid alcohol, nicotine and


certain foods, eat within 3 hours of bedtime
Elevate head 6-8 inches

Mild or Intermittent Symptoms


May be treated with liquid antacids or H2
blockers
Persistent Symptoms or Erosive Esophagitis
PPIs for 4-6 weeks
MTB S2CK

p. 245

GERD/Treatment

Barretts Esophagus

5% will not respond to PPIs


Patients require surgery to tighten LES:

Due to long-standing GERD


Change to columnar metaplasia
Typically takes 5 years

Nissen fundoplication
Stomach wrapped around LES

Endocinch
Scope used to place a suture around LES

Local heat or radiation of LES


Causes scarring to tighten LES
Last resort
MTB S2CK

p. 245

Diagnosis
Biopsy via endoscopy
Only way to assess presence of Barretts
esophagus
MTB S2CK

p. 245

Barretts Esophagus/Treatment

Gastritis

Finding
Barretts alone
(metaplasia)
Low grade dysplasia

Inflammation or erosion of gastric lining


Sometimes called gastropathy

High grade dysplasia

Management
PPIs and rescope every 2
3 years
PPIs and rescope every
6 12 months
Ablation with endoscopy,
photodynamic therapy,
radiofrequency ablation,
or surgical removal

3 levels: mild, moderate and severe


With or without hemorrhage

Many causes

Alcohol
NSAIDs
Helicobacter pylori
Portal HTN
Stress
Burns, trauma, sepsis, multiorgan failure

MTB S2CK

p. 246

MTB S2CK

p. 246

Gastritis/Presentation

Gastritis/Diagnosis

Often with GI bleeding without pain


Severe, erosive gastritis can present with
epigastric pain
Look for NSAIDs or alcoholism in history
No unique
q p
physical
y
findings
g

The most accurate


test is...
EGD

You cannot answer the most likely diagnosis


question from history and physical alone

MTB S2CK

p. 246

What is good
about this test?

Test

Endoscopic
biopsy
Serology

H. py
pylori testing
g
Treated if associated
with gastritis

MTB S2CK

Testing for Helicobacter pylori


Invasive test
(endoscopy)

Inexpensive,
excludes infection
if negative

Lacks specificity,
cant distinguish
current/previous
infection

Urea C13 or C14 Positive only in


breath testing active infection,

Requires expensive
equipment

H. Pylori
stool antigen

Requires stool
sample

Treat with PPIs


H2 blockers, sucralfate, and liquid antacids
arent effective as PPIs

Sucralfate is an inert substance (aluminum


hydroxide complex) that coats the stomach.
If sucralfate is presented as a choice, its
nearly always the wrong answer.

noninvasive

MTB S2CK

p. 246 247

Source: commons.wikimedia.org

p. 246

Gastritis/Treatment
What is bad
about this test?

Most accurate of
all tests

Positive only in
active infection,
noninvasive

Capsule endoscopy is
not appropriate for upper
GI bleeding if endoscopy
is one of the choices

MTB S2CK

p. 247

Gastritis

Peptic Ulcer Disease (PUD)

Stress ulcer prophylaxis is indicated in:


Mechanical ventilation
Burns Curlings ulcers
Head trauma Cushings ulcers
Coagulopathy

PUD refers to both duodenal ulcer and


gastric ulcer disease
Endoscopy is key to diagnosis and
treatment
MCCs H. pylori and NSAIDs

MTB S2CK

p. 247

Note: Alcohol and tobacco dont cause ulcers.


They delay healing of ulcers

MTB S2CK

Peptic Ulcer Disease

p. 247 248

Peptic Ulcer Disease/Presentation

PUD presents with recurrent episodes of


epigastric pain
Abdominal Pain
(dull, sore, gnawing)
Pain that is improved
with eating

Pain that is worsened


by eating

Duodenal Ulcer

Source: commons.wikimedia.org

Source: commons.wikimedia.org

MTB S2CK

p. 248

Peptic Ulcer Disease/Diagnosis

Peptic Ulcer Disease/Treatment

Upper endoscopy: most accurate test


Allows for intervention and biopsy

PUD responds to PPIs...


But must treat for H. pylori

Ulcer on endoscopy
Biopsy positive for H. pylori

Duodenal Ulcer

Clip or epinephrine
injection

Radiologic Testing
Poor sensitivity and no histology testing
MTB S2CK

p. 248

Gastric Ulcer

Bleeding ulcer

H. pylori in more than 80%


to 90% of cases
Antibiotic treatment

Gastric Ulcer

H. pylori in 50% to
70% of cases

Cancer is present in 4% of those with GU but


in none of those with DU.
MTB S2CK

p. 248 249

10

Peptic Ulcer Disease/Treatment

Peptic Ulcer Disease/Treatment

H. pylori Treatment

56-year-old woman comes to clinic with epigastric pain from


endoscopically confirmed duodenal ulcer, which is
unresponsive to several weeks of PPI, clarithromycin, and
amoxicillin.
What is the most appropriate next step?

Biopsy positive for H. Pylori


PCN allergy?

PPI combined with


clarithromycin and amoxicillin

Replace amoxicillin
with metronidazole

30-60 day post therapy


testing for eradication
MTB S2CK

p. 243

Peptic Ulcer Disease/Treatment


Treatment of Refractory Ulcers
If initial therapy doesnt resolve the DU then
detecting persistent H. pylori and switching the
antibiotics to metronidazole and tetracycline is
appropriate or adding bismuth
For those with refractory
y GU,, a repeat
p
endoscopy is done to exclude cancer

a.
b.
c.
d.
e.
f.
g.

Refer for surgery


g y Too invasive
Switch the PPI to ranitidine PPI is superior to ranitidine
Abdominal CT scan CT cant detect H. pylori
Capsule endoscopy Cant detect H. pylori
Urea breath testing
Vagotomy Not necessary and too invasive
Add sucralfate Always the wrong answer

MTB S2CK

p. 249

Gastric Ulcers vs. Duodenal Ulcers

GU is routinely biopsied
Routinely
repeating the
endoscopy to
confirm healing is
standard with GU

GU vs. DU

GU is associated
with cancer in
4%

GU pain is more often


worsened by food

MTB S2CK

p. 249

MTB S2CK

p. 250

Non ulcer Dyspepsia

Non ulcer Dyspepsia

Non-ulcer (functional) dyspepsia is epigastric


pain that has no identified etiology
MCC of epigastric pain in U.S.

The best initial therapy is with PPIs


NUD < 45-55 years

NUD > 45-55 years

Empiric PPI therapy

PPIs and EGD to r/o


cancer

Symptoms persist and H.


Pylori is present =
treat for H. pylori
MTB S2CK

p. 250

MTB S2CK

p. 250

11

Gastrinoma (Zollinger Ellison Syndrome)

Gastrinoma/Diagnostic Tests

Look for patient with ulcers


that are:
1. Large > 1 cm
2. Multiple
3. Past 3rd portion of
duodenum
4. Presents with:
Diarrhea
Abdominal pain
Anemia
Heme-positive stools

After endoscopy confirms an ulcer, diagnosis made using


one of the following three:
1. High gastrin levels after antisecretory therapy (PPIs/H2)
2. High gastrin levels despite a high gastric acid output
3. Persistent high gastrin levels despite injecting secretin
(most accurate)

MTB S2CK

Source: commons.wikimedia.org

p. 251

MTB S2CK

p. 251

Source: Niket Sonpal MD

Gastrinoma/Diagnostic Tests

Gastrinoma/Treatment

What is the next best step after diagnosing ZE?


Exclude metastatic disease

Local disease is removed surgically

Somatostatin receptor scintigraphy +


Endoscopic ultrasound

Metastatic disease requires lifelong


PPIs to block acid production

Ultrasound, CT, and MRI of abdomen have


poor sensitivity and are never the most
accurate test for diagnosing ZE
MTB S2CK

p. 251

MTB S2CK

p. 251

Diabetic Gastroparesis
64-year-old patient with diabetes for 20 years comes to
the office with several months of abdominal fullness,
intermittent nausea, constipation, and a sense of
bloating. On physical examination, a splash is heard
over the stomach.
What is the next step?
a. Abdominal CT scan Can only diagnose static conditions
b. Colonoscopy Wrong end!
c. Erythromycin
d. Upper endoscopy Wouldnt reveal anything
e. Nuclear gastric emptying study Most accurate test for
MTB S2CK

p. 252

diabetic gastroparesis,
but rarely used

Long standing diabetes

Autonomic neuropathy
from high glycemic
index

+
Resultant dysmotility
from an inability to
sense stretch in the GI
tract

=
Diabetic
gastroparesis
MTB S2CK

Nausea, vomiting, and


early satiety

p. 252

12

Diabetic Gastroparesis
The patients most common complaint is abdominal
discomfort with eating large or small meals
Nausea &
vomiting

Bloating &
constipation

Upper and Lower


Symptoms

Anorexia

Early satiety

Neurologic Symptoms

Endoscopy - large
amount of retained
food

Succussion
splash

Signs elicited by exam

Colon
Gastrointestinal Bleeding
Diarrhea
Irritable Bowel Syndrome
Inflammatory Bowel Disease
Diverticular Disorders
Colon Cancer Screening

Gastrointestinal Bleeding
69-year-old woman comes to ED with multiple red/black stools. Past
medical history significant for aortic stenosis. Pulse 115/ minute.
BP 94/62 mmHg. Examination otherwise normal.
What is the next step in management?
a.
b.
c.
d.
e.
f.
g.
h.

Colonoscopy
Nasogastric tube placement
Upper endoscopy
Patient is unstable,
Bolus of normal saline
Stablized first, THEN
CBC
diagnose
Bolus of 5% dextrose in water
Consult gastroenterology
D5w doesnt stay intravascular
Check for orthostasis

MTB S2CK

p. 252 253

Esophagitis

Upper GI
bl di
bleeding

Gastritis

Varices

Duodenitis
Cancer

MTB S2CK

Gastrointestinal Bleeding

p. 253

Gastrointestinal Bleeding/Physical Findings


Severity of Blood Loss Based on Hemodynamics

Most
common:
Diverticulosis

Lower GI
bl di
bleeding

Polyps

Most
common:
Ulcer disease

Physical Finding

Hemorrhoids

Orthostasis
Pulse > 100/minute
Systolic BP < 100 mmHg

Percentage of blood loss


15 20%
30%
30%

Remember: orthostasis is defined as...


IBD

UGIB
Cancer

MTB S2CK

p. 253

>10-point rise in pulse when going from supine to


sitting or standing up or BP drop of 20 points or
more when sitting up
MTB S2CK

p. 253

13

Variceal Bleeding

Gastrointestinal Bleeding/Diagnostic Tests

For acute severe bleeding

Vomiting
blood +/
black stool

Cirrhosis

Spider
angiomata
and caput
medusa
MTB S2CK

Variceal
bleeding

Replace fluids and check Hct, platelet count, and


coagulation tests (PT or INR)
Asterixis

For variceal bleeding


Octreotide and urgent endoscopy to control the
bleed by banding

Palmar
erythema
Splenomegaly

p. 253 254

MTB S2CK

p. 254

Additional Diagnostic Tests for GI Bleeding

Gastrointestinal Bleeding/Treatment

Test

Indication

Stabilization Treatment

Nuclear bleeding scan

Endoscopy unrevealing in a massive


acute hemorrhage
Specific site of bleeding needs
to be identified prior to surgery or
embolization of vessel; used onlyy in
massive, nonresponsive bleeding
Small bowel bleeding when upper and
lower endoscopy dont show etiology
Not useful in GI bleeding
Shows ischemia in severe bleeding

Fluid replacement

Angiography

Capsule endoscopy
CT or MRI of abdomen
EKG

MTB S2CK

p. 254

1-2 liters an hour

Packed RBCs
Hct < 30 in those who are older or suffer from CAD

MTB S2CK

p. 255

Gastrointestinal Bleeding/Treatment

Gastrointestinal Bleeding/Treatment

Stabilization Treatment

Treat Underlying Cause


Endoscopy to determine diagnosis and
administer treatment (band varices,
cauterize ulcers, inject epinephrine into
bleeding gastric vessels)
IV PPI for upper GI bleeding
Surgery to remove site of bleeding if fluids,
blood, platelets, and plasma will not control
bleed

Fresh frozen plasma


INR > 1.3

Platelets
< 50,000 when bleeding

MTB S2CK

p. 255

MTB S2CK

p. 255

14

Gastrointestinal Bleeding/Treatment

Esophageal and Gastric Varices


Octreotide (somatostatin)
Banding
Transjugular intrahepatic portosystemic
shunting
h ti (TIPS)
Propranolol

Diarrhea/Types
Lactose
intolerance

Carcinoid
syndrome

Diarrhea

Antibioticassociated
diarrhea

MTB S2CK

Chronic
pancreatitis
Malabsorption

p. 255

Diarrhea/Antibiotic Associated

Diarrhea/Antibiotic Associated

Clindamycin associated with highest


incidence of antibiotic-associated diarrhea
and Clostridium difficile
Blood and white cells may be present in stool
Presents several days or weeks after start of
antibiotics
ANY antibiotic can potentially cause diarrhea

Best initial test is...


Stool C. diff. toxin

Most accurate test is...


C. diff. PCR

Best initial therapy is...


Metronidazole

If theres no response to metronidazole, next step


in management is to...
Switch to oral vancomycin or fidaxomicin
IV vancomycin is always wrong for
antibiotic-associated diarrhea since it
will not pass bowel wall.

MTB S2CK

p. 255

MTB S2CK

p. 255 256

Diarrhea/Antibiotic Associated
75-year-old man is admitted to hospital with pneumonia.
Several days after start of antibiotics, he has diarrhea. Stool C.
diff toxin is positive, and hes started on metronidazole, which
leads to resolution of diarrhea over a few days. Two weeks
later diarrhea recurs and C. diff toxin is positive again.
What is the next step?
a.
b.
c.
d.
e.

Retreat with metronidazole orally


Use vancomycin orally Repeat metronidazole before switching
Sigmoidoscopy and treat only if pseudomembranes are found
Not necessary
Intravenous metronidazole Only if patient
cannot
take
orally
Wait for stool culture
C. diff. doesnt grow in culture
f. Intravenous vancomycin
Repeat metronidazole before switching /
IV vancomycin does not work
MTB S2CK

Diarrhea after
antibiotic Use
Yes
C. difficile
positive?
No
Consider
alternative causes

Yes
T t with
Treat
ith
metronidazole
No
Switch to oral
vancomycin or
fidaxomicin

Improvement?
Yes
Continue
metronidazole
until end of
course

p. 256

15

Diarrhea/Malabsorption

Diarrhea/Malabsorption

All present with steatorrhea:


Oily
Greasy
Stools
Floating
Foul smelling

All forms of fat malabsorption present with


deficiency of fat-soluble vitamins (A, D, E, and K)
Hence, they can all present with...

Causes:
Celiac disease
Whipples disease
Chronic pancreatitis
MTB S2CK

Deficiency

Manifestation

Vitamin D
Vitamin K
Vitamin B12

Hypocalcemia
Bleeding, easy bruising
Anemia, hypersegmented
neutrophils, neuropathy
Vitamin B12 needs an intact bowel wall
and pancreatic enzymes to be absorbed

p. 256

MTB S2CK

p. 256 257

Diarrhea/Diagnostic Tests

Diarrhea/Malabsorption

Celiac disease
Best initial test is...

Whipples Disease
Arthralgias
Ocular findings
Neurologic abnormalities (dementia, seizures)
Fever
Lymphadenopathy
L
h d
th

Anti-tissue transglutaminase

Most accurate diagnostic test is...


Small bowel biopsy
Flattening of villi
Can exclude lymphoma

Source: commons.wikimedia.org

Other tests
IgA antigliadin antibody
Antiendomysial antibody

MTB S2CK

p. 257

MTB S2CK

p. 257

Diarrhea/Malabsorption

Chronic Pancreatitis/Diagnostic Tests

Whipples disease and


tropical sprue
Most accurate
diagnostic test is...

Abdominal X-ray
50% to 60% sensitive
for calcification of
pancreas and very
specific when test is
abnormal

Bowel wall biopsy


showing specific
organism
Treat with ceftriaxone
or TMP/SMZ

MTB S2CK

p. 257

Source: commons.wikimedia.org

MTB S2CK

p. 257 258

16

Chronic Pancreatitis/Diagnostic Tests

Chronic Pancreatitis/Diagnostic Tests

Abdominal CT scan
80% to 90% sensitive
for pancreatic
calcification

Secretin stimulation testing


Most accurate diagnostic test
Place NG tube; normal pancreas releases large
volume of bicarbonate-rich fluids after IV secretin

D-xylose testing
Old test to distinguish pancreatitis from bowel wall
abnormalities
D-xylose normal in pancreatic disorders

MTB S2CK

p. 257 258

MTB S2CK

Malabsorption/Treatment

p. 257 258

Carcinoid Syndrome

Disease

Specific Treatment

Chronic pancreatitis

Enzyme replacement

Celiac disease

Avoid gluten containing


foods (wheat, oats, rye,
barley)

Whipples disease

Ceftriaxone, TMP/SMX

Tropical sprue

TMP/SMX, tetracycline

Presentation
Flushing
Wheezing
CV murmurs tricuspid regurgitation
Diarrhea
Best initial diagnostic test is...
Urinary 5-hydroxyindoleacetic acid (5-HIAA) test
Treatment
Octreotide

MTB S2CK

p. 258

MTB S2CK

p. 258 259

Lactose Intolerance

Irritable Bowel Syndrome (IBS)

Presents like malabsorptive diarrhea, except...

Pain syndrome with either diarrhea, constipation, or


both
Pain of IBS is...

Intermittently
No weight loss
Normal vitamin levels

Best initial diagnostic test is...


Stool osmolality test

Most accurate test is...


Cessation of symptoms after diet change

Best treatment is...


Stop eating milk products and consider exogenous
lactase therapy
MTB S2CK

p. 259

Relieved by BM
Less at night
Relieved by a change in bowel habit (e.g., diarrhea)

No specific diagnostic test


Diagnosis of exclusion in association with complex of
symptoms

Not associated with...


Weight loss
Blood or white cells in stool
MTB S2CK

p. 259

17

Irritable Bowel Syndrome/Treatment

Inflammatory Bowel Disease

Fiber in diet
Antispasmodic agents

Crohns disease (CD) and ulcerative colitis


(UC) present with...
Diarrhea and abdominal pain
Blood in stool
anemia
Weight
g loss
Fever

Hyoscyamine
Dicyclomine

Tricyclic
y
antidepressants
p
Amitriptyline

Antimotility agents
Loperamide for diarrhea
Lubiprostone (chloride-channel activator)
Increases BM frequency
MTB S2CK

p. 259

MTB S2CK

Inflammatory Bowel Disease

p. 259

Inflammatory Bowel Disease

Extraintestinal manifestations in both CD and UC


Arthralgias
Uveitis, iritis
Skin manifestation (erythema nodosum, pyoderma
gangrenosum)
Sclerosing cholangitis (more frequent in UC)

Erythema Nodosum

Risk of cancer associated with CD and UC


Both forms of IBD can lead to colon cancer
Cancer risk with duration
CD involving colon has same cancer risk as UC
MTB S2CK

Pyoderma Gangrenosum

Source: commons.wikimedia.org

Source: commons.wikimedia.org

p. 259 260

Inflammatory Bowel Disease/Differences

Inflammatory Bowel Disease/Diagnostic Tests

Crohns disease

Ulcerative colitis

Most accurate test is...

Skip lesions
Transmural granulomas
Fistulas and granulomas
Masses and obstruction
Perianal disease

Curable by surgery
Entirely mucosal
No fistulas, no abscesses
No obstruction
No perianal disease

Endoscopy

Frequent question: When should screening occur?


Source: commons.wikimedia.org

Answer: After 8 to 10 years of colonic involvement,


with colonoscopy every 1 to 2 years.
MTB S2CK

p. 260

Source: commons.wikimedia.org

CD thats mainly in small bowel, radiologic tests


such as barium studies will detect lesions
MTB S2CK

p. 260

18

Inflammatory Bowel Disease/Diagnostic Tests

ANCA and ASCA Results in IBD


Test

Crohns disease Ulcerative colitis

Antineutrophil cytoplasmic
antibody (ANCA)

Negative

Positive

Antisaccharomyces
cerevesiae antibody (ASCA)

Positive

Negative

Inflammatory Bowel Disease/Treatment

Acute exacerbations of either CD or UC are


treated with steroids
Chronic maintenance of remission is with 5ASA derivatives (mesalamine)
Asacol for UC
Pentasa for CD
Perianal CD
Ciprofloxacin and metronidazole

MTB S2CK

p. 260

MTB S2CK

p. 260 261

Inflammatory Bowel Disease/Treatment

Diverticulosis

Azathioprine and 6-mercaptopurine are used to


wean patients off steroids
Fistulae and severe unresponsive disease
Anti-tumor necrosis factor (TNF) agents (infliximab)
Surgery only if theres no response to anti-TNF
agents
NOTE
Neither form of IBD is routinely treated with surgery
UC can be cured, however, with colectomy
In CD, surgery is used exclusively for bowel
obstruction as it recurs at surgical site

Outpocketings of colon
Where arteries meet mucosa
Vegetarians
At risk?
Asymptomatic
Infection?

MTB S2CK

p. 261

MTB S2CK

p. 261

Diverticulosis

Diverticulitis

Most accurate test is...


Colonoscopy
Barium studies
Fiber Fiber Fiber

Diagnosis
LLQ pain and tenderness
Fever
Leukocytosis
Palpable mass sometimes occurs
Symptoms such as nausea, constipation,
and bleeding can be present, but are
nonspecific

MTB S2CK

p. 261

Source: commons.wikimedia.org

MTB S2CK

p. 261

19

Diverticulitis

Diverticulitis

Best initial test is...


CT scan of the abdomen
98% accuracy
Colonoscopy and barium enema are
dangerous
g
in acute diverticulitis
Increased risk of perforation because
infection weakens colonic wall

Source: commons.wikimedia.org

MTB S2CK

p. 261

Diverticulitis/Treatment
Treatment
Antibiotics that cover E. coli and anaerobes that
are present in bowel such as...

Ciprofloxacin combined with metronidazole


Amoxicillin/clavulanate
Ticarcillin/clavulanate
Piperacillin/tazobactam

Surgery is for those with...


No response to medical therapy
Frequent recurrences of infection
Perforation, fistula formation, abscess, strictures, or
obstruction
MTB S2CK

261 262

Which of the following is the most effective method of


screening for colon cancer?
a. Colonoscopy
b. Sigmoidoscopy
All less sensitive
c. Fecal occult blood testing (FOBT) than colonoscopy
d Barium enema
d.
e. Virtual colonoscopy with CT scanning Low sensitivity;
misses small
f. Capsule endoscopy
For small bowel bleeding

MTB S2CK

polyps

p. 262

Colon Cancer/Frequency of Screening

Colon Cancer/Frequency of Screening

Routine testing
Patients should have a colonoscopy every 10
years beginning at age 50

Hereditary nonpolyposis colon cancer


syndrome (HNPCC) comprises...
3 family members
2 generations
1p
premature ((< 50))

Single
g family
y member with colon cancer?
Begin 10 years earlier than the age at which
the family member developed their cancer or
age 40, whichever is younger
Screen every 10 years if relative > 60 or
every 5 years if relative < 60
MTB S2CK

p. 262

Start screening at age 25 with colonoscopy


every 1 to 2 years

MTB S2CK

p. 262

20

Colon Cancer/Frequency of Screening

Colon Cancer/Frequency of Screening

Familial adenomatous polyposis (FAP)


Presence of thousands of polyps with
abnormal genetic test known as
adenomatous polyposis coli (APC) test
Start screening with sigmoidoscopy at age
12 every year

Previous adenomatous polyp


Patient should have colonoscopy...
Every 3 to 5 years

Previous history of colon cancer


Patient should have colonoscopy...
1 year after resection
Then at 3 years
Then every 5 years

MTB S2CK

p. 263

MTB S2CK

p. 263

Other Polyposis Syndromes


Peutz-Jeghers syndrome
Multiple hamartomatous polyps, melanotic spots on
lips and skin, frequency of breast cancer, gonadal
and pancreatic cancer

Pancreas & Liver

Gardner syndrome
Colon cancer associated with: osteomas, desmoid
tumors, and other soft tissue tumors

Turcot syndrome
Colon cancer in association with CNS malignancy

Juvenile polyposis
Colon cancer in association with multiple
hamartomatous polyps
MTB S2CK

p. 263

Acute Pancreatitis
Drug allergy

Acute Pancreatitis/Presentation
Most
common
1. GB stones
2. ETOH

Ductal
obstruction

Pancreatitis

Trauma

Hypertriglyceridemia
Hypercalcemia

MTB S2CK

p. 263 264

Drug toxicity

Scorpion
sting

Acute epigastric pain + tenderness + nausea/vomiting


=
Pancreatitis
Pain intensity is subjective and doesnt correlate
with degree of organ damage
In severe cases theres hypotension and fever
The pain of pancreatitis goes straight through to
the back like a spear stabbed into the abdomen.
Cholecystitis pain goes around the side to the
back.

Infection

MTB S2CK

p. 264

21

Acute Pancreatitis/Diagnostic Tests


Which of the following is associated with the worst
prognosis in pancreatitis?
a. Elevated amylase
Levels do not correlate
with severity
b. Elevated lipase
c. Intensity of the pain Pain doesnt predict
d Low calcium
d.
e. C-reactive protein (CRP) rising
Elevated with all inflammation

MTB S2CK

p. 264

Acute Pancreatitis/Diagnostic Tests

Best initial tests are...


Amylase and lipase
Most specific diagnostic test is
CT scan
Labs
CBC: leukocytosis, drop in Hct over time with
rehydration
Elevated LDH and AST
Hypoxia
Hypocalcemia
Elevated urinary trypsinogen activation peptide
MTB S2CK

p. 264 265

Acute Pancreatitis/Diagnostic Tests

Imaging
CT or MRI scan are best
Also detect pseudocysts

ERCP
Can help determine etiology (stones, stricture, tumor)

Plain X
X-ray
ray
Sentinel loop of bowel (air-filled piece of small bowel
in LUQ)
Limited utility

Ultrasound has very poor accuracy


Overlying bowel blocks precise imaging
MTB S2CK

Source: Niket Sonpal, MD

NOTE: Abdominal CT scan is always performed with IV and


oral contrast to better define and outline abdominal structures.

p. 265

Acute Pancreatitis/Treatment

Acute Pancreatitis/Treatment

> 30% necrosis on CT or MRI, add antibiotics


(e.g., imipenem)
Infected, necrotic pancreatitis should be
resected with surgical debridement to
prevent ARDS and death
Pseudocysts are drained with a needle if
they are enlarging or painful

NPO (no food)


IV hydration
Analgesia
PPIs pancreatic stimulation from acid
entering
g duodenum

MTB S2CK

p. 265

MTB S2CK

p. 265

22

Acute Pancreatitis/Complication

Liver Disease
Spider
angiomata and
palmar
erythema

Hepatorenal
syndrome

Hepatopulmonary
syndrome

Portal hypertension
leading to varices

Chronic
Li
Liver
Disease

Thrombocytopenia

Coagulopathy

Asterixis and
encephalopathy
h l
th

Hypoalbuminemia &
edema and ascites

Source: commons.wikimedia.org

MTB S2CK

Ascites

p. 265

Ascites

Paracentesis is performed with...


New-onset ascites
Abdominal pain and tenderness
Fever
Low albumin
L
lb i iin th
the ascitic
iti fluid?
fl id?
Portal HTN from cirrhosis is the etiology

SAAG: Correlating Level with Specific Diseases


< 1.1 g/dL

> 1.1 g/dL

Infections
Cancer
Nephrotic syndrome

Portal HTN
CHF
Hepatic vein thrombosis
Constrictive pericarditis

Serum ascites albumin gradient (SAAG)


The difference or gradient of albumin between
serum and ascitic fluid
MTB S2CK

p. 266

MTB S2CK

p. 266

Spontaneous Bacterial Peritonitis

Spontaneous Bacterial Peritonitis

Infection without perforation of bowel

Best initial test is...


Cell count with > 250 neutrophils

Causative organisms

E. coli (most common)


Other gram-negative bacilli
Pneumococcus
Anaerobes (rare)

Most accurate test is...


Fluid culture, but takes too long for results
Gram
G
stain
t i iis negative/LDH
ti /LDH nonspecific
ifi
Treatment: cefotaxime or ceftriaxone
NOTE: SBP frequently recurs. When the ascites
fluid albumin level is quite low, use prophylactic
norfloxacin or TMP/SMX.

MTB S2CK

p. 266

MTB S2CK

p. 266

23

Treatment of Specific Features of Cirrhosis

Alcoholic Liver Disease

Feature

Treatment

Ascites and edema

Spironolactone and other diuretics. Serial


pericentesis for large volume ascites.
FFP and/or platelets only if bleeding occurs

Diagnosis of exclusion
No specific therapy
Most accurate test is...

Coagulopathy and
thrombocytopenia
Encephalopathy
Hypoalbuminemia
lb
Spider angiomata and palmar
erythema
Varices
Hepatorenal syndrome
Hepatopulmonary syndrome

Liver biopsy

Lactulose, neomycin, or rifaximin


f therapy
h
No specific
No specific therapy

Source: commons.wikimedia.org

Alcohol and drugs causing liver disease give a

Propranolol and banding via endoscopy


Somatostatin (octreotide), midodrine
No specific therapy

greater elevation in AST compared to ALT.


Viral hepatitis: Higher ALT than AST.
Binge drinking: Sudden rise in GGTP.

MTB S2CK

p. 267

MTB S2CK

p. 267

Primary Biliary Cirrhosis (PBC)

Primary Biliary Cirrhosis (PBC)

PBC is most likely diagnosis with...


Woman in 40s or 50s
Fatigue and itching
Normal bilirubin
Elevated alkaline phosphatase

Most unique features of PBC are...


Xanthelasma/xanthoma
Osteoporosis
Most accurate test is...
Liver biopsy
Most accurate blood test:
Antimitochondrial antibody
Treatment with...
Ursodeoxycholic acid

MTB S2CK

p. 267

MTB S2CK

Primary Biliary Cirrhosis (PBC)

p. 267 268

Primary Sclerosing Cholangitis


Pruritus
Elevated alkaline
phosphatase
GGTP
Elevated bilirubin level

Treatment
Cholestyramine
Ursodeoxycholic acid

Most accurate test is...


ERCP, not liver biopsy
Shows beading, narrowing,
or strictures in biliary
system
Source: commons.wikimedia.org

MTB S2CK

p. 268

Source: commons.wikimedia.org

24

Primary Sclerosing Cholangitis

Alpha 1 Antitrypsin Deficiency


Look for...
Combination of...

PSC doesnt improve or resolve with


resolution of IBD.
Even after a colectomy in UC, patient may
still progress to needing a liver
transplantation.

Liver disease
Emphysema (COPD)

Young patient (< 40)


Nonsmoker
Treatment
Replace enzyme exogenously

MTB S2CK

p. 268

MTB S2CK

Hemochromatosis

p. 268

Hemochromatosis

Genetic disorder leading to overabsorption of


iron in duodenum

Amenorrhea
Skin
Darkening

Mutation: C282y gene


Erectile
dysfunction

Presentation
Patient in their 50s with mild increases in AST
and alkaline phosphatase

MTB S2CK

p. 268

Best initial test is...


Iron studies
Increased serum iron
and ferritin
Decreased iron binding
capacity

Most accurate test is...


Liver biopsy
Increased iron

MTB S2CK

p. 269

Cardiomegaly
Fatigue and joint
pain (pseudogout)

MTB S2CK

Hemochromatosis

Hemochromatosis

Diabetes

p. 268 269

Chronic Hepatitis B and C


EKG
Conduction defects
Echocardiogram
Dilated or restrictive
cardiomyopathy

Both are associated with...


Cirrhosis
Liver cancer
PAN

Best therapy
Phlebotomy

MTB S2CK

p. 269

25

Chronic Hepatitis B and C


Chronic hepatitis B
Surface antigen
positive > 6 months
Hepatitis B DNA PCR
is the best way to
determine viral
replication activity
Liver biopsy for fibrosis
Biopsy tracks progress

MTB S2CK

Chronic Hepatitis B and C/Treatment


Chronic hepatitis C
80% have chronic
infection
Never symptomatic
when illness contracted
Hepatitis C DNA PCR
Determines disease
activity

p. 270

Chronic hepatitis B
Adefovir
Lamivudine
Telbivudine
Entecavir
Tenofovir
T
f i
Interferon

MTB S2CK

Chronic hepatitis C
Combination of...
Interferon +
Ribavirin +
Telapavir
( Boceprevir)
(or
B
i)

p. 270

Hepatitis Treatment Side Effects

Wilson Disease/Presentation

Drug

Adverse Effects

Interferon

Arthralgias, thrombocytopenia,
depression, leukopenia

Ribavarin

Anemia

Adefovir

Renal dysfunction

Lamivudine

None

Bocepevir

Anemia

Decrease in ceruloplasmin causes buildup of


copper
Neurological symptoms
Psychosis, tremor, dysarthria, ataxia, or
seizures
Coombs negative hemolytic anemia
Renal tubular acidosis or nephrolithiasis

Telaprevir

Rash

MTB S2CK

p. 270

Wilson Disease/Diagnosis

MTB S2CK

p. 270 271

Wilson Disease

Best initial test is...


Slit-lamp examination for Kayser-Fleischer
rings
Most accurate diagnostic
g
test is...
Abnormally increased amount of copper
excretion into urine after giving penicillamine

MTB S2CK

p. 271

MTB S2CK

p. 271

26

Wilson Disease/Treatment

Autoimmune Hepatitis

Penicillamine chelates copper and removes it


Additional therapies are...
Zinc: interferes with intestinal copper absorption
Trientine: alternate copper-chelating compound

Look for...
Young women
Signs of liver
inflammation
Positive ANA

Decreased ceruloplasmin level is not the most


accurate test. This is the most common wrong
answer. All plasma proteins can be decreased in those
with liver dysfunction and cirrhosis.

More specific tests:


Liver-kidney
microsomal antibodies
Anti-smooth muscle
antibodies

MTB S2CK

p. 271

MTB S2CK

Autoimmune Hepatitis

Most accurate test:


Liver biopsy
Treatment
Prednisone and/or
azathioprine

p. 271

Nonalcoholic Steatohepatitis
Nonalcoholic Fatty Liver Disease
Extremely common cause of mildly abnormal liver
function tests

Source: commons.wikimedia.org

Disorder is associated with:


Obesity
Diabetes
Hyperlipidemia
Corticosteroid use
Most accurate test: biopsy
Management: weight loss
MTB S2CK

p. 272

Nonalcoholic Steatohepatitis

Source: commons.wikimedia.org

27

Anemia
Presentation
Diagnostic Tests
y p
Symptoms
Mean Corpuscular Volume
Microcytic Anemia
Macrocytic Anemia
Normocytic Anemia
Treatment

Hematology
Dr. Conrad Fischer, MD
Associate Professor of Medicine
Touro College of Medicine
New York City

Anemia

Hematocrit and Symptoms

Presentation
All forms of anemia present with identical
symptoms if they have the same hematocrit (Hct)
Symptoms based on severity, not etiology
What is the most likely diagnosis?

Hematocrit

Expected Symptoms

> 30 35%

None

25 30%

Dyspnea (worse on
exertion),
), fatigue
g

20 25%

Lightheadedness, angina

< 20 25%

Syncope, chest pain

Cannot be answered using symptoms alone

Diagnostic Tests
Best initial test to evaluate anemia?
Always a complete blood count (CBC)

MTB S2CK

p. 203

Anemia/Diagnostic Tests

Ultimately, cardiac ischemia from


anemia proves fatal. Myocytes cant
distinguish between:
Anemia
Hypoxia
CAD
Carbon monoxide poisoning

MTB S2CK

p. 203

Mean Corpuscular Volume

Mean corpuscular volume (MCV)


First clue to knowing etiology
Mean Corpuscular
Volume
(Normal 80100 fL)

Smaller?

Larger?

Microcytosis

Macrocytosis

All of these conditions result in decreased


oxygen delivery to tissues
MTB S2CK

p. 203 204

MTB S2CK

p. 204

Microcytosis

Microcytosis

Causes of low MCV:


Iron deficiency
Thalassemia
Sideroblastic anemia
Anemia of chronic disease

Similarities among microcytic anemias


Low reticulocyte count
Only alpha thalassemia has 3 genes deleted
elevated reticulocyte count
Microcytic anemias are due to production
problems

MTB S2CK

p. 204

Microcytosis

Routine blood smear


Not effective in telling the difference between
types of microcytosis
All hypochromic
All potentially give target cells

MTB S2CK

p. 204

Nearly synonymous with

MTB S2CK

reticulocyte counts

p. 204

Macrocytic Anemia/Etiology
Causes of high MCV
B12 & folate deficiency
Alcoholism
Sideroblastic anemia
Liver disease or hypothyroidism
Medications
M di ti
(
(e.g.,
zidovudine
id
di or phenytoin)
h
t i )
Antimetabolite medications: azathioprine, 6mercaptopurine, hydroxyurea
Myelodysplastic syndrome (MDS)

MTB S2CK

p. 204

Normocytic Anemia

Anemia/Treatment

Acute blood loss & hemolysis


rapid drop in
Hct (no time for MCV change)
Blood loss leads to iron deficiency & microcytosis
Hemolysis increases reticulocyte count

Treatment
If severe: give packed RBCs
Answering the question At what Hct do I
transfuse a patient? depends on the following
factors:

Reticulocytes raise the MCV


Reticulocytes slightly larger than normal cells

MTB S2CK

p. 204

1. Symptomatic? Transfuse.
2. Hct very low in elderly person? Heart disease?
Transfuse.

MTB S2CK

p. 205

Anemia/Treatment

Blood Products

Symptomatic from anemia means:


SOB
Lightheaded, confused, and sometimes
syncope
Hypotension and tachycardia
Chest pain

Remember:
No transfusion if young & asymptomatic

MTB S2CK

p. 205

Packed Red Blood Cells (PRBCs)


Concentrated form of blood

Whole blood - 150 mL plasma = PRBCs


Hct of PRBCs: 70% - 80%
Removal of plasma doubles Hct
Each unit PRBCs raises Hct by 3 points/unit

MTB S2CK

p. 205

Blood Products

Blood Products

Fresh Frozen Plasma (FFP)


Replaces clotting factors for elevated prothrombin
time, aPTT, or INR
Most important if actively bleeding
FFP used as replacement with plasmapheresis

Cryoprecipitate
Used to replace fibrinogen
Some utility in disseminated intravascular
coagulation (DIC)
Provides high amounts of clotting factors in small
plasma volume

Whole blood is never correct


Whole blood is divided into either PRBCs or FFP
MTB S2CK

p. 205

MTB S2CK

p. 205

Microcytic Anemia

Microcytosis = MCV < 80 fL

Microcytic Anemia
Etiology
Presentation
Diagnostic Tests
Treatment

Microcytic, hypochromic anemia seen on blood smear.


MTB S2CK

p. 206

Copyright James Van Rhee.


Used with permission.

Microcytic Anemia/Etiology

Microcytic Anemia/Etiology

Iron deficiency
Blood loss
One teaspoon (5 mL/day) blood loss leads to iron
deficiency over time
Body only needs very tiny amount of iron

Chronic disease
Initially MCV is normal, then decreases
Unclear etiology
Any cancer or chronic infection
Clear mechanism only in renal failure deficiency of
erythropoietin
Hemoglobin synthesis will not occur because iron
does not move forward
Iron is locked in storage or trapped in macrophages
or in ferritin

1 to 2 mg/day

Menstruating women need a little more


2-3 mg/day

Pregnant women need 5-6 mg/day


Duodenum absorbs only about 4 mg/day
MTB S2CK

p. 206

MTB S2CK

p. 206

Microcytic Anemia/Etiology

Microcytic Anemia/Etiology

Sideroblastic anemia
Can be macrocytic when associated with
myelodysplasia (MDS)
Most common cause: Alcohol effect on marrow
Less common causes...

Thalassemia
Extremely common cause of microcytosis
Most with thalassemia trait are asymptomatic

Lead poisoning
Isoniazid
Vitamin B6 deficiency

MTB S2CK

p. 206

MTB S2CK

p. 206

Microcytic Anemia/Presentation

Microcytic Anemia/Presentation

You cant distinguish these anemias


based on symptoms
Might have a suggestion from history

How to answer What is the most likely


diagnosis? for anemia
Feature in the History
Most likely diagnosis?
Blood loss (GI bleeding)
Menstruation
Cancer or chronic infection
Rheumatoid arthritis
Alcoholic
Asymptomatic

MTB S2CK

p. 206

MTB S2CK

Iron deficiency
Iron deficiency
Chronic disease
Chronic disease
Sideroblastic
Thalassemia

p. 206

Microcytic Anemia/Diagnostic Tests

Microcytic Anemia/Diagnostic Tests

Peripheral smear not useful!


All hypochromic
All can be associated with target cells

Target cells: most common with


thalassemia

MTB S2CK

p. 207

MTB S2CK

p. 207

Microcytic Anemia/Iron Studies

Microcytic Anemia/Iron Studies

Unique features and diagnoses of iron studies

Iron deficiency

Unique Feature
Low ferritin
High iron
Normal iron studies

MTB S2CK

Diagnosis
Iron deficiency
Sideroblastic anemia
Thalassemia

p. 207

Copyright James Van Rhee.


Used with permission.

Low ferritin = Iron deficiency


However, one third of patients with iron deficiency
will have normal or increased ferritin
Ferritin is an acute phase reactant
This means any infection or inflammation can
raise the ferritin level

MTB S2CK

p. 207

Microcytic Anemia/Iron Studies

Microcytic Anemia/Iron Studies

Both iron deficiency and anemia of chronic disease


are associated with low serum iron
Iron deficiency: increase in total iron binding
capacity (TIBC)
TIBC measures unbound sites on transferrin
Open sites on transferrin = increased capacity
capacity or
number of unbound sites

Chronic disease
Serum iron: low in circulation
Iron trapped in storage
Ferritin (stored iron): elevated or normal
Circulating iron: decreased
Major
M j diff
difference iis TIBC is
i low
l

MTB S2CK

p. 207

MTB S2CK

p. 207

Microcytic Anemia/Iron Studies

Unique Laboratory Features

Sideroblastic anemia
Only microcytic anemia elevated circulating iron
level

Iron deficiency
Red cell distribution of width (RDW) increased

Thalassemia
Genetic disease with normal iron studies

Newer cells are more iron deficient


Newer cells are smaller in iron deficiency
As body runs out of iron, newer cells have less Hb &
get p
g
progressively
g
y smaller

Elevated platelet count common


Single most accurate test is...
Bone marrow biopsy for stainable iron (decreased)
Rarely done, but most accurate

MTB S2CK

p. 207

MTB S2CK

p. 207

Unique Laboratory Features

Unique Laboratory Features

Sideroblastic anemia
The most accurate test is...
Prussian blue staining
for ringed sideroblasts
Basophilic stippling can
occur in any cause of
sideroblastic anemia

Thalassemia
Most accurate test is...

MTB S2CK

p. 207 208

Hb electrophoresis!

MTB S2CK

p. 208

Source: commons.wikimedia.org

Unique Laboratory Features

Electrophoresis Findings

Thalassemia
Most accurate test for alpha thalassemia

Alpha thalassemia

Beta thalassemia

One gene deleted: normal


Two genes deleted: mild anemia,
normal electrophoresis
Three genes deleted: moderate
anemia with Hb H (beta 4
tetrads), increased reticulocytes
Four genes deleted: gamma 4
tetrads or hemoglobin Bart; CHF
causes death in utero

Inc. Hb F and A2
N/A

Genetic studies
3-gene deletion alpha thalassemia
Hb H (beta-4 tetrads)
Increased reticulocyte count

All thalassemia types have normal RDW

MTB S2CK

p. 208

MTB S2CK

Beta thalassemia intermedia


Normal Hb F
No transfusion dependence
N/A

p. 208

Microcytic Anemia/Treatment

Microcytic Anemia/Treatment

Iron deficiency
Replace iron with oral ferrous sulfate
If insufficient, patients get IM iron
Chronic disease
Correct underlying disease
Only
O l end-stage
d t
renall ffailure
il
responds
d tto
erythropoietin
Sideroblastic anemia
Correct the cause
Some respond to vitamin B6 (pyridoxine)

Thalassemia
Trait not treated

MTB S2CK

p. 208

Beta thalassemia major (Cooley anemia)


Chronic transfusion lifelong
Iron
I
overload
l d managed
d with
ith d
deferasirox
f
i
((orall iiron
chelator)
Deferoxamine is parenteral

MTB S2CK

Macrocytic Anemia
Etiology
Presentation
Diagnostic Tests
Treatment

p. 209

73-year-old man in office with fatigue progressively worse over


several months. He is short of breath when he walks up one
flight of stairs. He drinks 4 vodka martinis a day. He complains
of numbness and tingling in his feet.
Physical: decreased sensation in feet.
Hct: 28%
MCV: 114 fL (elevated)
What is the next step? 1st is peripheral smear
Once hypersegmented neutrophils are seen,
a. Vitamin B12 level
THEN get B12 & folate levels
b. Folate level
c. Peripheral blood smear
d. Schilling test To see if B12 deficiency is due to pernicious anemia
e. Methylmalonic acid level
Confirms diagnosis of B12 deficiency
when B12 levels are equivocal

MTB S2CK

p. 209

Macrocytic Anemia

Macrocytic Anemia/Etiology

Megaloblastic
Hypersegmented neutrophils

Vitamin B12 deficiency is caused by...


Pernicious anemia
Pancreatic insufficiency
Dietary deficiency (vegan/strict vegetarian)
Crohns disease damaging terminal ileum
Blind loop syndrome (gastrectomy or gastric
bypass for weight loss)
Diphyllobothrium latum

Many factors raise MCV


Only B12 & folate
deficiency &
antimetabolite
medications cause
hypersegmentation

Source: Alireza Eghtedar, MD

MTB S2CK

p. 209 210

MTB S2CK

p. 209

Macrocytic Anemia/Etiology

Macrocytic Anemia/Presentation

Folate deficiency
Dietary deficiency (goats milk has no folate
and limited iron/B12)
Psoriasis & skin loss or turnover
Drugs:
g p
phenytoin,
y
, sulfa

Alcohol
Gives macrocytosis & neurological problems
Will not give hypersegmented neutrophils

MTB S2CK

p. 209

MTB S2CK

p. 209

Macrocytic Anemia/Presentation

Macrocytic Anemia/Diagnostic Tests

B12 deficiency
Can give any neurological abnormality
Peripheral neuropathy most common
Dementia least common
Posterior column damage to position & vibratory
sensation or subacute
subacute combined degeneration
degeneration of
cord is classic
Look for ataxia

Labs common to both B12 & folate deficiency


Megaloblastic anemia
Increased LDH & indirect bilirubin levels
Decreased reticulocyte count
Hypercellular bone marrow
Macroovalocytes
M
l
t
Increased homocysteine levels
Only B12 deficiency is associated with increased
methylmalonic acid level

MTB S2CK

p. 210

MTB S2CK

p. 210

Macrocytic Anemia/Diagnostic Tests


B12 & folate deficiency
Identical hematologically on blood smear

73-year-old woman with decreased position & vibratory


sensation of lower extremities. Hct: 28%, MCV: 114 fL,
and hypersegmented neutrophils.
B12 level decreased, but near the borderline of normal.
What is the next step in management?
a.
b.
c.
d.
e.
f.

Methylmalonic acid level


Confirms cause as pernicious
Anti-intrinsic factor antibodies anemia after B12 deficiency
confirmed
Anti-parietal cell antibodies
Schillings test Rare test of etiology; premature
Folate level Folate doesnt give neurological deficits
Homocysteine level Doesnt add anything

Source: Alireza Eghtedar, MD

MTB S2CK

p. 210

MTB S2CK

p. 210 211

Macrocytic Anemia/Diagnostic Tests

Macrocytic Anemia/Diagnostic Tests

Tested facts about macrocytic anemia


Schilling test is never the right answer
Pernicious anemia is confirmed with antiintrinsic factor and anti-parietal cell
antibodies
Red cells are destroyed as they leave the
marrow, so reticulocyte count is low

Tested facts about macrocytic anemia


B12 & folate deficiency can cause
pancytopenia as well as macrocytic anemia
Pancreatic enzymes are needed to absorb B12

MTB S2CK

p. 211

They free it from carrier (R) protein

Neurological abnormalities will improve as long as


they are minor (e.g., peripheral) and short duration

MTB S2CK

p. 211

Macrocytic Anemia/Treatment

Replace what is deficient


Folate replacement corrects hematologic
problems of B12 deficiency, but not the
neurological problems

Which of the following is a complication of B12 or


folate replacement?
a. Seizures Not associated with treatment
b. Hemolysis Cells are produced rapidly, not hemolyzed
c Hypokalemia
c.
High K+ from massive tissue or
d. Hyperkalemia
cellular breakdown
e. Diarrhea
Cause hypokalemia, but not associated
with B12 or folate replacement therapy

MTB S2CK

p. 211

MTB S2CK

p. 211

Hemolytic Anemia

Hemolytic Anemia Part 1


Sickle Cell Disease
Hereditary Spherocytosis
Autoimmune Hemolysis
y

All forms lead to:


Sudden in Hct
Increased LDH, indirect bilirubin & reticulocytes
Decreased haptoglobin
Slight increase in MCV
Reticulocytes
R ti l
t > normall cells
ll

Hyperkalemia

MTB S2CK

p. 211 212

Sickle Cell Disease

Sickle Cell Disease/Presentation

Chronic, well-compensated hemolytic anemia


Reticulocyte count always high
Acute painful vaso-occlusive crisis is caused by...
Hypoxia
Dehydration
I f ti
Infection
Cold temperatures

Look for...
African American
Sudden, severe pain in chest, back & thighs
May have fever

MTB S2CK

p. 212

Rare for adult to present with acute crisis


without clear history of sickle cell disease

MTB S2CK

p. 212

Sickle Cell Disease/Manifestations

Sickle Cell Disease/Diagnostic Tests

Bilirubin gallstones
Increased infection
from autosplenectomy

Best initial test...


- Peripheral smear

Encapsulated
organisms

Osteomyelitis
y
Most commonly from
Salmonella

Stroke
Enlarged heart with
hyperdynamic features
and systolic murmur
Skin ulcers
Avascular necrosis of
femoral head

Retinopathy

MTB S2CK

p. 212

Most accurate test..


- Hemoglobin
electrophoresis
Trait (AS disease) does
not give sickled cells

MTB S2CK

p. 212 213

Sickle Cell Disease/Treatment


Which is found on smear in sickle cell disease?
a.
b.
c.
d
d.
e.

Basophilic stippling Associated with lead poisoning


Howell-Jolly bodies
Bite cells Seen with G6PD deficiency
Fragmented red cells seen with
S hi t
Schistocytes
t
intravascular hemolysis
Morulae

1. Oxygen/hydration/analgesia
2. Fever or white cell count higher than usual?
Antibiotics given! Ceftriaxone, levofloxacin, or
moxifloxacin
3. Folic acid in everyone
4. Pneumococcal vaccine: autosplenectomy
5. Hydroxyurea: prevent recurrences, increases Hb F

Seen inside neutrophils in Ehrlichia infections

Dont wait for results of testing!


Start antibiotics with fever.
No spleen = Overwhelming infection = Death!!
MTB S2CK

p. 213

MTB S2CK

p. 213

10

Sickle Cell Disease/Treatment

Exchange transfusion for severe vasoocclusive crisis with:


Acute chest syndrome
Priapism
Stroke
Visual disturbance from retinal
infarction

43-year-old man with sickle cell disease admitted with


acute pain crisis. Only med is folic acid. Hct on
admission: 34%. On 3rd hospital day, hematocrit
drops to 22%.
What is the initial test?
y count Not best way to gauge severity of
a. Reticulocyte
b. Peripheral smear hemolysis
Causes aplastic crisis which freezes growth of
c. Folate level marrow
d. Parvovirus B-19 IgM level
Most accurate test for parvovirus B-19 is
e. Bone marrow
PCR for DNA. IVIG is best initial therapy.
Marrow shows giant pronormoblasts, but this isnt better than
reticulocyte count at measuring severity of hemolysis

MTB S2CK

p. 214

MTB S2CK

p. 214

Sickle Cell Trait

Hereditary Spherocytosis

Patient is heterozygous for sickle gene (AS)


Only manifestation is inability to
concentrate urine (isosthenuria)
Clinically asymptomatic
Normal CBC
Normal smear
Hematuria sometimes occurs
No treatment for sickle cell trait

Genetic defect in
cytoskeleton of RBCs
Leads to abnormal
round shape
Loss of normal flexibility
characteristic of
biconcave disc that
allows red cells to bend
in spleen

MTB S2CK

p. 214

MTB S2CK

p. 214 215

Hereditary Spherocytosis/Presentation

Hereditary Spherocytosis/Diagnosis

Look for...
Recurrent episodes of hemolysis in a
young child or newborn
Intermittent jaundice
Splenomegaly
S l
l
Family history of anemia or hemolysis
Bilirubin gallstones

Low MCV
Increased mean corpuscular
hemoglobin concentration (MCHC)
Negative Coombs test

MTB S2CK

p. 214 215

Most acc
accurate
rate test:
test Osmotic fragility
fragilit
Cells are placed in slightly hypotonic solution
Increased swelling leads to hemolysis

MTB S2CK

p. 215

11

Hereditary Spherocytosis/Treatment

Autoimmune Hemolysis

1. Chronic folic acid replacement supports red


cell production
2. Splenectomy stops hemolysis but doesnt
eliminate spherocytes

Also known as warm or IgG hemolysis


Idiopathic in 50%
Clear causes are...

MTB S2CK

p. 215

Chronic lymphocytic leukemia (CLL)


Lymphoma
Systemic lupus erythematosus (SLE)
Drugs: penicillin, alpha-methyldopa, rifampin,
phenytoin

MTB S2CK

p. 215

Autoimmune Hemolysis/Diagnostic Tests

Autoimmune Hemolysis/Diagnostic Tests

Most accurate diagnostic test is...


Coombs test

Autoantibodies remove small amounts of red cell


membrane and lead to smaller membrane
Forces cell to become round
Biconcave discs have greater surface than sphere

Detects IgG on surface of red cells


Direct & indirect tests tell basically the same
thing
Indirect associated with greater amount of
antibody

Smear doesnt show fragmented cells because


destruction occurs inside spleen or liver, not in
blood vessel
MTB S2CK

p. 215 216

Autoimmune Hemolysis/Treatment

MTB S2CK

Autoimmune Hemolysis/Treatment

1. Glucocorticoids (prednisone) best initial


therapy
2. Recurrent episodes: splenectomy
3. Severe, acute hemolysis not responding to
prednisone can be controlled with intravenous
immunoglobulin
g
((IVIG))
4. Rituximab when splenectomy doesnt control
hemolysis

MTB S2CK

p. 216

p. 215 216

Alternate treatments to
diminish need for steroids:
Cyclophosphamide
Cyclosporine
Azathioprine
Mycophenolate mofetil

MTB S2CK

p. 216

12

Cold Agglutinin Disease

Hemolytic Anemia Part 2


Cold Agglutinin Disease
G6PD Deficiency
HUS and TTP
PNH

IgM antibodies against red cells in association with


Epstein-Barr virus, Waldenstrm macroglobulinemia, or
Mycoplasma pneumoniae
Presentation
Symptoms in colder parts of body
Numbness or mottling of nose, ears, fingers, and toes
Symptoms resolve on warming body part

Diagnosis
Direct Coombs test positive only for complement
Smear normal in most
May show spherocytes
MTB S2CK

Cold Agglutinin Disease

p. 216

Cold Agglutinin Disease/Treatment

Treatment
1. Stay warm
2. Administer rituximab
3. Cyclophosphamide, cyclosporine, or other
immunosuppressive agents stop production of
antibody
4. Plasmapheresis in some

Cryoglobulins often mixed up with cold agglutinins.


Both are IgM and dont respond to steroids.
Cryoglobulins are associated with:
Hepatitis C
Joint pain
Glomerulonephritis

Steroids and splenectomy


usually dont work
MTB S2CK

p. 216 217

MTB S2CK

p. 217

G6PD Deficiency

G6PD Deficiency/Presentation

X-linked recessive
Inability to generate glutathione reductase and
protect red cells from oxidant stress
Most common oxidant stress is infection
Other causes: dapsone, quinidine, sulfa drugs,
primaquine nitrofurantoin
primaquine,
nitrofurantoin, and fava beans

Look for...

African American or Mediterranean males


Sudden anemia & jaundice
Normal-sized spleen
With infection
i f ti or drug
d

G6PD deficiency is X-linked recessive,


it manifests almost exclusively in males

MTB S2CK

p. 217

MTB S2CK

p. 217

13

G6PD Deficiency/Diagnosis

G6PD Deficiency

Best initial test is...


Peripheral smear

Treatment
Avoid oxidant stress
Nothing reverses hemolysis

Look for Heinz bodies


and bite cells

Most accurate test...


G6PD level
1 to 2 months after
acute episode of
hemolysis

MTB S2CK

p. 217

MTB S2CK

p. 218

HUS and TTP

HUS and TTP

Hemolytic uremic syndrome (HUS) and thrombotic


thrombocytopenic purpura (TTP) are different
versions of the same basic disease

Both characterized by:

HUS
Associated with E. coli
0157:H7
More frequent in children

TTP
Associated with
ticlopidine clopidogrel
ticlopidine,
clopidogrel,
cyclosporine, and AIDS
Neurological disorders
(confusion and seizures)
Fever
More common in adults

Intravascular
hemolysis with
fragmented red cells
(schistocytes)
Thrombocytopenia
Renal insufficiency

Source: commons.wikimedia.org

MTB S2CK

p. 218

MTB S2CK

p. 218

HUS and TTP

Paroxysmal Nocturnal Hemoglobinuria

Diagnosis
No one specific test diagnoses either disorder
Normal PT/aPTT
Negative Coombs test

Clonal stem cell defect with increased sensitivity to


complement in acidosis

Treatment
Severe cases treated with plasmapheresis or plasma
exchange

MTB S2CK

p. 218

Etiology
Deficiency of complement regulatory proteins CD 55 and
59, also known as decay accelerating factor (DAF)
Gene for phosphatidylinositol glycan class A (PIG
(PIG-A)
A) is
defective
overactivation of complement system
During sleep, relative hypoventilation leads to pCO2
and acidosis
This does nothing to an unaffected person
In PNH it leads to hemolysis and thrombosis
MTB S2CK

p. 218

14

Paroxysmal Nocturnal Hemoglobinuria

Paroxysmal Nocturnal Hemoglobinuria

Presentation
Episodic dark urine with first urination of day from
hemoglobin
Pancytopenia and iron deficiency anemia

Pancytopenia
Most accurate test is...
Decreased CD55 & CD59

Ham test and sucrose hemolysis are never


the correct answer
Flow cytometry is another way of saying
CD55/CD59 testing

Thrombosis MCC of death


MTB S2CK

p. 218

MTB S2CK

p. 218 219

Paroxysmal Nocturnal Hemoglobinuria

1. Prednisone
2. Allogeneic bone marrow transplant is the
only method of cure
3. Eculizumab inactivates C5 in complement
pathway and decreases red cell destruction;
its a complement inhibitor
4. Folic acid and iron replacement with
transfusions as needed

MTB S2CK

Hematologic Malignancies
Aplastic Anemia
Polycythemia Vera
Essential Thrombocytosis
Myelofibrosis

p. 219

Aplastic Anemia

Aplastic Anemia

Pancytopenia of unclear etiology


Any infection or cancer can invade marrow causing
decreased production or hypoplasia
Other causes:

Fatigue of anemia
Infections from low white cell counts
Bleeding from thrombocytopenia

Radiation and toxins (e.g., toluene, insecticides (DDT),


and benzene)
Drug effect: sulfa, phenytoin, carbamazepine,
chloramphenicol, alcohol, chemotherapy
SLE
PNH
Infection: HIV, hepatitis, CMV, EBV
B12 and folate deficiency
Thyroid-inhibiting medications (e.g., propylthiouracil
(PTU) and methimazole)
MTB S2CK

p. 219

Most accurate test is...


is
Bone marrow biopsy
Aplastic anemia is confirmed by excluding all
causes of pancytopenia

MTB S2CK

p. 219

15

Aplastic Anemia/Treatment

Aplastic Anemia/Treatment

Treat any underlying cause thats identified


THEN...
Blood transfusion, antibiotics for infection, and
platelets for bleeding

True aplastic anemia treated with allogeneic bone


marrow transplantation (BMT) if young enough
and matched

Aplastic anemia acts as autoimmune disorder


T cells attack patients own marrow
Cyclosporine inhibits T cells
This brings the marrow back to life!

MTB S2CK

p. 219

When too old for BMT (> 50) or no matched donor


y
y g
globulin ((ATG)) and
treatment is anti-thymocyte
cyclosporine
Tacrolimus is alternative to cyclosporine

MTB S2CK

p. 219 220

Polycythemia Vera

Polycythemia Vera

Unregulated overproduction of all 3 cell lines


Red cell overproduction is most prominent
Mutation in JAK2 protein which regulates marrow
production
Red cells grow wildly despite low erythropoietin

Symptoms of hyperviscosity from increased red cell


mass such as:
Headache, blurred vision, and tinnitus
Hypertension
Fatigue
Splenomegaly
Bleeding from engorged blood vessels
Thrombosis from hyperviscosity

MTB S2CK

p. 220

MTB S2CK

Polycythemia Vera
Hematocrit markedly
elevated > 60%
Platelets and WBC count
Erythropoietin
Total red cell mass

MTB S2CK

p. 220

p. 220

Polycythemia Vera
You must exclude hypoxia
as cause of erythrocytosis
Oxygen levels: normal
Vitamin B12 levels are
elevated for unclear
reasons
it s
Iron levels because its
been used to make red cells

Most accurate test...


JAK2 mutation (95%)

Increased basophils
All myeloproliferative disorders

Small number converted to AML

MTB S2CK

p. 220

16

Essential Thrombocytosis (ET)

Essential Thrombocytosis (ET)

Markedly elevated platelet count > 1 million


Leads to both thrombosis & bleeding
Very difficult to distinguish from an elevated platelet
count as a reaction to another stress (e.g.,
infection, cancer, or iron deficiency)

< 60 & asymptomatic


with platelets < 1.5
million

MTB S2CK

p. 221

Best initial therapy...


Hydroxyurea

Anagrelide with red cell


No treatment necessary suppression from
hydroxyurea
> 60 & thromoboses or
platelet count > 1.5
Aspirin for
million
erythromelalgia (painful,
Begin treatment
red hands in ET)

MTB S2CK

p. 221

Myelofibrosis
Older persons with pancytopenia
Bone marrow shows marked fibrosis
Blood production shifts to spleen & liver, which become
markedly enlarged
Look for teardrop-shaped cells and nucleated red
blood cells on smear
Thalidomide and lenalidomide: tumor necrosis factor
inhibitors that increase bone marrow production
Occasional patient < 50-55, allogeneic bone marrow
transplantation is attempted

MTB S2CK

Leukemia
Acute Leukemia
Chronic Myelogenous Leukemia
Leukostasis Reaction
Myelodysplastic Syndrome
Chronic Lymphocytic Leukemia
Hairy Cell Leukemia

p. 221

Acute Leukemia/Presentation

Acute Leukemia/M3

Most frequently tested acute leukemia is M3,


known as acute promyelocytic leukemia (APL)
M3 is associated with disseminated intravascular
coagulation (DIC)

History of myelodysplasia suggests acute leukemia


Signs of pancytopenia (fatigue, infection, bleeding)
Even if WBC is normal or increased
Infection is common presentation
Leukemic cells (blasts) dont function normally

M3

DIC

No distinct clinical presentation bet


between
een the 3
subtypes of acute lymphocytic leukemia (ALL)

MTB S2CK

p. 221

MTB S2CK

p. 221

17

Acute Leukemia/Diagnosis

Acute Leukemia/Diagnosis

Best initial test is...


Blood smear

Auer rods
Eosinophilic inclusions
Associated with acute
promyelocytic leukemia
(M3)

Shows blasts

Most accurate test is...


Flow cytometry
Distinguishes different subtypes of acute leukemia
Detects specific CD subtypes associated with each
type of leukemia

Myeloperoxidase
Characteristic of acute myelocytic leukemia (AML)

MTB S2CK

p. 221

MTB S2CK

p. 221 222

Acute Leukemia/Treatment

Acute Leukemia/Treatment

Both AML & ALL treated initially with chemotherapy


to remove blasts from peripheral blood smear
Known as inducing remission
The question is: proceed to BMT after remission or
give more chemotherapy
Prognosis poor? go straight to BMT
Prognosis good? more chemotherapy

The best indicator of prognosis in acute leukemia is...

MTB S2CK

p. 222

Acute Leukemia/Treatment

1. Add all-trans-retinoic acid (ATRA) to those


with M3 (acute promyelocytic leukemia)
1. Add intrathecal chemotherapy such as
methotrexate to ALL treatment; prevents
relapse of ALL in CNS

MTB S2CK

p. 222

Cytogenetics
Assesses specific chromosomal characteristics
found in each patient
Good cytogenetics = less chance of relapse =
more chemotherapy
Bad cytogenetics = more chance of relapse =
immediate BMT

MTB S2CK

p. 222

Acute Leukemia/Treatment

Most tested facts for acute leukemia are...


M3 (promyelocytic leukemia) gives DIC
Add ATRA to M3
Auer rods = AML
Add intrathecal methotrexate to ALL

MTB S2CK

p. 222

18

Chronic Myelogenous Leukemia/Presentation

Chronic Myelogenous Leukemia/Diagnosis

Look for...
High WBC count: all neutrophils
Vague symptoms of fatigue, night sweats, and
fever from hypermetabolic syndrome
Splenomegaly: early satiety, abdominal fullness,
and LUQ pain
Pruritus is common after hot baths/showers

Determine if its a reaction to another


infection, stress (leukemoid reaction), or
leukemia
CML may give small numbers of blasts
Should be < 5%
Basophils increased

Histamine release from basophils

MTB S2CK

p. 223

MTB S2CK

p. 223

Chronic Myelogenous Leukemia/Diagnosis

Chronic Myelogenous Leukemia/Treatment

If leukocyte alkaline phosphatase score (LAP) is


a choice, then this is first

Imatinib (Gleevec),
dasatinib, or nilotinib are
the best initial therapy
Only BMT cures CML

Leukemic cells dont have normal amounts of


alkaline phosphatase; LAP score low in CML

If LAP score is not a choice, or the question is


What is the most accurate test? then answer
BCR-ABL

Never the first therapy

Can be done by PCR on peripheral blood

MTB S2CK

p. 223

MTB S2CK

p. 223

Myelodysplastic Syndrome (MDS)


54-year-old man with SOB, blurry vision, confusion, and
priapism. WBC count 225,000/L. Predominantly
neutrophils with about 4% blasts.
What is next step in management of this case?
a.
b.
c.
d.
e.
f.
g.

Leukapheresis
In acute leukostasis reaction,
BCR-ABL
BCR
ABL testing
i
important
t t to
t remove excessive
i
Bone marrow biopsy
white cells from the blood than
Bone marrow transplant
to establish a specific
Consult hematology/oncology diagnosis, no matter what the
etiology
Flow cytometry
Hydroxyurea
Will lower the cell count, but not as
rapidly as leukapheresis

MTB S2CK

p. 223 224

Preleukemic disorder in those > 60


Pancytopenia despite hypercellular bone
marrow
Most never develop AML
Infection & bleeding lead to death before leukemia
occurs

MTB S2CK

p. 224

19

Myelodysplastic Syndrome

Myelodysplastic Syndrome/Diagnosis

Many present with asymptomatic pancytopenia


on routine CBC
Symptoms can be...
Fatigue & weight loss
Infection
Bleeding
Sometimes splenomegaly
No single pathognomonic finding in history or
physical examination

CBC: anemia with increased


MCV, nucleated red cells,
and small number of blasts
Marrow: hypercellular
Prussian blue stain shows
ringed sideroblasts
Severity based on
percentage of blasts

MTB S2CK

p. 224

MTB S2CK

p. 224 225

Myelodysplastic Syndrome/Treatment

Chronic Lymphocytic Leukemia

CLL is a clonal proliferation of normal, mature-appearing B


lymphocytes that function abnormally
Occurs over age 50 in 90% affected
Most asymptomatic at presentation
Most common symptom is fatigue
Other symptoms:

Transfuse blood products as needed


Erythropoietin: about 20% response
Lenalidomide for those with 5q deletion
Decreases transfusion dependence

Lymphadenopathy (80%)
Spleen or liver enlargement (50%)
Infection

No unique physical findings


Richter phenomenon:
Conversion of CLL into
high-grade lymphoma
Occurs in 5%
MTB S2CK

p. 225

MTB S2CK

p. 225

Chronic Lymphocytic Leukemia/Diagnosis

Chronic Lymphocytic Leukemia/Treatment

WBC usually > 20,000/ L


80% to 98% lymphocytes
Half are
hypogammaglobulinemic
Anemia &
thrombocytopenia occur
from marrow infiltration or
autoimmune warm IgG
antibodies

Stage 0 (elevated WBC) & stage I


(lymphadenopathy); no treatment
Stage II (hepatosplenomegaly), stage III (anemia),
and stage IV (thrombocytopenia) treat with
fludarabine
If choices list fludarabine and rituximab,, this is the
best initial therapy for advanced-stage disease (II,
III, IV) or any patient who is symptomatic (severe
fatigue, painful nodes)
PCP prophylaxis is indicated in CLL

MTB S2CK

p. 225

Source: commons.wikimedia.org

Smudge cell:
Lab artifact
Nucleus crushed by
cover slip

MTB S2CK

p. 226

20

Chronic Lymphocytic Leukemia/Treatment

Hairy Cell Leukemia

Refractory cases: cyclophosphamide


(more efficacy, but more toxic)

Middle-aged men with:

Mild cases: chlorambucil

Severe infection: IVIG

Pancytopenia
Massive splenomegaly
Monocytopenia
Inaspirable dry tap, despite
hypercellularity of marrow

The most accurate test is...


Autoimmune thrombocytopenia
or hemolysis: prednisone

Tartrate resistant acid


phosphatase (TRAP) or
CD11c

Source: commons.wikimedia.org

Treat with cladribine

MTB S2CK

p. 226

MTB S2CK

p. 226

Non Hodgkin Lymphoma

Lymphoma
Non Hodgkin Lymphoma
Hodgkin Disease
Multiple Myeloma
MGUS
Waldenstrm Macroglobulinemia

Proliferation of lymphocytes in nodes and spleen


NHL most often widespread at presentation
NHL and CLL are extremely similar
NHL is solid mass and CLL is liquid or circulating

MTB S2CK

p. 226

Non Hodgkin Lymphoma

Non Hodgkin Lymphoma

Painless lymphadenopathy
May involve pelvic, retroperitoneal, or mesenteric
structures
Nodes not warm, red, or tender
B symptoms:

Best initial test is...

Fever,
Fever weight loss
loss, drenching night sweats

Advanced stages in 80% to 90% of cases

MTB S2CK

p. 227

Excisional biopsy

CBC normal in most cases


High LDH correlates with worse severity
Staging determines intensity of therapy
Typical staging procedures are:
CT scan of chest, abdomen, and pelvis
Bone marrow biopsy

MTB S2CK

p. 227

21

Non Hodgkin Lymphoma

Non Hodgkin Lymphoma/Treatment

Staging
Stage I: 1 lymph node group
Stage II: 2+ lymph node groups on same
side of diaphragm
Stage III: both sides of diaphragm
Stage IV: widespread disease

Local disease (Stage Ia and IIa): local radiation


Advanced disease (Stage III and IV, any B
symptoms): combination chemotherapy with
CHOP and rituximab (antibody against CD20)
C = cyclophosphamide
H = adriamycin (doxorubicin or hydroxydaunorubicin)
O = vincristine (oncovin)
P = prednisone
Mucosal Associated Lymphoid Tissue

Most common wrong answer is needle


aspiration of node. Aspiration not enough because
individual lymphocytes appear normal.

MTB S2CK

p. 227

MTB S2CK

Hodgkin Disease

Normal
lymphocyte
Reed Sternberg
cell

p. 228

p. 227

Differences between HD and NHL

Presentation, diagnostic tests, B symptoms, and


staging of Hodgkin disease (HD) are same as NHL
Hodgkin disease has Reed-Sternberg cells on
pathology

MTB S2CK

Lymphoma of stomach is associated with


Helicobacter pylori
Treat with clarithromycin and amoxicillin

Source: commons.wikimedia.org

Hodgkin Disease

Non Hodgkin Lymphoma

Local, Stage I, and Stage II in


80 90%

Stage III and Stage IV in 80 90%

Centers around cervical area

Disseminated

Reed Sternbergg cells on


pathology

No Reed Sternbergg cells

Pathologic classification:
Lymphocyte predominant has
best prognosis
Lymphocyte depleted has
worst prognosis

Pathologic classification:
Burkitt and immunoblastic
have worst prognosis

MTB S2CK

p. 228

Hodgkin Disease/Treatment

Hodgkin Disease/Treatment

Stage Ia and IIa: local radiation and Chemo


Stage III and IV or anyone with B symptoms:
ABVD
A = adriamycin (doxorubicin)
B = bleomycin
Relapses after radiation
V = vinblastine
therapy are treated with
D = dacarbazine
chemotherapy. Relapses
after chemotherapy are
treated with extra high dose
chemotherapy and bone
marrow transplantation.

Complications of Radiation and Chemotherapy


Radiation increases the risk of solid tumors such
as breast cancer or lung cancer
Screening for breast cancer earlier after treatment
Radiation increases premature coronary
disease
The risk of acute leukemia, MDS, and NHL from
chemotherapy is about 1% per year

MTB S2CK

p. 228

MTB S2CK

p. 228

22

Which of the following is most useful to determine


dosing of chemotherapy in HD?
The other choices arent as
a. Echocardiogram
accurate in determining left
b. Bone marrow biopsy
ventricular function
c. Gender
d MUGA or nuclear ventriculogram
d.
e. Hematocrit
f. Symptoms

MTB S2CK

p. 228 229

Hodgkin Disease/
Adverse Effects of Chemotherapy

Chemotherapeutic Agent

Toxicity

Doxorubicin

Cardiomyopathy

Vincristine

Neuropathy

Bleomycin

Lung fibrosis

Cyclophosphamide

Hemorrhagic cystitis

Cisplatin

Renal and ototoxicity

MTB S2CK

p. 229

Multiple Myeloma

Multiple Myeloma

Most common presentation is...

Abnormal proliferation of plasma cells


Unregulated production of useless immunoglobulin
Usually IgG or IgA
Immunoglobulins dont fight infection, but clog up
the kidney
IgM is a separate disease called Waldenstrm
macroglobulinemia

MTB S2CK

p. 229

Bone pain from pathologic fractures

Pathologic fracture: bone breaks under normal use


Due to osteoclast activating factor (OAF), which
attacks the bone causing lytic lesions
OAF
hypercalcemia
Infection common because abnormal plasma cells
dont make immunoglobulins effective against
infection

MTB S2CK

p. 229

Multiple Myeloma

Multiple Myeloma

Hyperuricemia: increased turnover of nuclear


material of plasma cells
Anemia: from infiltration of marrow with plasma
cells
Renal failure: from immunoglobulins and BenceJones p
protein in kidney;
y; hypercalcemia
yp
and
hyperuricemia also damage the kidney
Renal failure and infection are MCCs of death

First test done is X-ray of affected bone shows


lytic lesions
Serum protein electrophoresis (SPEP) shows an
IgG (60%) or IgA (25%) spike of a single type or
clone.
One clone = Monoclonal or M spike
p
Fifteen percent have light chains or Bence-Jones
protein only

MTB S2CK

p. 229

MTB S2CK

p. 229 230

23

Multiple Myeloma
Additional abnormalities
Hypercalcemia
Bence-Jones protein on urine immunoelectrophoresis
Beta-2 microglobulin levels correspond to severity of
disease
Smear with rouleaux
Elevated BUN and creatinine
Bone marrow biopsy: >10% plasma cells defines
myeloma
Elevated total protein with normal albumin

MTB S2CK

p. 230

Multiple Myeloma

Rouleaux
IgG paraprotein sticks to
red cells causing them
to adhere to each other
in a stack or roll

MTB S2CK

p. 230

Source: commons.wikimedia.org

Multiple Myeloma

M-spike on SPEP doesnt mean IgM


Myeloma has a decreased anion gap.
IgG is cationic.
Increased cationic substances will
increase chloride and bicarbonate levels.
This decreases the anion gap.

69-year-old woman admitted with severe back pain that


suddenly worsened--pop felt when she coughs and
tenderness over ribs. X-ray: lytic lesions. Calcium is 2
points above normal, Hct 27%, creatinine elevated.
UA: trace protein, but 24-hour urine show 5 grams of
protein.
What do you expect on technetium bone scan?
a. Normal
b. Lytic lesions at site of fractures
c. Increased uptake diffusely
d. Decreased uptake Nuclear bone scan shows increased
uptake with osteoblastic activity,
which is absent in myeloma.

MTB S2CK

p. 230

Why the difference between the protein on urinalysis


and 24-hour urine?
a.
b.
c.
d.
e.

False positive 24-hour urine is common in myeloma


Calcium in urine creates false negative urinalysis
Uric acid creates a false positive 24-hour
24 hour urine
Bence-Jones protein isnt detected by dipstick.
IgG in urine inactivates urine dipstick

MTB S2CK

p. 230

What is the single most accurate test for myeloma?


a. Skull X-rays Will show lytic lesions, but not as specific
b. Bone marrow biopsy Nothing besides myeloma is
associated with > 10% plasma
c. 24-hour urine
Of those with an M-spike
cells on bone marrow biopsy
of immunoglobulins, 99%
d. SPEP
dont have myeloma.
e. Urine
U i immunoelectrophoresis
i
l t h
i
Most IgG spikes are
(Bence-Jones protein)
from monoclonal
gammopathy of
unknown significance
that doesnt progress or
need treatment.

Bence-Jones protein is detected by urine


immunoelectrophoresis. Urine dipstick
detects only albumin.
MTB S2CK

p. 230

MTB S2CK

p. 230 231

24

Multiple Myeloma/Treatment
Best initial therapy is...
Combination of steroids with lenalidomide,
bortezomib, or melphalan

Most effective therapy under age 70 is autologous


bone marrow transplant with stem cell support

Myeloma therapy is in a state of rapid


flux due to numerous advances

MTB S2CK

Monoclonal Gammopathy of
Unknown Significance
IgG or IgA spike on SPEP is common in older
patients
Evaluate with bone marrow biopsy to exclude
myeloma
Monoclonal gammopathy of unknown significance
(MGUS) has small numbers of plasma cells
No therapy for MGUS
1% a year transform into myeloma
The quantity of immunoglobulin in the spike is main
correlate of risk for myeloma:
More MGUS = More myeloma

p. 231

MTB S2CK

p. 231

Waldenstrm Macroglobulinemia

Waldenstrm Macroglobulinemia

Overproduction of IgM
Malignant B cells lead to hyperviscosity

Presents with...
Lethargy
Blurry
Bl
vision
i i and
d vertigo
ti
Engorged blood vessels in eye
Mucosal bleeding
Raynaud phenomenon

MTB S2CK

p. 231

Anemia common
IgM spike on SPEP results in hyperviscosity
No bone lesions
Plasmapheresis is best initial therapy
Removes IgM and viscosity
L
Long-term
t
treatment
t t
t with
ith chlorambucil
hl
b il or
fludarabine and prednisone
Control cells that make abnormal Igs
Decrease means of production

MTB S2CK

p. 231

Bleeding Disorders

Bleeding Disorders
ITP
Von Willebrand Disease
Hemophilia
Factor XI Deficiency
DIC
Thrombophilia
Heparin Induced Thrombocytopenia

First step in evaluation is determining if bleeding is


from platelets or clotting factors
Platelet Bleeding
Superficial
Epistaxis, gingival,
petechiae, purpura,
gums, vaginal bleeding

Factor Bleeding
Deep
Joints and muscles

Bleeding in brain or GI system can be from either


platelet or clotting factor deficiency
MTB S2CK

p. 232

25

Immune (Idiopathic) Thrombocytopenic Purpura (ITP)

Look for...
Isolated thrombocytopenia
Normal Hct
Normal
N
l WBC countt

Normal-sized spleen

MTB S2CK

23-year-old woman comes to ED with increased


menstrual bleeding, gum bleeding when she brushes
her teeth, and petechiae.
Platelet count: 17,000/L (low)
What is the next step in therapy?
a. Bone marrow biopsy
b. Intravenous immunoglobulins
c. Prednisone
d. Antiplatelet antibodies
e. Platelet transfusion

p. 232

MTB S2CK

Prednisone is more
important than checking for
increased megakaryocytes
or antiplatelet antibodies,
which is characteristic of
ITP.

p. 232

ITP

ITP/Treatment

Diagnosis of exclusion

Presentation

Management

No bleeding, count > 30,000

No treatment

Occasional tests are:

Mild bleeding, count < 30,000

Glucocorticoids

Antiplatelet antibodies lack specificity, limited


benefit
Ultrasound or CT scan to exclude
hypersplenism
Megakaryocytes are elevated in number

Severe bleeding (GI/CNS),


count < 10,000

IVIG, Anti Rho (anti D)

Recurrent episodes, steroid


dependent

Splenectomy
Romiplostim or eltrombopag for
recurrences after splenectomy

Splenectomy or steroids not


effective

Rituximab, azathioprine,
cyclosporine, mycophenolate

MTB S2CK

p. 232

ITP

p. 233

Von Willebrand Disease (VWD)

Before splenectomy, give vaccination to:


Neisseria meningitidis
Hemophilus influenzae
Pneumococcus

MTB S2CK

MTB S2CK

p. 233

Most common inherited bleeding disorder


Von Willebrand factor (VWF) level or function
Autosomal dominant
Look for bleeding related to platelets (epistaxis,
gingival, gums) with normal platelet count
Markedly worsened after aspirin
aPTT elevated in 50% of patients

MTB S2CK

p. 233

26

Von Willebrand Disease (VWD)

Hemophilia

Diagnosis
Bleeding time: increased duration of bleeding
VWF (antigen) level may be decreased
Ristocetin cofactor assay: detects VWF
dysfunction

Look for...
Delayed joint or muscle
bleeding in male child

Treatment
Initial therapy: DDAVP (desmopressin), which
releases subendothelial stores of VWF
If no response, use factor VIII replacement or VWF
concentrate
MTB S2CK

p. 233

Bleeding delayed
because primary
hemostatic plug is with
platelets

Prothrombin time (PT)


normal
aPTT prolonged

MTB S2CK

Most accurate test is...


Specific assay for
factor VIII or IX

Mixing studies with


normal plasma correct
aPTT to normal
Treat mild cases with
DDAVP
Severe bleeding treated
with replacement of
specific factor

p. 234

Factor XI Deficiency

Disseminated Intravascular Coagulation (DIC)

Most of time, no increase in bleeding with factor


XI deficiency
With trauma or surgery, theres increased bleeding

DIC doesnt occur in otherwise healthy people


Look for a definite risk such as...

Look for...
Normal PT with p
prolonged
g aPTT
Mixing study: corrects aPTT to normal
Treatment
Use FFP to stop bleeding

MTB S2CK

p. 234

Sepsis
Burns
Abruptio placenta or amniotic fluid embolus
Snake bites
Trauma resulting in tissue factor release
Cancer

Theres bleeding related to both clotting factor


deficiency as well as thrombocytopenia

MTB S2CK

p. 234

Disseminated Intravascular Coagulation (DIC)

Disseminated Intravascular Coagulation (DIC)

Diagnostic Tests
Elevated PT and aPTT
Low platelet count
Elevated D-dimer & fibrin split products
Decreased fibrinogen level (has been
consumed)

Treatment
Replace platelets < 50,000/ L as well
as clotting factors with FFP
Heparin has no benefit
Cryoprecipitate
C
i it t may be
b effective
ff ti tto
replace fibrinogen levels if FFP doesnt
control bleeding

MTB S2CK

p. 234

MTB S2CK

p. 235

27

Hypercoaguable States/Thrombophilia

Heparin Induced Thrombocytopenia

MCC:

HIT more common with unfractionated heparin

Factor V Leiden mutation

No difference in intensity of
anticoagulation
Warfarin to INR of 2
2-3
3 for 6 months

MTB S2CK

p. 235

Can still occur with LMW heparin

Presents 5 to 10 days after start of heparin with a


marked drop in platelet count (> 30%)
Both venous and arterial thromboses occur
Venous clots more common

HIT rarely leads to bleeding


Platelets just precipitate out
Diagnostic Tests
HIT confirmed with ELISA for platelet factor 4
(PF4) antibodies or serotonin release assay
MTB S2CK

p. 235

Heparin Induced Thrombocytopenia

Antiphospholipid Syndromes

Treatment
Immediately stop all heparin-containing products
Cant just switch unfractionated heparin to LMW
heparin
Direct thrombin inhibitors

Two main syndromes - lupus anticoagulant and


anticardiolipin antibody
Both cause thrombosis
Anticardiolipin antibodies associated with multiple
spontaneous abortions
Antiphospholipid (APL) syndromes specifically
cause thrombophilia with abnormal aPTT

Argatroban,
Argatroban lepirudin,
lepirudin and bivalirudin

After direct thrombin inhibitor is started, use warfarin

MTB S2CK

p. 235

MTB S2CK

p. 235

Antiphospholipid Syndromes
Best initial test is...
Mixing study

Because its a circulating inhibitor, aPTT remains


elevated after mix
Most accurate test for lupus anticoagulant is...
Russell viper venom test

Treatment
Treat with heparin and warfarin as you would for
any cause of DVT or PE
APL syndrome may require lifelong anticoagulation

MTB S2CK

p. 235

28

Introduction to Antibiotics
Principles of Answering Questions
Beta lactam Antibiotics
Fluoroquinolones
Aminoglycosides
Doxycycline
Trimethoprim/Sulfamethoxazole
Beta lactam/Beta lactamase Combinations
Specific Organism Groups and Their Treatments

Infectious Diseases
Conrad Fischer, MD
Associate Professor of Medicine
Touro College of Medicine
New York City

Introduction to Antibiotics

Introduction to Antibiotics

Organisms associated with diseases


dont change
But antibiotics that treat them change

Principles of Answering Infectious


Diseases Questions
The radiologic test is never the most
accurate test
Risk factors for an infection aren
arentt as
important as individual presentation
Beta-lactam antibiotics have greater
efficacy than other classes

M
Mostt iimportant
t t thing
thi
Antibiotics associated with each group
of organisms

MTB S2CK

p. 3

MTB S2CK

p. 3

Beta lactam Antibiotics

Penicillins

Penicillins, Cephalosporins, Carbapenems,


Aztreonam

Ampicillin and amoxicillin

Penicillins
Penicillin ((G,, VK,, benzathine))
Viridans group streptococci
Streptococcus pyogenes
Oral anaerobes
Syphilis
Leptospira
MTB S2CK

p. 3

Cover same organisms as penicillin


And
E. coli
Lyme disease
Gram-negative bacilli (few)

MTB S2CK

p. 3

Penicillins

Penicillins

Answer as Best Initial Therapy for


Gram-negative bacteria
covered by amoxicillin
H. influenzae
E. coli
Listeria
Proteus
Salmonella

MTB S2CK

p. 3

Penicillins

Oxacillin, Cloxacillin, Dicloxacillin, and


Nafcillin
Skin infections: Cellulitis
Endocarditis, meningitis, and bacteremia
from staphylococci
Osteo & Septic arthritis only when proven
sensitive
Not against Methicillin-Resistant
Staphylococcus aureus (MRSA) or
Enterococcus
MTB S2CK

p. 4

Otitis media
Dental infection and endocarditis
prophylaxis
Lyme disease limited to rash, joint, or 7th
CN involvement
UTI in pregnant women
Listeria monocytogenes
Enterococcal infections
MTB S2CK

p. 4

Penicillinase resistant penicillins (PRPs)

Methicillin is never right


Causes allergic interstitial nephritis
Methicillin sensitive or
resistant means oxacillin
sensitive or resistant.

MTB S2CK

p. 4

Penicillins

Piperacillin, Ticarcillin, Azlocillin, Mezlocillin

Piperacillin, Ticarcillin
Gram-negative bacilli (e.g., E. coli, Proteus)
enterobacteriaciae & pseudomonads
Best initial therapy

Also for: streptococci and anaerobes


BUT
NOT the answer, infection exclusively from
these single organisms

Cholecystitis & ascending cholangitis


Pyelonephritis
Bacteremia
Hospital-acquired and ventilator-associated
pneumonia
Neutropenia and fever
MTB S2CK

p. 4

Use narrower agent


Combined with a beta-lactamase inhibitor
such as tazobactam or clavulanic acid
MTB S2CK

p. 4

Cephalosporins

Cephalosporins

Cross-reaction penicillin & cephalosporins is

Very small (3% to 5%)


All cephalosporins, in every class, will cover group
A, B, and C streptococci, viridans group
streptococci, E. coli, Klebsiella, and Proteus
mirabilis

If the case describes a rash to penicillin


Answer cephalosporins
If the case describes anaphylaxis
You must use a non-beta-lactam
antibiotic

Listeria, MRSA, and Enterococcus


are resistant to all forms of
cephalosporins.
MTB S2CK

p. 4

First Generation

Cefazolin, Cephalexin, Cephradrine,


Cefadroxyl
1st-gen cephalosporins treat
Staphylococci
Methicillin sensitive = oxacillin sensitive =
cephalosporin sensitive

Streptococci (except Enterococcus)


Some gram-negative bacilli such as E. coli, but
not Pseudomonas
Osteomyelitis, septic arthritis, endocarditis,
cellulitis
MTB S2CK

p. 5

MTB S2CK

p. 5

Second Generation

Cefotetan, Cefoxitin, Cefaclor, Cefprozil,


Cefuroxime, Loracarbef
Cover same organisms as 1st-gen
cephalosporins
AND
Add coverage for anaerobes & more
gram-negative bacilli

MTB S2CK

p. 5

Second Generation

Third Generation

Cefotetan, Cefoxitin, Cefaclor, Cefprozil,


Cefuroxime, Loracarbef
Cefotetan or cefoxitin
Best initial therapy for pelvic inflammatory disease
(PID) combined with doxycycline
Warning
Cefotetan and cefoxitin risk of bleeding and give
a disulfiram-like reaction with alcohol
Cefuroxime, loracarbef, cefprozil, cefaclor
Respiratory infections (e.g., bronchitis, otitis media,
and sinusitis)

Ceftriaxone, Cefotaxime, Ceftazidime


Ceftriaxone
First-line for pneumococcus, including
partially insensitive organisms

MTB S2CK

p. 5

Meningitis
Community-acquired pneumonia (in
combination with macrolides)
Gonorrhea
Lyme involving heart or brain
Avoid ceftriaxone in neonates because of
impaired biliary metabolism
MTB S2CK

p. 5

Third Generation

Fourth Generation

Cefotaxime

Cefepime
Better staphylococcal coverage compared
with the 3rd-generation cephalosporins
Neutropenia and fever
Ventilator-associated pneumonia

Superior to ceftriaxone in neonates


Spontaneous bacterial peritonitis
C ft idi
Ceftazidime
h
has pseudomonal
d
l coverage

MTB S2CK

p. 5

MTB S2CK

Carbapenems

Ceftaroline: only cephalosporin to cover


MRSA

Imipenem, Meropenem, Ertapenem,


Doripenem
Cover gram-negative bacilli, including many
thatre resistant
Anaerobes
Streptococci and staphylococci
Neutropenia and fever

p. 5

Monobactams

Aztreonam
Only monobactam used

Exclusively
Gram-negative bacilli
Including
I l di Pseudomonas
P
d

No cross-reaction with penicillin

Ertapenem differs from other


carbapenems. Ertapenem does
not cover Pseudomonas
MTB S2CK

p. 6

Fluoroquinolones

Ciprofloxacin, Gemifloxacin, Levofloxacin,


Moxifloxacin

Community-acquired pneumonia,
including penicillin-resistant
pneumococcus
Gram-negative bacilli including most
pseudomonads
Cipro: NOT for pneumococcus

MTB S2CK

p. 6

MTB S2CK

p. 6

Fluoroquinolones

Ciprofloxacin: Cystitis and pyelonephritis


Moxifloxacin: Not for urine/cystitis
Diverticulitis and GI infections
Combined with metronidazole
Don
Dontt cover anaerobes except moxifloxacin
Moxifloxacin as single agent for
diverticulitis without metronidazole

MTB S2CK

p. 6

Fluoroquinolones

Fluoroquinolones

Quinolones cause
Bone growth abnormalities in children
and pregnant women
Tendonitis and achilles tendon rupture
Gatifloxacin removed because of
glucose abnormalities

Source: Grook Da Oger, commons.wikimedia.org

MTB S2CK

p. 6

Aminoglycosides

Gentamicin, Tobramycin, Amikacin


Gram-negative bacilli (bowel, urine,
bacteremia)
Synergistic with beta-lactam
beta lactam antibiotics
for enterococci and staphylococci
No effect against anaerobes (need
oxygen to work)
Nephrotoxic and ototoxic
MTB S2CK

p. 6

MTB S2CK

p. 6

Doxycycline

Chlamydia
Lyme
Rash
Joint
7th CN
palsy
MRSA skin

MTB S2CK

p. 7

Target-shaped rash of Lyme disease or erythema migrans. Source: Nishith Patel.

Doxycycline

Trimethoprim/Sulfamethoxazole

Rickettsia
Syphilis: Primary & secondary ONLY if
allergic to penicillin
Borrelia, Ehrlichia, and Mycoplasma
Ad
Adverse
effects
ff t
Tooth discoloration (children)
Fanconi syndrome (Type II RTA
proximal), photosensitivity
Esophagitis

Cystitis
Pneumocystis pneumonia treatment and
prophylaxis
MRSA of skin and soft tissue (cellulitis)
Rash
Hemolysis (G6PD deficiency)
Marrow suppression (folate antagonist)

MTB S2CK

p. 7

Nitrofurantoin has one indication:


Cystitis, especially in pregnant women.
MTB S2CK

p. 7

Beta lactam/Beta lactamase Combinations

Amoxicillin/clavulanate
Ticarcillin/clavulanate
Ampicillin/sulbactam
Piperacillin/tazobactam
Beta lactamase adds staphylococci effect
Beta-lactamase
Sensitive Staph only

Gram Positive Cocci: Staphylococci and


Streptococci

Best initial therapy:


Oxacillin, cloxacillin, dicloxacillin, nafcillin
First-generation cephalosporins: Cefazolin,
cephalexin
Fluoroquinolones
Macrolides (azithromycin, clarithromycin,
erythromycin) are third-line
Less efficacy than oxacillin or cephalosporins
Erythromycin more toxic

MTB S2CK

p. 7

MTB S2CK

p. 7

Oxacillin (Methicillin) Resistant Staph

Anaerobes

Oral (above diaphragm)


Penicillin (G, VK, ampicillin, amoxicillin)
Clindamycin
Metronidazole (GI)

Vancomycin
Linezolid: Reversible bone marrow toxicity
Daptomycin
Tigecycline
Ceftaroline

Piperacillin, carbapenems, and


2nd-generation cephalosporins
also cover anaerobes

Minor MRSA infections of skin are


treated with:
TMP/SMX
Clindamycin
Doxycycline
MTB S2CK

p. 7

Gram Negative Bacilli (E. coli, Klebsiella,


Proteus, Pseudomonas, Enterobacter, Citrobacter)

Bowel (peritonitis, diverticulitis)


Urinary tract (pyelonephritis)
Cholecystitis or Cholangitis
Quinolones
Aminoglycosides
Carbapenems
Piperacillin, ticarcillin
Aztreonam
Cephalosporins
MTB S2CK

p. 8

MTB S2CK

p. 8

Man admitted with E. coli bacteremia.


Which of the following is the most appropriate
therapy?
a. Vancomycin
b. Linezolid
c. Quinolones, aminoglycosides, carbapenems,
piperacillin, ticarcillin, or aztreonam
d. Doxycycline
They dont cover grame. Clindamycin
negative bacilli.
f. Oxacillin

MTB S2CK

p. 8

Central Nervous System (CNS) Infections

Central Nervous System Infections


Meningitis
g
Encephalitis

All CNS infections give


Fever
Headache
Vomiting
Seizures

MTB S2CK

Meningitis/Definition

Clues to Answering the


Most Likely Diagnosis Question

Symptom

Diagnosis

Stiff neck
Photophobia
Meningismus

Meningitis

Confusion

Encephalitis

Focal neurological
findings

Abscess

MTB S2CK

p. 9

Infection/inflammation of CNS covering

Source: SVG by Mysid, original by SEER


Development Team commons.wikimedia.org

p.

Meningitis/Etiology: 4 bugs 95% of cases

Meningitis/Presentation

Streptococcus pneumonia (60%)


Group B streptococci (14%)
Haemophilus influenzae (7%)
Neisseria meningitidis (15%)
Listeria (2%)

Fever
Headache
Neck stiffness
(nuchal rigidity)
Photophobia

Staphylococcus with recent


neurosurgery

Patient with Neck stiffness. Source: Sophian, Abraham: Epidemic


cerebrospinal meningitis (1913), St. Louis, C.V Mosby ,
commons.wikimedia.org

S. Pneumoniae Source: CDC/Dr. M.S. Mitchell

MTB S2CK

p. 9

MTB S2CK

p. 9

Meningitis/Presentation

What is the Most Likely Diagnosis?

Acute bacterial (presents in several hours)


Focal abnormalities: 30%
If confusion occurs, you wont be able to
answer What is the most likely diagnosis?
without a CT and lumbar puncture (LP)
Cryptococcal meningitis: Slow several weeks

AIDS with <100


CD4 cells/ L

p. 9

Camper/hiker
Rash
R h target-like
lik
shape
Joint pain
Facial palsy
Tick remembered
in 20%

MTB S2CK

Meningitis/Diagnostic Tests

Source: commons.wikimedia.org

p. 10

Camper/hiker
Rash moves
from arms/legs
t trunk
to
t
k
Tick remembered
in 60%

Rocky Mountain
spotted fever
(Rickettsia)

Neisseria

Viral

Pulmonary
TB in 85%

Tuberculosis

p. 9

Cerebrospinal Fluid Evaluation

Best initial test and Most accurate test


Lumbar Puncture

MTB S2CK

Adolescent,
petechial rash

None
Cryptococcus

Lyme disease
MTB S2CK

Stiff Neck
Photophobia

Cryptococcus,
Tuberculosis Viral
Lyme, Rickettsia
10s100s
10s100s
10s100s
1000s,
Lymphocytes Lymphocytes
Neutrophils Lymphocytes
Bacterial

Cell
count

Protein Elevated
level

Possibly
elevated

Markedly
elevated

Usually
normal

Glucose Decreased
level

Possibly
decreased

May be low

Usually
normal

Stain
Stain:
and
5070%;
Culture Culture:
90%

Negative

Negative

Negative

MTB S2CK

p. 10

Meningitis/Diagnostic Tests

Meningitis/Diagnostic Tests

When Is Head CT the Best Initial Test?


If before the LP, there is

Neuro exam needs patient who


understands and follows instructions
and answers questions

Papilledema
Seizures
Focal
neurological
abnormalities
Confusion

Blurred
disc
margin

You cannot do an
accurate neurologic
exam if the patient is
severely confused.

Papilledema is a blurred, fuzzy disc margin from increased intracranial pressure. Source:
Conrad Fischer, MD

MTB S2CK

p. 10

MTB S2CK

p. 10

Meningitis/Diagnostic Tests

Bacterial Antigen Detection


(Latex Agglutination Tests)

If theres a contraindication to immediate LP,


giving antibiotics is best initial step in
management

Better
B
tt to
t treat
t t and
d
decrease the accuracy of a
test than to risk permanent
brain damage.
MTB S2CK

p. 10

Lyme and
Rickettsia

Acid fast stain and culture


on 3 high-volume lumbar
punctures
Centrifuge to concentrate
the organisms
TB has high CSF protein
Uncentrifuged sample of
CSF: 10% sensitivity

MTB S2CK

India ink is
60% to 70%
sensitive

p. 11

Meningitis/Treatment

Cryptococcus

Specific serologic
testing, ELISA,
western blot, PCR

Similar accuracy to Gram stain


If positive, theyre extremely specific
If negative, could still have infection
Not sufficiently sensitive to exclude bacterial
meningitis
i iti
When is a bacterial antigen test indicated?
Those receiving antibiotics prior to LP
Culture may be falsely negative
MTB S2CK

What is the Most Accurate Diagnostic Test?


Tuberculosis

Viral

Diagnosis of
exclusion

Cryptococcal
antigen: > 95%
sensitive and
specific

p. 11

Bacterial: Ceftriaxone, Vancomycin, and Steroids


Base treatment answer on cell count
Culture
Needs 2 to 3 days
Never available when treatment decision is made
G
Gram
stain
t i
Good if positive
False negative: 30% to 50%
Protein & Glucose
Nonspecific
Doesnt allow treatment decision
MTB S2CK

p. 11

Meningitis/Treatment

Listeria Monocytogenes

Steroids (dexamethasone)
Lowers mortality only in S. pneumoniae
Give when thousands of neutrophils present
No culture results for several days

Resistant to cephalosporins
Sensitive to penicillins
Add Ampicillin to Ceftriaxone and Vancomycin if
case describes risk factors for Listeria.
Risks

Thousands of neutrophils on
CSF = ceftriaxone, vancomycin,
and steroids. Add ampicillin if
immunocompromised for
Listeria.
MTB S2CK

p. 11

Elderly
Neonates
Steroid use
AIDS or HIV
Immunocompromised (includes alcoholism)
Pregnant

MTB S2CK

p. 11

Neisseria meningitidis

Neisseria meningitidis

Respiratory isolation
Rifampin or Ciprofloxacin to close contacts
Close contacts
Major respiratory fluid contact
Household contacts
Kissing
Sharing cigarettes or eating utensils

Routine school and work contacts are


not close contacts
Healthcare workers qualify only if they

MTB S2CK

p. 12

Intubate patient
Perform suctioning
Or
Have contact with respiratory secretions

MTB S2CK

p. 12

Meningitis

Man comes to emergency department with fever,


severe headache, neck stiffness, and
photophobia. He has weakness in his left arm
and leg.
Whats the next step in management?
a. Ceftriaxone, Vancomycin, and Steroids
b. Head CT Focal neurological deficits,
initiate therapy
c. Ceftriaxone
Almost always
d. Neurology consultation wrong
e. Steroids Not sufficient

MTB S2CK

p. 12

Consultation is almost always a wrong answer on


USMLE Step 2 CK

What is the most common


neurological
l i ld
deficit
fi it off
untreated bacterial
meningitis? Eighth cranial
nerve deficit or deafness.
MTB S2CK

p. 12

Encephalitis
Acute onset
Fever, and
Confusion

Many causes
Herpes simplex
((most common))

Must do head CT
first because of
confusion

What is the most accurate test of herpes


encephalitis?
a.Brain biopsy Less accurate than PCR
b.PCR of CSF
c MRI
c.MRI

The radiologic test is never


the most acc
accurate
rate test

d.Viral culture of CSF Most accurate test for


genital, skin lesions

e.Tzanck prep Best initial test for genital lesion


Useless, 95%

f. Serology for herpes (IgG, IgM) population


MTB S2CK

p. 12

MTB S2CK

p. 13

positive

10

Encephalitis/Treatment

Acyclovir
Best initial therapy (Herpes encephalitis)
Famciclovir & Valacyclovir
IV unavailable

A woman with herpes encephalitis confirmed by PCR


gets 4 days of acyclovir. Her creatinine level rises.
Whats the most appropriate next step in
management?
a. Stop acyclovir Important to treat
a Reduce the dose of acyclovir and hydrate
a.
b. Switch to oral famciclovir or valacyclovir

Foscarnet
Acyclovir-resistant herpes
MTB S2CK

Insufficient for herpes


encephalitis
> Nephrotoxicity than acyclovir

c. Switch to foscarnet

p. 13

MTB S2CK

p. 13

Otitis Media

Head and Neck Infections


Otitis Media
Sinusitis
Pharyngitis
Influenza (The Flu)

Redness
Immobility
Bulging
Decreased light
reflex
Pain
Decreased hearing
Fever

Otitis media. Source: www.sharinginhealth.ca


MTB S2CK

p. 13

Otitis Media/Diagnostic Tests/Treatment

Which of the following is the most sensitive


physical finding for otitis media?
a. Redness
b. Immobility
c. Bulging
d. Decreased light reflex
e. Decreased hearing
Theyre less sensitive
MTB S2CK

p. 13 14

Tympanocentesis:
Sample of fluid for culture
Most accurate diagnostic test
Choose tympanocentesis if:
Multiple recurrences
No response to multiple antibiotics

Radiologic tests for otitis are always the


wrong answer
MTB S2CK

p. 14

11

Otitis Media/Treatment

Amoxicillin: Best initial therapy


If no response to amoxicillin or it recurs,
answer:
Amoxicillin/clavulanate
Azithromycin
A ith
i or clarithromycin
l ith
i
Cefuroxime or loracarbef
Levofloxacin, gemifloxacin, moxifloxacin
Quinolones are relatively
contraindicated in children
MTB S2CK

p. 14

34-year-old woman with facial pain, discolored nasal


discharge, bad taste in mouth, fever and facial
tenderness.
Which of the following is the most accurate diagnostic
test?
a.Sinus
a
Sinus biopsy or aspirate
Radiologic test never the most
b.CT scan
accurate test
c.X-ray
d.Culture of the discharge Always the wrong
answer for sinusitis
e.Transillumination
Doesnt provide precise
microbiological
diagnosis
MTB S2CK p. 14

Sinusitis

Culture of nasal discharge is always


the wrong answer for sinusitis

Use Sinus Biopsy or Aspirate


Infection frequently recurs
No response to different empiric therapies

MTB S2CK

p. 14

34-year-old woman with facial pain, discolored nasal


discharge, bad taste in her mouth, and fever. She has
facial tenderness.
What is the most appropriate next step, action, or
management?
a.
a
b.
c.
d.
e.

Excellent for resistant g


gram-positive,
p

Linezolid doesnt cover Haemophilus


CT scan Diagnosis is clear, radiologic testing
X-ray
unnecessary
Amoxicillin and decongestant
Erythromycin and decongestant

MTB S2CK

Poor coverage for


S. pneumoniae

p. 15

Pharyngitis

Pharyngitis/Diagnostic Tests

Pain on swallowing
Enlarged lymph nodes in
neck
Exudate in pharynx
Fever
No cough and no hoarseness

Best initial test: Rapid strep test


An office/clinic-based test
Finds group A beta-hemolytic streptococci in
minutes
Negative test excludes disease and no antibiotics
are needed
Positive rapid strep test = positive pharyngeal
culture
Small vesicles or ulcers: HSV or herpangina
Membranous exudates: Diphtheria, Vincent angina,
or EBV

Streptococcal pharyngitis. Source: James Heilman,


MD

These features make the likelihood of streptococcal


pharyngitis exceed 90%

MTB S2CK

p. 15

MTB S2CK

p. 15

12

Pharyngitis/Treatment

Influenza (The Flu)

1. Penicillin or amoxicillin is best initial therapy


2. Penicillin allergic:

Cephalexin (reaction only rash)

Clindamycin or macrolide (anaphylaxis)

Streptococcal pharyngitis treated to


prevent rheumatic fever

Arthralgias/myalgias
Cough
Fever
Headache
Sore throat
Nausea, vomiting, or
diarrhea, especially
in children

Source: Mikael Hggstrm

MTB S2CK

p. 15

MTB S2CK

p. 16

Influenza (The Flu)

Influenza (The Flu)/Treatment

The most appropriate next step in


management depends on time course
from presentation
Within 48 hours of onset
Perform nasopharyngeal swab or wash
Rapidly detects antigen associated with
influenza

< 48 hours of symptoms:


Oseltamivir, zanamivir
Neuraminidase inhibitors shorten duration of
symptoms
Treats both influenza A and B
> 48 hours of symptoms:
Symptomatic treatment only
Analgesics, rest, antipyretics, hydration

Oseltamivir and zanamivir dont


successfully treat complications of
influenza such as pneumonia.
MTB S2CK

p. 16

MTB S2CK

p. 16

Blood and WBCs in Stool

Infectious Diarrhea
Blood and WBCs in Stool
No Blood or WBCs in Stool

When you see...in the history the answer


is.
Poultry
Salmonella
Campylobacter: MCC associated with
Guillain-Barre Syndrome
E. coli 0157:H7Hemolytic Uremic
Syndrome (HUS)
Shigella: 2nd most common association
with HUS
MTB S2CK

p. 16

13

Blood and WBCs in Stool

Blood and WBCs in Stool

Shellfish and cruise ships


Vibrio parahaemolyticus:
Shellfish, liver & skin disease
Vibrio vulnificus:
Hemochromatosis blood transfusions
Hemochromatosis,
Yersinia: High affinity for iron
Antibiotics, white and red cells in stool
Clostridium difficile

The Best initial test is:


Blood and/or fecal
leukocytes
Wont determine specific
organism
Stool lactoferrin: Greater
sensitivity
iti it & specificity
ifi it
than stool leukocytes

MTB S2CK

p. 16

WBCs in a stool sample (Methylene Blue Stain). Source:


Bobjgalindo

Lactoferrin: Better answer than fecal leukocytes if


one of the choices
Most accurate test is stool culture
MTB S2CK

p. 16

No Blood No WBCs in Stool

Treatment

Viral
Giardia:

Mild disease: Oral fluid replacement


Severe disease: Fluid replacement and
ciprofloxacin

Camping/hiking and unfiltered fresh water

Cryptosporidiosis:
AIDS <100 CD4 cells
Test acid fast stain

Bacillus cereus: Vomiting


Staphylococcus: Vomiting

MTB S2CK

p. 16 17

Scombroid
Most rapid onset
Wheezing, flushing, rash
Found in fish
Treat with antihistamines

Severe infectious diarrhea means:


Hypotension
Tachycardia
Fever
Abdominal pain
Bloody diarrhea
Metabolic acidosis
MTB S2CK

p. 17

Disease Specific Treatment


Organism

Treatment

Disease
GiardiaSpecific Treatment

Metronidazole,
Tinidazole

Cryptosporidiosis

Treat underlying AIDS,


Ni
Nitazoxanide
id

Viral

Fluid support as needed

B. cereus,
Staphylococcus

Fluid support as needed

MTB S2CK

Hepatitis
Acute Hepatitis
Chronic Hepatitis
p

p. 17

14

Acute Hepatitis/Definition/Etiology

Acute Hepatitis

Infection or inflammation of liver


Majority of acute cases are viral hepatitis A/B
Hepatitis C
Rarely presents with acute infection
Found on blood tests for liver function
Evaluation of cirrhosis
Hepatitis D
Exclusively as coinfection of hepatitis B
IDUs
MTB S2CK

p. 17

Hepatitis E
Worst in pregnancy
East Asia
S
Sex,
Blood,
Bl d M
Mom: H
Hepatitis
titi B
B, C
C, D
Food and water (enteric): Hepatitis A and E
You Ate hepatitis A & you Eat hepatitis E

MTB S2CK

p. 18

Acute Hepatitis/Presentation

Acute Hepatitis/Diagnostic Tests

No way to detect specific type from


symptoms
All forms present with:
Jaundice
Fever,
F
weight
i ht loss,
l
and
d fatigue
f ti
Dark urine
Hepatosplenomegaly
Nausea, vomiting, abdominal pain

Increased direct bilirubin


Increased ratio of alanine aminotransferase
(ALT) to aspartate aminotransferase (AST)
Increased alkaline phosphatase

MTB S2CK

p. 18

Aplastic anemia is a rare


complication of acute hepatitis

MTB S2CK

p. 18

Disease Specific Diagnostic Tests

Which correlates with increased mortality?


a.
b.
c.
d
d.
e.

Bilirubin
Prothrombin time
ALT
AST
Alkaline phosphatase

MTB S2CK

p. 18

All can be
markedly elevated
and better fast,
except PT

Hepatitis A, C, D, and E: Best initial diagnostic


test
IgM antibody: Acute infection
IgG antibody: Resolution of infection
Hepatitis C: Disease activity
PCR RNA level = Amount of active viral
replication
Hepatitis C PCR level = First thing to change
PCR drop = Improvement with treatment
PCR rises = Treatment failure
MTB S2CK

p. 18

15

Serologic Patterns

Serologic Patterns

Acute or chronic
infection

Resolved, old, past


infection

Surface
antigen

Positive

Negative

e antigen

Positive

Vaccination

Window period

Surface
antigen

Negative

Negative

Negative

e antigen

Negative

Negative

Core antibody Positive IgM or IgG

Positive IgG

Core antibody Negative

Surface
antibody

Positive

Surface
antibody

MTB S2CK

Negative

p. 19

MTB S2CK

Which becomes abnormal first after acquiring


hepatitis B infection?
a.
b.
c.
d.
e.
f.

Bilirubin
Viral replication rises after S Ag
e-antigen
Surface antigen
Measure of bodys
Core IgM antibody response to infection
ALT
Anti-hepatitis B e-antibody Resolution

MTB S2CK

Which of the following is the most direct correlate with


the amount/quantity of active viral replication?
a.
b.
c.
d.
e.
f.

Bilirubin
e-antigen
Doesnt tell quantity
Surface antigen
Core IgM antibody
Measure of bodys
response to infection
ALT
Anti-hepatitis B e-antibody

Will appear prior to


resolution of all DNA
polymerase activity

MTB S2CK

Which indicates you cant transmit infection


(i.e., active infection has resolved)?
a. Bilirubin normalizes
b No e-antigen
b.
e antigen found

Normalize long before viral


replication stops.
No viral replication

c. No surface antigen found


d. No core IgM antibody found
e. ALT normalizes
f. Anti-hepatitis B e-antibody
MTB S2CK

p. 19

starting

p. 19

p. 19 20

No serological
evidence of disease
Will appear prior to
resolution of all DNA
polymerase activity

Positive

Positive IgM, then


IgG
Negative

p. 19

Which is the best indication of the need for treatment


in chronic disease?
a.
b.
c.
d.
e.
f.

Bilirubin
At least some active disease,
e-antigen
it might be on the way to
Surface antigen
spontaneous resolution and
Core IgM antibody wouldnt benefit
ALT
Measure of bodys
Anti-hepatitis B e-antibody response to infection

MTB S2CK

p. 20

16

Acute hepatitis/Treatment
Which of the following is the best indicator that a
pregnant woman will transmit infection to her child?
At least some active disease, it might
a. Bilirubin
be on the way to spontaneous
b. e-antigen
resolution and wouldnt benefit.
c. Surface antigen
Perinatal transmission: 10% if
d. Core IgM antibody positive surface antigen, but eantigen is negative; 90% when
e. ALT
both positive
f. Anti-hepatitis B, e-antibody
Measure of the bodys response to
infection
MTB S2CK

p. 20

Chronic Hepatitis/Treatment
Chronic hepatitis B = Surface antigen > 6 months
e-antigen = Elevated level of DNA polymerase
Treat when BOTH surface and e-antigen are positive
Entecavir, or
Adefovir, or
T
Tenofovir,
f i or
Lamivudine, or
Telbivudine, or
Interferon (joint & muscle pain, depression, the flu)
Interferon is an injection
Interferon has most adverse effects
MTB S2CK

p. 21

Role of Liver Biopsy

Hepatitis A and E:
Resolve spontaneously over weeks
Almost always benign conditions
Only acute hepatitis C
Hepatitis B:
gets medical therapy
Ch i iin 10%
Chronic
No treatment for acute disease
Hepatitis C:
Use interferon, ribavirin and either telaprevir or
boceprevir!!!
Treatment decreases likelihood of chronic infection
MTB S2CK

Chronic Hepatitis/Treatment
Adverse effects of interferon:
Arthralgia/myalgia
Leukopenia & thrombocytopenia
Depression & flu-like symptoms

Goal of hepatitis therapy:


Reduce DNA polymerase
Convert e-antigen to anti-hepatitis e-antibody

MTB S2CK

p. 21

p. 21

Chronic Hepatitis

Fibrosis is a strong indication to begin


therapy for either hepatitis B or C right away
Fibrosis + Active Viral replication will
progress to cirrhosis
Cirrhosis: Irreversible
Who will progress?
Old terms chronic active or chronic
persistent hepatitis are unhelpful
DNA polymerase level is helpful
MTB S2CK

p. 21

ALT levels arent a good


indication of chronic
hepatitis activity. You can
have significant infection
with normal transaminase
levels.

MTB S2CK

p. 21

17

Treatment of Chronic Hepatitis C

Treatment of Chronic Hepatitis C

Theres no way to determine the duration


of infection with hepatitis C, since theres
no equivalent of surface antigen test
Most patients dont have acute symptoms
PCR RNA viral load is elevated, patients
If PCR-RNA
should be treated with interferon and
ribavirin and either boceprevir or telaprevir

Three Drugs!
80% resolution with 3 drugs
Interferon AND Ribavirin
AND either
Telaprevir OR Boceprevir

Ribavirin causes anemia


MTB S2CK

p. 21

MTB S2CK

p. 21 22

Urethritis

Sexually Transmitted Diseases


Urethritis
Cervicitis
Pelvic Inflammatory Disease (PID)
Syphilis
Genital Warts (Condylomata Acuminata)
Pediculosis (Crabs)
Scabies

Urethritis/Diagnostic Tests
Best initial test
Men: Urethral swab for Gram stain & WBCs
Intracellular gram-negative diplococci = Neisseria
gonorrhoeae
Urine for nucleic acid amplification test (NAAT)
detects gonorrhea and Chlamydia
NAAT or DNA probe: Most accurate test
Other causes of urethritis:
Mycoplasma genitalium
Ureaplasma
Women: Self-administered vaginal swab
MTB S2CK

p. 22

Look for urethral discharge to answer


What is the most likely diagnosis?
Both urethritis & cystitis give:
Dysuria
Frequency
Burning
Cystitis: No discharge

MTB S2CK

p. 22

Urethritis/Treatment

Combine:
One drug for gonorrhea & one for Chlamydia
Quinolones arent the best initial therapy
because of resistance

MTB S2CK

Gonorrhea

Chlamydia

Cefixime
Ceftriaxone

Azithromycin
Doxycycline

p. 22

18

Cervicitis

PID/Presentation

Cervical discharge
Inflamed strawberry cervix
Testing & treatment are identical to
previous description for urethritis
Except
E
t self-administered
lf d i i t d vaginal
i l swab
b
for NAAT

Lower abdominal tenderness


Lower abdominal pain
Fever
Cervical motion tenderness
Leukocytosis

Always to exclude pregnancy first!!

MTB S2CK

p. 22

MTB S2CK

p. 22 23

PID/Diagnostic Tests

Laparoscopy in PID

Cervical swab for:


Culture
DNA probe or nucleic acid amplification
(NAAT)
Tests clarifyy need for treating
gp
partner
Culture preferred to determine resistance

The most accurate test for PID


Only rarely needed
Laparoscopy is answer if:
Diagnosis is unclear
Symptoms persist despite therapy
Recurrent episodes for unclear reasons

Cervical testing isnt


the most accurate
test for PID.
MTB S2CK

p. 23

PID/Treatment

Combination of medications for gonorrhea


and Chlamydia
Inpatient: Cefoxitin or cefotetan combined
with doxycycline
Outpatient: Ceftriaxone and doxycycline
(possibly with metronidazole)
Anaphylaxis to penicillin?:
Levofloxacin and metronidazole (outpatient)
Clindamycin and gentamicin (inpatient)
MTB S2CK

p. 23

MTB S2CK

p. 23

Ulcerative Genital Disease/


What Is the Most Likely Diagnosis?
Often impossible to determine specific
diagnostic, visual characteristics
Question must have sufficient
evidence to g
give answer

All genital ulcers can have inguinal


adenopathy
MTB S2CK

p. 23

19

Presentation of STDs

Diagnostic Tests

History and physical findings

Most likely diagnosis

Diagnosis

Diagnostic Test

Painless ulcer

Syphilis

Syphilis

Painful ulcer

Chancroid (Haemophilus
ducreyi)

Dark field microscopy (Best in primary)


VDRL or RPR (75% sensitive in primary)
FTA or MHA TP (confirmatory)

Chancroid (Haemophilus
ducreyi)

Stain & culture (special media)

Lymph nodes tender and


suppurating

Lymphogranuloma
venereum

Lymphogranuloma
venereum

Complement fixation titers in blood


Nucleic acid amplification testing on
swab

Vesicles prior to ulcer


and painful

Herpes simplex

Herpes simplex

Tzanck prep: Best initial test


Viral culture: Most accurate test

MTB S2CK

p. 23

MTB S2CK

Diagnostic Tests

p. 24

Treatment
Diagnosis

If dark-field is positive for


spirochetes no further
testing for syphilis is
necessary
necessary.

Treatment

Syphilis

One dose IM benzathine penicillin


Doxycycline, if penicillin allergic

Chancroid
Azithromycin (single dose)
(Haemophilus ducreyi)
Lymphogranuloma
venereum
Herpes simplex

MTB S2CK

p. 24

MTB S2CK

Doxycycline
Acyclovir, valacyclovir, famciclovir
Foscarnet for acyclovir resistant
herpes

p. 24

Syphilis/Presentation
Woman comes to clinic with multiple painful genital
vesicles.
What is the next step in management?
a. Acyclovir orally
y
topically
p
y Worthless
b. Acyclovir
If the presentation is diagnostic testing is

c. Tzanck prep not necessary

Most accurate test, but not necessary, if

Primary Syphilis
Painless genital ulcer
with heaped-up
indurated edges (it
becomes painful if it
becomes secondarily
infected with bacteria)
Painless adenopathy

d. Viral culture vesicles are clear

Chancres heal spontaneously even


without treatment. Penicillin
prevents later stages.

e. Serology Cant distinguish genital infection from oral


infection in the past

f. PCR
MTB S2CK

p. 24

Richard Usatine, M.D. Used with permission

PCR encephalitis, not genital


MTB S2CK

p. 25

20

Syphilis/Presentation

Syphilis/Presentation

Secondary Syphilis
Rash (palms and
soles)
Alopecia areata
Mucous patches
Condylomata lata

Tertiary Syphilis
Neurosyphilis
Meningovascular (stroke from vasculitis)
Tabes dorsalis (loss of position and vibratory sense,
incontinence, cranial nerve)

Tabes Dorsalis . Source: CDC/Susan Lindsley

MTB S2CK

p. 25

phil.cdc.gov

Syphilis/Presentation

Tertiary Syphilis
Neurosyphilis
General paresis (memory and personality
changes)
Argyll Robertson pupil

Aortitis (aortic regurgitation, aortic


aneurysm)
Gummas (skin and bone lesions)
MTB S2CK

p. 25

Syphilis

p. 25

Sensitivity of Diagnostic Tests by Stage


Test

VDRL or RPR

FTA ABS

Primary

75%85%

95%

Secondary

99%

100%

Tertiary

95%

98%

MTB S2CK

p. 25

Syphilis/Treatment

False positive VDRL/RPR


Infection, older age, injection drug
use, AIDS, malaria, antiphospholipid
syndrome, and endocarditis
Titers of VDRL or RPR are
reliable at > 1:8
Lower titer is more often falsely
positive
High titers (> 1:32) are rarely
false positive
MTB S2CK

MTB S2CK

p. 25

Primary & secondary syphilis:


Single IM injection of penicillin
Oral doxycycline: Penicillin allergy
Tertiary syphilis:
IV penicillin
Desensitize to penicillin if allergic

MTB S2CK

p. 25

21

Syphilis

Genital Warts (Condylomata Acuminata)/


Diagnosis

Jarisch-Herxheimer reaction
Fever, headache, myalgias after treatment
Give aspirin & antipyretics (itll pass)

Desensitization the answer for


neurosyphilis and pregnant women

MTB S2CK

p. 26

MTB S2CK

Genital Warts (Condylomata Acuminata)/


Treatment

Remove by physical means:


Cryotherapy with liquid nitrogen
Surgery for large ones
Laser
Melt with podophyllin or trichloroacetic
acid
Imiquimod: Apply locally
(immunostimulant leads to sloughing off of
lesion)
MTB S2CK

Papillomavirus
Diagnose based on
the visual
appearance
Wrong answers:
Biopsy
Serology
Stain, smear, or
Condylomata acuminata (genital warts). Source: Farshad
culture
Bagheri, MD.

p. 26

p. 26

Pediculosis (Crabs)

Hair-bearing areas
(axilla, pubis)
Itchy
Visible on surface
Treat with permethrin;
li d
lindane
iis equall iin
efficacy, but more toxic

MTB S2CK

Pediculosis pubis. Source: commons.wikimedia.


Used with permission

p. 26

Scabies

Scabies/Treatment

Web spaces between


fingers and toes or at
elbows
Around nipples near
the genitals
Burrows visible (they
dig) but smaller than
pediculosis
Scrape & magnify

Permethrin
Widespread disease: Oral Ivermectin
Severe disease needs repeat dosing

MTB S2CK

p. 27

Scabies burrow under the skin and must be scraped out to


establish a diagnosis. Source: Conrad Fischer, MD.

MTB S2CK

p. 27

22

Urinary Tract Infections (UTI)

Urinary Tract Infections


Cystitis
Pyelonephritis
y
p
Acute Prostatitis
Perinephric Abscess

Present with dysuria (frequency, urgency,


burning), maybe fever
Urinalysis shows WBCs
E. coli MCC
Quinolones Best initial therapy
Anatomic defects lead to UTIs:
Stones
Strictures
Tumor or prostate hypertrophy
Diabetes
MTB S2CK

p. 27

Urinary Tract Infections

Cystitis

Foley catheter is a foreign body


Neurogenic bladder is an obstruction

Presents with dysuria and:


Suprapubic pain/discomfort
Mild/absent fever

Frequency = Multiple episodes of micturation


Polyuria = Increase in volume of urine

Men have anatomic abnormalities


Women most often dont
Best initial test: Urinalysis with > 10 WBCs
Most accurate test: Urine culture

MTB S2CK

p. 27

MTB S2CK

p. 27 28

Cystitis/Treatment

Trimethoprim/sulfamethoxazole (TMP/SMZ)
if local resistance is low
Ciprofloxacin
Cephalexin
Nitrofurantoin ((Pregnant
g
women))
All beta-lactam antibiotics are
considered safe in pregnancy

MTB S2CK

p. 28

36 year old generally healthy woman comes with


urinary frequency and burning.
UA: > 50 WBC
What is the next step in management?

a.
b.
c.
d.
e.

TMP/SMZ for 3 days


TMP/SMZ for 7 days With anatomic abnormality
Urine culture
No need of culture
Ultrasound of urinary system or imaging when
symptoms of cystitis
CT scan of urinary system

MTB S2CK

p. 28

are clear & WBCs in


urine

23

Pyelonephritis

Pyelonephritis

Dysuria with:
Flank or CVA tenderness
High fever
Occasional abdominal pain
UA with high WBCs
Imaging studies (CT or sonogram) look
for anatomic abnormalities causing
infection

Treat with:
Ampicillin & gentamicin
Ciprofloxacin
Change based on culture/sensitivity

MTB S2CK

p. 28

Any drugs for gram-negative


bacilli would be effective for
pyelonephritis

MTB S2CK

p. 28

Acute Prostatitis

Perinephric Abscess

Dysuria with:
Perineal pain
Tender prostate on examination
Diagnostic yield of urine culture greatly
increased with p
prostate massage
g
Treat same as you would for pyelonephritis
Longer duration
TMP/SMZ or ciprofloxacin for 6 to 8 weeks
for chronic prostatitis

Look for:
Pyelonephritis not resolving with appropriate
therapy
When drug choice & dose are correct failure
of infection to resolve is an anatomic problem
Do sonogram or CT scan
Drainage of fluid collection is mandatory
Culture of infected fluid is essential to guide
therapy

MTB S2CK

p. 29

MTB S2CK

p. 29

Endocarditis/Definition

Endocarditis

Heart valve infection


Leads to fever & murmur
Diagnosis:
Positive blood cultures
Vegetations on echocardiogram

MTB S2CK

p. 29

24

Endocarditis/Etiology

Endocarditis/Etiology

Rare on normal heart valves (except


IDUs)
Risk proportional to degree of damage
of valves
Regurgitant & stenotic lesions have
increased risk
Prosthetic valves: Highest risk

MTB S2CK

p. 29

MTB S2CK

Endocarditis/
What Is the Most Likely Diagnosis?

Fever
New murmur or
change in a murmur
Complications of
endocarditis

Source: Splarka
http://en.wikipedia.org/wiki/Splinter_hemorrhages

p. 29 30

p. 29

Endocarditis/Presentation/
What Is the Most Likely Diagnosis?

Splinter hemorrhages
Janeway lesions (flat
and painless)

MTB S2CK

Can develop on normal valves if:


Severe bacteremia
Highly pathogenic organisms
(Staphylococcus aureus)
Dental procedures with blood confer a small
risk of endocarditis
Surgery of mouth & respiratory tract confers
no risk unless severe valve disease is
present
Artificial valve or cyanotic heart disease
Endoscopy confers no risk even with biopsy

Osler nodes (raised and painful)


Roth spots in eyes
Brain (mycotic aneurysm)
Kidney (hematuria, glomerulonephritis)
Conjunctival petechiae
Splenomegaly
Septic emboli to lungs

MTB S2CK

p. 30

Endocarditis/Diagnostic Tests

Best initial test Blood culture (95%99%


sensitive)
If positive do an Echo
Transthoracic echo (60% sensitive, but 95%
specific)
Transesophageal
T
h
l echo
h (95% sensitive
iti and
d
specific)
EKG rarely shows atrioventricular (AV) block if
dissection of conduction system occurs (5%
10%)

A man comes into the ED with fever and a murmur.


Blood cultures grow Streptococcus bovis.
Transthoracic echocardiography shows vegetation.
What is the next step in the management?
a.
b
b.
c.
d.
e.

Transthoracic showed

Colonoscopy
vegetation.
Transesophageal echocardiogram
Will not show diverticuli
CT of the abdomen
Repeat the blood cultures No point, already positive
Surgical valve replacement Premature

Fever + murmur =
endocarditis

MTB S2CK

p. 30

MTB S2CK

p. 30 31

25

How to Diagnose
Culture Negative Endocarditis

Endocarditis/Treatment

1. Oscillating vegetation on echocardiography


2. Three minor criteria:
Fever
Risk: IDU or prosthetic valve
Embolic phenomena

MTB S2CK

p. 31

Empiric:
Vancomycin and gentamicin
When culture results are available
Treat based on sensitivities

MTB S2CK

Endocarditis/Treatment

p. 31

Endocarditis/Treatment

Organism

Treatment

Organism

Viridans streptococci

Ceftriaxone for 4 weeks

Vancomycin

Staphylococcus aureus
(sensitive)

Oxacillin, nafcillin, or
cefazolin

Staphylococcus
epidermidis or resistant
Staphylococcus
Enterococci

Ampicillin & gentamicin

Fungal

Amphotericin and valve


replacement

MTB S2CK

p. 31

Endocarditis

MTB S2CK

p. 31

p. 31

Endocarditis

Treatment of Resistant Organisms


Add aminoglycoside & extend duration of
treatment
When Is Surgery the Answer?
CHF or ruptured valve or chordae tendineae
Prosthetic valves
Fungal endocarditis
Abscess
AV block
Recurrent emboli while on antibiotics
MTB S2CK

Treatment

Add rifampin for prosthetic


valve endocarditis with
Staphylococcus.
The single strongest indication
for surgery is acute valve
rupture and CHF.
MTB S2CK

p. 31 32

26

Treatment of Culture Negative Endocarditis

Prophylaxis for Endocarditis

Most common culture negative = Coxiella


HACEK: Acronym for organisms difficult to
culture causing endocarditis

Use prophylaxis if theres BOTH:


1. Significant cardiac defect

Haemophilus aphrophilus
Haemophilus parainfluenza
Actinobacillus
Cardiobacterium
Eikenella
Kingella

Prosthetic valve
Previous endocarditis
Cardiac transplant recipient with valvulopathy
Unrepaired cyanotic heart disease

AND!
2. Risk of bacteremia
Dental work with blood
Respiratory tract surgery that produces
bacteremia

Use ceftriaxone for HACEK group


MTB S2CK

p. 32

MTB S2CK

p. 32

Prophylaxis for Endocarditis

Prophylaxis for Endocarditis

Amoxicillin prior to procedure


If penicillin allergic use clindamycin,
azithromycin, or clarithromycin
Procedures and anatomic abnormalities
that do not need prophylaxis are:
Flexible endoscopy, even with biopsy,
ERCP
Obstetrical and gynecologic procedures

Urology procedures (including prostate


biopsy)
Mitral valve prolapse, even with murmur
Mitral regurgitation, mitral stenosis,
aortic regurgitation
regurgitation, aortic stenosis
stenosis,
hypertrophic obstructive
cardiomyopathy (HOCM), atrial septal
defect (ASD)

MTB S2CK

p. 32

MTB S2CK

p. 32

Lyme Disease/Definition

Lyme Disease

Arthropod-borne
Spirochete: Borrelia burgdorferi
What is the most common manifestation?
Fever & Rash
What is the most common manifestation if
untreated?
Joint pain
Also: Cardiac or neurologic disease
MTB S2CK

p. 33

27

Lyme Disease/Etiology

Lyme Disease/Etiology

Transmitted by deer
tick (Ixodes
scapularis)
Tick very small
Only 20% patients
recall bite

Patients recall being outdoors


Hiking or camping
Experimental models:
Tick must be attached for at least 24
hours in order to transmit organism

Richard Usatine, M.D. Used with permission

MTB S2CK

p. 33

MTB S2CK

p. 33

Lyme Disease/Etiology

Lyme Disease/Presentation

Ixodes tick
Northeast states:
Connecticut (where the town of Lyme gave
the disease its name)
Massachusetts
New York
New Jersey

Rash:
Most common manifestation (85% - 90%)
Occurs 5-14 days after bite
Fever often present

MTB S2CK

p. 33

cdc.gov

MTB S2CK

p. 33

Lyme Disease/Presentation

Lyme Disease/Presentation

Erythema migrans
Round, red
Pale center
Target or bulls-eye

Joint pain 60% without treatment


Oligoarthritis = few joints
Joint fluid: ~ 25,000 WBCs/ L
Similar number to gout, pseudogout and
many infections
i f ti
The knee is the most commonly
affected joint in Lyme disease.

Target-shaped rash of Lyme disease or erythema migrans.


Source: Nishith Patel.

MTB S2CK

p. 33 34

MTB S2CK

p. 33

28

Lyme Disease/Presentation

Lyme Disease/Presentation

Neurological manifestations 10% 15%


CNS or peripheral
Meningitis, encephalitis, or CN palsy

Cardiac: 4% - 10%
Any part of myocardium or pericardium
Myocarditis or ventricular arrhythmia

Transient AV block is the most


common cardiac manifestation in
Lyme disease.

Bells palsy or 7th CN is most


common neurological
manifestation of Lyme disease.

MTB S2CK

p. 33

MTB S2CK

p. 33

Lyme Disease/Diagnostic Tests

Lyme Disease/Treatment

Typical rash doesnt need serologic testing to


start treatment
Serologic testing for Lyme is essential for:
Joint
Neurologic
g
Cardiac manifestations
Most 7th CN palsy, arthralgia & AV block are
not caused by Lyme
Testing is with IgM, IgG, ELISA, Western
blot, and PCR testing

Manifestation

Treatment

Asymptomatic tick bite

No treatment
routinely

Rash

Doxycycline
Amoxicillin

Joint, 7th CN palsy

Doxycycline
Amoxicillin

Cardiac and neurologic


manifestations other than 7th CN
palsy

IV ceftriaxone

MTB S2CK

p. 34

MTB S2CK

p. 34

Asymptomatic Tick Bite

Bites without symptoms rarely need


treatment
Who DOES need treatment?
Single dose of doxycycline
Within 72 hours of tick bite when:

HIV/AIDS

Ixodes scapularis clearly identified as tick


Tick attached for > 24-48 hours
Engorged nymph-stage tick
Endemic area
MTB S2CK

p. 34

29

HIV/AIDS/Definition

A retrovirus infecting CD4 (T-helper)


cells
CD4 cells drop from normal (600-1000)
at a rate of 50 to 100/year untreated
5 and 10 years before clinical
manifestations
Not HIV itself that leads to death
Low CD4 leads to illness

HIV life cycle

Source: nih.gov

MTB S2CK

p. 35

HIV/AIDS/Etiology

Transmitted by:
IDU
Sex, particularly men who have sex with men
Transfusion (extremely rare since 1985)
Perinatal
Needle stick or blood-contaminated sharp
instrument injury
Kissing is not proven to
transmit HIV.
MTB S2CK

p. 35

Risk of Transmission of HIV


Without Prophylactic Treatment
Mode of
transmission
Vaginal transmission

Percentage of risk with each event


1:30001:10,000 for insertive intercourse
1:1000 for receptive intercourse

Oral sex

1:1000 for receptive fellatio with ejaculation


Unclear for insertive fellatio or cunnilingus

Needle stick injury

1:300

Anal sex

1:100 for receptive anal intercourse

Mother to child

25%30% perinatal transmission without


medication

MTB S2CK

p. 35

HIV/AIDS/Presentation

HIV/AIDS/Diagnostic Tests

Best initial test: ELISA test


Confirmed with Western blot
Infected infants:
Diagnose with PCR or viral culture
ELISA testing is unreliable in infants
Maternal HIV antibodies are present for
up to 6 months after delivery

Infections when CD4 drops < 50/ L


PCP < 200/ L
CD4 > 200/ L (few infections occur)
Infections > 200/ L are:
Varicella zoster (shingles)
Herpes simplex
TB
Oral and vaginal candidiasis
Bacterial pneumonia

MTB S2CK

p. 35

MTB S2CK

p. 36

30

Viral Load Testing (PCR RNA level)

Viral Load Testing (PCR RNA level)

The goal of therapy is to drive down


viral load
Undetectable levels (< 50/ L): CD4 will
rise
CD4 rises = Opportunistic infections
stop
Life expectancy when viral load is
undetectable by PCR-RNA is equal in
duration to HIV-negative person

Measures response to therapy


Decreasing levels good
Detect treatment failure
Rising levels bad
Diagnoses HIV in babies

MTB S2CK

p. 36

MTB S2CK

p. 36

Viral Resistance Testing (Genotyping)

HIV/AIDS/Treatment

Perform prior to initiating antiretroviral


medications
Decreases likelihood of starting
medication to which patients virus is
resistant
Resistance testing: Evaluates treatment
failure
Guides choice of medications
Select 3 drugs from 2 different classes to
which patients virus is susceptible

For HIV: Treatment is initiated when:


CD4 < 500/ L
or
Viral load is very high (>100,000/ L)
or
Opportunistic infection occurs

MTB S2CK

p. 36

HIV/AIDS/Treatment

MTB S2CK

p. 36

Choice of Initial Antiretroviral Medication


Initial drug regimen:

Treatment failure first


manifests with rising PCRRNA viral load

MTB S2CK

p. 36

Emtricitabine, Tenofovir, and


Efavirenz
USMLE Step 2 CK doesnt
doesn t test
dosing

MTB S2CK

p. 36

31

Choice of Initial Antiretroviral Medication

Antiretroviral First line Medications by Class

Treatment failure:
Rising viral load
CD4 count decreases or fails to rise
CD4 changes are slower than changes in
viral load
Alternate Drug Regimens
If emtricitabine/tenofovir/efavirenz cannot be
used because of resistance alternate
regimens are based on combination of 3
drugs from at least 2 different classes

Nucleoside and
nucleotide reverse
transcriptase
inhibitors (RTIs)

Non
nucleoside
RTIs

Protease inhibitors

Zidovudine
Didanosine
Stavudine
Lamivudine
Emtricitabine
Abacavir
Tenofovir

Efavirenz
Etravirine
Nevirapine

Ritonavir
Saquinavir
Nelfinavir
Amprenavir
Fosamprenavir

MTB S2CK

p. 37

MTB S2CK

p. 37

Additional Classes of Second line Agents

Postexposure Prophylaxis

Used with drug resistance to multiple classes of


first-line agents
Entry inhibitors:

Enfuvirtide
Maraviroc

Integrase inhibitor:
Raltegravir

Lopinavir
Atazanavir
Indinavir
Tipranavir
Darunavir

All significant needle stick injuries


Sexual exposures
Bites
Give 4 weeks of combination therapy
Urine & stool arent an indication for
postexposure prophylaxis (PEP)

USMLE Step 2 CK wont require you to


know details of efficacy differences
between classes.
MTB S2CK

p. 37

Postexposure Prophylaxis

Postexposure prophylaxis
isnt routinely indicated for
needle stick injury if HIV
status of needle is unknown

MTB S2CK

p. 37

Adverse Effects of HIV Medications


Drug

Adverse effect

Zidovudine

Anemia

Stavudine and didanosine

Peripheral neuropathy and pancreatitis

Abacavir

Hypersensitivity, Stevens Johnson


Reaction

Protease inhibitors

Hyperlipidemia, hyperglycemia

Indinavir

Nephrolithiasis

Tenofovir
MTB S2CK

p. 37

MTB S2CK

Renal insufficiency
p. 38

32

Prevention of Perinatal Transmission

Prevention of Perinatal Transmission

Pregnant Patients
If already on antiretroviral medications
Continue same treatment
If pregnant and not already on
medications
di ti
Start combination antiretrovirals
Only Efavirenz is NOT used in
pregnancy

After Delivery for Mother


CD4 > 500 and she doesnt need them
Stop after delivery

MTB S2CK

p. 38

MTB S2CK

Indications for Antiretrovirals


During Pregnancy
Action

Patient on antiretrovirals at
the time of pregnancy

Continue same medications,


except switch efavirenz to
protease inhibitor

Not on antiretrovirals,
CD4 low or viral load high

Initiate antiretrovirals
immediately. Continue after
delivery

Not on antiretrovirals,
CD4 high and viral load low

Antiretrovirals immediately,
stop them in mother after
birth

p. 38

p. 38

Cesarean Delivery for HIV Positive Mothers

Condition

MTB S2CK

CD4 < 500 or viral load high


Continue after delivery
Baby
Zidovudine during delivery (intrapartum) AND
For 6 weeks after to help prevent transmission

Viral load >1000 L: Perform Cesarean


delivery
Everyone: Intrapartum zidovudine
Fully controlled HIV (viral load
undetectable) gives <1%
transmission.

MTB S2CK

p. 39

33

Diagnostic Tests
in Nephrology

Nephrology
Emma Holliday,
Holliday MD
Resident Physician
Radiation Oncology
University of Texas MD Anderson Cancer Center

Urinalysis
White Blood Cells
Hematuria
Cytoscopy
Casts

Urinalysis

Dipstick vs. Urinalysis

Measures:

Both give some quantitative values


Dipstick described as direct (e.g., 300 mg protein)
or scale number: 0, 1+, 2+, 3+, or 4+
Microscopic urinalysis reports number of cells/highpowered field

Protein
WBCs or leukocyte esterase
RBCs
Specific gravity and pH
Nitrites

Pyuria with positive nitrites = UTI

MTB S2CK

p. 299

Dont worry about the precise scale


Every USMLE Step 2 CK test comes with a range
of normal values attached to assess severity

MTB S2CK

p. 299

Urinalysis/Protein

Urinalysis/Protein

Renal tubules secrete slight amounts of TammHorsfall protein

2 - 10% of population has transient proteinuria


Benign reasons for proteinuria:

Should be < 30 to 50 mg/24 hours

Greater amounts of protein associated with either


tubular disease or glomerular disease
Very large amounts of protein excreted with
glomerular disease
1+ proteinuria ~ 1g/day
2+ proteinura ~ 2g/day
3+ proteinuria ~ 3g/day
4+ proteinuria ~ 4g/day
MTB S2CK

p. 300

Orthostatic proteinura or physical exertion


If symptoms of
proteinuria - best
initial test?

Urinalysis/Dipstick

If positive, order urine


protein to creatinine ratio
or 24 hour urine collection
Most accurate for
determining
etiology?
MTB S2CK

Renal biopsy

p. 300

Urinalysis/Proteinuria

Urinalysis/Microalbuminuria

Limitations of dipstick for proteinuria:


Only detects albumin
Cant detect Bence Jones protein

Microalbuminuria

Present in multiple myeloma


Best test for Bence Jones proteinuria is UPEP (urine
protein electrophoresis)

Cant detect very low amounts of protein

Tiny amounts of protein too small to


detect on UA

An important screening test in diabetic


patients
Should be performed yearly

Long-term microalbuminuria
Leads to worsening renal function in
diabetic patient and should be treated
MTB S2CK

p. 301

MTB S2CK

p. 301

Pyuria
Diabetic patient is evaluated, UA shows no protein.
Microalbuminuria is detected (level between 50 - 300
mg/24 hours).
Whats the next best step in management?
a. Enalapril
b Kidney biopsy Extreme! We know the etiology already
b.
c. Hydralazine Less effective & with more adverse effects
d. Renal consultation NEVER consult on Step 2
e. Low-protein diet Bad for glycemic control
f. Repeat UA annually and treat when trace protein is
Starting early will delay disease progression
detected
MTB S2CK

p. 301

White blood cells in urine:


Inflammation
Infection
Allergic interstitial nephritis

Neutrophils cant be distinguished from eosinophils


on UA
Neutrophils indicate infection (UTI or urethritis)
Eosinophils indicate allergic or acute interstitial
nephritis (NOT NSAID-induced renal disease)
Wright and Hansel
stains detect
eosinophils in the urine.
MTB S2CK

p. 301

Hematuria
Etiology:
Stones
Coagulopathy
Infection (cystitis, pyelonephritis)
Cancer
Cancer
C
T
Treatments
t
t (cyclophosphamide)
( l h
h id )
Trauma
Glomerulonephritis
False positive = hemoglobinuria or myoglobinuria

Woman is admitted with trauma and dark urine. The


dipstick is markedly positive for blood.
What is the best initial test?
a. Microscopic examination of urine
Overkill - best test to look for bladder mass
b. Cystoscopy
Best test for hydronephrosis
c Renal ultrasound
c.
d. Renal/bladder CT scan Most accurate test for stones
e. Abdominal X-ray Best test for ileus or to detect free air
f. Intravenous pyelogram Never the right answer - slow
test uses nephrotoxic contrast

Dysmorphic RBCs = glomerulonephritis


MTB S2CK

p. 301 302

MTB S2CK

p. 302

When Is Cystoscopy the Answer?

Casts

When theres hematuria without


infection or prior trauma and:

Microscopic collections of material clogging up the


tubules and being excreted in urine

The renal ultrasound or CT doesnt


show etiology
Bladder sonography shows mass for
possible biopsy

Hyaline
Broad, waxy
Granular, muddybrown

Cystoscopy is the most


accurate test of the
bladder.
MTB S2CK

Type of cast
Red cell
White cell
Eosinophil

p. 302

MTB S2CK

p. 302 303

Association
Glomerulonephritis
Pyelonephritis
Acute interstitial
nephritis
Dehydration
Chronic renal disease
Acute tubular necrosis
(are dead tubular
casts)

Acute Kidney Injury

Acute kidney injury (AKI)

Acute Kidney Injury


Acute Tubular Necrosis
Hepatorenal
p
Syndrome
y
Atheroemboli
Acute (Allergic) Interstitial Nephritis
Analgesic Nephropathy

Decrease in creatinine clearance


Results in sudden rise in BUN and creatinine

The definition is not based on specific values


of BUN and creatinine
Categories:
Prerenal azotemia (decreased perfusion)
Postrenal azotemia (obstruction)
Intrinsic renal disease (ischemia and toxins)
MTB S2CK

Acute Kidney Injury/Presentation

Acute Kidney Injury


Prerenal

Intrinsic Renal

ATN
Hypotension
- Toxins
- Sepsis
NSAIDs
- Anaphylaxis
AG, ampho
Cis,
Cis cyclosporine
- Bleeding
- Prolonged ischemia
- Dehydration AIN
Hypovolemia
- PCN, sulfa
Rhabdo/hemoglobinuria
- Diuretics
Contrast
- Burns
Crystals
- Pancreatitis Bence-Jones proteins
Post-strep infection
- pump fxn
- Low albumin
- Cirrhosis
MTB S2CK p. 303 304
artery stenosis
Renal

p. 303

Post Renal

BPH/Prostate cancer
Ureteral stone
Cervical cancer
Urethral stone
Neurogenic bladder
Retroperitoneal
fibrosis (chemo or
XRT)

AKI usually = asymptomatic rise in BUN and


creatinine; when symptomatic:
Nausea and vomiting
Fatigue/malaise
Weakness
SOB, edema (fluid overload)

Very severe disease presents with:


Confusion
Arrhythmia from hyperkalemia and acidosis
Sharp, pleuritic chest pain from pericarditis
MTB S2CK

p. 304 305

Acute Kidney Injury/Diagnostic Tests

Acute Kidney Injury/Diagnostic Tests

The best initial test is...

What is the best initial imaging test?

BUN and creatinine

Nonfunctional kidneys creatinine rises


~ 1 point/day (1 mg/dL)
If the BUN:creatinine
BUN creatinine ratio is > 20
20:1
1 the
etiology is either prerenal or postrenal
damage
Intrinsic renal disease ratio closer to
10:1
MTB S2CK

p. 305

Renal sonogram because it isnt invasive and


doesnt need contrast

Kidney biopsy is rarely the right answer for


AKI

MTB S2CK

p. 305

Acute Kidney Injury/Diagnostic Tests

Acute Kidney Injury/Diagnostic Tests

When cause is unclear next best diagnostic step


is:

Urine Sodium and Fractional Excretion of Sodium


Normal kidney:

Urinalysis
Urine sodium (UNa)
Fractional excretion of sodium (FENa) or urea
(FEUrea)
Urine osmolality

If all of these are choices always go with


urinalysis first

MTB S2CK

p. 306

blood pressure
aldosterone
aldosterone
sodium reabsorption
in the urine (< 20)

sodium

Damaged kidney (e.g., ATN):


Kidneys cant reabsorb Na appropriately (> 20)
Prerenal azotemia:
low UNa (below 20) =
low FENa (less than 1%)

MTB S2CK

You can answer all the


questions on USMLE Step
2 CK without knowing the
mathematical formula for
FENa.

p. 306

Acute Kidney Injury/Diagnostic Tests

Urine Osmolality
Normal kidney:
intravascular volume
ADH
ADH
water reabsorption at
collecting duct
urine osmolarity

Damaged kidney (e.g., ATN):


Kidneys cant concentrate urine
appropriately - will see Osm similar to
blood (~300mOsm)
MTB S2CK

p. 306

20-year-old African-American man has screening test


for sickle cell. Hes found to be heterozygous (trait or
AS) for sickle cell.
Sickle cell trait rarely
What is the best advice for him?experience pain crises
a.
b.
c.
d.
e.

Nothing needed until he has a painful crisis


Avoid dehydration
y
Hydroxyurea Only if > 4 pain crises per year.
Folic acid supplementation Indicated if hemolysis
Pneumococcal vaccination present (high RBC
Indicated if functionally
asplenic

MTB S2CK

turnover)

p. 307

Classification of Acute Renal Failure


Test

Prerenal
azotemia

Acute tubular
necrosis

BUN:Creatinine

> 20:1

< 20:1

Urine Sodium (UNa)

< 20 mEq/L

> 20 mEq/L

FFractional
ti
l excretion
ti
of sodium (FENa)

< 1%

> 1%

Urine osmolality
(UOsm)

> 500
mOsm/kg

< 300
mOsm/kg

MTB S2CK

p. 307

Patient presents with fever and acute LLQ abdominal


pain. Blood cultures grow E. coli and Candida albicans.
Patient started on vancomycin, metronidazole,
gentamicin, and amphotericin. CT scan reveals
diverticulitis. After 36 hours, her creatinine rises
dramatically.
Which of the following is most likely the cause of her renal
insufficiency?
a. Vancomycin Need 5-10 days exposure
b. Gentamicin Need 5-10 days exposure
c. Contrast media
d. Metronidazole Hepatically excreted
e. Amphotericin Need 5-10 days exposure
MTB S2CK

p. 308

Acute Tubular Necrosis/Definition

Injury to kidneys from ischemia and/or toxins


result in sloughing of tubular cells into urine
Proteinuria isnt significant since protein, not
tubules, spill into urine when glomeruli are
damaged

Acute renal failure and a toxin in history are


your clues to What is the most likely
diagnosis? question for ATN
MTB S2CK

p. 307

74-year-old blind man admitted with obstructive uropathy


and chest pain. History of hypertension and diabetes.
Creatinine drops from 10 mg/dL to 1.2 mg/dL 3 days after
catheter placement. The stress test shows reversible
ischemia.
This is still
What is the most appropriate management? considered

a. Coronary artery calcium score on CT scan experimental


a
b. One to two liters of normal saline hydration prior and
during angiography
Evidence not as strong
c. N-acetylcysteine prior to angiography
d. Mannitol during angiography Doesnt help
e. Furosemide during angiography Loop diuretics are dangero
worsen it
f. Intravenous sodium bicarbonate before and during
angiography Evidence not as strong

MTB S2CK

p. 308

Extra Difficult Question

Extra Difficult Question

A patient with mild renal insufficiency undergoes


angiography and develops 2 mg/dL rise in creatinine
from ATN despite the use of saline hydration before
and after procedure.
What do you expect to find on laboratory testing?

A patient with extremely severe myeloma with a


plasmacytoma is admitted for combination
chemotherapy. Two days later creatinine rises.
What is the most likely cause?

a.
b.
c.
d.

Urine sodium
FENa
Urine specific gravity
8 (low)
>1%
1.035 (high)
58 (high)
>1%
1.005 (low)
5 (very low)
<1%
1.040 (very high)
45 (high)
>1%
1.005 (low)
Spasm of afferent arteriole leads to reabsorption of Na
(and thus water)
very concentrated urine ( specific
gravity)

MTB S2CK

p. 309

a.
b
b.
c.
d.
e.

Cisplatin Takes 5-10 days


Hyperuricemia
Can cause renal insufficiency
Bence-Jones proteinuria but would not get worse with
treatment
Hypercalcemia
Hyperoxaluria Oxalate crystals can be present in
urine in cases of bowel resection

MTB S2CK

p. 309

Extra Difficult Question

Extra Difficult Question

What wouldve prevented this event?


Allopurinol, hydration, and rasburicase
Given prior to chemotherapy to prevent
renal failure from tumor lysis syndrome

Suicidal patient ingests an unknown substance and


develops renal failure 3 days later. Her calcium level
is low and urinalysis shows an abnormality.
What did she take?

MTB S2CK

p. 309

a.
b.
c.
d.
e.
f.

Aspirin Doesnt affect Ca levels or cause crystals


p
Hepatotoxic
Acetaminophen
p
Ethylene glycol
Constrict afferent arteriole
ATN and
Ibuprofen
papillary necrosis
Opiates Can cause FSGS, but not AKI
Methanol Affects retinal inflammation not AKI

MTB S2CK

p. 310

Acute Tubular Necrosis/Toxins

Acute Tubular Necrosis

3 things increase risk of toxic/insult ATN:


Hypoperfusion of kidney
Underlying renal insufficiency
Hypertension
Diabetes
Older age

Summary of Causes:
Slow onset (5 - 10 days)

We lose 1% of renal
function every year past
age of 40.
MTB S2CK

p. 310

Mag

Contrast

Best prevented with saline hydration


N-acetylcysteine and sodium bicarbonate arent
consistently proven as beneficial
MTB S2CK

p. 310

Rhabdomyolysis

Summary of Causes:
Hemoglobin and myoglobin
Hyperuricemia
Ethylene Glycol
Multiple Myeloma
NSAIDs

Etiology?

p. 310

risk for aminoglycoside and cisplatin

Immediate toxicity (24 48 hrs)


media

Acute Tubular Necrosis

MTB S2CK

Drug-related injury

Aminoglycosides, amphotericin, cisplatin,


vancomycin, acyclovir, cyclosporine
Dose dependent

Trauma, prolonged immobility, snake


bites, seizures, and crush injuries

The best initial test?


UA (dipstick AND microscopic analysis)

Characteristic findings?
Blood is positive on dipstick but NO
RBCs are seen on microscopic exam
MTB S2CK

p. 311

Rhabdomyolysis

Rhabdomyolysis

Most specific lab test?

Treat with:
Saline hydration
Mannitol
Bicarbonate

Urine myoglobin

Abnormal lab findings?


Creatine phosphokinase (CPK)
Hyperkalemia
H
k l i
Hyperuricemia
Hyperphosphatemia
Hypocalcemia
MTB S2CK

p. 311

Dont treat hypocalcemia


in rhabdomyolysis if
asymptomatic. In recovery, the
calcium will come back
out of muscles.
MTB S2CK

p. 311

Acute Tubular Necrosis/Treatment


A man comes to the ED after a triathlon followed by
status epilepticus. He takes simvastatin at triple the
recommended dose. His muscles are tender and
urine is dark. IV fluids are started.
What is the next best step in management?
a.
b.
c.
d.
e.

CPK level Would suggest


gg
rhabdo as cause,, but not life
threatening complications.
EKG
Potassium replacement Would be fatal
Urine dipstick Same reason as A
Urine myoglobin Same reason as A

MTB S2CK

p. 311 312

No therapy proven to benefit ATN


Patients should be managed with
Hydration, if theyre volume depleted and
correction of electrolyte abnormalities

Diuretics increase urine output, but dont


change overall outcome
More urine output with
diuretics doesnt mean
renal failure is reversing.
MTB S2CK

p. 312

Acute Tubular Necrosis/Treatment

Acute Tubular Necrosis/Treatment

Answering treatment questions for ATN is


based on recognizing the most common
wrong answers:

When Is Dialysis the Answer?


Dialysis is initiated if theres:
Fluid overload
Encephalopathy
Pericarditis
Metabolic acidosis
A- acidosis
E- electrolytes
Hyperkalemia

Low-dose dopamine
Diuretics
Mannitol
Steroids

All these are ineffective in reversing ATN

I- intoxications
O- overload of
volume
U-uremia

Correct underlying cause


MTB S2CK

p. 312

MTB S2CK

p. 312

Acute Tubular Necrosis/Rhabdomyolysis

Hepatorenal Syndrome

Patient develops ATN from gentamicin. Shes


vigorously hydrated and treated with high doses of
diuretic, low-dose dopamine, and calcium acetate as
a phosphate binder. Urine output increases, but she
still progresses to end-stage renal failure. She also
becomes deaf.
What caused her hearing loss?

Renal failure developing secondary to liver


disease
Kidneys normal
Look for:

a.
b.
c.
d.
e.

Hydrochlorothiazide No otoxicity
Dopamine No ototoxicity
Furosemide
Chlorthalidone No ototoxicity
Calcium acetate No ototoxicity

MTB S2CK

Severe liver disease (cirrhosis)


New-onset renal failure with no other explanation
Very low urine sodium (> 10 15 mEq/dL)
FeNa < 1%
Elevated BUN:creatinine ratio (> 20:1)

Treat with:
Albumin, midodrine, octreotide

p. 313

MTB S2CK

Atheroemboli/Etiology

p. 313

Atheroemboli/Etiology
Livedo reticularis

Can occur during catheter procedures:


Cardiac catheterization, angiogram

Emboli can go to:


Kidney, leading to AKI
Eye
Cutaneous vessles
livedo reticularis

Lab findings:
Eosinophilia/Eosinophiluria,

compliment,

ESR

Most accurate test:


Biopsy

Peripheral pulses
are normal in
atheroemboli
atheroemboli.
Theyre too small to
occlude vessels
such as the radial or
brachial artery.

cholesterol crystals
Source: Farshad Bagheri, MD

MTB S2CK

p. 313 314

MTB S2CK

p. 314

Acute (Allergic) Interstitial Nephritis

Acute (Allergic) Interstitial Nephritis

Antibodies and eosinophils attack cells lining


tubules
Reaction to drugs (70%), infection, and
autoimmune disorders

Presentation: What Is the Most Likely Diagnosis?


Look for acute renal failure (rising BUN and
creatinine) with:
Fever (80%)
Rash (50%)
Arthralgias
Eosinophilia and eosinophiluria (80%)
BUN/Cr ratio < 20:1
WBC and RBC in urine
What is the most accurate test?
Wright/Hansel stain to identify eosinophiluria

Penicillins and cephalosporins


Sulfa drugs (including diuretics like furosemide and
thiazides which are sulfa derivatives)
thiazides,
Phenytoin
The medications that cause AIN
Rifampin
are the same as those that cause:
Quinolones
Drug allergy and rash
Allopurinol
Stevens-Johnson syndrome
PPI
Toxic epidermal necrolysis
Hemolysis
MTB S2CK

p. 314 315

MTB S2CK

p. 315 316

Acute (Allergic) Interstitial Nephritis

Treatment
AIN usually resolves spontaneously with
stopping drugs or controlling infection
Severe disease is managed with dialysis,
which may be temporary
When creatinine continues to rise after
stopping the drug, giving glucocorticoids
(prednisone, hydrocortisone,
methylprednisolone) is the answer

MTB S2CK

p. 316

Analgesic Nephropathy

Analgesic nephropathy presents with:


ATN from direct toxicity to tubules
AIN
Membranous glomerulonephritis
Papillary necrosis

MTB S2CK

p. 316

Analgesic Nephropathy

Papillary Necrosis

Presentation
Vascular insufficiency of kidney from inhibiting
prostaglandins

Sloughing off renal papillae


Etiology
NSAIDs or sudden vascular insufficiency

Prostaglandins dilate the afferent arteriole


NSAIDs constrict the afferent arteriole and
decrease renal perfusion
p
Asymptomatic in healthy patients
When patients are older and have underlying
renal insufficiency from diabetes and/or
hypertension, NSAIDs can tip them over into
clinically apparent renal insufficiency due to
direct toxicity to tubules
MTB S2CK

p. 316

Papillary Necrosis

Clinical presentation:
Fever, hematuria, and sudden onset flank
pain
Looks like pyelonephritis

Best initial test:


UA

Most accurate test:


CT scan

Death of cells in papillae and their dropping


off the internal structure of kidney

Occurs with underlying kidney damage:


Sickle Cell
DM
Urinary obstruction
Chronic pyelonephritis
MTB S2CK

p. 316

Differences between Pyelonephritis


and Papillary Necrosis
Pyelonephritis

Papillary necrosis

Onset
Symptoms

Few days
Dysuria

Urine culture
CT scan

Positive
Diffuse
swollen kidney
Antibiotics, such as
ampicillin/gentamicin
or fluoroquinolones

Few hours
Necrotic material
in urine
Negative
Bumpy contour
of kidney interior
No treatment

Treatment

Treatment:
No specific therapy
MTB S2CK

p. 317

MTB S2CK

p. 317

Glomerular Diseases
Goodpasture Syndrome
IgA Nephropathy
Postinfectious Glomerulonephritis
Alport Syndrome
Polyarteritis Nodosa
Lupus Nephritis
Amyloidosis
Nephrotic Syndrome

Tubular Disease

Glomerular Diseases

Acute
Caused by toxins
Not nephrotic
No biopsy
No steroids
or immunosuppressives

MTB S2CK

Chronic
Immune mediated
All nephrotic
Need biopsy
Often steroids,
cyclophosphamide,
mycophenolate

p. 318

Glomerular Diseases/Diagnostic Tests

Goodpasture Syndrome

All forms of glomerulonephritis have:


UA with hematuria
Dysmorphic red cells (deformed as they
squeeze through an abnormal glomerulus)
Red cell casts
Urine sodium and FENa are low
Proteinuria
The amount of

Lung + kidney involvement


No upper respiratory tract involvement
Diagnosis
Best initial test:

proteinuria is the
main difference between
glomerulonephritis and
nephrotic syndrome.
MTB S2CK

Antiglomerular basement membrane


antibodies
Kidney biopsy in

Most accurate test:


Lung or kidney biopsy

MTB S2CK

p. 318

Goodpasture Syndrome

Goodpasture syndrome
shows linear deposits.

p. 319

Goodpasture Syndrome

Diagnosis
Anemia from hemoptysis
CXR abnormal, but not diagnostic
Treatment
Plasmapheresis
Steroids
Cyclophosphamide

MTB S2CK

p. 319

Commons.wikimedia. Used with permission

MTB S2CK

p. 319

10

IgA nephropathy (Berger disease)

Postinfectious Glomerulonephritis

MCC of acute glomerulonephritis in U.S.


Presentation: Gross hematuria 1 to 2 days
after an upper respiratory tract infection
IgA levels in 50%
Most accurate test = kidneyy biopsy
p y
No treatment: will resolve, progress to
ESRD
Steroids and ACE-inhibitors may help severe
proteinuria

Most common infection is Streptococcus


Follows throat infection or skin infection
(impetigo) by 1 to 3 weeks
Presentation

MTB S2CK

p. 319 320

Cola-colored urine
Edema (periorbital)
HTN
Oliguria

MTB S2CK

p. 320

Postinfectious Glomerulonephritis

Diagnostic Tests
1st - UA
glomerulonephritis
nd
2 - Antistreptolysin O (ASO) titers and
anti-DNAse antibody titers
Most
M t accurate
t - Biopsy
Bi

Commons.wikimedia, James Heilman, MD.


Used with permission

Postinfectious Glomerulonephritis

MTB S2CK

p. 320

Postinfectious Glomerulonephritis

Treatment
Management of strep infection does not
reverse glomerulonephritis
Use supportive therapies such as:
Antibiotics
A tibi ti
Diuretics to control fluid overload
<5% of those with PSGN
will progress.
MTB S2CK

p. 320

Commons.wikimedia. Used with permission

MTB S2CK

p. 320

11

Alport Syndrome

Polyarteritis Nodosa

Congenital defect of collagen


Results in glomerular disease combined
with:

Definition
Systemic vasculitis of small and
medium-sized arteries
Spares lungs
Associated
A
i t d with
ith hepatitis
h
titi B

Sensorineural hearing loss


Visual disturbance from loss of collagen
fibers that hold lens of eye in place

PAN is nonspecific.
Theres no single finding
that allows you to
answer the most likely
diagnosis question.

Theres no specific therapy to reverse


this defect of type IV collagen
MTB S2CK

p. 320

MTB S2CK

p. 321

Polyarteritis Nodosa/Presentation

Polyarteritis Nodosa/Diagnostic Tests

The most common organ systems involved are:


GI

Blood tests will show:


Anemia and leukocytosis
ESR and C-reactive protein
ANCA: Not present in most cases
ANA and rheumatoid factor: Sometimes present in
low titer
Angiography
Renal, mesenteric, or hepatic artery showing
aneurysmal dilation
Biopsy
Most accurate if at symptomatic site

Abdominal pain, bleeding, nausea, and vomiting


occur

Neuro
Peripheral neuropathy or mononeuritis
mononeuritis multiplex.
multiplex.

Skin
Petechiae, purpura, ulcers, livedo reticularis

Cardiac
Stroke or MI, particularly in young person

MTB S2CK

p. 321

MTB S2CK

p. 321

Polyarteritis Nodosa

Lupus Nephritis

Treatment
Prednisone and cyclophosphamide
mortality
Treat hepatitis B when its found

SLE can give any degree of renal involvement


Normal
Mild, asymptomatic proteinuria
Membranous glomerulonephritis
Glomerulosclerosis
scars kidneys without
inflammation leading to ESRD requiring dialysis
Diagnosis
Biopsy (tells severity)
Treatment
Steroids, cyclophosphamide, mycophenolyate

MTB S2CK

p. 321 322
MTB S2CK

p. 322

12

Amyloidosis

Amyloidosis

Amyloid is an abnormal protein produced in


association with:
Myeloma
Chronic inflammatory diseases
Rheumatoid arthritis
Inflammatory bowel disease
Chronic infections
Amyloid, HIV

Most accurate test = biopsy


Apple-green birefringence with Congo
red staining
Best treatment = control underlying
disease
2nd line Treatment = melphalan and
prednisone

nephropathy,
polycystic kidneys, and
diabetes give large
kidneys on sonogram
and CT scan.
MTB S2CK

p. 322

MTB S2CK

Amyloidosis

p. 322

Nephrotic Syndrome/Definition
Measure of the severity of proteinuria in association
with any form of glomerular disease
> 3.5 g/24 hrs

Liver can no longer increase the production of


albumin to compensate for urinary losses
Massive proteinuria leads to:
Edema (periorbital)
Hyperlipidemia
Thrombosis:
From urinary loss of natural anticoagulants protein C,
protein S, and antithrombin
MTB S2CK

p. 322

Katsumi M. Miyai, MD, PhD. Regents of the


University of California. Used with permission

MTB S2CK

p. 322

Nephrotic Syndrome/Etiology

Nephrotic Syndrome/Diagnostic Tests

MCC = diabetes and hypertension


Any of the glomerular diseases just described
Other associations are:
Cancer (solid organ): membranous
Children: minimal change disease
Injection drug use and AIDS: focal-segmental
NSAIDs: minimal change disease and
membranous
SLE: Any of them

Best initial test = urinalysis


Next best test = albumin/creatinine ratio or
24-hour urine protein collection
Most accurate test = biopsy:

MTB S2CK

p. 323

Focal-segmental
Focal segmental
Membranous
Membranoproliferative
Minimal change
Mesangial
MTB S2CK

p. 323

13

Nephrotic Syndrome/Diagnostic Tests

Nephrotic Syndrome/Treatment

By definition, nephrotic syndrome is:


Hyperproteinuria (> 3.5 g/24 hours)
Hypoproteinemia
Hyperlipidemia
Edema

Treatment
Best 1st therapy = glucocorticoids
2nd line = cyclophosphamide
ACE inhibitors or ARBs to control proteinuria
Edema managed with salt restriction and
diuretics
Hyperlipidemia is managed with statins as
you would any form of hyperlipidemia

MTB S2CK

p. 324

MTB S2CK

p. 324

End Stage Renal Disease/Definition

End Stage Renal Disease


Etiology
Presentation
Manifestations
Treatment
Kidney Transplantation

Not defined as a particular BUN or creatinine


Loss of renal function
symptoms and
laboratory abnormalities known as uremia
Uremia is defined as the presence of:
Metabolic acidosis
Fluid overload
Encephalopathy
Hyperkalemia
Pericarditis

Peritoneal dialysis and


hemodialysis are equally
effective at removing
wastes from the body.

Acute indications for dialysis


MTB S2CK

p. 324 325

End Stage Renal Disease

End Stage Renal Disease

Manifestations of Renal Failure


Anemia: Normocytic, normochromic

Manifestations of Renal Failure (contd)


Bleeding:

Loss of EPO

No degranulation

Hypocalcemia: Cant absorb Ca from GI tract


25-hydroxy-vitamin D

1,25-dihydroxy-vitamin D

Osteodystophy: Demineralized bones, soft/weak


Secondary hyperparathyroidism

Hyperphosphatemia: Cant excrete


PTH

release of PO4 from bones

Hypermagnesemia: Cant excrete


MTB S2CK

p. 325

cant aggregate

Infection:
No degranulation

cant fight infection

Accelerated atherosclerosis:
Abnormal WBCs cannot clear lipid accumulation
from arteries

Pruritus:
Unclear reasoning; urea accumulating in skin
causes itching
MTB S2CK

p. 325

14

End Stage Renal Disease

Treatment of ESRD Manifestations

Manifestations of Renal Failure (contd)


Endocrinopathy:

Manifestation

Treatment

Anemia

Erythropoietin replacement and


iron supplementation
Replace vitamin D and calcium
DDAVP increases platelet function
Dialysis and ultraviolet light
Oral binders, see Hyperphos Rx
Restriction of high Mg foods, laxatives,
and antacids
Dialysis
Dialysis, estrogen & testosterone
replacement

Women are anovulatory


Men have testosterone
erectile
dysfunction
Hyperinsulinemia and insulin resistance
Either hyper- or hypoglycemic

MTB S2CK

Hypocalcemia
Bleeding
Pruritus
Hyperphosphatemia
Hypermagnesemia
Atherosclerosis
Endocrinopathy

p. 325

MTB S2CK

p. 325

End Stage Renal Disease

End Stage Renal Disease

Treatment of Hyperphosphatemia
Medications:
Calcium acetate
Calcium carbonate
Use sevelamer and
Sevelamer
lanthanum when calcium
level is high.
Lanthanum

Kidney Transplantation
Only 50% of ESRD patients will be
suitable for transplantation
The donor doesnt have to be alive or
related although these are both better
related,
HLA-identical, related donor
kidneys last 24 years on average.

Never use aluminum containing


phosphate binders. Aluminum
causes dementia.
MTB S2CK

p. 326

MTB S2CK

End Stage Renal Disease

Thrombotic thromobocytopenic purpura


and Hemolytic Syndrome

Survival by Method
1 year

3 years

5 years

Living, related donor

95%

88%

72%

Deceased donor

90%

78%

58%

Dialysis alone

Variable

Variable

30 40%

Diabetics on dialysis

Variable

Variable

20%

MTB S2CK

p. 326

p. 326

Different variants, same disease


TTP is associated with HIV, cancer, and
drugs (e.g., cyclosporine, ticlopidine, and
clopidogrel)
HUS is MC in children and the most
frequently tested association is E. coli
0157:H7 and Shigella

MTB S2CK

p. 326

15

Thrombotic thromobocytopenic purpura


and Hemolytic Syndrome

Thrombotic thromobocytopenic purpura


and Hemolytic Syndrome

Both TTP and HUS are associated with:


Intravascular hemolysis
Renal insufficiency
Thrombocytopenia

TTP also
l is
i associated
i d with:
ih
Neurological symptoms
Fever

The hemolysis is visible on smear with


schistocytes, helmet cells, and fragmented
red cells
MTB S2CK

p. 326

MTB S2CK

p. 327

Source: Abhay Vakil, MD.

Thrombotic thromobocytopenic purpura and


Hemolytic Syndrome
Most cases of HUS from E. coli will resolve
spontaneously
Plasmapheresis is generally urgent in TTP
Severe HUS also needs urgent plasmapheresis
If plasmapheresis is not one of the choices
choices, use
infusions of fresh frozen plasma (FFP)
Steroids dont help

Cystic Disease
Simple versus Complex Cysts
Polycystic
y y
Kidneyy Disease

Platelet transfusion is never


the correct choice for TTP or
HUS
MTB S2CK

p. 327

Benign (Simple) vs. Malignant (Complex) Cysts


Simple Cyst

Complex Cysts

Echogenicity

Echo free

Mixed echogenicity

Walls

Smooth, thin

Irregular, thick

Demarcation

Sharp

Transmission

Good through
to back

Lower density
on back wall
Debris in cyst

MTB S2CK

p. 326

Polycystic Kidney Disease

Polycystic kidney disease (PCKD)


presents with:
Pain
Hematuria
Stones
No therapy exists to
Infection
prevent or reverse cysts
of any type.
HTN

MTB S2CK

p. 328

16

What is the MCC of death from PCKD?


a.
b.
c.
d
d.
e.

Sodium & Potassium


Disorders

Only 10-15% have cerebral

Intracerebral hemorrhage aneurysms


Stones Can occur but do not lead to death
Infection Pyelo is more common but rarely fatal
potential. Doesnt
Doesn t progress
Malignancy No malignant potential
Renal failure to RCC

MTB S2CK

Hypernatremia
Hyponatremia
Hyperkalemia
Hypokalemia

p. 328

Hypernatremia/Etiology

Hypernatremia/Etiology

Loss of free water


Examples are:

Diabetes Insipidus (DI)


High urinary volume water loss
Insufficient or ineffective ADH
Symptoms

Sweating
Burns
Fever
F
Pneumonia (insensible losses from
hyperventilation)
Diarrhea
Diuretics
MTB S2CK

p. 328

p. 328

Hypernatremia/Diagnostic Tests

Central DI

Nephrogenic DI

Definition

Loss of ADH
production

Loss of ADH
effect

Etiology

CNS disorders:
Stroke
Tumor
Trauma
Hypoxia
Infection

Lithium
Demeclocycline
Chronic kidney
disease
Hypokalemia
Hypercalcemia

p. 328 329

Sodium disorders
cause CNS problems

If uncorrected, severe hypernatremia causes


coma and irreversible brain damage
MTB S2CK

Diabetes Insipidus (DI)

MTB S2CK

Confusion
Disorientation
Lethargy
Seizures

To distinguish DI from other causes of


hypernatremia look for:
Increased urine volume
Decreased urine osmolality
Decreased
D
d urine
i sodium
di
Increased urine volume
despite dehydration and
hyperosmolality of
blood suggests DI
MTB S2CK

p. 329

17

Diagnosing DI

Central DI vs. Nephrogenic DI


Water deprivation test

Best initial test?


Continued
high volume
dilute urine

urine volume
osmolality

Diabetes
Insipidus

Psychogenic
Polydipsia

Next best test?

ADH administration test


urine volume
osmolality

Central DI
MTB S2CK

Continued high
volume dilute urine

Nephrogenic DI

**Most
accurate test
= ADH level
NDI
CDI

p. 329

CDI

NDI

Polyuria and nocturia

Yes

Yes

Urine osmolality and sodium

Low

Low

Positive water deprivation test

Yes

Yes

Response to ADH

Yes

No

ADH level

Low

High

MTB S2CK

p. 329

Hypernatremia/Treatment

Hypernatremia/Treatment

Fluid loss:

Complications of Therapy
Cerebral edema: sodium levels brought
down too rapidly
Cerebral edema presents with
worsening confusion and seizures

Correct underlying cause of fluid loss

CDI:
Replace ADH (vasopressin also known as
DDAVP)

NDI:
Correct potassium and calcium
Stop lithium or demeclocycline
Give hydrochlorothiazide or NSAIDs for those
still having NDI despite these interventions
MTB S2CK

p. 330

Hyponatremia/Etiology

MTB S2CK

p. 330

Hyponatremia/Etiology

Hypervolemia
MCC of hyponatremia with
hypervolemic state are:

Intravascular volume depletion


CHF, nephrotic syndrome, cirrhosis

CHF
Nephrotic syndrome
Cirrhosis

Increased ADH levels


Stimulated by baroreceptors
in atria and carotids

Free water reabsorption


Sodium concentration drops
MTB S2CK

p. 330

MTB S2CK

p. 330

18

Hyponatremia/Etiology

Hyponatremia/Etiology

Hypovolemia
Sweating
Burns
Fever
Pneumonia
Diarrhea
Diuretics

Addisons disease
Loss of adrenal function
loss of
aldosterone
Aldosterone causes Na+ reabsorption
If the body
y loses aldosterone,, it loses Na+

MTB S2CK

p. 330

MTB S2CK

p. 331

Hyponatremia/Etiology

Hyponatremia/Presentation

Euvolemic Hyponatremia
MCCs:

Hyponatremia presents entirely with CNS


symptoms:
Confusion
Lethargy
Disorientation
Seizures
Symptoms of hyponatremia
Coma

Pseudohyponatremia (hyperglycemia)
Psychogenic polydipsia
Hypothyroidism
H
th idi
Syndrome of inappropriate ADH (SIADH)
release

MTB S2CK

p. 331

Hyponatremia/Diagnostic Tests

SIADH
High urine osmolality
High urine sodium
Low uric acid level and BUN
Most accurate test is a high ADH level

MTB S2CK

p. 332

are dependent on how fast


it occurs.

MTB S2CK

p. 331

Response to Hyponatremia
Normal levels

SIADH

Urine
osmolality

Low
(< 100 mOsm/kg)

High

Urine
sodium

Low (< 20 mEq/L)

High (> 40 mEq/L)

MTB S2CK

p. 332

19

Hyponatremia/Treatment

The treatment answer isnt


based on sodium level;; its
based on the symptoms.

MTB S2CK

p. 332

Hyponatremia/Treatment

Clinical Manifestations of Hyponatremia by Severity


Degree of
hyponatremia

Specific
manifestation

Management

Mild
hyponatremia

No symptoms

Restrict fluids

Moderate

Minimal confusion

Saline and loop


diuretics

Severe

Lethargy, seizures,
coma

Hypertonic saline,
conivaptan,
tolvaptan

MTB S2CK

p. 332

Central Pontine Myelinolysis

Complications of Treatment
Goal: increase in Na is 0.5 to 1 mEq/hour (12
to 24 mEq/day)
If the sodium level is brought up to normal
too rapidly
central pontine myelinolysis
(osmotic demyelinization occurs)

MTB S2CK

p. 332

Hyperkalemia

MTB S2CK

p. 332

Hyperkalemia/Etiology

Typically defined as [K+] > 5mEq/L

Severe hyperkalemia can


stop the heart in seconds if
the level is high enough.

K > 5mEq/L

Pseudohyperkalemia

Hemolysis
Leukocytosis
Thrombocytosis
Repeat the blood
sample

MTB S2CK

p. 332

MTB S2CK

Decreased
Excretion

Renal failure
Acute or chronic
Low aldosterone state
ACE
inhibitors/ARBs
RTA IV
Drugs
Addison disease

Increased
Release from
Tissues

Cell lysis
Low insulin
Acidosis
Drugs
Beta blockers
Digoxin
Heparin

p. 333

20

Hyperkalemia/Presentation

Hyperkalemia/Diagnostic Tests

Potassium disorders interfere with muscle


contraction and cardiac conductance
Look for:

If hyperkalemia is suspected
Most urgent test:

Weakness
Paralysis when severe
Ileus (paralyzes gut muscles)
Cardiac rhythm disorders

EKG

The EKG in severe hyperkalemia shows:


Peaked T waves
Wide QRS
PR interval prolongation

Hyperkalemia does not


cause seizures.
MTB S2CK

p. 333

MTB S2CK

Hyperkalemia/Treatment

Hyperkalemia/Treatment

K > 5mEq/L

Pseudohyperkalemia?

When theres hyperkalemia and an


abnormal EKG, the most appropriate
next step is clearly calcium chloride or
gluconate.

EKG
Changes?
Yes

No treatment needed

MTB S2CK

No

Calcium chloride or
calcium gluconate
Insulin and glucose,
inhaled beta agonist
Give bicarbonate if
acidosis is the cause
Consider hemodialysis

MTB S2CK

Hypokalemia/Etiology

MTB S2CK

Shift into cells

Alkalosis
Insulin

agonists
(stimulate Na/K
ATPase
pump)

p. 334 335

p. 334

Hypokalemia/Presentation

K < 3.5 mEq/L

Very Rare
Kidney can
adjust excretion

Sodium = CNS symptoms


Hyperkalemia = muscular
and cardiac symptoms

Kayexalate
Loop diuretics

p. 334

Decreased
Intake

p. 333 334

Renal Losses

aldosterone
Conns

Volume
Cushings
Bartters
Licorice
Hypomagnesemia
RTA I and II

GI
Losses

Vomiting
Diarrhea
Laxatives

Hypokalemia leads to problems with


muscular contraction and cardiac
conduction
Potassium is essential for proper
neuromuscular contraction
Hypokalemia presents with:
Weakness
Paralysis
Loss of reflexes
MTB S2CK

Muscular abnormalities may


be so severe as to cause
rhabdomyolysis.

p. 335

21

Hypokalemia/Presentation

Hypokalemia/Treatment

EKG findings
U waves are the most characteristic finding
of hypokalemia
Other findings are ventricular ectopy (PVCs),
flattened T waves, and ST depression

No maximum rate of oral potassium


replacement
GI tract cannot absorb potassium faster than the
kidneys can excrete it

IV potassium replacement, however, can cause


a fatal arrhythmia if its
it s done too fast
You must allow time for potassium to equilibrate
into cells

Hypokalemia does not


cause seizures.

MTB S2CK

p. 335

MTB S2CK

Patient is admitted with vomiting and diarrhea from


gastroenteritis. His volume status is corrected with IV
fluids and diarrhea resolves. His pH is 7.40 and
serum bicarbonate has normalized. Despite vigorous
oral and IV replacement, his potassium level fails to
rise.
What should you do?
a.
b.
c.
d.
e.

Consult nephrology Dont consult except for procedures!


Magnesium level
Parathyroid hormone level Mg needed for PTH release
Intracellular pH level Not MCC
24-hour urine potassium level Doesnt tell you

MTB S2CK

IV potassium replacement
must be very slow.

p. 335 336

underlying cause or
change tx

p. 335

Woman with ESRD and G6PD deficiency skips dialysis


for a few weeks. She experiences a crush injury during a
MVA. She is taking dapsone and has recently eaten fava
beans.
What is the most urgent step?
a.
b
b.
c.
d.
e.
f.
g.
h.

Initiate dialysis
Must look for EKG changes 1st
EKG
Bicarbonate administration
Take 15-20min to work
Insulin administration
Kayexalate Take hours to work
Urine dipstick
Doesnt address life threatening
CPK levels
complications
Urine myoglobin

MTB S2CK

p. 336

Non anion gap metabolic acidosis (NAGMA)

Acid Base Disturbances


Renal Tubular Acidosis
Urine Anion Gap
p
Metabolic Acidosis
Metabolic Alkalosis
Respiratory Acidosis and Alkalosis

The anion gap is:


Na+ - (Cl- + HCO3- )
Normal anion gap is 6 12
The difference in + & is due to negative
charges
g on albumin
Elevated gaps means increased acids
(negatively changed) present

MTB S2CK

p. 336

22

Non anion gap metabolic acidosis

Renal Tubular Acidosis (RTA)

What causes NAGMA?

Distal RTA (Type I)


Pathology in distal tubule
Inability to generate enough bicarbonate

RTA
Diarrhea

Why is the gap normal in these?


They
y have elevated Cl ((hyperchloremic
yp
metabolic acidosis)

The anion gap increases from ingested


substances (e.g., ethylene glycol or
methanol), or organic acids (e.g., lactate) that
are anionic and drives chloride levels down
MTB S2CK

Inadequate bicarb
acid cannot be excreted
p
pH of urine is high,
g ,p
pH of blood is low

Caused by factors damaging distal tubule


Amphotericin
SLE or Sjgren syndrome

MTB S2CK

p. 336 337

p. 337

Renal Tubular Acidosis

Distal RTA (Type I)/Diagnostic Tests

Distal RTA (Type I)


Symptoms to look out for?

Best initial test

Calcium oxalate kidney stones


Due to alkaline urine

Distal RTA calcifies


kidney parenchyma
(nephrocalcinosis)

MTB S2CK

No acid into the


tubule makes the
urine basic

p. 337

Urine pH (> 5.5)

Most accurate test


Infuse ammonium chloride (acid)
Normal person
excretes acid
urine pH
decreases
Distal RTA
cannot excrete acid
urine pH
remains high

Treatment
Replace HCO3
MTB S2CK

p. 337

Renal Tubular Acidosis

RTA Type II/Diagnostic Tests

Proximal RTA (Type II)


Pathology in proximal tubule
Inability to absorb filtered bicarb
Caused by factors damaging proximal tubule:

Proximal RTA (Type II)


Urine pH: Low (< 5.5)

Amyloidosis
Myeloma
Fanconi syndrome
Acetazolamide
Heavy metals
MTB S2CK

p. 337

Because distal tubule absorbs bicarb normally


If you give bicarb: urine pH increases

Osteomalacia
Chronic metabolic acidosis leaches calcium
out of the bones and become soft

MTB S2CK

p. 337

23

RTA Type II/Treatment

RTA Type IV/Definition/Diagnostic Tests

Treatment:
MASSIVE doses of bicarbonate

Hyporeninemic, Hypoaldosteronism (Type IV


RTA)
Pathology in distal nephron
Problem is decreased amount or effect of
aldosterone

Because proximal tubule doesnt absorb


it normally

Thiazide diuretics
Cause volume depletion
reabsorption

MTB S2CK

bicarb

Both proximal and distal


RTA are hypokalemic.
Potassium is lost in the
urine.

p. 338

RTA Type IV/Treatment

Fludrocortisone is the
steroid with the highest
mineralocorticoid or
aldosterone-like effect.

p. 338

Proximal (Type II) Distal (Type I)

Type IV

Variable

High > 5.5

< 5.5

Blood potassium Low


level

Low

High

Nephrolithiasis

No

Yes

No

Diagnostic test

Administer
bicarbonate

Administer acid Urine salt


loss

Treatment

Thiazides

Bicarbonate

MTB S2CK

p. 338

Urine Anion Gap/Definition

Fludrocortisone

p. 338

Metabolic Acidosis

Normal anion gap (612):

NAGMA

RTA and diarrhea

Elevated anion gap (>12):

UAG

RTA

MTB S2CK

Urine pH

To promote Na+ reabsorption, K+ and H+


secretion

Positive

Caused by diabetes
Best test
High urine Na even after Na-restricted diet
Hyperkalemia

Types of Renal Tubular Acidosis (RTA)

Treatment:
Fludrocortisone

MTB S2CK

Loss of Na+, retention of K+ and H+

Negative

The anion gap is increased if there are


unmeasured anions driving bicarbonate
level down
Respiratory alkalosis from
hyperventilation
compensates for all forms
of metabolic acidosis.

Diarrhea

MTB S2CK

p. 339

24

Causes of Metabolic Acidosis with


an Increased Anion Gap
Cause

Test

Treatment

Lactate

Hypotension or
hypoperfusion

Blood lactate
level

Correct
hypoperfusion

Ketoacids

DKA, starvation

Acetone level

Oxalic acid

Ethylene glycol
overdose

Crystals on UA

MTB S2CK

Causes of Metabolic Acidosis with


an Increased Anion Gap
Cause

Test

Treatment

Formic acid

Methanol
overdose

Inflamed retina

Fomepizole,
dialysis

Insulin & fluids

Uremia

Renal failure

BUN, creatinine Dialysis

Fomepizole,
dialysis

Salicylates

Aspirin overdose Aspirin level

MTB S2CK

p. 339

Metabolic Acidosis

Alkalinize urine

p. 339

You cannot determine the etiology of metabolic


acidosis from the ABG.
Metabolic problems
always show
compensation.

Metabolic Alkalosis
Key laboratory finding?
Elevated serum bicarbonate level
Respiratory compensation?
Decreased respiratory rate
CO2 retention
respiratory acidosis
Etiology
GI loss: vomiting or nasogastric suction
aldosterone: primary, Cushing, ACTH, volume
contraction, licorice
Diuretics
Milk-alkali syndrome
Hypokalemia

Metabolic Alkalosis/Etiology

Respiratory Acidosis and Alkalosis

Arterial Blood Gas in Metabolic Alkalosis


The ABG in metabolic alkalosis will always
have:

Minute ventilation is a more precise


measure of respiratory status than
respiratory rate
Minute ventilation = RR x TV

Arterial Blood Gas in Metabolic Acidosis


The arterial blood gas (ABG) in metabolic acidosis
will always have:
Decreased pH < 7.4
Decreased pCO2 indicating respiratory alkalosis as
compensation
p
Decreased bicarbonate

MTB S2CK

p. 339

Increased pH > 7.40


Increased pCO2 (compensatory respiratory
acidosis)
Increased bicarbonate

MTB S2CK

p. 339 340

Hyperventilation may occur


with a tiny tidal volume.This
does not increase minute
ventilation.

Metabolic derangements
kill patients with cardiac
arrhythmia. They also alter
potassium levels.
MTB S2CK

p. 340

MTB S2CK

p. 340

25

Causes of Respiratory Acidosis and Alkalosis


Respiratory alkalosis

Respiratory acidosis

Decreased pCO2

Increased pCO2

Increased minute ventilation

Decreased minute ventilation

Metabolic acidosis as

Metabolic alkalosis as

Compensation

Compensation

Anemia

COPD/emphysema

Anxiety

Drowning

Pain

Opiate overdose

Fever

Interstitial lung disease

Kyphoscoliosis

Pulmonary emboli

Sleep apnea/morbid obesity

MTB S2CK

Nephrolithiasis
Etiology
Treatment
Long term Management
Metabolic Acidosis and Stone Formation

1 antitrypsin deficiency

p. 340

Nephrolithiasis/Pearls

Most common stone?


Calcium oxalate

Forms more frequently in?


Alkaline urine

Most
M t common risk
i k ffactor?
t ?
Overexcretion of calcium in urine

MTB S2CK

p. 340

46-year-old man comes to the ED with excruciating


left flank pain radiating to groin. He has some blood in
urine.
What is the next step?
a.
b.
c.
d.
e.

Ketorolac
X-ray
Sonography Provide pain relief before diagnostic tests
Urinalysis
Serum calcium level

MTB S2CK

p. 341

Nephrolithiasis/Treatment
What is the most accurate diagnostic test for
nephrolithiasis?
a.
b.
c.
d.
e.

CT scan
X-ray 10-20% false negative. Misses uric acid stones
Sonography Less accurate
Urinalysis
y
May
ay show
s o hematuria,
e a u a, not
o spec
specific
c to
o cause
Intravenous pyelogram Requires contrast, takes hours

The best initial therapy for acute renal


colic is with:
Analgesics
Hydration
Imaging
I
i
CT noncontrast
t t
Cystine stones are
managed with surgical
removal, alkalinizing urine.

Uric acid stones are not


detectable on X-ray, but
visualized on CT.
MTB S2CK

p. 341

MTB S2CK

p. 341

26

Nephrolithiasis/Treatment

Etiology of the stone determined with:


Stone analysis
Serum calcium, sodium, uric acid, PTH,
magnesium, and phosphate levels
24-hour
24 h
urine
i ffor volume,
l
calcium,
l i
oxalate, citrate, cystine, pH, uric acid,
phosphate, and magnesium
Fat malabsorption increases
stone formation.
MTB S2CK

p. 341

Woman with her first episode of renal colic has a 1.8


cm stone in the left renal pelvis. No obstruction.
Normal BUN and creatinine.
What is the next step in management?
a.
a
b.
c.
d.
e.

Wait for it to pass; hydrate and observe If < 5mm


Lithotripsy
Surgical removal If > 2cm
Hydrochlorothiazide Decreases calcium in urine
Stent placement Only releives hydronephrosis

MTB S2CK

p. 342

Nephrolithiasis

50% with kidney stones will recur over


the next 5 years
UTI gives struvite stones
(magnesium/ammonium/pho
sphate). Remove them
surgically.

MTB S2CK

p. 342

A man with a calcium oxalate stone is managed with


lithotripsy. Stone is destroyed and passes. Urinary
calcium level is increased.
Besides increasing hydration, which is most likely to
benefit this patient?
a.
b.
c.
d.
e.

Calcium restriction Can rate of oxalate stones


Hydrochlorothiazide
Furosemide Increases urinary calcium
Stent placement Only relieves obstruction
Increased dietary oxalate Can rate of oxalate stones

MTB S2CK

p. 342

Nephrolithiasis

Metabolic acidosis removes calcium


from bones and increases stone
formation
In addition, metabolic acidosis
decreases citrate levels
Citrate binds calcium, making it
unavailable for stone formation

MTB S2CK

p. 342

Urinary Incontinence
Stress incontinence

Urge incontinence

Symptoms

Older woman with


painless leakage with
coughing, laughing, or
lifting heavy objects

Sudden pain in bladder


followed immediately by
the overwhelming urge
to urinate

Test

Have patient stand


and cough; observe
for leakage

Pressure measurement
in half full bladder;
manometry

MTB S2CK

p. 343

27

Urinary Incontinence

Treatment

Stress incontinence

Urge incontinence

1. Kegel exercises

1. Bladder training
exercises
2. Local anticholinergic
therapy
1. Oxybutynin
y
y
2. Tolterodine
3. Solifenacin
4. Dariferancin
3. Surgical tightening of
urethra

2. Local estrogen
cream

3. Surgical
tightening of
urethra
MTB S2CK

Hypertension
Definition
Etiology
gy
Presentation
Diagnostic Tests
Treatment
Hypertensive Crisis

p. 343

Hypertension/Definition

Hypertension/Etiology

Systolic pressure > 140 mmHg


Diastolic pressure > 90 mmHg

Most common etiology? (95%)

Special population with lower threshold?


Diabetics

Definition of hypertension?
> 130/80

MTB S2CK

p. 343

Essential hypertension

Common secondary causes?

Renal artery stenosis


Glomerulonephritis
Coarctation of aorta
Acromegaly
Pheochromocytoma
Hyperaldosteronism
Cushing syndrome or any cause of hypercortisolism
including therapeutic use of glucocorticoids
Congenital adrenal hyperplasia
MTB S2CK

p. 343 344

Hypertension/Presentation

Hypertension/Presentation

Typically asymptomatic
Symptoms of end organ damage:

Presentation of Secondary Hypertension


Renal artery stenosis:

Coronary artery disease


Cerebrovascular disease
CHF
Visual disturbance
Renal insufficiency
Peripheral artery disease
MTB S2CK

p. 344

Bruit is auscultated at flank and is


continuous throughout systole and diastole

Glomerulonephritis
p
Coarctation of aorta:
Upper extremity > lower extremity BP

MTB S2CK

p. 344

28

Hypertension/Presentation

Hypertension/Diagnostic Tests

Acromegaly
Pheochromocytoma: Episodic
hypertension with flushing
Hyperaldosteronism: Weakness from
hypokalemia

Other tests to perform in hypertensive


patients
EKG
Urinalysis
Glucose
Gl
measurements
t to
t exclude
l d
concomitant diabetes
Cholesterol screening

MTB S2CK

MTB S2CK

p. 344

p. 344

Hypertension/Treatment

Hypertension/Drug Therapy

Best initial therapy?


Lifestyle modifications:

Best first drug? (all-comers)

Weight loss (*most effective*)


Sodium restriction
Dietary modification
Exercise
Tobacco cessation

Hydrochlorothiazide

If BP > 160/100?
Defines Stage 2 HTN
Start 2 medications immediately

Lifestyle modifications are


tried for 3 to 6 months
before medications are started.
MTB S2CK

p. 344 345

MTB S2CK

p. 345

Hypertension

Hypertension

If diuretics dont control BP, the most


appropriate next step in management is:

Medications for refractory hypertension:


Central-acting alpha agonists

ACE inhibitor
Angiotensin receptor blocker (ARB)
Beta blocker (BB)
Calcium-channel blocker (CCB)

Alpha methyldopa, clonidine

Peripheral-acting alpha antagonists


Prazosin, terazosin, doxazosin

Direct-acting vasodilators
Hydralazine, minoxidil

MTB S2CK

p. 345

MTB S2CK

p. 345

29

Hypertension/Compelling Indications

Hypertensive Crisis

If this is in the history

This is the best initial therapy

Coronary artery disease

BB, ACE, ARB

Diabetes mellitus

ACE, ARB

Benign prostatic

Alpha blockers

Defined as HTN with SYMPTOMS


Confusion
Blurry vision
Dyspnea
Chest
Ch t pain
i

hypertrophy
Depression and asthma

Avoid BBs

Hyperthyroidism

BB first

Osteoporosis

Thiazides

MTB S2CK

p. 345

MTB S2CK

p. 345 346

Hypertensive Crisis

The best initial therapy?


Labetolol or nitroprusside
2nd line agents?
Enalapril
CCBs:
Diltiazem
Verapamil

Esmolol
MTB S2CK

p. 346

30

Stroke
Definition
Etiology
Presentation
Diagnostic Tests
Treatment

Neurology
Conrad Fischer, MD
Associate Professor of Medicine
Touro College of Medicine
New York City

Stroke/Definition

Stroke/Etiology

Sudden onset neurological deficit


Death of brain tissue
3rd most common cause of death in the
USA
Risk
Ri k ffactors
t

Bleeding (15%)
Blockage of flow (85%)

Hypertension
Diabetes
Hyperlipidemia
Tobacco
MTB S2CK

Thrombosis
Embolus
Heart
H t
Atrial fibrillation
Valvular heart disease
DVT via Patent foramen ovale

Carotid stenosis

p. 273

MTB S2CK

Stroke/Presentation

p. 273

Stroke/Presentation

Middle cerebral artery (MCA)


>90% of all strokes
Weakness or sensory loss
Opposite (contralateral) side from stroke

Contralateral homonymous hemianopsia


Eyes look toward the side of lesion
Aphasia
If stroke on same side as speech center
On left in 90%
MTB S2CK

p. 273

Source: commons.wikimedia.org

MTB S2CK

p. 273

Stroke/Presentation

Stroke/Diagnostic Tests

Anterior cerebral artery (ACA)

Best initial test for stroke is...


CT scan of the head
With or without contrast?...
Without contrast
Most accurate test for stroke is...
MRI

Personality/cognitive defects (e.g., confusion)


Urinary incontinence
Leg > arm weakness

Posterior cerebral arteryy (PCA)


(
)
Loss of consciousness
Ipsilateral (same side) sensory loss of face, 9th,
and 10th CNs
Contralateral sensory loss of limbs
Limb ataxia
MTB S2CK

p. 274

MTB S2CK

p. 274

Stroke/Diagnostic Tests

Stroke/Treatment

CT
Done first
Excludes
hemorrhage
Prior to treatment

Nonhemorrhagic
Best initial therapy when LESS than 3 hours since
onset...
Thrombolytics

More than 3 hours since onset...


Aspirin

If the patient is already on aspirin...


ADD dipyridamole
OR
SWITCH to clopidogrel
Source: Mohammad Maruf, MD

MTB S2CK

p. 274

MTB S2CK

p. 274 275

Stroke/Treatment

Stroke/Evaluation of Causes & Treatment

Hemorrhagic
Best initial
treatment...
Nothing

Echocardiogram
Damaged valves?
Surgical replacement
Thrombi?
Heparin followed by warfarin to INR of
2-3

MTB S2CK

p. 275

MTB S2CK

p. 275

Stroke/Evaluation of Causes & Treatment

Stroke/Evaluation of Causes & Treatment

Electrocardiogram
Atrial fibrillation or atrial flutter
Warfarin as long as arrhythmia persists
no P wave
A fib
P wave
NSR
Source: J Heuser
Source: Kjetil Lenes

Stroke/Evaluation of Causes & Treatment

Holter monitor

Stroke/Evaluation of Causes & Treatment

Holter monitor

Detect arrhythmias
If initial EKG is normal: Do Holter monitor
Detect atrial arrhythmias
G t sensitivity
Greater
iti it th
than EKG

MTB S2CK

Source: Macro987

p. 275

Stroke/Evaluation of Causes & Treatment

Carotid duplex ultrasound


Carotid stenosis is a frequent cause of
emboli
Surgery?
Symptomatic
S
t
ti and
d >70%
70% stenosis
t
i
Endarterectomy superior to carotid
angioplasty

Stroke
Patient presents with:
Sudden onset unilateral weakness
Facial droop
+/- Speech deficits

Best Initial Test:


Head CT without contrast
No acute process

Likely ischemic stroke


MTB S2CK

Acute hemorrhage

Likely hemorrhagic stroke

p. 275

Stroke

< 3 hours from


onset of
symptoms
Treatment:
Consider TPA, if
no hx of bleeding

MTB S2CK

Stroke

Likely ischemic stroke

p. 274 275

> 3 hours from


onset of
symptoms

Likely hemorrhagic stroke


Symptoms last
< 24 hours,
then resolve

Treatment:
Aspirin
ADD dipyridamole or
switch to clopidogrel
if already on aspirin

Transient
ischemic
attack

Treatment:
Control blood pressure
Optimal systolic BP is between 140-160 mmHg
If > 170 mmHg,
mmHg use nicardipine,
nicardipine enalaprilat,
enalaprilat or labetalol
Reverse anticoagulation
If patient is on warfarin
fresh frozen plasma, vitamin K
If patient is on heparin
protamine sulfate

Perform further studies


to asses etiology

Stroke

Goals!
Diabetes

Headache

Hemoglobin A1C <7%

Types
Physical
y
Examination
Diagnostic Tests
Treatment
Trigeminal & Postherpetic Neuralgia

Hypertension
BP <140/90 mmHg

LDL
<100 mg/dL if carotid stenosis is the cause

Tobacco smoking
Must stop!
MTB S2CK

p. 276

Headache

Headache/Tension Headache

Types
Tension
Migraine
Cluster
Giant cell (temporal) arteritis
Pseudotumor cerebri

Constant pressure
Mild to moderate pain, mainly bilateral
Lasts 4-6 hours
Physical exam
Nothing!
Diagnostic tests
None (all normal)
Treatment
NSAIDs and other analgesics

MTB S2CK

p. 276

MTB S2CK

p. 276

Headache/Migraine

Headache/Migraine

Visual disturbance
Photophobia aura
May be related to:
Food/menses
Precipitated by emotions
Ph i l exam
Physical
Rare cases: aphasia, numbness, dysarthria,
weakness
Diagnostic tests
All normal
Scan head the first time, then stop

Treatment
Abortive

MTB S2CK

p. 276

Triptans or ergotamine

Prophylactic (preventive)
> 3 migraines/month
Propranolol

Other preventive medications:


Calcium-channel blockers
Tricyclic antidepressants
SSRIs
Botulinum toxin injections
MTB S2CK

p. 277

Headache/Cluster Headache

Headache/Cluster Headache

Symptoms

Treatment
Abortive
Triptans
Ergotamine
100% oxygen
Preventive
Verapamil
Lithium
Prednisone

Frequent, short duration, high intensity

Men 10X MORE than women!!!


Physical examination
Red,, tearing
g eye
y with rhinorrhea
Horner syndrome occasionally

Diagnostic tests
Scan head first time
No need for subsequent imaging with recurrences
MTB S2CK

p. 276

MTB S2CK

Headache/Giant Cell (Temporal) Arteritis

p. 277

Headache
Headache

Symptoms
Visual disturbance, jaw claudication
Muscle pain, fatigue, and weakness

Physical exam
Visual loss, temporal area tenderness

Diagnostic tests
Elevated ESR
Most accurate test?

Biopsy!

Bilateral band-like
pressure
Lasts 4-6 hours
Normal P/E

Tension
Headache
H
d h
Treatment
NSAIDs
Acetaminophen

Treatment
p. 276

Migraine
Treatment
Avoid triggers
NSAIDs
5-HT1 agonists
3 attacks/month

Prednisone
MTB S2CK

+/- Aura, photophobia


Related to
food/emotions/menses
Rare: aphasia,
numbness, dysarthria

MTB S2CK

p. 389.4

Prophylaxis
Propranolol
Sodium valproate

Episodic pain
Unilateral periorbital
intense pain
Lacrimation
Eye-reddening
Nasal stuffiness
Lid ptosis

Cluster
Headache
Acute Treatment
Sumitriptan
Octreotide
Oxygen
Prophylaxis
Verapamil
Prednisone
Sodium valproate

Headache/Pseudotumor Cerebri

Cranial Nerve 6 Palsy

Associated with
Obesity
Oral contraceptives
Vitamin A toxicity
Mimics brain tumor: nausea & vomiting
Physical exam
Papilledema
Diplopia: 6th CN (abducens) palsy
MTB S2CK

p. 276

Headache/Pseudotumor Cerebri

Headache/Pseudotumor Cerebri

Diagnostic tests
CT or MRI
Normal
Done to exclude intracranial mass
Lumbar puncture
Increased pressure
CSF
Normal

Treatment
Acetazolamide +/- furosemide
Weight loss
Steroids
Repeated lumbar puncture
Ventriculoperitoneal shunt

MTB S2CK

p. 277

MTB S2CK

p. 277

Headache/Trigeminal Neuralgia

Headache/Trigeminal Neuralgia

Idiopathic
5th CN
Severe, overwhelming knife-like facial pain
Precipitated by
Chewing
Touching the face
Pronouncing words in which the tongue
strikes the back of front teeth
No specific diagnostic test

Treatment
Carbamazepine or oxcarbazepine
Baclofen
Lamotrigine
Surgical decompression when failing
medications

MTB S2CK

p. 278

MTB S2CK

p. 278

Headache/Postherpetic Neuralgia

Headache/Postherpetic Neuralgia

Residual pain
following resolution
of herpes zoster
(shingles) vesicular
lesions
Shingles
Shi l iis a painful
i f l
dermatomal rash
that occurs in 15%
with prior varicella
zoster (chickenpox)
infection

Acute Treatment:
Acyclovir, famciclovir, or valacyclovir
reduce the incidence of pain
Steroids do not help

MTB S2CK

p. 278

Shingles rash caused by herpes zoster virus. Source: NIAID

MTB S2CK

p. 278

Headache/Postherpetic Neuralgia

Prevention of Herpes Zoster (Shingles)

Treatment

Zoster vaccine
Everyone > 60
High dose varicella vaccine
Decreases reactivation of varicella into
zoster
t

Tricyclic antidepressants (amitriptyline)


Gabapentin or pregabalin
Carbamazepine
Phenytoin
Topical capsaicin
Antiepileptic medications
Have some beneficial effect in neuropathic pain
(e.g., postherpetic neuralgia or peripheral
neuropathy)

None work in > 50% to 70%


MTB S2CK

p. 278

MTB S2CK

p. 278

Classification of Seizures

Seizures
Classification
Diagnostic Tests
Treatment
Management

Partial
Absence (petit mal)
Generalized (tonic-clonic)
Status
S
a us epilepticus
ep ep cus

MTB S2CK

p. 279

Partial Seizures

Absence Seizures

Focal to one part of the body

Also referred to as petit mal


Consciousness briefly impaired
Patient often remains upright and often
appears normal or stares into space
Absence
Ab
seizures
i
occur more often
ft in
i
children

Limited to arm or leg

Partial seizures can be:


Simple
Intact consciousness

Complex
Loss of consciousness

MTB S2CK

p. 279

MTB S2CK

p. 280

Generalized Seizures

Generalized Seizures/Causes

Also referred to as tonic-clonic


Generalized seizure
Varying phases
Muscular rigidity (tonic)
Followed by jerking of muscles for
several minutes (clonic)

MTB S2CK

p. 280

Hyponatremia or hypernatremia
Hypoxia
Hypoglycemia
Any CNS infection
Encephalitis, meningitis, abscess
Any CNS anatomic abnormality
Trauma, stroke, tumor

MTB S2CK

p. 278

Generalized Seizures/Causes

Seizures/Diagnostic Tests

Electroencephalogram (EEG)
The right answer after the other tests
are done
If CT or MRI are normal
No
N point
i t iin EEG if there
th
is
i a clear
l
metabolic, toxic, or anatomic defect
causing the seizure

Hypocalcemia
Uremia (elevated creatinine)
Hepatic failure
Withdrawal
Alcohol, barbiturate, and
benzodiazepine
Cocaine toxicity
Hypomagnesemia
(rare)
MTB S2CK

p. 278

MTB S2CK

p. 279

Delerium, Stupor, and Coma

Treatment of Status Epilepticus

Altered consciousness
Unresponsiveness to stimuli
From metabolic, toxic, and CNS infections
Also called
Confusion
Difficulty
Diffi lt with
ith arousal
l
Obtundation
When severe enough, a seizure occurs
Confusion is to coma and seizure,
as angina is to myocardial infarction

The only seizure treatment that is clear


Best initial therapy...
Benzodiazepine
Lorazepam
Diazepam intravenously

MTB S2CK

p. 279

MTB S2CK

p. 279

Treatment of Status Epilepticus

Treatment of Status Epilepticus

If seizure persists...
Phenytoin or fosphenytoin
Fosphenytoin = phenytoin efficacy
Fosphenytoin has fewer adverse effects

1. Benzodiazepines did NOT work


2. Fosphenytoin did NOT work
3. Use phenobarbital

Ph
Phenytoin
t i
Hypotension and AV block
Fosphenytoin
No BP or cardiac effect
Can be given more rapidly
MTB S2CK

p. 279

MTB S2CK

p. 279

Treatment of Status Epilepticus

Treatment of Status Epilepticus

What if barbiturates do NOT work?

To summarize...
1. Benzodiazepine
2. Fosphenytoin
3. Phenobarbital
4. General anesthesia

1. Neuromuscular blocking agents allow intubation:


Succinylcholine
Vecuronium
Pancuronium

2. General anesthesia
Midazolam or propofol
Place on ventilator before propofol, which can
stop breathing
MTB S2CK

p. 278

MTB S2CK

p. 280

Epilepsy Treatment/Indications

Choice of Antiepileptic Drugs

Antiepileptic drugs
Not long term for single seizure
When should you start after a single
seizure?
Presentation
P
t ti in
i status
t t epilepticus
il ti
Abnormal EEG
Family history of seizures

MTB S2CK

p. 280

Status epilepticus treatment is clear


Epilepsy long-term treatment is not clear
No medication is clearly superior to the others
Phenytoin, valproic acid, and carbamazepine all
have nearly equal efficacy
Gabapentin,
Gabapentin topiramate,
topiramate lamotrigine,
lamotrigine
oxcarbazepine, or levetiracetam
Ethosuximide is the best therapy for absence
seizures
You cannot be asked to choose between them
based on efficacy
MTB S2CK

p. 280

Choice of Antiepileptic Drugs

Discontinuance of Antiepileptic Medication

If not controlled with single agent, an


alternate medication should be tried
If still not controlled, add a second drug
If multiple medications do not work:
Surgery!

Wait until seizure-free for 2 years

MTB S2CK

p. 280

38-year-old man evaluated for seizures achieves partial


control with second medication. He drives to work daily.
What do you do about his ability to drive?
a. Confiscate his license Rules vary state to state
Cannot
b. Allow him to drive if he is seizure-free for 1 year prevent
c. Allow him to drive as long as his seizure history is
noted
t d on hi
his lilicense Rules vary state to state
d. Recommend that he find an alternate means of
transportation
e. Do not let him leave the office unless he is picked up by
someone; no further driving Cannot incarcerate
f. Allow him to drive as long as he is accompanied
Being accompanied does not prevent seizures
MTB S2CK

Sleep deprivation EEG:


The best way to detect possibility of
recurrence
Elicits abnormal activity with more
sensitivity
This is NOT a 100% sensitive test!
MTB S2CK

p. 281

Subarachnoid
Hemorrhage
Definition/Etiology
Diagnostic Tests
Treatment

p. 281

10

Subarachnoid Hemorrhage

Circle of Willis

Rupture of aneurysm
Usually in Circle of
Willis (anterior)
Aneurysms found in
2% of autopsies
Majority never
rupture
Cause of rupture not
clear
MTB S2CK

p. 281

MTB S2CK

p. 281

Subarachnoid Hemorrhage/Diagnosis

Subarachnoid Hemorrhage/Diagnosis

What is the most likely diagnosis?


Look for...

How does SAH differs from meningitis?


Very sudden in onset
Loss of consciousness

Sudden onset of severe headache with


meningeal irritation (stiff neck, photophobia)
and fever!

Loss of consciousness in 50%


Sudden increase in intracranial pressure
Focal neurological complications occur in 30%

MTB S2CK

p. 281

MTB S2CK

p. 281

Subarachnoid Hemorrhage/Diagnosis

Subarachnoid Hemorrhage/Diagnosis

Best initial test is...


CT without contrast
(95% sensitive)
Most accurate test is...
Lumbar puncture
showing blood

LP necessary only for 5% with falsely


negative CT
CSF in SAH has both increased WBCs and
RBCs
WBC can mimic meningitis
Ratio of WBCs to RBCs will be normal in
SAH

Source: Saba Ansari, MD

MTB S2CK

p. 282

MTB S2CK

p. 282

11

Subarachnoid Hemorrhage/Diagnosis

Subarachnoid Hemorrhage/Diagnosis

WBC count > normal 1:500?

Angiography
Determines site of aneurysm
Guides lesion repair
MRA
Diagnosis based on...
CT and sometimes LP
Only angiography can tell location

Suspect meningitis

Xanthochromia
Yellow CSF
From breakdown red cells in CSF

EKG

May show large or inverted T waves


Suggestive of myocardial ischemia
Cerebral T waves
From excessive sympathetic activity
MTB S2CK

p. 282

Subarachnoid Hemorrhage/Diagnosis

MTB S2CK

p. 282

Subarachnoid Hemorrhage/Treatment

Nothing reverses hemorrhage


Nimodipine (calcium-channel blocker)
prevents subsequent ischemic stroke

MTB S2CK

p. 282

MTB S2CK

p. 283

Subarachnoid Hemorrhage/Treatment

Subarachnoid Hemorrhage/Treatment

Embolization (coiling)
Catheter clogs up site of bleeding
Prevents repeated hemorrhage
Interventional neuroradiologist places
platinum wire
p
Embolization superior to surgical clipping for
survival and complications

Ventriculoperitoneal shunt
SAH associated with hydrocephalus
Shunt only if hydrocephalus develops

MTB S2CK

p. 283

MTB S2CK

p. 283

12

Subarachnoid Hemorrhage/Treatment

Subarachnoid Hemorrhage/Treatment

Seizure prophylaxis
Phenytoin is not routine
If the question asks Which of the
following is most likely to decrease
mortality? ...
mortality?

Consultation is only right when you want


to do a procedure that isnt given as a
choice.

Phenytoin is not the answer

MTB S2CK

p. 283

MTB S2CK

A woman in the ED with a severe headache one day


prior to admission. Temp 103 F, nuchal rigidity &
photophobia. Head CT is normal.
LP CSF WBCs: 1,250 RBCs: 50,000
Whats the next step?

Spine Disorders
Anterior Spinal Artery Infarction
Subacute Combined Degeneration of the Cord
Spinal Trauma
Brown Squard Syndrome
Syringomyelia

Angiography
g g p y For location of SAH, this is not SAH
50 000 RBC should
50,000
h ld only
l
Ceftriaxone and vancomycin
give 50-100 WBCs
Nimodipine Prevent SAH stroke 1,250 WBCs is infection
Embolization Permanent SAH fix
Surgical clipping Worse alternative to embolization
Useless for blood
Repeat CT scan with contrast
Neurosurgical consultation Dont consult unless you
REALLY want a procedure
MTB S2CK p. 283
not listed
a.
b.
c.
d.
e.
f.
g.

Anterior Spinal Artery Infarction

Loss of all function except posterior column


Position and vibratory sensation intact
Flaccid paralysis below the level of infarction
Loss of deep tendon reflexes (DTRs) at level of
infarction
Evolves into spastic paraplegia several weeks later
Loss of pain and temperature
Extensor plantar response

p. 284

Subacute Combined Degeneration


of the Cord

B12 deficiency or neurosyphilis


Position & vibratory sensation lost
Everything else intact

No specific therapy

MTB S2CK

p. 284

MTB S2CK

p. 284

13

Spinal Trauma

Brown Squard Syndrome

Acute limb weakness and/or sensory


disturbance
Below level of injury
Severity in proportion to degree of injury
Sphincter
p
function impaired
p
Loss of DTRs at level of injury
Followed by hyperreflexia below level of
trauma
Treat with glucocorticoids

Unilateral hemisection of spinal cord


Due to injury
Knife cutting half the cord
On physical exam...

MTB S2CK

p. 284

Ipsilateral
Motor
Position
Vibration

Contralateral
Pain
Temperature

No treatment
MTB S2CK

p. 284

Syringomyelia

Syringomyelia

Fluid-filled, dilated central canal of spinal


cord
Widening bubble or cavitation damages
neural fibers passing near center of spine
Caused by tumor or severe trauma to spine

What is the most likely diagnosis?


Look for:
Loss of pain & temperature bilaterally
Across the upper back and both arms
Look for the phrase capelike
capelike distribution
distribution
Loss of reflexes
Muscle atrophy

MTB S2CK

p. 285

MTB S2CK

p. 285

Syringomyelia
The most accurate
test is...
MRI

CNS Abscess & Disease


Brain abscess
Neurocutaneous diseases

The best treatment


is...
Surgical
Removal of tumor
Drain cavity
Source: Mohammad Maruf, MD.

MTB S2CK

p. 285

14

Brain Abscess/Definition & Etiology

Brain Abscess/Presentation

Collection of infected material in brain


parenchyma
Acts as space-occupying lesion
Spreads from contiguous infection
Starts in sinuses,, mastoid air cells,, or otitis
media
Any bacteremia

Headache, nausea, vomiting, fever,


seizures, and focal neurological findings
No way to distinguish brain abscess
from cancer without a biopsy
Cancer gives fever
fever, too

Pneumonia and endocarditis cause bacteremia,


which can lead to a brain abscess

MTB S2CK

p. 285

MTB S2CK

p. 286

Brain Abscess/Diagnostic Tests

Brain Abscess/Diagnostic Tests

Best initial test is...


Head CT or MRI
Most accurate test is...
Brain biopsy

Scan shows a ring or


contrast-enhancing
lesion
Surrounding edema
Mass effect
Cancer and infection
are indistinguishable
based on imaging
study alone
Source: Nishith Patel

MTB S2CK

p. 286

Source: Nishith Patel

MTB S2CK

p. 286

Brain Abscess/Microbiology

Brain Abscess/Treatment

Biopsy is essential
Only biopsy distinguishes abscess from cancer
Only way to know sensitivity of organism
Abscesses can be...
Staphylococci, streptococci, Gram-negative bacilli,
and anaerobes
Frequently mixed (polymicrobial)
Treat for...
6 to 8 weeks intravenously
Followed by 2 to 3 more months orally

Empiric therapy
Penicillin + metronidazole +
ceftriaxone (or cefepime)
Vancomycin (alternative to penicillin)
Use vancomycin if theres been recent
neurosurgery

MTB S2CK

p. 286

MTB S2CK

p. 287

15

Neurocutaneous Diseases

Tuberous Sclerosis

Tuberous sclerosis
Neurofibromatosis (von Recklinghausen
Disease)
Sturge-Weber Syndrome

Neurological abnormalities
Seizures, slowly progressive mental deterioration

Skin
Adenoma sebaceum (reddened facial nodules)
Shagreen patches (leathery plaques on trunk)
Ash leaf (hypopigmented) patches

MTB S2CK

p. 287

Retinal lesions
Cardiac rhabdomyomas
No specific treatment
Control seizures
MTB S2CK

p. 287

Neurofibromatosis
(von Recklinghausen Disease)

Neurofibromatosis
(von Recklinghausen Disease)

Neurofibromas: soft,
flesh-colored lesions
attached to peripheral
nerves
8th CN tumors
Cutaneous
hyperpigmented lesions
(caf au lait spots)
Meningioma and gliomas

No specific treatment
Lesions affecting 8th CN may require
surgical decompression to preserve
hearing

Source: Mohammad Maruf, MD.

MTB S2CK

p. 287

MTB S2CK

p. 287

Sturge Weber Syndrome


Port-wine stain of face
Seizures
CNS
Homonymous hemianopsia
Hemiparesis
Mental subnormality

Skull X-ray
Calcification of angiomas

No treatment
Control seizures
MTB S2CK

Movement Disorders Part 1


Essential Tremor
Parkinsonism
Restless Leg Syndrome
Huntington Disease
Tourette Disorder
Multiple Sclerosis

p. 288

16

Essential Tremor

Essential Tremor

Occurs at both rest and intention (i.e., reaching for


things)
Tremor greatest in hands, but can affect head
Affects some manual skills
Handwriting or use of computer keyboard

Caffeine makes it worse


Treat with propranolol

Tremor at rest and exertion improved with a drink of


alcohol is key to diagnosis
MTB S2CK

p. 288

Parkinsonism/Definition

Parkinsonism/Etiology

Head trauma from boxing


Use of antipsychotic medications

Loss of substantia nigra


Decrease in dopamine
Movement disorder
Tremor and gait disturbance

Thorazine

Encephalitis
Reserpine
Metoclopromide

No test for parkinsonism


Diagnosis is entirely on clinical presentation

MTB S2CK

p. 288

Parkinsonism/Presentation

MTB S2CK

p. 288

Pill Rolling Tremor

Look for 50-60 (or older) with...


Tremor
Muscular rigidity
Bradykinesia (slow movements)
Shuffling gait with unsteadiness on turning
Tendency
T d
to
t fall
f ll
Cogwheel rigidity (slowing of movement on passive
flexion or extension of extremity)
Facial expression limited (hypomimia)
Writing is small (micrographia)
MTB S2CK

p. 289

17

Parkinson Facies

Parkinsonism/Presentation
Postural instability or orthostatic hypotension
Inability of pulse and BP to reset
Lightheadedness when getting up from
seated position

The most frequent Parkinsonism question is:


Treatment
Know the drugs!
MTB S2CK

p. 289

Parkinsonism/Treatment

Parkinsonism/Treatment

Mild disease
Anticholinergic medications relieve tremor
and rigidity
Benztropine and trihexyphenidyl
Adverse effects
Dry mouth
Worsening prostate hypertrophy
Constipation
More frequent in older patients

Mild disease
Amantadine
Increases release of dopamine from
substantia nigra
Definitely
y the answer in older patients
p
((> 60))
intolerant of anticholinergic medications

MTB S2CK

p. 289

MTB S2CK

p. 289

Parkinsonism/Treatment

Parkinsonism/Treatment

Severe disease
Inability to care for themselves, orthostatic
Dopamine agonists
Pramipexole
Ropinirole
Bromocriptine and cabergoline are older
agents
Infrequent use because of adverse effects

Severe disease
Levodopa/carbidopa
Single most effective medication
Associated with on/off phenomena
Episodes of insufficient dopamine (off)
( off )
characterized by bradykinesia
The on effect is too much dopamine
resulting in dyskinesia

MTB S2CK

p. 289

MTB S2CK

p. 289

18

Parkinsonism/Treatment

Parkinsonism/Treatment

Severe disease
COMT inhibitors (tolcapone, entacapone)
Extends duration of levodopa/carbidopa
Blocks metabolism of dopamine
Used only with levodopa/carbidopa
Use when there are on/off phenomena to
even out dopamine level
When response to therapy is inadequate

Severe disease
MAO inhibitors (rasagiline, selegiline)

MTB S2CK

p. 289

As single agent or an adjunct to levodopa/carbidopa


Block metabolism of dopamine

Deep brain stimulation


Electrical stimulation
Highly effective in some patients

MTB S2CK

p. 289

Spasticity
70-year-old man with extremely severe
parkinsonism comes to ED with psychosis and
confusion developing at home. Hes maintained on
levodopa/carbidopa, ropinirole, and tolcapone.
Whats the most appropriate next step in
management?
a.
b.
c.
d.
e.
MTB S2CK

Dontt stop meds!


Don

Stop levodopa/carbidopa Severe Parkinsonism


Start clozapine
Stop ropinirole Dont stop meds severe parkinsonism
Stop tolcapone Dont stop meds severe parkinsonism
Start haloperidol More adverse effects than clozapine
p. 290

Painful, contracted muscles


From damage to CNS
Often associated with MS
No single treatment always effective
Baclofen and tizanadine (central-acting
alpha agonist) may work

MTB S2CK

p. 290

Restless Legs Syndrome

Huntington Disease (HD)

Hereditary disease
CAG trinucleotide repeat sequences on
chromosome 4

Uncomfortable sensation in legs


Creepy and crawly at night
Bed partner who is being kicked at night
Discomfort worsened by caffeine
Relieved by moving legs
Treat with dopamine agonists (e.g.,
pramipexole)

MTB S2CK

p. 290

MTB S2CK

p. 291

19

Huntington Disease/Presentation

Huntington Disease

What is the most likely diagnosis?


HD is the answer when you see...
Choreaform movement disorder (dyskinesia)
Dementia
Behavior changes
Irritability, moodiness, antisocial behavior
Onset between 30 - 50
Often with family history of HD

Diagnosis
Symptom triad
Movement
Memory
Mood

MTB S2CK

p. 291

Genetic testing >99% accurate

MTB S2CK

p. 291

Huntington Disease

Tourette Disorder

Treatment
No treatment can reverse HD
Dyskinesia treated with tetrabenazine
Psychosis treated with haloperidol or
quetiapine
ti i

Idiopathic disorder
Vocal tics, grunts, and coprolalia
Motor tics (sniffing, blinking, frowning)
Obsessive-compulsive behavior
No specific diagnostic test
Treat with neuroleptics
e.g. Fluphenazine, clonazepam, pimozide
ADHD drugs
Methylphenidate

MTB S2CK

p. 291

MTB S2CK

p. 291

Multiple Sclerosis

Multiple Sclerosis/Presentation

Idiopathic disorder
Exclusively CNS (brain and spinal cord)
More common in white women living in
colder climates

What is the most likely diagnosis?


Look for:
Multiple neurological deficits of CNS
Affects any aspect of CNS
Most common
presentation:
Blurry vision
Visual disturbance
from optic neuritis

MTB S2CK

p. 291

MTB S2CK

p. 291

NEUR_02_19

20

Multiple Sclerosis/Presentation

Multiple Sclerosis/Diagnostic Tests

After optic neuritis, the most common abnormalities


are motor and sensory
Other findings

MRI: Best initial AND Most accurate test

Fatigue
Spasticity and hyperreflexia
Cerebellar deficits

Least common findings


Cognitive defects
Dementia
Sexual dysfunction
Source: Saba Ansari, MD

MTB S2CK

p. 292

MTB S2CK

p. 292

Multiple Sclerosis/Diagnostic Tests

Multiple Sclerosis/Treatment

Lumbar puncture: CSF


Mild elevation in protein
< 50 - 100 WBCs

The best initial therapy for acute


exacerbations of MS...
High-dose steroids

Oligoclonal bands: Found in 85% of MS


Oligoclonal bands: Not specific to MS

Oligoclonal bands are the answer in 3% - 5% with


equivocal (nondiagnostic) MRI

MTB S2CK

p. 292

MTB S2CK

p. 292

Multiple Sclerosis/Treatment

Multiple Sclerosis/Treatment

Drugs that prevent relapse and progression


Glatiramer (copolymer1)
Beta-interferon
Natalizumab
Mitoxantrone
Fingolimod
Dalfampridine
Azathioprine and cyclophosphamide (rarely
used)

A new MS treatment is started.


MRI shows new, multiple white matter
hypodense lesions.
What is it?
Progressive
g
multifocal
leukoencephalopathy (PML)
What is the cause?
Natalizumab

MTB S2CK

p. 292

MTB S2CK

p. 293

21

Amyotrophic Lateral Sclerosis (ALS)

Movement Disorders Part 2


Amyotrophic Lateral Sclerosis
Charcot Marie Tooth Disease
Peripheral Neuropathy
Facial Nerve Palsy
Guillain Barr Syndrome
Myasthenia Gravis

Exclusively a loss of upper and lower


motor neurons

Idiopathic
MTB S2CK

p. 293

Amyotrophic Lateral Sclerosis (ALS)


What is the most likely diagnosis?
Look for...
Weakness starting in 20s to 40s
Combination of upper and lower motor neuron loss
Initial presentation...
Difficulty
Diffi lt chewing
h i and
d swallowing
ll i
Decrease in gag reflex
Pooling of saliva in pharynx and weak cough
Frequent episodes of aspiration
Death in patients within 3 to 5 years after diagnosis
Most commonly due to respiratory failure
MTB S2CK

p. 293

Kinetic Tremor

Presentation of Amyotrophic Lateral Sclerosis


Upper Motor Neurons

Lower Motor Neurons

Weakness
Weakness
Spasticity
Wasting
Hyperreflexia
Fasciculations
Extensor plantar responses

MTB S2CK

p. 293

Fasciculations

22

Amyotrophic Lateral Sclerosis/Diagnostic Tests

Amyotrophic Lateral Sclerosis/Treatment

Electromyography

Riluzole
Reduces glutamate buildup in neurons
Delays progression
Baclofen
Treats spasticity
CPAP and
d BiPAP
Help respiratory difficulties secondary to muscle
weakness
Tracheostomy
Maintenance on ventilator necessary when disease
advances

Loss of neural innervation in multiple


muscle groups
CPK levels

MTB S2CK

p. 293

MTB S2CK

p. 293

Charcot Marie Tooth Disease

Peripheral Neuropathy

Genetic disorder
Lose both motor and sensory innervation

MCC: Diabetes mellitus


Other causes: uremia, alcoholism, and
paraproteinemias (e.g., MGUS)
Best initial therapy

Distal weakness & sensory loss


Wasting in legs
Decreased DTRs
Tremor

Pregabalin, gabapentin

Foot deformity with high arch common (pes cavus)


Legs look like inverted champagne bottles
Most accurate test: Electromyography
No treatment
MTB S2CK

p. 294

Tricyclic antidepressants
Most seizure medications effective in some people
Phenytoin
Carbamazepine
Lamotrigine

MTB S2CK

p. 294

Specific Peripheral Nerve Neuropathies

Specific Peripheral Nerve Neuropathies

Nerve

Precipitating event

Presentation

Nerve

Precipitating event

Presentation

Ulnar nerve

Biker, pressure on palms of


hands, medial elbow trauma

Tibial
nerve

Worsens with walking

Pain/numbness in
ankle & sole of foot

Radial nerve

pp arm,
Pressure on inner/upper
use of crutches, Saturday night
palsy (falling asleep on arm)

Wasting of
hypothenar
eminence,
4th/5th digit pain
p
Wrist drop

Peroneal
nerve

High boots, pressure on the


back of knee

Weak foot with


decreased
dorsiflexion and
eversion

Lateral
cutaneous n.
of the thigh

Obesity, pregnancy, sitting with


crossed legs

Pain/numbness
of outer aspect
of thigh

Median
nerve

Typists, carpenters,
working with hands

Thenar eminince,
pain/numbness
in 1st 3 fingers

MTB S2CK

p. 294

MTB S2CK

p. 294

23

Facial (Seventh Cranial) Nerve Palsy

Facial (Seventh Cranial) Nerve Palsy

Also known as Bells palsy


Mostly idiopathic
Some identified causes

Paralysis of entire side of face


Stroke paralyzes only lower half of face
Upper half of face innervate from both cerebral
hemispheres

Lyme, sarcoidosis, herpes zoster, and


tumors

Look for statement the face feels stiff


or pulled to one side
MTB S2CK

p. 294

Difficulty closing eye


If they can wrinkle forehead on affected side worry
about stroke

Cannot wrinkle forehead = Bell's palsy!

MTB S2CK

p. 295

Facial (Seventh Cranial) Nerve Palsy

Facial (Seventh Cranial) Nerve Palsy

Hyperacusis
Sounds are extra loud
7th CN supplies stapedius muscle
Acts as a shock absorber on ossicles of
middle ear

The most accurate test is...


Electromyography and nerve
conduction studies
But...no test is usually done

Taste disturbance
7th CN supplies sensation of taste to anterior
two-thirds of tongue
MTB S2CK

p. 295

MTB S2CK

p. 295

Facial (Seventh Cranial) Nerve Palsy

The best initial therapy is...


Prednisone
But...
60% of patients have full recovery even
without treatment

Acyclovir does not help

MTB S2CK

p. 278

38-year-old with pain near his ear followed by


weakness of one side of his face. Both upper and
lower parts of face are weak. Sensation is intact.
What is the most common complication?
a.
b.
c.
d.
e.
f.

Corneal ulceration
Aspiration pneumonia Gag reflex and cough are normal
Sinusitis Nasal discharge & face pain
Otitis media Ear pain, decreased hearing
Deafness Sounds are actually extra loud
Dental caries Cavities do not paralyze your face!

MTB S2CK

p. 295

24

Guillain Barr Syndrome

Guillain Barr Syndrome

Acute inflammatory polyneuropathy


Autoimmune damage of multiple peripheral nerves

What is the most likely diagnosis?


Look for:
Weakness of legs that ascends from feet
Moves toward chest
Loss of DTRs
Some
S
have
h
mild
ild sensory di
disturbance
t b
Respiratory muscle weakness

No CNS involvement
Circulating antibody attacks myelin sheaths of
peripheral nerves
Associated with Campylobacter jejuni

GBS hits diaphragm!


Autonomic dysfunction with hypotension, HTN, or
tachycardia can occur
MTB S2CK

p. 296

MTB S2CK

p. 296

Guillain Barr Syndrome

Guillain Barr Syndrome

CSF = increased protein + normal cell count

Tests of respiratory muscle involvement


Inspiration is the active part of breathing
and the patient loses the strength to inhale

Most specific diagnostic test


Nerve conduction studies/electromyography
Decrease in propagation of electrical impulses along
nerves

Decrease in FVC
Decrease in peak inspiratory pressure

PFTs tell who might die from GBS

MTB S2CK

p. 296

MTB S2CK

p. 296

Guillain Barr Syndrome

Treatment
Intravenous immunoglobulin (IVIG) or
plasmapheresis are equal in efficacy

Prednisone is a wrong answer for GBS,


doesnt help
Combining IVIG and plasmapheresis is
incorrect
MTB S2CK

p. 296

Woman comes with bilateral leg weakness over last


few days. No knee jerk or ankle jerk reflexes.
Weakness started in feet and progressed up to calves
and thighs.
Which of the following is the most urgent step?
a.
b.
c.
d.
e.

Pulmonary function testing


Arterial blood gas Dont wait for inc CO2 on ABG
Nerve conduction study Most accurate, but not next
Lumbar puncture For Dx, not severity assessment
Peak flow meter Assesses expiratory flow

MTB S2CK

p. 296

25

Myasthenia Gravis

Myasthenia Gravis/Presentation

Muscular weakness from antibodies


against acetylcholine receptors at the
NMJ

What is the most likely diagnosis?


Look for...

Double vision and difficulty chewing


Weakness of limb muscles worse at end of the day
Ptosis
Weakness with sustained activity
Normal pupillary responses

Extraocular muscles & mastication (masseter) are


often the only 2 muscular activities universally done
by people (e.g., watching TV and eating)

MTB S2CK

p. 297

MTB S2CK

p. 297

Myasthenia Gravis/Diagnostic Tests

Myasthenia Gravis/Diagnostic Tests

Best initial test is...


Acetylcholine receptor antibodies
(80% 90% sensitive)
Better answer than edrophonium testing
Edrophonium

Most accurate test is...


Electromyography

Short-acting inhibitor of acetylcholinesterase


Temporary bump up in acetylcholine levels
Associated with a clear improvement in motor
function that lasts for a few minutes

MTB S2CK

p. 297

Shows decreased strength with repetitive stimulation

Questions often ask What


What imaging test should be
done?
Answer...
Chest something!
Chest X-ray, CT, or MRI are done to look for
thymoma or thymic hyperplasia
MTB S2CK

p. 297

Myasthenia Gravis/Treatment

Myasthenia Gravis/Treatment

Best initial treatment is...


Neostigmine or pyridostigmine
Longer acting versions of edrophonium

If meds dont control disease, the most


appropriate next step in management is...
Thymectomy if < 60
If > 60 prednisone is used
Azothioprine,
p
, cyclophosphamide,
y p
p
, or
mycophenolate are used in order to get
patient off steroids

MTB S2CK

p. 297

MTB S2CK

p. 298

26

Myasthenia Gravis/Treatment

Acute myasthenic crisis


Severe, overwhelming disease
Profound weakness
Respiratory involvement
Treated with IVIG or plasmapheresis

MTB S2CK

p. 298

27

Pregnancy

Obstetrics and
Gynecology

Definitions
Signs and Diagnosis of Pregnancy
Ph i l i Ch
Physiologic
Changes iin P
Pregnancy

Jason M. Franasiak, MD
Chief Resident Physician
Obstetrics & Gynecology
University of North Carolina

Pregnancy
27-year-old woman with nausea and vomiting for 2 weeks.
Symptoms worsen in the morning, but occur at any time during
the day. She has a decrease in appetite. Her last menstrual
period (LMP) was 6 weeks ago. Physical examination is
unremarkable.
Which is the best next step?

Pregnancy Symptoms

Amenorrhea
Breast tenderness
Nausea and vomiting
Fatigue

The surge in estrogen, progesterone, and betahuman chorionic gonadotropin (beta-HCG) leads to
many symptoms of pregnancy

p for anemia,, but


a. Complete
p
blood count Helpful
not next step
b. Beta-HCG
c. HIDA scan Not initial work-up for nausea/vomitting
d. Comprehensive metabolic panel Important, but must
diagnose cause
e. Urinalysis Important, but must
diagnose cause

MTB S2CK

p. 441

MTB S2CK

Definitions

p. 441

Dating Methods
Developmental age (DA): Days since fertilization
Gestational age (GA): Days/weeks since the LMP

Embryo
Fetus

On average, 2 weeks longer than DA

Infant

Ngele rule: Estimate day of delivery


Take the first day of LMP, subtract 3 months then
add 7 days

Bi th
Birth
8 weeks
gestation

1 year

Example: LMP = 7 / 1 / 2010


-3 +7

Fertilization

EDD = 4 / 8 / 2011
MTB S2CK

p. 441

MTB S2CK

p. 442

Trimester Breakdown
1st Trimester

2nd Trimester

Fertilization

3rd Trimester
Delivery

24 weeks DA
26 weeks GA

12 weeks DA
14 weeks GA

Genetic triple or
quad screen
Fetal movement at
16-20 weeks GA
Anatomic ultrasound
at 18-20 weeks GA

FIRST screening
Fetal heart tones
with doppler

MTB S2CK

Term Lengths

Frequent visits
Monitoring for
labor

p. 442

Previable

Preterm

Term

Fertilization
24 weeks
GA

MTB S2CK

37 weeks
GA

Postterm
42 weeks
GA

p. 442

Gravidity/Parity

G6P2124
Living
Children

Gravity

Abortions

Number of
Pregnancies
Parity

Full-term
birth

Preterm
birth

F-PAL = Full-term (F); Preterm (P);


Abortions (A); Living Children (L)
MTB S2CK

p. 442 443

Sign

Physical Finding

Goodell sign
Ladin sign

Softening of cervix
Softening of uterine
midline
Blue discoloration of
vagina and cervix
Small blood vessels/
reddening of palms
Hyperpigmentation of
the face, worse with sun
Hyperpigmentation
down abdomen midline

Telangiectasias/
palmar erythema
Chloasma
Linea nigra

MTB S2CK

p. 444

a. Quickening
b. Goodell sign
c. Ladin sign
d. Linea nigra
e. Chloasma

MTB S2CK

Signs of Pregnancy

Chadwick sign

20-year-old woman presents to the office because


she believes shes pregnant. Her sexual partner
usually wears a condom, but didnt 2 weeks ago. She
is now 4 weeks late for her menstruation.
Which of the following is a first sign of pregnancy?
Not felt until second trimester
Seen at 6 weeks gestation
Seen in second trimester
Seen at 16 weeks gestation

p. 443

Diagnostic Evaluation
Time from Conception
4 6 weeks
6 weeks
6 8 weeks
1st trimester
16 weeks
2nd trimester

Beta-hCG
Urine and serum testing
all highly sensitive
Produced by cytotrophoblast or syncytiotrophoblast
in placenta
First trimester
Doubling every 48 hours for first 4 weeks
Urine pregnancy tests are positive 4 weeks
following the first day of LMP
Peak levels at 10 weeks gestation
Levels drop in 2nd trimester

MTB S2CK

p. 444

Diagnostic Evaluation

Physiologic Changes in Pregnancy

Step 2 Key Points

Cardiovascular changes
Cardiac output increases 30-50%
Lower blood pressure

Ultrasound confirms
intrauterine
pregnancy
At 5 weeks, or a
beta-HCG of 1500
IU/L, a gestational
sac should be seen
on ultrasound

Decreased afterload

Increased blood volume

Gestational
Sac
Yolk Sac
Image: X.Compagnion , commons.wikimedia.org

MTB S2CK

p. 444

MTB S2CK

p. 444

Physiologic Changes in Pregnancy

Physiologic Changes in Pregnancy

Gastrointestinal changes
Morning sickness

Genitourinary and Renal Changes

Caused by increase in estrogen, progesterone,


and HCG made by placenta

Gastroesophageal reflux
Lower
L
esophageal
h
l sphincter
hi t h
has d
decreased
d ttone
Displacement of stomach by uterus

Constipation
Motility in large intestine decreased

MTB S2CK

p. 445

Increase in the size of


kidney and ureters
Increased risk of
pyelonephritis
Increased incidence of
stress urinary
incontinence

Increase in GFR
Decrease in
BUN/creatinine

MTB S2CK

Image: Nevit Dilmen, WikiCommons

p. 445

Physiologic Changes in Pregnancy

Physiologic Changes in Pregnancy

Hematology Changes
Increased...

Hematology Changes
Hypercoagulable state

RBCs
Plasma
Coagulation factors

Anemia

Physiologic
hydronephrosis
in pregnancy

Fibrinogen (Factor I) increases 50% along with fibrin


split products (Factors VII, VIII, IX, X)
Protein C and S (inhibitors of coagulation) decrease
No increase in PT,, PTT,, or INR
Recall: Virchow triad
Hypercoagulation

Plasma volume > RBCs

Endothelial Damage
MTB S2CK

p. 445

MTB S2CK

Stasis

p. 445

Prenatal Care/First Trimester

Prenatal Care
First Trimester
Second Trimester
Third Trimester
Other Screening Tests

Visits every 4 to 6 weeks


Between 11 and 14 weeks
Ultrasound confirms gestational age and check for
nuchal translucency

A thickened or enlarged
nuchal translucency is
an indication of Down
syndrome.

MTB S2CK

Image: Dr. Wolfgang Moroder WikiCommons

p. 445

Prenatal Care/Second Trimester


17-year-old woman presents for routine prenatal
checkup at 12 weeks.
Which of the following is the most accurate method to
establish gestational age?
a. Ultrasound
b. Beta-HCG
c. Pelvic exam
d. Fundal height
e. LMP

MTB S2CK

Levels varyy widely


y
May change with multiple gestations
May change with multiple gestations
May be unreliable

p. 445 446

Genetic Screening: Risk of chromosomal anomalies


15-20 weeks gestation
Triple screen
Maternal serum alpha-fetoprotein (MSAFP)
beta-HCG
Estriol

Quad screen
Inhibin A added to triple screen
Banana Sign

MTB S2CK

p. 446

Prenatal Care/Second Trimester

Prenatal Care/Second Trimester

Increased MSAFP may indicate dating error, neural


tube defect, or abdominal wall defect

Landmarks in 2nd trimester:


Auscultation of fetal heart rate

Ultrasound at 16 weeks
showing banana sign
created by compression
of cerebellum in
posterior fossa due to
neural tube defect.

16 to 20 weeks: Quickening
Fetal movement first detected by mother
Multiparous
M lti
may experience
i
earlier
li

Banana Sign

Anatomic ultrasound
18 to 20 weeks

Image: Dr. W. Moroder, WikiCommons

MTB S2CK

p. 446

MTB S2CK

p. 446

Prenatal Care/Third Trimester

Third Trimester Testing

Visits are every 2 to 3 weeks until 36 weeks

Week
27

Weekly visits after 36 weeks


Braxton-Hicks Contractions
Oft
Often occur during
d i 3rd trimester
ti
t
Sporadic and dont cause cervical dilation
If they become regular, the cervix should be checked
to rule out preterm labor before 37 weeks

MTB S2CK

p. 446

Test
Complete blood
count

24 28 Glucose challenge

36

MTB S2CK

Action
If hemoglobin < 11,
replace iron orally
* with stool softener
If glucose > 140 at one
hour perform oral
hour,
glucose challenge test
Treat if positive

Cervical cultures for


Chlamydia and gonorrhea
Prophylactic antibiotics
Rectovaginal culture for
during labor
group B Streptococcus
p. 446

Prenatal Care/Third Trimester

Prenatal Care/Other Screening Tests

Glucose challenge test: Fasting or nonfasting


ingestion of 50 g of glucose and serum glucose
check 1 hour later

Chorionic Villus Sampling


Done at 10 to 13 weeks
Obtains fetal karyotype
Catheter into intrauterine cavity to aspirate
chorionic villi from placenta

Glucose challenge test: Fasting serum glucose,


ingestion of 100 g of glucose
Serum glucose checks at 1, 2, and 3 hours
Elevated glucose during any two of these tests is
abnormal
MTB S2CK

p. 447

MTB S2CK

p. 447

Image: National Human Genome


Research Institute, WikiCommons

Prenatal Care/Other Screening Tests

Prenatal Care/Other Screening Tests

Amniocentesis
Done after 15-20 weeks
Obtains fetal karyotype
Needle placed transabdominally into
amniotic
i ti sac and
d withdraw
ithd
amniotic
i ti
fluid

Percutaneous umbilical blood sample


In cases of Rh isoimmunization and
when a fetal CBC is needed
Needle placed transabdominally into
uterus to get blood from umbilical cord

MTB S2CK

p. 447

MTB S2CK

p. 447

Ectopic Pregnancy
Risk Factors
Presentation
Diagnosis
Management

29-year-old woman with history of chlamydia and


abnormal vaginal bleeding presents with left lower
quadrant abdominal pain for the past eight hours. Her
LMP was 6 weeks ago. Her temperature is 99 F, heart
rate is 100 bpm, blood pressure is 130/80 mmHg, and
respiratory rate is 13 breaths/minute.
Which of the following is the most likely diagnosis?
a. Ectopic pregnancy
b. Menstrual cramps Altered menstrual pattern present
c. Diverticulitis Does not cause vaginal bleeding
d. Ovarian torsion
e. Ovarian cyst
MTB S2CK

p. 447 448

Ectopic Pregnancy

Ectopic Pregnancy/Risk Factors

Pregnancy implants outside the uterus


Most commonly occurs in ampulla of fallopian tube

Risk Factors
Previous ectopic pregnancies (strongest risk factor)
Pelvic inflammatory disease (PID)
Intrauterine devices (IUD)

70-80%

Source: Kaplan Lecture Notes

MTB S2CK

p. 448

Source: Nevit Dilmen, wikimedia commons

MTB S2CK

p. 448

Ectopic Pregnancy

Ectopic Pregnancy

Presentation

Differential diagnosis
Abortion
Acute appendicitis
Adnexal torsion
Corpus luteum cyst
rupture

Period of amenorrhea
Unilateral lower abdominal or pelvic pain
Vaginal bleeding
If ruptured
ruptured, can be hypotensive with
peritoneal irritation

MTB S2CK

Source: Gloecknerd, wikimedia commons

Diverticulitis
Endometriosis
Gastroenteritis
PID
UTI

p. 448

Ectopic Pregnancy

Ectopic Pregnancy/Transvaginal Ultrasound

Diagnostic Tests
Beta-hCG
Confirms pregnancy

Bladder

Uterus

Ultrasound
Locates implantation
p
site

Laparoscopy:
Invasive test and treatment to visualize and
remove the ectopic pregnancy

MTB S2CK

Image: Courtesy of Jason Franasiak, MD

p. 448

Ectopic Pregnancy/Treatment

Ectopic Pregnancy/Medical Treatment

Ectopic
Confirmed

Not
Ruptured

Ruptured
Medical
Treatment
Stable

Surgery
(Laparoscopy)

MTB S2CK

Surgical
Treatment

Unstable
IV fluids,
blood products,
dopamine

p. 448

Ectopic Pregnancy
Exclusion Criteria for Methotrexate
Hemodynamically unstable patients
Signs of impending or ongoing ectopic mass rupture
Clinically important abnormalities in baseline hematologic,
renal, or hepatic laboratory values
Immunodeficiency, active pulmonary disease, or peptic ulcer
disease
Hypersensitivity to Methotrexate
Coexistent viable intrauterine pregnancy
Breastfeeding
Unwilling or unable to be compliant with post-therapeutic
monitoring
Do not have timely access to a medical institution
MTB S2CK

Ectopic
pregnancy

p. 449

Baseline labs:
CBC
LFTs
Kidney function
-hCG

Methotrexate given,
-hCG checked 4 & 7 days later

< 15% drop


in -hCG

2nd dose of methotrexate,


-hCG checked day 4 & 7

Persistently
high -hCG

MTB S2CK

p. 449

> 15% drop


in -hCG

> 15% drop


in -hCG

Observe for
side effects,
no other Rx
necessary

Surgical
treatment

Ectopic Pregnancy/Surgical Treatment


Surgical
Treatment

Salpingostomy
Salpingectomy

ostomy = cut
ectomy = remove

Images: Courtesy of Jason Franasiak, MD

MTB S2CK

p. 449 450

Ectopic Pregnancy
Mothers who are Rh negative should receive anti-D
Rh immunoglobulin (RhoGAM) to prevent hemolytic
disease

MTB S2CK

Abortion

p. 450

Abortion
20-year-old woman presents to ED for vaginal bleeding and
lower abdominal pain for one day. She states that shes 15
weeks pregnant. T 99.0 F, HR 100 bpm, BP 110/75 mmHg,
and RR 12/min. Pelvic exam, blood present in vault.
Ultrasound shows intrauterine bleeding, products of
conception, and dilated cervix.
Which is the most likely diagnosis?

a. Complete abortion
b. Incomplete abortion
c. Inevitable abortion
d. Threatened abortion
e. Septic abortion
MTB S2CK

A pregnancy that ends < 20 weeks


gestation or fetus weighs 500g
80% of spontaneous abortions occur prior
to 12 weeks gestation

We are going to review


the definitions in this
section

p. 450

MTB S2CK

Abortion

p. 450

Abortion

Maternal factors that increase risk of abortion


Maternal age
Anatomic abnormalities

MTB S2CK

p. 450

Source: Ed Uthman, MD., commons.wikimedia.org

Source: RadsWiki, commons.wikimedia.org

Abortion

Abortion

Maternal factors that increase risk of


abortion
5

6
3

1.
2.
3.
4.
5.
6.

Dye injector
Cervix
Uterus
Adhesions
Right Tube
Left Tube

Maternal Age
Anatomic abnormalities
Infections
Immunological factors (e.g. Anti-phospholipid
syndrome or SLE)
Endocrinological factors
Malnutrition
Trauma

Source: Floranerolia, commons.wikimedia.org

MTB S2CK

p. 450

Abortion/Presentation

Abortion/Diagnostic Tests

Signs/Symptoms
Cramping abdominal pain
Vaginal bleeding
Hypotension
Tachycardia

CBC
Blood type and Rh screen
Pelvic ultrasound

MTB S2CK

You cannot answer the most


likely diagnosis question about
abortion without an ultrasound

p. 450

MTB S2CK

Abortion/Types
Complete

Abortion

Incomplete

No products
of conception
Follow up
in office

p. 451

Inevitable

Threatened

Products of conception
intact, but intrauterine
bleeding present and
dilation of cervix

Missed

Septic

Death of fetus,
but all products
of conception
present in the
uterus

Mothers whore Rh negative should also


receive anti-D Rh immunoglobulin at this
time

D&C/Medical
Some
products of
conception
Dilation &
Curettage
(D&C) / Medical
MTB S2CK

p. 451

Products of
conception
intact, intrauterine
bleeding, No
dilation
of cervix
Bed rest,
pelvic rest

D&C/M di l
D&C/Medical
Infection of
uterus and
surrounding
areas
D&C and IV
Antibiotics
(levofloxacin &
metronidazole)

MTB S2CK

p. 451

Multiple Gestations/Presentation

Multiple Gestations

Signs/Symptoms
Exponential growth of uterus
Rapid weight gain by mother
Elevated beta-HCG and Maternal
S
Serum
Al
Alpha-Fetoprotein
h F t
t i (MSAFP)
Fertility drugs increase
multiple gestations
MTB S2CK

Multiple Gestations/Diagnostic Tests

p. 451

Multiple Gestations

Ultrasound visualizes fetus


Monozygotic 1 egg and Identical twins
1 sperm
Same gender
that splits Same physical characteristics
Same blood type
Fingerprints differ
Dizygotic

MTB S2CK

2 eggs and Fraternal twins


2 sperm
Different or same sex
They resemble each other
As any siblings would

p. 452

Multiple Gestations/Diagnostic Tests

Source:Trlkly , commons.wikimedia.org

Multiple Gestations

For monozygotic twins, amnionicity and


chorionicity depend on timing of
cleavage

10

Multiple Gestations

Multiple Gestations

Complications

Source: Kevin Dufendach, commons.wikimedia.org

Preterm Labor

Spontaneous abortion of one fetus


Premature labor and delivery
Placenta previa
Anemia

MTB S2CK

p. 452

28-year-old woman 28th week of pregnancy with


severe lower back pain. The pain is cyclical and is
increasing in intensity. On physical examination she
seems to be in pain. T 98.9 F, HR 104 bpm, BP
135/80 mmHg, RR 15/min. Cervix is 3 cm dilated.
Which of the following is the most likely diagnosis?
a. Premature rupture of membranes Membranes intact
b. Preterm labor
c. Cervical incompetence Presents with contractions
d. Preterm contractions Cervical dilation present

MTB S2CK

p. 452

Preterm Labor

Preterm Labor/Risk Factors

Preterm births occur < 37 weeks gestation

Spontaneous: preterm labor


Indicated: maternal or fetal

Preterm labor
Activation of hypothalamic-pituitary-adrenal
yp
p
y
(HPA) axis
Decidual hemorrhage
Inflammation
Uterine distension

Premature rupture of membranes


Multiple gestation
Previous history of preterm labor
Placental abruption
Maternal factors
Uterine anatomical abnormalities
Infections

MTB S2CK

p. 452 453

11

Preterm Labor/Risk Factors

MTB S2CK

Preterm Labor/Risk Factors

Source: RadsWiki, commons.wikimedia.org

p. 453

Preterm Labor/Presentation

Contractions

MTB S2CK

Source: Ed Uthman, , commons.wikimedia.org

p. 453

Preterm Labor/Evaluation

Cervical
change

Initial evaluation
Gestational age
Fetal weight
Presenting fetal part

Occurring between 20 and 37 weeks


gestation

MTB S2CK

p. 453

Preterm Labor/Evaluation

MTS2CK

p. 453

Preterm Labor

Preterm labor

Indications for delivery

Maternal severe HTN


Maternal cardiac disease
Maternal cervical dilation > 4 cm
Maternal hemorrhage (abruptio placenta, DIC)
Fetal death
Chorioamnionitis
Preterm rupture of membranes (34 weeks)
Fetal distress
Intrauterine growth restriction with reverse diastolic flow

When any of these is present, answer delivery


MTS2CK

p. 453

Stop delivery if
Delivery if
EGA 24-33 wks
EGA 34-37 wks
EFW 600-2,500
600-2 500 g EFW > 2,500
2 500 g
Betamethasone
& Tocolytics
MTS2CK

p. 453

12

Preterm Labor/Corticosteroids

Preterm Labor/Tocolytics

Corticosteroids
Betamethasone or dexamethasone
Effects of betamethasone begin within 24 hours
and peak at 48 hours

Goal
Delay preterm labor
Allows time for steroids to work
Transport to specialist unit

Goal
Decrease risk of RDS and neonatal mortality

Agents
Magnesium sulfate

Corticosteriods increase surfactant and


thus mature fetal lungs
MTS2CK

p. 453

MTS2CK

Preterm Labor

Preterm Labor/Tocolytics

Magnesium toxicity leads to


respiratory depression &
cardiac arrest, check deep
tendon reflexes often!

Toxicity: Give calcium gluconate

MTS2CK

p. 454

p. 454

Goal
Decrease contractions
Given to allow time for steroids to work
Agents
Magnesium sulfate
Calcium-channel blockers
-adrenergic agents
Prostaglandin synthetase inhibitors
MTS2CK

p. 454

Preterm Labor/Tocolytics

Premature Rupture of
Membranes
Ductus
Arteriosus

Source:
commons.wikimedia.org

13

Premature Rupture of Membranes (PROM)

Premature Rupture of Membranes

The rupture of chorioamniotic membrane


before onset of labor

Diagnosis
Sterile speculum examination
Fluid pools in posterior fornix
Fluid turns nitrazine paper blue
When dry, fluid has ferning pattern
Source: Elizabeth August, MD

MTS2CK

p. 454

MTS2CK

p. 454

Premature Rupture of Membranes

Premature Rupture of Membranes/Treatment

PROM can lead to


Preterm labor
Cord prolapse
Placental
ace a ab
abruption
up o
Chorioamnionitis

Before 32 weeks gestation


Corticosteroids
Antibiotics

>37 weeks gestation, unknown GBS, > 18 hours rupture


Penicillin administered for prophylaxis

34-37
34
37 weeks gestation, unknown GBS
Initiate Penicillin

Known GBS negative


NO Antibiotics

PROM = Do fewer exams!

<34 weeks gestation, unknown GBS


Erythromycin and Penicillin initiated

MTS2CK

p. 454 455

MTS2CK

p. 455

Third Trimester Bleeding

DDx

Third Trimester Bleeding

Vulva (trauma, varicose veins)


Vagina (lacerations)
Cervix (polyp, cervicitis, carcinoma)
Ut i
Uterine
Uterine rupture
Placenta previa
Vasa previa
Placental abruption

14

Third Trimester Bleeding/Previa

Placenta previa
Abnormal implantation of placenta over
internal cevical os

24-year-old woman in her 32nd week of pregnancy


presents to ED. She woke up in bed in a pool of
blood. Denies contractions or pain. HR 105 bpm, BP
110/70 mmHg.
Which is the next step?
Never do digital exam w/o

Risk
Ri k F
Factors
t
Previous uterine scar
Multiple gestations
Previous placenta previa
MTS2CK

p. 456

a. Digital vaginal exam


knowing placental location
b. Transabdominal ultrasound
c. Immediate vaginal delivery
Gather information first
d. Immediate cesarean delivery
e. Transvaginal ultrasound Abdominal US is safer

MTS2CK

Third Trimester Bleeding/Previa

p. 455

Third Trimester Bleeding/Previa

Digital vaginal exam contraindicated in


third trimester vaginal bleeding
Can cause
Separation between placenta and uterus
Severe hemorrhage

Source: Jason Franasiak, MD

Third Trimester Bleeding/Previa

Presentation
PAINLESS vaginal bleeding
Usually presents > 28 weeks

MTS2CK

Third Trimester Bleeding/Previa Types

MTS2CK

p. 456

Vasa
Previa

Partial
Complete

Marginal

Partial covering
of the internal
cervical os, but
covers more than
th
the marginal

Diagnosis
Transabdominal ultrasound
Placenta location

p. 455

Complete
covering of
the internal
cervical os
MTS2CK

Marginal
covering of
the internal
cervical os

Lowlying
Placenta

Fetal vessel
present over
cervical os

Placenta thats
implanted in lower
segments of uterus
but not covering
internal cervical os

p. 456

15

Third Trimester Bleeding/Previa Types


Full
moon

Half
moon

Third Trimester Bleeding/Previa Types

Crescent
moon
Umbilical
cord
Placenta
Fetal
vessels
Internal os
Source: Sigrid de Rooij, comons.wikimedia.org

MTS2CK

p. 456

MTS2CK

p. 457

Third Trimester Bleeding/Treatment

Third Trimester Bleeding/Accreta

Treatment of placenta previa


Strict pelvic rest
Type and screen, CBC
Fetal considerations
Delivery by C-section

Placental invasion (based on depth)


Accreta
Increta
Percreta

Labor
Severe hemorrhage
Fetal distress
MTS2CK

p. 457

Third Trimester Bleeding

MTS2CK

p. 457 458

Third Trimester Bleeding/Accreta

Placental invasion (based on depth)


Accreta
Increta
Percreta
Risk Factors
Placenta previa
Prior uterine scars
MTS2CK

p. 458

MTS2CK

p. 457

16

Third Trimester Bleeding/Abruption

Placental Abruption

Placental abruption
Abnormal, premature separation of placenta from
uterus
Effects
Complete
Partial
Life-threatening
g
bleeding
Premature delivery
Uterine tetany
DIC
And
Hypovolemic shock
MTS2CK

Minor bleed
No clinical
signs/symptoms

p. 458

Placental Abruption/Etiology

Placental Abruption/Presentation

Risk factors
Maternal HTN (chronic, preeclampsia,
eclampsia)
Prior placental abruption
Tobacco
T b
and/or
d/ cocaine
i use
Trauma

Clinical presentation
Vaginal bleeding
Severe abdominal PAIN (uterine
tenderness)
Contractions
C t ti
Possible fetal distress

MTS2CK

p. 458

MTS2CK

Placental Abruption/Presentation

p. 458

Placental Abruption/Diagnostic Tests

Diagnosis
Transabdominal ultrasound
Clinical scenario

Late Decelerations

Placenta previa = painLESS vaginal bleeding


Source: Jason Franasiak, MD

Placental abruption = painFUL vaginal bleeding


MTS2CK

p. 459

17

Placental Abruption/Diagnostic Tests

Source: Nevit Dilman commons.wikimedia.org

Placental Abruption/Treatment

Delivery plan
Cesarean delivery
Uncontrollable hemorrhage
Fetal distress

Placental Abruption
Type

Description

Complications

Concealed

Blood within uterine


cavity
Placenta more likely
to be completely
detached

Serious complications:
DIC
Uterine tetany
Fetal hypoxia
Fetal death
Sheehan syndrome
(postpartum
hypopituitarism)

External

Blood drains through Usually smaller with minimal


cervix
complications
Placenta more likely
to be partially
detached

MTS2CK

p. 459

Placental Abruption/Treatment

Mothers who are Rh negative should also


receive anti-D Rh immunoglobulin at
this time

Vaginal delivery
Placental separation is limited
Fetal heart tracing is assuring
Fetal death prior to presentation

MTS2CK

p. 459

MTB S2CK

p. 451

Uterine Rupture

Uterine Rupture

Spontaneous complete transection of uterus from


endometrium to serosa
Usually occurs during labor

Life threatening to mother and baby


Life-threatening

Immediate delivery!

MTS2CK

p. 459

18

Uterine Rupture/Risk Factors

Uterine Rupture

Risk factors
Previous C-section
Classical: higher risk of uterine rupture
Low transverse

Trauma
Uterine myomectomy
Uterine overdistention
Polyhydramnios
Multiple gestations

Types of cesarean scars. Source: Elizabeth August, MD.

Placenta percreta
MTS2CK

p. 460

MTS2CK

p. 460

Uterine Rupture/Presentation

Uterine Rupture/Treatment

Clinical presentation:
Extreme abdominal pain
Abnormal bump in abdomen
Lack of uterine contractions
Regression of fetus

Treatment
Emergent laparotomy and delivery
Repair of uterus or hysterectomy
Future management
Early delivery via C
C-section
section
Uterine rupture requires
immediate laparotomy and
delivery of the fetus

MTS2CK

p. 460

MTS2CK

p. 461

Rh Incompatibility

Rh incompatibility

Rh Incompatability

Mother RhD negative


Baby RhD positive
Leads to isoimmunization
Rh isoimmunization
Fetal RBCs cross placenta
Maternal antibodies to RhD antigen are
made
MTS2CK

p. 461

19

Rh Incompatibility

Rh Incompatibility

Clinical significance
1st pregnancy: mild anemia/hyperbilirubinemia
2nd pregnancy: maternal antibodies attack the
second Rh positive baby

Hemolytic Disease of Newborn


Fetal anemia
Extramedullary production of fetal RBCs

Hemolysis

Neurotoxicity from hyperbilirubinemia

Leads to hemolysis of fetal RBCs

High output cardiac failure

Hemolytic disease of newborn

MTS2CK

heme and bilirubin levels

p. 461

MTS2CK

Rh Incompatibility

Hydrops fetalis

p. 461

Rh Incompatibility

Rh Antibody Screening
Rh Negative
Antibody titer
Sensitized
Further
Monitoring
MTS2CK

Unsensitized

Rh Positive
No further
screening

Antibody screen: if mother is Rh or Rh+


Antibody titer: how many maternal
antibodies to Rh+ blood

Repeat at 28 weeks
and give Rhogam as
indicated

p. 461

MTS2CK

p. 461

Rh Incompatibility

Rh Incompatibility

Rhogam indications for Rh unsensitized


patients
28 weeks gestation
Delivery
Procedures (amniocentesis)
(
)
Bleeding (abortion, abruption)

Antibody Screen Positive


Antibody titer
Titer 1:4
Sensitized
Titer 1:16
consider treatment
algorithm
l ith

Unsensitized = no anti-Rh antibodies present

MTS2CK

p. 461 462

MTS2CK

p. 462

20

Rh Incompatibility

Rh Incompatibility
Antibody titer

Antibody titer 1:16


Fetus is Rh Antigen Positive

1:16

Determine paternal Rhesus type

Homozygote
Positive

Middle Cerebral Artery Dopplers

Homozygote
g
Negative

Heterozygote
or unsure
paternity

Treatment
algorithm

Peak MCA Velocity is


>1 5 MOM
>1.5
(Multiples of median)

No Treatment
Fetal genotyping
performed on samples
of chorionic villi,
amniocytes or fetal
blood

MTS2CK

Cordocentesis with
transfusion if fetal Hct
is <30%

p. 462

MTS2CK

Rh Incompatibility

p. 462.3

Rh Incompatibility

Source: Jason Franasiak, M.D.


Source: Jason Franasiak, M.D.

Rh Incompatibility
Antibody titer 1:16
No MCA Doppler Capability

Amniocentesis for fetal cells to be evaluated under


spectrophotometer (evaluated bilirubin)
Indeterminate

Repeat
amniocentesis
2-3 weeks

Affected

Repeat
amniocentesis
1-2 weeks

Hypertension

Transfusion
zone
Fetus probably is anemic
Do percutaneous umbilical blood
sample (fetal hematocrit)
Fetal Hct is <30%
Perform an intrauterine transfusion

MTS2CK

p. 462.3

21

Hypertension/Chronic
29-year-old woman G2P1 in her 30th week of pregnancy
presents for routine prenatal visit. Her wedding ring is
getting too tight. BP 150/100 mmHg, HR 92 bpm, RR 12,
T 99 F. Urine 1+ protein. LFTs: normal.
Which is the most likely diagnosis?
a. Chronic hypertension
b. Gestational hypertension No proteinuria
c. HELLP syndrome No laboratory abnormalities
d. Preeclampsia
e. Eclampsia Pre-eclampsia + seizures

Chronic HTN
BP 140/90 mmHg before 20 weeks
gestation
Treatment
Labetalol, nifedipine, or methyldopa
ACE inhibitors and ARBs cause fetal
malformations
Dont use during pregnancy

MTS2CK

p. 463

Hypertension/Gestational

Gestational HTN
BP 140/90 mmHg after 20 weeks
gestation
No proteinuria
Treatment
T t
t
Labetalol, nifedipine, or methyldopa
Only during pregnancy

MTS2CK

p. 463

MTS2CK

p. 463

Hypertension/Preeclampsia
Mild preeclampsia Severe preeclampsia
Hypertension
Proteinuria

Edema
Mentall status
changes
Vision changes
Impaired liver
function

MTS2CK

Generalized
Yes

No
No

Yes
Yes

p. 463

Hypertension/Preeclampsia

Hypertension/Preeclampsia

Preeclampsia Risk Factors


Nulliparity
Multiple gestation
Advanced maternal age
Chronic HTN
Renal disease
History of preeclampsia

Mild
BP > 140/90 mmHg
Proteinuria 1+ to 2 +

MTS2CK

p. 463

At term

Preeclampsia
Severe
BP > 160/110
Proteinuria 3+ to 4+

Preterm

IInduce
d
delivery

MTS2CK

> 160/110
Dipstick 3+; 24 hour
urine > 5 g

> 140/90
Dipstick 1+ to
2+; 24 hour
urine > 300 mg
Hands, feet, face
No

1. Betamethasone
a. Mature fetus
lungs
2. Magnesium sulfate
a. Seizure
prophylaxis

1. Prevent Eclampsia
p
a. Magnesium
sulfate
2. Control BP
a. Hydralazine
3. Delivery
a. Preterm
b. Term

p. 464

22

Hypertension/Preeclampsia

The only definitive treatment


in preeclampsia is delivery

Hypertension/Eclampsia

Eclampsia
Tonic-clonic seizures occurring in
patient with preeclampsia
Treatment
Stabilize
St bili mother
th
Seizure control: magnesium
BP control: hydralazine and labetalol

Deliver baby
MTS2CK

p. 464

MTS2CK

p. 464

Hypertension HELLP

HELLP Syndrome
Hemolysis, Elevated Liver enzymes,
Low Platelets

Gestational Diabetes

Treatment
Stabilize mother
BP control: hydralazine and labetalol

Deliver baby
MTS2CK

p. 464

Pregestational Diabetes
28-year-old woman in her 27th week of gestation
presents for routine prenatal visit. No complaints. T 99 F,
BP 120/80 mmHg, HR 87 bpm. The patient is given 50
mg of glucose. An hour later, blood glucose 145 mg/dL.
Which is the best next step?
a. Treat with insulin The 1h GTT is a screening test
b. Treat with sulfonylurea
c. Do a fasting blood glucose level A 3h GTT is needed
d. Perform oral glucose tolerance test

MTS2CK

p. 464 465

Pregestational diabetes
Diabetes before pregnancy
Either Type 1 or Type 2 DM

MTS2CK

p. 465

23

Pregestational Diabetes

Pregestational Diabetes

Maternal Complications
Preeclampsia
Spontaneous abortion
Increased rate of infection
Increased postpartum hemorrhage

Fetal Complications
Congenital anomalies
Macrosomia

MTS2CK

p. 465

Shoulder dystocia

Preterm
P t
labor
l b

MTS2CK

p. 465

Pregestational Diabetes/Evaluation

Pregestational Diabetes

Additional prenatal testing


EKG
24-hour urine for baseline renal function
HbA1C
Ophthalmological exam

DM type

Route of administration

Insulin type

Type 1
Type 2

Insulin pump
Subcutaneous insulin

NPH
NPH, lispro

MTS2CK

p. 465

MTS2CK

Pregestational Diabetes/Fetal Testing


Age (weeks)
32-36

>36

37

38-39

Lecithin/sphingomyelin
ratio (L/S ratio)

NST: fetal well-being


Ultrasound: fetal size

L/S ratio: assess fetal


lung maturity test
(if mat
mature
re
delivery)
deli er )

Twice-weekly testing; one


NST and one biophysical
prole (BPP)
NST: fetal well-being
BPP: amount of amniotic uid
and fetal well- being
p. 466

p. 465

Gestational Diabetes/Complications

Weekly nonstress test


(NST) and ultrasound

MTS2CK

Oral Medications
Metformin and glyburide

Continue
testing, IOL at
39 weeks

Gestational diabetes (GDM)


Glucose intolerance identified during pregnancy
Complications
Preterm birth
Fetal
F t l macrosomia
i
Birth injuries from fetal macrosomia
Neonatal hypoglycemia
Development of overt Type 2 DM postpartum

MTS2CK

p. 466

24

Gestational Diabetes/Complications

Gestational Diabetes/Evaluation

Erbs
Palsy

Diagnosis
Routine screening between 24 and 28
weeks GA
1-hour glucose tolerance test
Positive
Positi e if > 140 mg/dL

3-hour glucose tolerance test


Klumpkes Palsy

MTS2CK

Gestational Diabetes

Diabetes/Treatment

Glucose load test


Non-fasting ingestion of 50g glucose
followed by
No
serum measurement one hour later
< 140 mg/dL
NO gestational
diabetes

All glucose levels


are normal

No further test

p. 466

> 140 mg/dL


Oral glucose tolerance test
Fasting ingestion of 100 mg of glucose
followed by serum glucose measurements at
1, 2, and 3 hours after ingestion
Elevation of serum glucose
at 1, 2, or 3 hours

Treatment
Diabetic diet and exercise (walking)
Medical management
Insulin
Oral hypoglycemics

Dont tell pregnant patients to lose weight. Its the


most common wrong answer. Once patients are put
on insulin they should follow fetal testing schedule
starting at 32 weeks.

Gestational diabetes
MTS2CK

p. 467

MTS2CK

p. 467

Diabetes/Treatment

Gestational diabetes needs formal 2-hour


GTT postpartum to screen Type 2 DM

MTS2CK

Fetal Growth Abnormalities


Intrauterine Growth Restriction
Macrosomia

p. 467

25

Intrauterine Growth Restriction

Intrauterine Growth Restriction/Etiology

Fetuses with intrauterine growth restriction (IUGR)


weigh in bottom 10% for gestational age

Types of IUGR
Type
Symmetric

Characteristic
Brain in proportion with rest of body
Occurs < 20 weeks gestation

Asymmetric Brain weight isnt decreased


Abdomen is smaller than head
Occurs > 20 weeks
MTS2CK

p. 467 468

Chromosomal abnormalities
Neural tube defects
Infections (viral, protozoans)
Multiple gestations
Maternal disease
HTN or renal disease
Malnutrition
Substance abuse
Hemoglobinopathies

MTS2CK

Intrauterine Growth Restriction

p. 468

Intrauterine Growth Restriction

Physical examination
Fundal height = gestational age in weeks
Example: a patients fundal height at
28 weeks should be 28 cm

Diagnosis
Ultrasound done to confirm gestational age and
fetal weight
MTS2CK

Source: Jason Franasiak, M.D.

p. 468

Intrauterine Growth Restriction

Intrauterine Growth Restriction

Source: Jason Franasiak, M.D.

Source: Jason Franasiak, M.D.

26

Intrauterine Growth Restriction

Intrauterine Growth Restriction

Complications
Premature labor
Stillbirth
Fetal hypoxia
Lower IQ
Seizures
Mental retardation

Treatment/Prevention
Quit smoking
Prevent maternal infection with
immunizations
Determine
D t
i optimal
ti l d
delivery
li
titime

MTS2CK

p. 468

MTS2CK

p. 468

Macrosomia/Risk Factors

Macrosomia/Diagnostic Tests

Macrosomia
Estimated birth weight > 4500 g

Physical examination

Risk factors
Maternal
M t
l diabetes
di b t or obesity
b it
Advanced maternal age
Post term pregnancy
Fetal genetic syndromes
MTS2CK

Example: a patients fundal height at


28 weeks should be 28 cm

Macrosomia
Fundal height

p. 468

MTS2CK

Macrosomia/Diagnostic Tests

Physical Examination
Fundal height 3 cm than GA

Fundal height = gestational age in weeks

3 cm greater than GA

p. 469

Intrauterine Growth Restriction

Ultrasound

Ultrasound confirms the estimated weight by


Femur length
Abdominal circumference
Head circumference and biparietal diameter

MTS2CK

p. 469

Source: Jason Franasiak, M.D.

27

Intrauterine Growth Restriction

Intrauterine Growth Restriction

Source: Jason Franasiak, M.D.

Source: Jason Franasiak, M.D.

Macrosomia

Shoulder Dystocia/ Clavicle Fracture

Complications
Shoulder dystocia
Birth injuries
Low Apgar scores
Hypoglycemia

MTS2CK

Source: Nevit Dilman, commons.wikimedia.org

p. 469

Gestational Diabetes/Complications

Macrosomia

Treatment
Induction of labor
Erbs
Palsy

Lungs mature
EFW < 4500 grams

Cesarean deli
delivery
er
EFW > 5000 grams

Klumpkes Palsy

MTS2CK

p. 469

28

Nonstress Test
Allows for evaluation of fetal well-being in utero

Labor and Delivery


Fetal Testing
Electronic Fetal Monitoringg
Physiologic Changes Before Labor
Induction of Labor

Source: Jason Franasiak, MD

MTS2CK

Nonstress Test

p. 469

Biophysical Profile

Biophysical profile (BPP)


Test
NST
Fetal Breathingg
Fetal Movement
Tone
Amniotic Fluid

Score
0 or 2
0 or 2
0 or 2
0 or 2
0 or 2

Source: Jason Franasiak, MD

MTS2CK

p. 469

MTS2CK

Electronic Fetal Monitoring

p. 470

Electronic Fetal Monitoring

Source: Jason Franasiak, MD

Source: Jason Franasiak, MD

MTS2CK

p. 470

MTS2CK

p. 470

29

Electronic Fetal Monitoring

Electronic Fetal Monitoring

Type

Description

Cause

Early
decelerations

Decrease in heart rate that


occurs with contractions

Head
compression

Variable
decelerations

Decrease in heart rate and


return to baseline with no
relationship to contractions

Umbilical cord
compression

Late
decelerations
(most serious
and dangerous)

Decrease in heart rate after


contraction started. No
return to baseline until
contraction ends

Fetal hypoxia

MTS2CK

p. 470

MTS2CK

p. 471

Electronic Fetal Monitoring

Electronic Fetal Monitoring

Physiological Changes Before Labor

Labor & Delivery

Early Changes
Lightening
Braxton-Hicks contractions
Bloody show

Stages of Labor
Stage 1
Onset of labor
full
dilation of cervix
Primipara: 618 h
p
210 h
Multipara:
Latent
phase

p. 471

MTS2CK

Stage 3

Full dilation of
cervix
delivery
of neonate
Primipara:
p
2h
Multipara: 1 h

Delivery of
neonate
delivery of
placenta
30 min

Active phase

Onset of labor
4 cm dilation
Primipara: 67 h
Multipara: 45 h
MTS2CK

Stage 2

4 cm dilation
full dilation
Primipara: 1.2 cm/h (minimum)
Multipara: 1.5 cm/h (minimum)

p. 471

30

Labor & Delivery

Labor & Delivery

Monitoring (Stage 1)
Maternal BP and pulse
Electronic fetal monitor: fetal HR and
uterine contractions
Examine
E
i cervix
i (every
(
2 hours)
h
)
Cervical dilation
Cervical effacement
Fetal station
MTS2CK

NOT
effaced
NO
dilation

Fully
effaced
1 cm
dilated

5 cm
dilation

Fully
dilated
at
10 cm

Source: Fred the Oyster, commons.sikimedia.org

p. 471

Labor & Delivery

Labor & Delivery

Stage 2
Cervix fully dilated to delivery
Internal vs. External Fetal Monitoring

MTS2CK

p. 472

Stages of fetal head descent. Source: Elizabeth August, MD.

Labor & Delivery

Labor & Delivery

Internal vs. External Fetal Monitoring

Internal vs. External Fetal Monitoring

Source: Jason Franasiak, MD


Source: Jason Franasiak, MD

Source: Jason Franasiak, MD

31

Labor & Delivery

Labor & Delivery

Stage 2
Cervix fully dilated to delivery

Stage 2: Cervix is fully dilated to delivery


Cardinal Movements of Labor

Internal vs. External Fetal Monitoring


Rupture of amniotic membranes
Spontaneous or artificial
Meconium

1.
2.
3
3.
4.
5.
6.
7.
MTS2CK

Engagement
Descent
Flexion
Internal Rotation
Extension
External Rotation
Expulsion
p. 472 473

Labor & Delivery

Operative Delivery

Stage 3
From delivery of neonate to placenta

Operative Vaginal Delivery


Forceps
Vacuum

Immediately after delivery


Repair lacerations of vagina
Signs of placental separation include:
Fresh bleeding from vagina
Umbilical cord lengthening
Uterine fundus rising
Uterus becoming firm
MTS2CK

C-section

p. 473

Operative Delivery

Induction of Labor/Medications

Induction of labor
Initiating labor via medical means
Medications
PGE2 used for cervical ripening
Oxytocin
Types of cesarean scars. Source: Elizabeth August, MD.

MTS2CK

p. 460

MTS2CK

p. 473

32

Induction of Labor/Medications

Mechanical means
Amniotomy
Foley Balloon

Complications of Labor
and Delivery
Prolonged Latent Stage
Protracted Cervical Dilation
Arrest Disorders
Malpresentation
Shoulder Dystocia
Postpartum Hemorrhage

Inspect for a prolapsed umbilical cord


after puncturing amniotic sac

MTS2CK

p. 473

Prolonged Latent Stage


22-year-old primipara in 39th week of pregnancy, with
intense abdominal pain thats intermittent. Gush of fluid
felt 3 hours ago. Cervix 3 cm dilated, 50% effaced, and
fetus head is felt at 2 station. The next 3 hours she
progresses, her cervix is 5 cm dilated, 60% effaced, and
fetal head at 1 station. Six hours after presentation, her
cervix is 5 cm dilated and 60% effaced, fetal head at 0
station.
Which is the most likely diagnosis?
a. Prolonged latent stage Cervix has not dilated
b. Protracted cervical dilation Cervix has not dilated
c. Arrest of descent Fetal head has descended
d. Arrest of cervical dilation
MTS2CK

p. 474

Prolonged latent stage


Latent phase 20 hours for primipara
Latent phase 14 hours for multipara
Etiology
Sedation
Unfavorable cervix
Uterine dysfunction with irregular/weak
contractions
MTS2CK

p. 474

Prolonged Latent Stage

Protracted Cervical Dilation

Treatment of Prolonged Latent Phase


Rest
Hydration

Stage 1 of labor
< 1.2 cm/hour in primipara
< 1.5 cm/hour in multipara

Most will convert to spontaneous


delivery in 6 to 12 hours

MTS2CK

p. 474

Etiology: 3 Ps
Power: strength and frequency of contractions
Passenger: size and position of fetus
Passage: size of pelvis

MTS2CK

p. 474 475

33

Protracted Cervical Dilation

Arrest Disorders/Types

Treatment of protracted cervical


dilation
Power

Arrest of Cervical Dilation


No cervical dilation for 2 hours

Pitocin
Amniotomy

Passenger/Pelvis

Arrest of Fetal Descent


No fetal descent for 1 hour

C-section
MTS2CK

p. 475

MTS2CK

p. 475

Arrest Disorders/Etiology

Etiology
Cephalopelvic disproportion
Cesarean delivery

Excessive sedation/anesthesia
Rest or reversal

Malposition
Time
Operative delivery (forceps)
Cesarean delivery

MTS2CK

p. 475

25-year-old woman in 35th week of gestation


presents for routine prenatal checkup. T 98 F, BP
130/90, HR 87, and RR 12. Her abdomen is gravid. A
hard circular surface is felt in proximal part of the
uterus.
Which is the next step?
a. External cephalic version Perform ultrasound first
b. Ultrasound
c. CT scan Ultrasound is used to determine position
d. X-ray

MTS2CK

p. 475

Malpresentation/Presentation

Malpresentation/Presentation

Presenting Part
Part of fetal body thats closest to
vaginal canal and will be engaged when
labor starts
Cephalic

Diagnosis
Physical Examination
Leopold maneuvers
Vaginal exam

Ultrasound
Ultraso nd

Head

Should always confirm suspected


diagnosis on physical examination
with ultrasound

Malpresentation
Foot or buttock
MTS2CK

p. 476

MTS2CK

p. 476.

34

Types of Breech Presentation

Type

Malpresentation/Presentation

Description

Frank breech Fetus hips are flexed with extended


knees bilaterally
Complete
b
breech
h

Fetus hips and knees are flexed


bil
bilaterally
ll

Footling
breech

Fetus feet are first


one leg (single footling)
or
both legs (double footling)

MTS2CK

p. 476

MTS2CK

Malpresentation/Presentation

MTS2CK

p. 477

p. 476

Source: Elizabeth August, MD.

Malpresentation/Presentation

Source: Elizabeth August, MD.

MTS2CK

p. 477

Source: Elizabeth August, MD.

Malpresentation/Treatment

Shoulder Dystocia

Treatment
External cephalic version
C-section

Shoulder dystocia
Entrapment of anterior shoulder behind
pubic symphisis after delivery of fetal
head

MTS2CK

p. 477

MTS2CK

p. 477

35

Shoulder Dystocia

Shoulder Dystocia/Risk Factors

Risk Factors
Maternal diabetes
Maternal obesity
Post-term pregnancy
History of prior shoulder dystocia
Any factor that indicates fetus is too big or
the pelvis is too small is a risk factor for
shoulder dystocia
Source: Elizabeth August, MD.

MTS2CK

p. 478

Shoulder Dystocia/Treatment

MTS2CK

p. 478

Shoulder Dystocia/Treatment

Treatment
1. McRoberts Maneuver

McRoberts maneuver. Source: Elizabeth August, MD.

MTS2CK

p. 478 479

MTS2CK

p. 478

Shoulder Dystocia/Treatment

Postpartum Hemorrhage/Etiology

Treatment

Postpartum hemorrhage
More than 500 mL after delivery
Early vs. late postpartum bleeding

1. McRoberts maneuver
2. Rubin maneuver
3. Woods maneuver
4. Delivery of posterior arm
5. Deliberate fracture of fetal clavicle
6. Zavanelli maneuver

MTS2CK

p. 478

Etiology
gy
a = without
Uterine atony
tony = contractions
Laceration
Retained products of conception
Coagulopathy
MTS2CK

p. 479

36

Postpartum Hemorrhage/Risk Factors

Risk factors for atony


Anesthesia
Uterine overdistention
Prolonged/rapid labor
Augmented labor
Uterine leiomyoma
Preeclampsia with magnesium therapy

MTS2CK

p. 479

Postpartum Hemorrhage/Treatment
Evaluation/Treatment
Examination of perineum, vagina, and cervix
Bimanual examination of uterus (+/- compression/massage)
Administer uterotonics (oxytocin, methylergonovine maleate,
15 methyl-PGF2alpha/Hemabate, misoprostol)
Operative management
Uterine arteryy embolization
D&C
Bakri balloon placement
Collapsed silicone ballon inserted in uterus, later filled with fluid

B-Lynch stitch
Uterine artery ligation
Hysterectomy
MTS2CK

p. 479

Postpartum Hemorrhage/Treatment

Postpartum Hemorrhage/Treatment

B-Lynch Suture

Evaluation/Treatment
Examination of perineum, vagina, and cervix
Bimanual examination of uterus
Administer uterotonics
Operative management

Uterine Artery Ligation

Blood products
PRBCs, FFP, Cryoprecipitate

Source: Niels Olson, commons.wikimedia.org


Modified: Jason Franasiak, MD

Postpartum Hemorrhage/Treatment
Product

Contents

Packed RBCs

RBCs, WBCs, plasma

FFP

Soluble plasma
proteins
p

Cryoprecipitate Factors VIII and XIII,


fibrinogen,
fibronectin, vWF
Platelets
Platelets, RBCs,
WBCs, plasma

Effect
Hct 3%
Hgb 1g/dL
fibrinogen 10
mg/dL
g/
fibrinogen 10
mg/dL

The Uterus
Premenstrual Syndrome
Menopause
AUB/DUB
Contraception

5000
10,000/mm3 per
unit

37

PMS/PMDD

PMS/PMDD

Premenstrual syndrome (PMS)


Physical, mood-related, and behavioral
changes

Over 200 symptoms attributed to PMS

Premenstrual Dysphoric Disorder (PMDD)


Severe form of PMS

MTB S2CK

p. 481

Common symptoms
Headache
Breast
B
t ttenderness
d
(mastodynia)
(
t d i )
Pelvic pain, bloating
Irritability, lack of energy

MTB S2CK

p. 481

PMS/PMDD

PMS/PMDD

Diagnostic criteria
Present for 2 consecutive cycles
Symptom-free in first part of cycle
Symptoms present in second half of cycle
Dysfunction in social or economic
performance

Treatment of PMS/PMDD
Lifestyle
Limit caffeine, alcohol, cigarettes, and chocolate
Aerobic exercise
Increase calcium and magnesium

Pharmacologic
NSAIDs
Severe: SSRIs

MTB S2CK

p. 481

MTB S2CK

p. 481

Menopause

Menopause

Menopause
Permanent cessation of menses
Due to permanent cessation of estrogen
production
Median
M di age off onset:
t 51

Physiology
Early

MTB S2CK

The oocytes
produce less
estrogen and
progesterone
t
LH and FSH start
to rise
Shortening of
menstrual cycles

Late
Changes in sex
hormones
Testosterone
Androstenedione
Estrone
Estradiol

p. 482

38

Menopause

Menopause

Symptoms
Menstrual irregularity
Sweats and hot flashes
Mood changes
Dyspareunia (pain during sexual intercourse)

Physical Examination
Decrease in breast size
Vaginal/cervical atrophy
Uterovaginal prolapse

Women are symptomatic for an average


of 12 months, but some experience
symptoms for years
MTB S2CK

p. 482

Atrophic vaginitis: vaginal epithelium is


estrogen dependent. The absence of
estrogen causes thinning, presents as
itching, burning, and/or dyspareunia.

MTB S2CK

Menopause/Osteoporosis

p. 482

Menopause
Hormone replacement therapy (HRT)
Estrogen +/- progesterone
Contraindications
Estrogen-dependent carcinoma
History of PE or DVT

HRT associated with endometrial


hyperplasia, can lead to endometrial
carcinoma

Vertebral body fractures. Source: Dr Robert CARLIER, CHU


Raymond Poincar, Garches, France, commons.wikimedia.org

Abnormal Uterine Bleeding


Menorrhagia

Hypomenorrhea

Heavy, prolonged
menstrual bleeding
Gushing of blood
Clots may be seen
Endometrial
hyperplasia
Uterine fibroids
Dysfunctional
uterine bleeding
Intrauterine device

MTB S2CK

p. 482 483

MTB S2CK

p. 482

Abnormal Uterine Bleeding


Metrorrhagia

Menometrorrhagia

Intermenstrual bleeding

Light menstrual
flow
May only have
spotting
Obstruction
(hymen, cervical
stenosis)
OCPs

Endometrial polyps
Endometrial/cervical
cancer
Exogenous estrogen
administration
d i i t ti

Postcoital
bleeding

Oligomenorrhea
Menstrual cycles > 35 days long

Irregular bleeding
Time intervals
Duration
u at o
Amount of bleeding

Pregnancy
Menopause
Significant weight loss
(anorexia)
Tumor
T
secreting
ti estrogen
t

Endometrial polyps
Endometrial/cervical cancer
Exogenous estrogen
administration
Malignant tumors

MTB S2CK

Bleeding after
intercourse
Cervical cancer
Cervical polyps
Atrophic vaginitis

p. 483

39

Abnormal Uterine Bleeding

Abnormal Uterine Bleeding


Uterine fibroid

Postcoital bleeding is cervical cancer until


proven otherwise

Any patient > 35 with abnormal bleeding


should undergo endometrial biopsy to
rule out endometrial carcinoma
Source: James Heilman, MD, commons.wikimedia.org

MTB S2CK

Abnormal Uterine Bleeding


Normal squamous cells

p. 482 483

Abnormal Uterine Bleeding


HPV infects cells causing
mild dysplasia

Anovulatory Uterine Bleeding


Estrogen produced
No progesterone
No ovulation
no corpus luteum
progesterone
Prevents withdrawal bleeding

no

Source: Ed Uthman MD, commons.wikimedia.org

Abnormal Uterine Bleeding

Dysfunctional Uterine Bleeding

Diagnosis/Evaluation
CBC
Pregnancy test
PT/PTT
Pelvic ultrasound
Endometrial biopsy
Pap smear
Thyroid studies
Prolactin levels

Dysfunctional uterine bleeding


(DUB)
Unexplained abnormal bleeding

Theres no specific test for DUB


Diagnosis by exclusion
MTB S2CK

p. 483

40

Dysfunctional Uterine Bleeding

Treatment of DUB
Oral contraceptive
pills (OCP)
Cyclic progesterone

Dysfunctional Uterine Bleeding

Long-term
Endometrial
ablation
Hysterectomy

Acute hemorrhage
D&C
IV estrogen

MTB S2CK

Source: Hic et nunc, commons.wikimedia.org

p. 484

Dysfunctional Uterine Bleeding

Contraception/Female Condoms

Advantages
Offer some protection against HIV and STDs
Under female control
Disadvantages
Not as effective as other methods
They are larger and bulkier than male
condoms

Source: Hic et nunc, commons.wikimedia.org

Contraception/Female Condoms

MTB S2CK

p. 484

Contraception/Vaginal Diaphragm

Source: : ka Grzywacz, commos.wikimedia.org

MTB S2CK

p. 484

Source: Axefan2, commons.wikimedia.org

MTB S2CK

p. 484

41

Contraception/Cervical Cap

Contraception/Vaginal Diaphragm

Advantages
Under female control
Disadvantages
Need to be fitted properly
Requires advance preparation
Improper placement or dislodging reduces
efficacy

Source: Ceridwen, commons.wikimedia.org


Source: Dake, commons.wikimedia.org

MTB S2CK

Contraception/Oral Contraceptive Pills

p. 484

Contraception/Oral Contraceptive Pills

Hormones
Combination of both estrogen and progestin
Progestin only
Use
21 days of active pill
7 days placebo
Menses occurs during 7 days of placebo pills
Source; Matthew Bowden, commons.wikimedia.org

MTB S2CK

p. 484

MTB S2CK

p. 484

Contraception/Oral Contraceptive Pills

Contraception/Vaginal Ring

Advantages
Effective with perfect use
Reduces rates of ovarian and endometrial
cancer
Easily
y reversible

A flexible vaginal ring inserted in vagina

Disadvantages
User dependent
Risk of thromboembolism
MTB S2CK

p. 484

Releases both estrogen and progesterone


Remains in place for 3 weeks

The vaginal
aginal ring has similar side effects
and efficacy to OCPs

MTB S2CK

p. 485

42

Contraception/Transdermal Patch

Contraception/Transdermal Patch

Transdermal patch
Combination of estrogen and progesterone
Placed on skin for 7 days

Patches shouldnt
sho ldnt be placed on breast
Side effects and efficacy = OCPs
Source : Keitei, commons.wikimedia.org

MTB S2CK

p. 485

MTB S2CK

p. 485

Contraception/Injectable

Contraception/Implantable

Depo-Medroxyprogesterone Acetate
(DMPA)

Implant contraceptive system


Releases progestin daily
Inhibits ovulation and thickens cervical
mucus

IM injection
Every 3 months

Contraception/Intrauterine Device

Contraception/Intrauterine Device

Intrauterine device (IUD)


Copper device
Levonorgestrel device

Most commonly used worldwide


Most effective aside from sterilization

Source: Gloecknerd, commons.wikimedia.org

MTB S2CK

p. 485

MTB S2CK

p. 485

43

Contraception/Intrauterine Device

Contraception/Sterilization

Tubal Ligation/Occlusion
Vasectomy

Source: Nevit Dilmen ,commons.wikimedia.org

MTB S2CK

p. 485

Vulva and Vagina/Labial Fusion

Vulva and Vagina

Fusion
Occurs when excess androgens are present
Exogenous
Endogenous

MCC of labial fusion is 21-B


hydroxylase deficiency
Treatment
Conservative
Reconstructive surgery
MTB S2CK

Vulva and Vagina/Epithelial Abnormalities


Abnormality
Lichen
sclerosus

Age group
affected
Any age
If postmenopausal,
increased concern
for cancer

Description
White, thin skin
from labia to
perianal area,
parchment

q
g p
patients
Squamous
Anyy age;
Patients with
cell
whove had chronic chronic irritation
hyperplasia vulvar pruritus
develop hyper
keratosis (raised
white lesion)
Lichen
planus
MTB S2CK

30s60s

p. 486

p. 485

Vulva and Vagina/Bartholin Gland Cyst


Treatment
Topical
steroids

Sitz baths or
lubricants
(relieve
pruritus)

Pruritic, polygonal, Topical


papular and
steroids
Purple

Bartholin Glands
Location: lateral sides of vulva
Function: secrete mucus
Bartholin Gland Cyst/Abscess
y
Presents with pain, tenderness, and
dyspareunia
Edema and inflammation with deep fluctuant
mass
MTB S2CK

p. 486

44

Vulva and Vagina/Bartholin Gland Cyst

Vulva and Vagina/Bartholin Gland Cyst


Clitoris
Labia Minora
Urethra
Skenes glands
g
Vagina
Bartholins glands

Source: Nicholasolan, commons.wikimedia.org

Bartholin Abscess
Simple incision and drainage (I&D)
Word catheter placement
4-6 weeks duration
Small rubber catheter, inflatable balloon tip is
inserted into cyst incision,
incision after the contents of
the cyst have been drained
Recurrence: marsupialization or excision
During I&D, fluid released should be
cultured for STDs (e.g., Neisseria gonorrhea
and Chlamydia trachomatis)
MTB S2CK

p. 486

Vaginitis/Risk Factors
19-year-old woman with vaginal pruritus and
discharge for one week. Discharge is green and
profuse. Shes had multiple sexual partners in past 2
months. LMP 2 weeks ago. Wet mount: motile
flagellates.
Which is the most likely diagnosis?
a. Chlamydia Dx w/ culture or DNA probe
b. Bacterial vaginosis Clue cells on wet mount
c. Neisseria gonorrhoeae
d. Candidiasis Hyphae on wet mount
e. Trichomonas vaginalis
MTB S2CK

p. 486 487

Antibiotic use (Lactobacillus normally


keeps vaginal pH < 4
4.5)
5)
Diabetes
Overgrowth of normal flora

p. 487

Vaginitis

Bacterial vaginosis

Candidiasis

Gardnerella
Vaginal
discharge with
fishy odor;
gray white
Saline prep:
clue cells
Metronidazole
or clindamycin

Candida albicans
White, cheesy
vaginal discharge
Potassium
hydroxide (KOH):
pseudohyphae;
vaginal culture is
most specific
Miconazole or
clotrimazole,
econazole, or
nystatin

p. 487

Risk factors

MTB S2CK

Types of Vaginitis

MTB S2CK

Vaginitis: spectrum of conditions

Trichomonas (most
common nonviral STD)

Trichomonas: partners need


to be treated

Trichomonas
vaginalis
Profuse, green,
frothy vaginal
discharge
Saline prep:
motile
flagellates
Treat both
patient and
partner with
metronidazole
MTB S2CK

p. 487

45

Vaginitis/Gardnerella

Vaginitis/Candida albicans

Clue cells. Source: Per Grinsted, commos.wikimedia.org

Vaginitis/Trichomonas vaginalis

Source: Nephron, commons.wikimedia.org

Vulva/Malignant Disorders

Paget Disease
Intraepithelial neoplasia
Most common in postmenopausal Caucasians
Presentation
Vulvar soreness and pruritus
Appears as a red lesion with superficial white
coating
Trichomonas. Source: cdc.gov

Trichomonas. Source: Alex Brollo,


commons.wikimedia.org

MTB S2CK

Vulva/Malignant Disorders

p. 487

Vulva/Malignant Disorders
Large cells with clear
cytoplasm in epidermis

Biopsy is needed for definitive diagnosis


Treatment
Wide local excision or vulvectomy
2 cm margin is optimal

Source: Nephron, commons.wikimedia.org

MTB S2CK

p. 487

46

Vulva/Malignant Disorders

Staging of Squamous Cell Carcinoma

Squamous Cell Carcinoma


Most common type of vulvar cancer

Stage

Findings

0
I
II
III

Carcinoma in situ
Limited to vaginal wall < 2 cm
Limited to vulva or perineum > 2 cm
Tumor spreading to lower urethra or anus,
unilateral
il
l lymph
l
h nodes
d present
Tumor invasion into bladder, rectum, or
bilateral lymph nodes
Distant metastasis

Presentation
Pruritus, bloody vaginal discharge, and
postmenopausal bleeding
Exam: ranges from a small ulcerated lesion to large
cauliflower-like lesion
A biopsy is essential for diagnosis

MTB S2CK

p. 488

IV
IVa

MTB S2CK

p. 488

Vulva/Malignant Disorders

Treatment
Unilateral: modified radical vulvectomy
Bilateral: radical vulvectomy

Uterine Abnormalities

Lymph nodes thatre involved must


undergo lymphadenectomy

MTB S2CK

p. 488

Uterine Abnormalities/Adenomyosis

Uterine Abnormalities/Adenomyosis

Adenomyosis
Invasion of endometrial glands into myometrium
Typically between ages of 35 and 50

Diagnosis
Clinical diagnosis
Physical examination

Risk factors
Endometriosis
Uterine fibroids
Presentation
Dysmenorrhea and menorrhagia
MTB S2CK

p. 488

Uterus is large, globular, boggy

MRI
Treatment
Hysterectomy - only definitive treatment

MTB S2CK

p. 488

47

Uterine Abnormalities/Adenomyosis

Uterine Abnormalities/Adenomyosis

Source: Hic et nunc, commons.wikimedia.org

Source: Hic et nunc, commons.wikimedia.org

Uterine Abnormalities/Endometriosis

Uterine Abnormalities/Endometriosis

Endometriosis
Endometrial tissue outside of endometrial cavity
Most common sites are ovary and pelvic
peritoneum

Presentation
Cyclical pelvic pain
Abnormal bleeding
Infertility

Endometriosis occurs in women of reproductive


age
More common if first-degree relative has
endometriosis

MTB S2CK

p. 488

Dysmenorrhea and dyspareunia


are common in endometriosis

MTB S2CK

Uterine Abnormalities/Endometriosis

Physical examination
Nodular uterus
Adnexal mass

p. 488 489

Uterine Abnormalities/Treatment

Mild disease
NSAIDs
Combined OCPs
Severe disease
Danazole
Leuprolide acetate (leupron)
Surgery

Source: Julie M. Hastings, Asgerally T. Fazleabas, commons.wikimedia.org

MTB S2CK

p. 489 490

MTB S2CK

p. 489

48

Uterine Abnormalities/Treatment

Ovarian Abnormalities

Source: Hic et nunc, commons.wikimedia.org

Polycystic Ovarian Syndrome/Symptoms

Polycystic Ovarian Syndrome/Diagnosis

Presentation
Amenorrhea or irregular menses
Hirsutism and obesity
Acne
Insulin resistance

Diagnostic test
Pelvic ultrasound
Elevated free testosterone
LH to FSH ratio > 3:1

MCC of androgen excess and


hirsutism
MTB S2CK

p. 489

MTB S2CK

Polycystic Ovarian Syndrome/Diagnosis

p. 489 490

Polycystic Ovarian Syndrome/Diagnosis

Uterus

Ovary

Source: Schomyny, commons.wikimedia.org

49

Polycystic Ovarian Syndrome/Treatment

Treatment
Weight loss
OCPs
Spironolactone (hirsutism)
Metformin (insulin resistance)
Clomiphene (infertility)

MTB S2CK

p. 490

50

Breast Cancer

Oncology

Presentation
Diagnosis
g
Genetic Tests
Treatment

Emma Holliday,
Holliday MD
Resident Physician
Radiation Oncology
University of Texas MD Anderson Cancer Center

Breast Cancer/Presentation

Breast Cancer/Presentation

Most often found by patient


Found in asymptomatic women

- On screening mammography
- By palpation of a mass
Hard immobile
Hard,
immobile, fixed to the chest wall

Painless lump
Skin changes
Nipple retraction
Nipple discharge

Source:http://www.4woman.gov/faq/cancerillustrations-with-t.gif

MTB S2CK

p. 347

MTB S2CK

p. 347

Breast Cancer/Diagnostic Tests

Breast Cancer/Diagnostic Tests

Biopsy is the best initial test

Mammography
Screen the general
population starting
at age 50

Different methods are:


Fine needle aspiration (FNA)
Best initial biopsy

Core needle biopsy


Tells receptor status

Open surgical biopsy


Most accurate test

MTB S2CK

p. 347

MTB S2CK

p. 347

When Is Ultrasound the Answer?


A woman finds a hard, nontender breast mass on
self-examination. There is no alteration of the mass
with menstruation. She is scheduled to undergo a
FNA biopsy.
Which of the following is most likely to benefit the
patient?
a. Mammography
b. BRCA testing
c. Ultrasound
d. Bone scan
e. PET scan
MTB S2CK

Confirms an extra risk of cancer

Clinically indeterminate mass lesions. It


tells cysts versus solid lesions
Answer ultrasound if the lesion:
Is painful
Varies in size or pain with menstruation

Tells cystic vs solid


To exclude bone metastases
To look for metastatic disease

p. 348

MTB S2CK

p. 348

When Is PET Scan the Answer?

When Is PET Scan the Answer?

Determines content of abnormal lymph nodes that


are not easily accessible to biopsy. Cancer
increases uptake on PET scan.

For example:

How do you tell the content of


an abnormal, inaccessible lesion
without biopsy? Try PET scan.

MTB S2CK

p. 348

When Is BRCA Testing the Answer?


BRCA associated with increased risk of breast
cancer, particularly within families
BRCA associated with ovarian cancer

80-year-old woman with biopsy-proven breast


cancer has no nodes with cancer in the axilla. The
primary lesion is small and the woman may not
need adjuvant
j
chemotherapy.
py Chest CT shows an
abnormal hilar lymph node.

MTB S2CK

p. 348

When Is Sentinel Lymph Node Biopsy


the Answer?
When you want to know how likely it is that the
breast cancer has spread

What is not clear is what to do when BRCA is


positive BRCA not shown to add mortality benefit
positive.

The precise utility of MRI for breast


cancer is not yet clear.
Source:http://www.cancer.gov/ncicancerbulletin/022211/page2

MTB S2CK

p. 348

MTB S2CK

p. 349

When Are Estrogen and Progesterone


Receptors Tested?
Estrogen receptor (ER) and progesterone receptor
(PR) testing is routine for all patients
Hormone manipulation therapy is done if either test
is positive

Breast Cancer/Treatment
Surgery Options
Lumpectomy
Modified Radical Mastectomy
Radical Mastectomy

Radical mastectomy is always the wrong answer


Lumpectomy + Radiation is JUST AS GOOD as
modified radical mastectomy
MTB S2CK

p. 349

MTB S2CK

p. 349

Hormonal Manipulation

Breast Cancer/Treatment

All ER or PR positive patients should receive:

Know the differences in side effects:

Tamoxifen
Raloxifene
Aromatase inhibitors (anastrazole, letrozole,
exemestane)
If all are among the answer choices, aromatase
inhibitors are the answer to the most likely to benefit
the patient question

MTB S2CK

p. 349

When Is Trastuzumab the Answer?


All breast cancers should be tested for HER
2/neu. This is an abnormal estrogen receptor
Those who are positive should receive antiHER 2/neu antibodies known as trastuzumab
Trastuzumab decreases the risk of recurrent
disease

Tamoxifen rarely gives endometrial


cancer and clots (tamoxifen is a selective
ER modifier).
)
Aromatase inhibitors give osteoporosis
(aromatase inhibitors inhibit estrogen effect
everywhere, even the good effects, like on
bone density).
MTB S2CK

p. 349

When Is Chemotherapy
the Answer?

Neoadjuvant means therapy given


BEFORE the definitive treatment. Goal is
to decrease cancer burden
Adjuvant means an additional therapy to
clean up presumed microscopic cancer
cells too small in amount to be detected

MTB S2CK

p. 350

MTB S2CK

p. 350

When Is Adjuvant Chemotherapy


the Answer?
Adjuvant chemotherapy is the answer when:
Lesions >1 cm
Positive axillary lymph nodes found
Use tamoxifen when
multiple first-degree
relatives have breast
cancer. It lowers the
risk of breast cancer.

MTB S2CK

Breast Cancer/Treatment

All of these definitely lower mortality:


Mammography
ER/PR testing, then tamoxifen/raloxifene
Aromatase inhibitors
Adjuvant chemotherapy
Lumpectomy and radiation
Modified radical mastectomy
Trastuzumab (anti-Her 2/neu)
Prophylaxis with tamoxifen

We do not know what


to do about BRCA
when it is positive.

p. 350

MTB S2CK

p. 350

Prostate Cancer/Presentation

Prostate Cancer
Presentation
Treatment
Screening

Obstructive
symptoms
Palpable mass
Elevated or rising
PSA

Source:http://www.cancer.gov/cancertopics/pdq/treatment/prostate/Patient/page2

MTB S2CK

p. 350

Prostate Cancer

Prostate Cancer/Treatment

Biopsy is the best


initial test and the
most accurate test.

Prostatectomy
Radiation therapy
Brachytherapy
Hormonal therapy
Watchful waiting

Half of men above age


80 have prostate cancer
on autopsy.
MTB S2CK

p. 350

Source:http://kidney.niddk.nih.gov/kudiseases/pubs/i
magingut/

MTB S2CK

p. 350 351

Prostate Cancer/Treatment

Gleason Grading

Prostatectomy has slight benefit over


radiation in terms of survival

Gleason Score =
Tumor Grade

Most common complications of


prostatectomy are:
Erectile dysfunction
Urinary incontinence

High Gleason grade


suggests greater
g
benefit of surgical
removal of the
prostate
Source:http://www.training.seer.cancer.gov/prostate/abstract-codestage/morphology.html

Get it out before it metastasizes if the Gleason


grade is high
MTB S2CK

p. 350 351

MTB S2CK

p. 351

Hormonal Manipulation in Prostate Cancer

Management That Is Definitely


Not Beneficial in Prostate Cancer

Flutamide

These answers are ALWAYS wrong:


Screening imaging study like ultrasound
Lumpectomy
Chemotherapy
Hormonal manipulation to prevent recurrences

Competitive inhibitor of testosterone & DHT

Leuprolide, goserelin
GNRH agonists: downregulates LH & FSH

Ketoconazole
K t
l
Suppresses testosterone

Orchiectomy
Stops endogenous production

MTB S2CK

p. 351

MTB S2CK

p. 351

Prostate Specific Antigen (PSA)

Prostate Specific Antigen (PSA)

PSA is controversial:
No mortality benefit with PSA
PSA is not to be routinely offered
Normal PSA does not exclude prostate cancer
High PSA doesnt always mean prostate cancer

If question specifically says, The patient


is requesting PSA to screen for cancer,
then the answer is do the test

The higher the PSA, the


greater the risk of cancer. PSA
corresponds to the volume of cancer.
MTB S2CK

p. 351

MTB S2CK

p. 351

Elevated PSA Algorithm


Elevated PSA
Palpable mass

No palpable mass

Biopsy
p y the mass

Transrectal ultrasound
Mass seen
Biopsy the
mass

MTB S2CK

Lung Cancer

No mass seen
Multiple blind
biopsies

p. 351

Lung Cancer

Surgery

Who can get surgery?


Size of the tumor alone does not determine
whether or not it can be resected
Large tumors can be resected with
Demonstration of suffficient residual lung
volume by PFTs
It is solitary and surrounding tissue is healthy

Small tumors cannot be resected if PFTs


indicate poor lung function
MTB S2CK

Source:http://www.cancer.gov/cancertopics/pdq/treatment/non- Source:http://www.cancer.gov/cancertopics/pdq/treatment/non
small-cell-lung/Patient/page4
-small-cell-lung/Patient/page4

Source:http://www.cancer.gov/cancertopics/pdq/treatment/n
on-small-cell-lung/Patient/page4

p. 352

Lung Cancer

Surgery is not possible in these cases:


Bilateral disease
Malignant pleural effusion
Heart, carina, aorta, or vena cava is involved
Small cell cancer is considered unresectable
in 95% of cases because it is metastatic or
spread outside one lung

MTB S2CK

Ovarian Cancer
Screening
Diagnosis
g
Treatment

p. 352

Ovarian Cancer

Ovarian Cancer/Diagnosis

No screening test for ovarian cancer

The initial test is an ultrasound or CT scan

Presentation:
Woman >50 years old
Increasing abdominal girth
No history of liver disease

Weight loss, fatigue


Source:http://www.cancer.gov/cancertopics/pdq/treatment/ovaria
nepithelial/Patient/page1

MTB S2CK

p. 352

MTB S2CK

Source: James Heilman, MD,


http://en.wikipedia.org/wiki/File:POvarianCA.png

p. 352

Ovarian Cancer/Diagnosis

Ovarian Cancer/Treatment

Most accurate test is biopsy

The only cancer in which removing large


amounts of locally metastatic disease
benefits patient
Remove all visible tumor and pelvic
organs and give chemotherapy

Source: http://en.wikipedia.org/wiki/File:Ovarian_carcinoma.JPG

MTB S2CK

p. 352

MTB S2CK

p. 352

Testicular Cancer/Presentation

Testicular Cancer
Presentation
Diagnosis
g
Treatment

Young men: 20-40 years old


Painless lump in the scrotum
Next best step?
Transillumination
Scrotal Ultrasound

Differential
ee a d
diagnosis?
ag os s
Epididymitis
Hematocele
Varicocele

MTB S2CK

p. 352

Testicular Cancer/Diagnosis

Testicular Cancer/Diagnosis
Seminoma
Has NORMAL AFP
Can have elevated
bHCG
LDH correlated with
disease burden

Diagnostic Testing
Remove the whole testicle
with inguinal orchiectomy
Do not cut the scrotum,
which
hi h can spread
d the
th
disease
Needle biopsy of the
testicle is always a wrong
answer

MTB S2CK

AFP

Embryonal
AFP and bHCG
AFP and bHCG

Teratoma
AFP and bHCG

Source:Ed Uthman, MD.


http://en.wikipedia.org/wiki/File:Seminoma_of_the_Testis.jpg

p. 352

Testicular Cancer/Treatment

Staging is performed with:

1st- orchiectomy
2nd- radiation
3rd- chemotherapy

CT scan of the abdomen, pelvis, and


chest
Lymphatic channels in the retroperitoneum
spread testicular cancer into the chest

p. 353

Yolk sac or
Endodermal sinus

Choriocarcinoma

Testicular Cancer/Staging

MTB S2CK

Non-seminoma

http://emedicine.medscape.com/article/437966-clinical#a0218

Testicular cancer is one of the only


malignancies in which chemotherapy can
cure widely metastatic disease, including
spread into the brain

MTB S2CK

p. 353

Prevention of Invasive Cervical Cancer

Cervical Cancer
Prevention/early detection
Clinical Presentation
Treatment

Human papillomavirus (HPV)


vaccine is given to all women
between ages 11 and 26
Pap smear is performed
starting at age 21
Everyy 3 yyears,, with
cytology, until 30 years old
After age 30, every 5 years,
if HPV testing added
Stop at age 65 with
adequate screening history
and low risk
MTB S2CK

Source:http://www.cdc.gov/cancer/dcpc/prevention/vaccination.htm

Source: Nephron,
http://en.wikipedia.org/wiki/File:Adenocarcinoma_on_pap_test_1.jpg

p. 353

Detection of Cervical Cancer

Detection of Cervical Cancer

Atypical squamous cells of undetermined


significance (ASCUS)
HPV testing
If positive
If negative

Pap smear does not lower


mortality as much as
mammography or colonoscopy.

colposcopy and biopsy


repeat Pap in 6 mo

Low-grade
g
and high-grade
g g
dysplasia
y p
Colposcopy and biopsy

MTB S2CK

p. 353

MTB S2CK

Cervical Cancer/Presentation
Asymptomatic
Detected on Pap

p. 353

Cervical Cancer
Symptomatic
Abnormal vaginal
bleeding
Post coital bleeding
Abnormal discharge
Pelvic p
pain or fullness
Dysuria

The management of early cervical cancer =


hysterectomy

Source: Hic et nunc, http://en.wikipedia.org/wiki/File:Scheme_hysterectomy-en.svg

Ophthalmology
Conjunctivitis
The Red Eye (Ophthalmologic Emergencies)
Cataracts
Diabetic Retinopathy
Retinal Artery & Vein Occlusion
Retinal Detachment
Macula Degeneration

Ophthalmology
Conrad Fischer, MD
Associate Professor of Medicine
Touro College of Medicine
New York City

Comparison of Viral and Bacterial


Conjunctivitis
Viral conjunctivitis
Bacterial conjunctivitis
Bilateral
Watery discharge
Easily transmissible
Normal vision
Itchy
Preauricular adenopathy
No specific therapy

MTB S2CK

Unilateral
Purulent, thick discharge
Poorly transmissible
Normal vision
Not itchy
No adenopathy
Topical antibiotics

Conjunctivitis
Viral conjunctivitis

Bacterial conjunctivitis

Source: phil.cdc.gov

David C Cogan Ophthalmic Pathology Collection

p. 497

Conjunctivitis

Etiologies of The Red Eye


Conjunctivitis

Uveitis

Glaucoma

Abrasion

Autoimmune
diseases

Pain

Trauma

Photophobia

Fixed
midpoint
pupil

Feels like
sand in
eyes

Slit lamp
examination

Tonometry

Fluorescein
stain

Topical
steroids

Acetazolamide
Mannitol
Pilocarpine
Laser
trabeculoplasty

No specific
therapy
Patch not
clearly
beneficial

Presentation

The must know subjects


in ophthalmology are:
The red eye (emergencies)
Diabetic retinopathy
Artery and vein occlusion
Retinal detachment

Discharge
Eye findings

Normal pupils
Most
accurate test

Clinical
diagnosis
Best initial
therapy

Topical
antibiotics
MTB S2CK

p. 497

MTB S2CK

p. 498

Glaucoma/Chronic Glaucoma

Most often asymptomatic


Diagnosed by screening
Confirmation with tonometry
Elevated intraocular pressure

Glaucoma/Chronic Glaucoma
Treat to decrease production of aqueous humor or
increase drainage
Prostaglandin analogues Topical carbonic
Latanaprost
anhydrase
Travoprost
inhibitors
Bimatoprost

Topical
p
beta blockers
Timolol
Carteolol
Metipranolol
Betaxolol
Or
Levobunolol

MTB S2CK

p. 498

Glaucoma/Acute Angle Closure Glaucoma

Look for
Sudden onset
Extremely painful, red eye hard to palpation
Walking into dark room precipitates pain
because of p
pupillary
p
y dilation
Pupil doesnt react to light because its stuck
Cup-to-disc ratio > 0.3

MTB S2CK

Dorzolamide
Brinzolamide

Alpha-2 agonists
Apraclonidine

Pilocarpine
Laser

p. 498

Glaucoma/Acute Angle Closure Glaucoma


Conjunctival
vessels dilated
at corneal edge

Hazy
cornea
Author:Jonathan Trobe, M.D. Source: commons.wikimedia.org

MTB S2CK

p. 498

Glaucoma/Acute Angle Closure Glaucoma

Herpes Keratitis

Diagnosis
Confirmed with tonometry

Infection of cornea
Eye is red, swollen, and painful, but
dont use steroids
Steroids markedly increase production
of virus

Treatment
IV acetazolamide
IV mannitol (osmotically draws of fluid out)
Pilocarpine & beta blockers constrict pupil &
enhance drainage
Laser iridotomy
MTB S2CK

p. 498

Fluorescein stain confirms dendritic


pattern

MTB S2CK

p. 499

Herpes Keratitis

Cataracts

Treatment
Oral acyclovir, famciclovir, or valacyclovir
Topical trifluridine or idoxuridine

No medical therapy for cataracts


Surgically remove lens & replace with new
intraocular lens
New lens may automatically have bifocal
capability
Early cataracts are diagnosed with an
ophthalmoscope or slit-lamp exam
Advanced cataracts visible on exam

Beware of steroid use for


herpes keratitis
Steroids worsen condition

MTB S2CK

p. 499

MTB S2CK

Cataracts

p. 499

Diabetic Retinopathy
Annual screening exams INDISPENSIBLE!!!!
Detects retinopathy before visual loss occurs
Nonproliferative or background retinopathy is
managed by controlling glucose level
Most accurate test is
Fluorescein angiography

Proliferative: treated with laser photocoagulation


Vascular endothelial growth factor inhibitors
(VEGF) injected in some patients to control
neovascularization
nih.gov

MTB S2CK

p. 499

Diabetic Retinopathy

Retinal Artery and Vein Occlusion

Vitrectomy may be necessary to remove a vitreal


hemorrhage obstructing vision

Both present with


Sudden onset monocular visual loss
Cant make diagnosis without retinal examination
No conclusive therapy for either condition

MTB S2CK

p. 499

New blood vessel formation obscures vision. Source: Conrad Fischer, MD.

MTB S2CK

p. 500

Retinal Artery Occlusion

Retinal Vein Occlusion

Retinal vein occlusion leads to extravasation of blood into the retina.


Source: Conrad Fischer, MD.

Retinal artery occlusion presents with sudden loss of vision and a pale retina and dark
macula. Source: Conrad Fischer, MD.

MTB S2CK

p. 500

MTB S2CK

p. 500

Retinal Artery and Vein Occlusion

Retinal Detachment

Treatment of artery occlusion attempted with...


100% oxygen
Acetazolamide to intraocular pressure
Thrombolytics

Caused by
Trauma
Extreme myopia (changes shape of eye)
Diabetic retinopathy
Anything that pulls on retina can detach it
Presents with
Sudden onset of painless, unilateral loss
of vision
Described as curtain coming down

Macula
Mac
la is described as
cherry redin artery
occlusion because the
rest of retina is pale
MTB S2CK

p. 500

MTB S2CK

p. 501

Retinal Detachment

Macular Degeneration

Reattachment by mechanical methods


Surgery, laser, cryotherapy
Injection of expansile gas pushes retina back up
against globe of eye

MTB S2CK

p. 501

Sudden painless loss of vision like a courtain coming down.


Source: Conrad Fischer, MD.

MCC of blindness in older persons in U.S.


Idiopathic
Atrophic (dry) type and neovascular (wet) type
Far more common in older patients
Bilateral
N
Normal
l external
t
l appearance off eye
Central vision lost

MTB S2CK

p. 501

Macular Degeneration

Macular Degeneration

Neovascular disease
More rapid
More severe
New vessels grow between retina and underlying
Bruchs membrane
Neovascular or wet type causes 90% of permanent
blindness from macular degeneration
Atrophic macular
degeneration has no
proven effective therapy
MTB S2CK

p. 501

MTB S2CK

p. 502

Macular degeneration can be diagnosed only by visualization


of the retina. Source: Conrad Fischer, MD.

Macular Degeneration

Best initial therapy for neovascular disease


VEGF inhibitors
Ranibizumab or bevacizumab

Injected directly into vitreous chamber


every few weeks
Over 90% stop progression
1/3 improve vision

MTB S2CK

p. 502

Routine Management
of the Newborn

Pediatrics

Physical Exam
Apgar Score
Eye Care
Routine Screening and Prevention

Ryan Close, MD MPH


Resident Physician
Internal Medicine Pediatrics
Hospital of the University of Pennsylvania

Apgar Score

Physical Exam

Adults
Physical exams start
with vital signs...Newborns
Heart Rate (HR)

60 100 BPM

120160 BPM

Respiratory Rate
(RR)

12 24 BrPM

40 60 BrPM

Systolic Blood
Pressure (SBP)

120 mmHg

65 mmHg

Diastolic Blood
Pressure (DBP)

80 mmHg

50 mmHg

MTB S2CK p. 403

Apgar Score

Scores calculated at 1 minute and 5 minutes


Score at 1 minute
Represents conditions during labor and
delivery
Indicates need for resuscitation
Score at 5 minutes
Represents effectiveness of resuscitation
efforts
Prognostic of survival
MTB S2CK p. 404

Important part of physical exam


Translates physical exam into a score
Used for management decisions

Provides both short- and long-term


prognostic indicators

MTB S2CK p. 404

Apgar Score
0 points

1 point

2 points

Appearance
Skin color

Blue all over Pink torso/blue


extremities

Pink all over

Pulse
Rate

< 60 bpm or 60 100 bpm


asystole

> 100 bpm

Grimace
G
i
N response Grimace/
No
Gi
/
Reflex & irritability
feeble cry

S
Sneeze,
cough,
h
loud cry

Activity
Muscle tone

None

Some flexion

Active
movement

Respiration
Breathing

Absent

Weak, irregular

Strong

MTB S2CK p. 404

Apgar Score

Low Apgar score isnt predictive of


cerebral palsy

Knowing differences in management if


Apgar score is low

MTB S2CK p. 404

Prevention/Eye Care
A 3.9 kg male infant whose Apgar scores were 9 and
10 at 1 and 5 minutes, respectively, presents five
days after delivery because of red eyes. The delivery
was without any complications.
What is the most likely diagnosis at day 5 of life?
a. Chemical irritation
b. Neisseria gonorrhoeae
c. Chlamydia trachomatis
d. Group-B Streptococci
e. Herpes simplex

A 28-year-old G1PO woman delivers a 3.9 kg male infant


whose Apgar scores are 9 and 10 at 1 and 5 minutes,
respectively. The delivery was uncomplicated, and both
mother and child are in no acute distress.
Whats the most appropriate first step upon delivery of this
patient?
a. Intubate the child No signs of respiratory distress / high Apgar score
b. Send cord blood for arterial blood gas (ABG) Patient is stable
c. Suction the mouth and nose
d. Nasogastric tube (NGT) placement No need for decompression
e. Give prophylactic antibiotics No evidence of infection or sepsis
What we are not told:
- Is this child term, or pre-term?
- Vital signs ?

MTB S2CK p. 403

- Erythromycin ointment
- Tetracycline ointment
- Silver nitrate drops

Chemical
Irritation
Due to
silver
nitrate
Developing
countries
Not an
allergy

Prevention/Eye Care

Chlamydia
trachomatis

Neisseria
gonorrhea
Gram
Gramnegative
diplococci
Prevent with
ointments

Treat with
ceftriaxone

Not effectively
prevented by
prophylaxis
ointments

Herpes
Simplex
Treat with
systemic
acyclovir
and topical
vidarabine

Treat with oral


erythromycin

MTB S2CK p. 404 405

Prevention/Eye Care

Prevention & Screening/Vitamin K Deficiency

A 3.9 kg male infant whose Apgar scores were 9 and


10 at 1 and 5 minutes, respectively, presents five
days after delivery because of red eyes. The delivery
was without any complications.
What is the most likely diagnosis at day 5 of life?
a. Chemical irritation
b. Neisseria gonorrhoeae
c. Chlamydia trachomatis
d. Group-B Streptococci
e. Herpes simplex

Vitamin K deficient bleeding = hemorrhagic disease


of newborn
Neonate colon lacks normal bacterial flora that
produces vitamin K
Vitamin K is responsible for clotting factors II, VII,
IX,, and X as well as proteins
p
C and S
Vit K
Inactive
Clotting
Factors

Active
Clotting
Factors

Treatment: one IM injection of vitamin K


Oral doses will not work
MTB S2CK p. 405

Prevention & Screening/Vitamin K Deficiency


Intrinsic Pathway

Prevention & Screening/Testing

Extrinsic Pathway

XIIa
XI

XIa
IX
VIII

TF

IXa

VIIa

VIIIa

VII
II

Xa
V

Va

Fibrinogen
IIa
Thrombin

Fibrin

Newborns should be screened for:


Phenylketonuria (PKU)
Congenital adrenal hyperplasia (CAH)
Biotinidase
Beta-thalassemia
Galactosemia
G l t
i
Hypothyroidism
Homocysteinuria
G6PD deficiency
Hearing test
MTB S2CK p. 405 406

Transient Conditions of the Newborn

The Big Transient Three

Medical Conditions
of the Newborn
Transient conditions
Polycythemia
Tachypnea
Hyperbilirubinemia
Delivery associated injuries
Newborn infections

Polycythemia

Tachypnea

Hyperbilirubinemia
MTB S2CK

p. 406 407

Transient Polycythemia of the Newborn

Transient Polycythemia of the Newborn

Polycythemia

Transient Polycythemia

Increased RBCs

Benign
Most often related to cord clamping

Epo!

Primary vs. Secondary


Primary: Normal or low EPO levels
Secondary: High EPO levels

Source: James Van Rhee

Hypoxia

Polycythemia of the Newborn


Can be very harmful

EPO!

Hyperviscosity, decreased perfusion, thromboses


LGA, SGA, and IDM
MTB S2CK

p. 406

MTB S2CK

p. 406

Transient Tachypnea of the Newborn (TTN)

Transient Tachypnea of the Newborn (TTN)

Tachypnea

The Takeaway:
Benign condition
Term infants
Delivered via C-section
Oxygen, antibiotics, and watch closely
Watch for

60 BrPM (40 60 is normal)

TTN or Respiratory Distress Syndrome II (RDS II)

Benign condition 1% to 2% of newborns


Causes
Excess remaining lung fluid
Pulmonary immaturity
Surfactant deficiency/insufficiency

Management

Sepsis

Oxygen and antibiotics


Work-Up: CBC and chest radiograph
MTB S2CK

p. 406 407

MTB S2CK

Transient Hyperbilirubinemia
Hyperbilirubinemia
Production > Elimination

p. 406 407

Transient Hyperbilirubinemia
increased TSB

From increased production, decreased elimination,


or both

Indirect = Unconjugated bilirubin


Direct = Conjugated bilirubin
(Total Bili) (Direct Bili) = Indirect Bili

Any hyperbilirubinemia < 24 hr of life


Evaluation
Any conjugated hyperbilirubinemia
Evaluation

Transient Hyperbilirubinemia
Benign and very common (~60% newborns)
Peaks at 2-3 days of life
Increased production of unconjugated bilirubin
MTB S2CK

p. 407

MTB S2CK

p. 407

Transient Conditions of the Newborn

Delivery Associated Injuries

The Big Transient Three

Types of injuries
Subconjunctival hemorrhage
Skull fracture
Scalp injuries
Brachial palsies
Clavicular
Cl i l fracture
f t
Facial nerve palsy

Polycythemia

Tachypnea

Microhemorrhages
Benign
Caput succedaneum
Cephalohematoma

Brief paralysis
of facial nerve

Hyperbilirubinemia
MTB S2CK

p. 406 407

MTB S2CK

p. 407

Brachial Plexus Injuries

Brachial Plexus Injuries

General
Macrosomic infants (e.g., IDM)
Shoulder dystocia
Duchenne-Erb Paralysis
90% of brachial palsies
C5 C6
Waiters tip
Cannot abduct or externally rotate
Klumpke Paralysis
C7 T1
Claw hand +/- Horner syndrome
MTB S2CK

p. 407 408

Clavicular Fracture

Physical Exam

Most common newborn fracture


Usually result of:

Physical exams start with vital signs...

Shoulder dystocia

Systolic
Blood
Pressure
(SBP)

Diastolic
Blood
Pressure
(DBP)

60-100 BPM 16-24 BrPM

120 mmHg

80 mmHg

120-160
BPM

65 mmHg

50 mmHg

Age Group

Heart Rate
(HR)

Adult
Newborns

Respiration
Rate (RR)

Best diagnostic tool:


Radiograph
g p

Most appropriate management:

40-60 BrPM

Immobilize

MTB S2CK

p. 408

Neonatal Sepsis

ToRCH Infections

Early

Late

GBS
E. Coli
Listeria

Staphylococci

E. Coli
GBS
IVF
Cultures
Antibiotics

Ampicillin
Gentamicin
Cefotaxime*
MTB S2CK

p. 433

Ampicillin
Gentamicin
Cefotaxime*

Type

Presentation

Diagnostic tests Treatment

Chorioretinitis, hydrocephalus,
Toxo
plasmosis ring enhancing lesion

Initial: IgM
Most accurate: PCR

Pyrimethamine
and sulfadiazine

Syphilis

Rash on palms/soles, snuffles,


frontal bossing, Hutchinson 8th
n. palsy, saddle nose

Initial: VDRL / RPR


Most accurate: FTA
ABS / Dark Field

Penicillin IV

Rubella

PDA, cataracts, deafness,


hepatosplenomegaly, low plts,
elevated bilirubins

Elevated maternal
rubella IgM with
clinical picture

Supportive

CMV

Periventricular calcifications,
microcephaly, chorioretinitis,
hearing loss

Initial: Urine/saliva
viral titers
Most accurate: PCR

Ganciclovir

Herpes

Week 1: Shock and DIC


Week 2: Vesicular skin lesions
Week 3: Encephalitis

Initial: Tzanck smear


Most accurate: PCR

Acyclovir and
supportive care

MTB S2CK

p. 434

Amniotic Fluid Abnormalities

Common Abnormalities
of the Newborn
Amniotic Fluid Abnormalities
Abdominal Abnormalities
Genitourinary Abnormalities

Amniotic Fluid
80% from mother
20% from infant
Problems
Polyhydramnios (too much)
Oligohydramnios (too little)

MTB S2CK

p. 408

Amniotic Fluid Abnormalities

Amniotic Fluid Abnormalities

Polyhydramnios
Too much fluid
Overproduction / Decreased resorption
CNS malformations

Polyhydramnios
GI malformations

MTB S2CK

p. 408

Esophageal atresia

MTB S2CK

p. 408

Amniotic Fluid Abnormalities

Abdominal Abnormalities

Oligohydramnios
Too little fluid (Low AFI)
Under production
Causes

A premature infant born at 28 weeks in respiratory


distress with grunting, nasal flaring, and use of
accessory muscles. Bowel sounds heard upon
auscultation of the back and chest X-ray shows air
fluid levels in the chest.
Which of the following is the most likely diagnosis?
a. Hydrocele
b. Gastroschisis
c. Diaphragmatic Hernia
d. Hiatal Hernia
e. Omphalocele

Post-term pregnancies
Renal agenesis and renal failure

Cord Compression
ACE-inhibitors
Potters syndrome
MTB S2CK

p. 408

MTB S2CK

p. 409

Abdominal Abnormalities

Abdominal Abnormalities
Which of the following is the most likely diagnosis?
a.
b.
c.
d
d.
e.

Urological issue not respiratory issue


Hydrocele
Abdominal wall defect
Gastroschisis
Diaphragmatic Hernia
M
More
common iin GERD seen in
i adults
d lt
Hi t l Hernia
Hiatal
H i
Omphalocele Abdominal wall defect

Source: Niket Sonpal, MD

MTB S2CK

p. 409

Abdominal Abnormalities
Diaphragmatic Hernia
Congenital defect in diaphragm
Two types: Bochdalek and Morgagni
LEFT side most common

Key findings:

Respiratory distress
Scaphoid abdomen
Bowel sounds in chest
Abnormal chest radiograph

First step in management

MTB S2CK

p. 409

Abdominal Abnormalities
Omphalocele
Midline wall defect
With sac covering
Associations

Imperforate anus
Congenital heart defects (50%)
Conjoined twins
Trisomy 18 (Edwards
syndrome)
Beckwith Wiedemann
syndrome

Gastroschisis

Lateral wall defect


No sac covering
Atresias
Surgery is necessary

Surgery is necessary

Intubation
MTB S2CK

p. 409

Abdominal Abnormalities

MTB S2CK

p. 410

Abdominal Abnormalities/Tumors
Wilms Tumor
Most common primary renal malignancy in peds
Presents with

An elevated AFP in abdominal wall defects


The MCC of elevated AFP is incorrect dating

Umbilical hernias: Similar to omphaloceles

Asymptomatic flank mass


Hematuria
Hypertension and aniridia

First step in evaluation: Abdominal ultrasound


MC diagnostic test: Computed tomography
Relationships with syndromes
WAGR, Denys-Drash, Beckwith-Wiedemann

Treatment: Surgery + chemotherapy + radiation


MTB S2CK

p. 410

MTB S2CK

p. 410

Abdominal Abnormalities/Tumors

Genitourinary Abnormalities

Neuroblastoma
Very common among children
Often involves adrenal gland
Presents with

Hydrocele
Painless, benign, fluid-filled
Cryptorchidism
Undescended testis, increased cancer risk
Surgical correction after 6 months
Hypospadias
Ventral surface opening, surgery
Epispadias
Dorsal surface opening, surgery

Painful abdominal mass


Neurological findings
Opso(myo)clonus

Diarrhea

Diagnostic keys
Urine catecholamines and their metabolites
Vanillyl Mandelic Acid (VMA)
Homovanillic acid (HVA)
MTB S2CK

p. 411

MTB S2CK

p. 411

Cyanotic Heart Defects

Cyanotic Heart Defects

Tetralogy of Fallot

Tetralogy of Fallot
Transposition of the Great Vessels
Hypoplastic Left Heart Syndrome
Truncus Arteriosus
Total Anomalous Pulmonary Venous
Return

Tetralogy of Fallot

The most common cyanotic heart


defect in children
Four aspects
p
Pulmonary stenosis
VSD
Overriding aorta
Right ventricular hypertrophy
MTB S2CK

p. 412

Tetralogy of Fallot/Signs & Symptoms

Tetralogy of Fallot

Cyanosis
Lips and extremities
Squatting
Increases systemic pressure
Shunts blood to pulmonary circulation
Holosystolic
H l
t li murmur
VSD
There are 3 holosystolic murmurs
Mitral regurgitation (MR)
Tricuspid regurgitation (TR)
Ventricular septal defect (VSD)
MTB S2CK

p. 412 413

Tetralogy of Fallot

Transposition of the Great Vessels

Two separate circulations


Right heart/systemic circulation
Left heart/pulmonary circulation
Defect dependent
Patent ductus arteriosus (PDA)
VSD
Atrial septal defect (ASD)
Most common cyanotic lesion of neonates
MTB S2CK

Transposition of the Great Vessels

p. 413

Transposition of the Great Vessels

Treatment
Prostaglandin E1
Surgery

MTB S2CK

p. 413

Hypoplastic Left Heart Syndrome

Hypoplastic Left Heart Syndrome

Syndrome
Absent pulses
Right ventricular heave
Mild cyanosis/gray

MTB S2CK

p. 414

Truncus Arteriosus

Truncus Arteriosus

One Great Vessel


The less common heart defect (2%)
NOT dependent on PDA
Mild cyanosis
Big problem: Pulmonary hypertension
Treat with
Surgery!

MTB S2CK

p. 414

Total Anomalous Pulmonary Venous Return

Abnormal pulmonary venous return


Pulmonary veins
Right atrium
PFO dependent
Right heart overload
Two types:
With obstruction
Without obstruction

MTB S2CK

p. 414 415

TAPVR 2
Sign/Symptoms

Tests

TAPVR with
obstruction

Early in life with


respiratory
distress and
severe cyanosis

CXR shows
pulmonary
edema
Echo is test of
choice

TAPVR without
obstruction

Presents later
Age 1 2 years
with heart
failure

S
Surgery
CXR shows
snowman sign
Echo is test of
choice

MTB S2CK

Treatment
Surgery

p. 415

10

Cyanotic Heart Defects


R

L Shunt

PDA dep

VSD

Surgery

TOF

Acyanotic Heart Defects

TGV
Hypoplastic LH
Truncus Art
TAPVR

Acyanotic Heart Defects

Heart murmur in a child

VSD

3-year old female brought in because her parents say


she wont eat anymore. Upon physical examination, a
loud pansystolic murmur is appreciated. The child
appears small for her age, but her records dont show
any maternal or delivery complications.

ASD

Which of the following is the most likely finding on EKG?

PDA
Coarctation of the aorta

a.
b.
c.
d.
e.

Right ventricular hypertrophy


RBBB
ST-segment elevation
QT-prolongation
P-wave inversion

MTB S2CK

Ventricular Septal Defect

p. 415

VSD, Murmurs, and Auscultation

There are 3 holosystolic murmurs...


MR
TR
VSD
VSD

VSD is the most common


congenital heart defect
MTB S2CK

p. 415

11

Ventricular Septal Defect


Pathophysiology

Ventricular Septal Defect

Left-to-Right shunt (opposite of cyanotic defects)


Pulmonary hypertension

Presentation
Dyspnea with distress
Loud pulmonic S2
High-pitched holosystolic murmur

Tests
CXR: Findings are not diagnostically helpful
Best initial test is an echocardiogram
Most diagnostic (definitive) test is a cardiac
catheterization
MTB S2CK

p. 416

Heart murmur in a child

VSD

3-year old female brought in because her parents say


she wont eat anymore. Upon physical examination, a
loud pansystolic murmur is appreciated. The child
appears small for her age, but her records dont show
any maternal or delivery complications.
Which of the following is the most likely finding on EKG?

a.
b.
c.
d.
e.

Right ventricular hypertrophy


RBBB
ST-segment elevation
QT-prolongation
P-wave inversion

MTB S2CK

p. 415

VSD & Eisenmenger Syndrome


3-year old female brought in because her parents say
she wont eat anymore. Upon physical examination, a
loud pansystolic murmur is appreciated. The child
appears small for her age, but her records dont show
any maternal or delivery complications.
Which of the following is the most likely finding on EKG?

a.
b.
c.
d.
e.

Right ventricular hypertrophy


RBBB ASD or ischemic disease
ST-segment elevation Myocardial infarction
Electrolyte disturbances
QT-prolongation
P-wave inversion Atrial arrhythmias

MTB S2CK

p. 415

12

Atrial Septal Defect

Atrial Septal Defect

Types of ASDs
Primum, secundum, and sinus venosus
2xs common in men

Usually asymptomatic
Fixed wide-splitting S2 vs. Physiologic splitting
Tests
Best initial test is an echocardiogram (bubble study)
Most diagnostic test is a cardiac catheterization

Prognosis
Most close without intervention

MTB S2CK

p. 416 417

Atrial Septal Defect

Patent Ductus Arteriosus

Without closure

Failure of closure after 12 hours


Left-to-Right shunt
Presentation

Atrial enlargement
Dysrhythmias
Embolic Risk

Treatment
Surgical

Machinery-like murmur
Wide pulse pressures
Bounding pulses

Tests
Best initial test is an echocardiogram
Most diagnostic test is a cardiac catheterization

Treatment
NSAIDS (indomethacin)
MTB S2CK

Patent Ductus Arteriosus

p. 417

Patent Ductus Arteriosus

13

!!
!
!

!!

Jaundice,

Hyperbilirubinemia

Gastroenterology,,Part,1,
Jaundice,)
Esophageal,)Duodenal,)Choanal)Atresia)
Pyloric)Stenosis)
Hirschsprungs)Disease)

Production > Elimination

Jaundice very common in newborns


Red flags:
Jaundice < 24 hrs of life ! Evaluation
Direct hyperbilirubinemia ! Evaluation
Hyperbilirubinemia after 3 weeks ! Evaluation

Jaundice never common in children


Physiologic Benign
Primary concern ! Kernicterus
MTB)S2CK))A))p.)419)

The,Approach,to,Jaundice,

The,Approach,to,Jaundice,

Child
or Newborn?

Child

Child
or Newborn?

Evaluate

Newborn
24 hrs
of life?

Evaluate

Newborn
Y

24 hrs
of life?

Evaluate

Evaluate

Direct vs.
Indirect

Direct vs.
Indirect

The,Approach,to,Jaundice,

The,Approach,to,Jaundice,

Direct vs.
Indirect

Direct

Child

Direct vs.
Indirect

Indirect

Direct

Indirect

The,Approach,to,Jaundice,

The,Approach,to,Jaundice,

Direct

Direct

Indirect

Evaluate

Sick
appearing?

N
Y

2 weeks of life

Breast milk
jaundice?

1 2 wks

Transient vs.
Breast feeding
jaundice

< 1 wk

Hepatocyte:
- Infection
- Sepsis
- Endocrine
- Genetic

Obstruction:
- Biliary
atresia

The,Approach,to,Jaundice,

Jaundice,

Kernicterus

Bilirubin encephalopathy
Basal gangalia, hippocampus, subthalamic
nucleus
Multiple neurological abnormalities

InD

Prevention
Phototherapy vs. exchange transfusion

MTB)S2CK))A))p.)419)
)

MTB)S2CK))A))p.)419)

Atresias,
Esophageal,Atresia,

Esophageal,Atresia,Types,
Choanal,Atresia,

Duodenal,Atresia,

)Blind)esophagus)

80 90%
of cases

MTB)S2CK))A))p.)420)A)424)

20% of all cases = unimportant

MTB)S2CK))A))p.)420)

Atresias,
Esophageal,Atresia,

Atresias,
Choanal,Atresia,

Duodenal,Atresia,

)Blind)esophagus)
)Presents)with:)

Esophageal,Atresia,

Choanal,Atresia,

)Blind)esophagus)
)Presents)with:)

)Buccopharyngeal))
)membrane)
)(+))resp)distress)
)Best)iniKal)test:)

Frothing, cough,
cyanosis, and
respiratory distress
with feeds

Frothing, cough,
cyanosis, and
respiratory distress
with feeds

No)resp)distress)at,
rest,
)IniKal)test)
)CXR)

No)resp)distress)at,
rest,
)IniKal)test)
)CXR)

)Concerns)
)AspiraKon)PNA)
)

Treatment of choice for aspiration PNA


is penicillin-like antibiotics (amoxicillinclavulanate, piperacillin-tazobactam) or
clindamycin

MTB)S2CK))A))p.)420))

Atresias,
Choanal,Atresia,

)Blind)esophagus)
)Presents)with:)

)Buccopharyngeal))
)membrane)
)(+))resp)distress)
)Best)iniKal)test:)

No)resp)distress)at,
rest,
)IniKal)test)
)CXR)

)Concerns)

Newborns are
obligate nose
breathers

Pass NG tube

)Most)diagnosKc:)
CT Scan

CT sees bone
better

)First)step)in))
)management:)
Secure airway!

)AspiraKon)PNA)
)

MTB)S2CK))A))p.)420)A)424)

Duodenal,Atresia,

Esophageal,Atresia,

Frothing, cough,
cyanosis, and
respiratory distress
with feeds

)Concerns)

Duodenal,Atresia,

Pass NG tube

)Most)diagnosKc:)
CT Scan

)First)step)in))
)management:)
Secure airway!

Duodenal,Atresia,
)Failed)duodenal))
)canalizaKon)
)NO,resp)distress))
)Bilious,vomiKng))
)IniKal)test)
AXR
Double-Bubble
Double-bubble +
distal gas =
volvulus

)AspiraKon)PNA)
)

Source: James C. Pascual

MTB)S2CK))A))p.)424A425)

Atresias,

Pyloric,Stenosis,

Esophageal,Atresia,

Choanal,Atresia,

)Blind)esophagus)

)Buccophayngeal))
)membrane)
)(+))resp)distress)
)Best)iniKal)step:)

)Presents)with:)
Frothing, cough,
cyanosis, and
respiratory distress
with feeds

No)resp)distress)at,
rest,
)IniKal)test)
)CXR)

)Concerns)
)AspiraKon)PNA)
)

MTB)S2CK))A))p.)424A425)

Pass NG tube

)Most)diagnosKc:)
CT Scan

)First)step)in))
)management:)
Secure airway!

Duodenal,Atresia,
)Failed)duodenal))
)canalizaKon)
)NO,resp)distress))
)Bilious,vomiKng))
)IniKal)test)
AXR
Double-Bubble

Hypertrophy of pyloric sphincter


Not atresia

Presents
3 weeks of age
Nonbilious vomiting ! projectile!
Small olive pit size epigastric mass
Dehydration contraction alkalosis

)TRISOMY)21)
)First)step)in))
)management:)
IVF
MTB)S2CK))A))p.)421)

Pyloric,Stenosis,

Pyloric,Stenosis,

Contraction Alkalosis

Tests

Loss of ECF

Fluid Loss
Vomiting

Loss of H+
Loss of K+
Loss of Cl-

Ultrasound: more
diagnostic than X-ray
Radiograph is helpful

RAAS
Metabolic Alkalosis
High pH, High Bicarb, High pCO2

Hypochloremic
Hypokalemic

Treatment
IVF
NG Tube
Pyloric myotomy

MTB)S2CK))A))p.)421)

Source: Gergory J. Hall

Hirschsprung,Disease,
Congenital lack of ganglionic cells in large bowel
KEY clinical feature
Failure or delayed passage of meconium
Progresses to large bowel obstruction (LBO)
Abdominal distention

Best initial diagnostic test: AXR


Most diagnostic test: Rectal suction biopsy
NOT barium study
NOT manometry

Treament
Surgery
MTB)S2CK))A))p.)422A423)

!
!
!
!
!

Bilious Vomiting

Meckels Diverticulum
Rule of 2s

Bilious
Vomiting
Duodenal
Atresia

Within 1st day


Initial test:
AXR

Volvulus

Intussusception

Within 1st year


Initial test:
AXR

Within 1st year


Initial test:
US
Doughnut

First Step
IVF!
Treament
Surgery

First Step:
IVF!
Treatment
Air Enema

Double

bubble
First Step
IVF!
Treament
Surgery
MTB S2CK

p. 424 426

Painless rectal
bleeding

MTB S2CK

Diarrhea and Gastroenteritis

Katsumi M. Miyai, M.D., Ph.D., Regents of the


University of California. Used with permission.

p. 426 427

Diarrhea and Gastroenteritis

The most important questions to ask:


Is this chronic or acute?
Infectious or not?
Bloody or not?
How sick is the child?

MTB S2CK

2% prevalence
2 years old
2 ft proximal to
ileocecal valve
2 inches long
2 types of ectopic tissue
Males 2x more affected
2% symptomatic

Infectious
Diarrhea

Viral

Parasites

Fungal

Giardia
Cryptosporidia

Candida spp
Histoplasma

Bacterial

Salmonella
Shigella
C difficile
C.
E. Coli
Campylobacter
Yersinia

p. 427

Diarrhea and Gastroenteritis

Diarrhea and Gastroenteritis

Infectious
Diarrhea

Viral

Rotavirus
Most
M t common
Winter
Symptoms:
Fever, emesis
NO blood
< 7 days
Viral prodrome
Vaccine

Adenovirus
Endemic
E d i
Year round
Symptoms:
Fever, emesis
NO blood
> 7 days
Viral prodrome

Small, round
Norwalk
N
lk
EPI demic
Symptoms:
Explosive
Cramping, pain
Short lived
1 2 days

Management
Hydration is key
Almost always the answer
Antibiotics for suspected bacterial
i f ti
infection
WBC or blood in stool
NEVER use antidiarrheal meds in
these patients
MTB S2CK

p. 428

15

Large for Gestational Age/Macrosomia

Endocrinology
Infants of Diabetic Mothers
Congenital
g
Adrenal Hyperplasia
yp p
(CAH)
(
)
Vitamin D Deficiencies

A 10.5-pound infant is born to a mother with Type I diabetes. Upon


examination of newborn, he is shaking, and a holosystolic murmur is
heard over precordium. The babys right arm is adducted and
internally rotated. His lab findings show an elevated bilirubin.
Which of the following is the most appropriate next step in
management?

a.
b.
c.
d.
e.

IV insulin
Blood sugar level
Serum calcium levels
Serum TSH
CT head and neck

MTB S2CK

Infants of Diabetic Mothers (IDM)

Maternal
Hyperglycemia

MTB S2CK

Fetal / Infant
Hyperglycemia

Infants of Diabetic Mothers (IDM)

Infant
Hyperinsulinemia

p. 430

IDM/Macrosomia
Macrosomia = weight 4500 g
Large for gestational age (LGA) = top 90th percentile
Causes in IDM
Oversupply of AAs, glucose, etc.
Insulin is a growth factor
Consequences
C
Trauma
Risk of C-section
Treatment
None Prevention!
MTB S2CK

p. 430

p. 429

Four primary issues...


Macrosomia
Hypoglycemia
Electrolyte abnormalities
Jaundice

MTB S2CK

p. 430

IDM/Hypoglycemia

Maternal
Hyperglycemia

Infant
Hyperglycemia

Infant
Hyperinsulinemia

Birth

MTB S2CK

p. 430

16

IDM/Hypoglycemia

IDM/Electroyte abnormalities

Direct consequence of maternal hyperglycemia


Hypertrophied pancreatic beta-cells
Insulin overproduction
Hyperresponsive
After birth:
No
N changes
h
iin iinsulin
li production/activity
d ti / ti it
Consequences
Severe hypoglycemia
Seizures
Treatment
Monitoring + Glucose

Hypocalcemia
Hypomagnesemia / Hyperphosphotemia
Twitching and Tremulousness
Cardiac arrhythmias
Calcium and Magnesium levels linked
Always
Al
check
h kb
both
th
Correct together

MTB S2CK

p. 430

MTB S2CK

p. 430

IDM/Jaundice
One-third of IDMs will develop jaundice
Unconjugated/Indirect hyperbilirubinemia
Overproduction of bilirubin
Resolving hematomas
Polycythemia
Treatment
T t
t
Phototherapy

MTB S2CK

p. 430

A 10.5-pound infant is born to a mother with Type I diabetes. Upon


examination of newborn, he is shaking, and a holosystolic murmur is
heard over precordium. The babys right arm is adducted and
internally rotated. His lab findings show an elevated bilirubin.
Which of the following is the most appropriate next step in
management?

a.
b.
c.
d.
e.

IDMs have hyperinsulinemia

IV insulin
Blood sugar level
Serum calcium levels
Serum TSH
CT head and neck

MTB S2CK

Check, but not most immed.


Good test, but not relevant
No concern trauma or bleed

p. 429

IDM/Other

Congenital Adrenal Hyperplasia (CAH)

Other abnormalities
Small left colon syndrome
Respiratory distress syndrome (RDS)
Cardiac abnormalities

Cholesterol
Pregnenolone

17-OHg
Pregnenolone

DHEA
Androstenediol

17-OHProgesterone

Deoxycortisol

Androstenedione

Estrone

Testosterone

Estradiol

17 -hydroxylase deficiency

Cortisol

11 -hydroxylase deficiency

21-hydroxylase deficiency
MTB S2CK

p. 430

17

CAH: 17 Hydroxylase Deficiency

Congenital Adrenal Hyperplasia


17 hydroxylase def

Cholesterol
Aldosterone

Cortisol

21 hyroxylase def

Aldosterone

Cortisol

Sex hormones
HYPERtensive

Classic S2CK: teenage girl


presents with delayed puberty
and incidentally found to have
elevated BP

Sex Development
p
Testosterone

Girls: nml at birth


Boys: pseudo
hermaphroditism

Estradiol

11 hydroxylase def

Electrolytes
Hypokalemia

17 -hydroxylase deficiency

MTB S2CK

p. 430

MTB S2CK

21 Hydroxylase Deficiency

p. 430

Congenital Adrenal Hyperplasia

Cholesterol
Aldosterone

Cortisol

17 hydroxylase def

21 hyroxylase def

Aldosterone

Cortisol

Sex hormones
HYPERtensive

Aldosterone

Cortisol

Sex hormones
HYPOtensive

Sex Development
p
Testosterone

Girls: nml at birth


Boys: pseudo
hermaphroditism

Estradiol

Electrolytes
Hypokalemia

21-hydroxylase deficiency
MTB S2CK

p. 430

MTB S2CK

11 Hydroxylase Deficiency
Cholesterol

11-DOC

Cortisol

Electrolytes
Hyponatremia
Hypochloremia
Hyperkalemia

17 hydroxylase def

21 hyroxylase def

11 hydroxylase def

Aldosterone

Cortisol

Sex hormones
HYPERtensive

Aldosterone

Cortisol

Sex hormones
HYPOtensive

Aldosterone

Cortisol

Sex hormones

11 DOC
HYPERtensive

Estradiol

Girls: nml at birth


Boys: pseudo
hermaphroditism

Electrolytes
Hypokalemia

11 -hydroxylase deficiency

p. 430

Girls: virilized
Boys: nml at birth

p. 430

Sex Development
p

MTB S2CK

Salt wasting Shock

Sex Development

Congenital Adrenal Hyperplasia

Aldosterone

Testosterone

11 hydroxylase def

MTB S2CK

Salt wasting Shock

Sex Development
Girls: virilized
Boys: nml at birth

Electrolytes
Hyponatremia
Hypochloremia
Hyperkalemia

Sex Development
Girls: virilized
Boys: nml at birth

Few electrolytes
abnormalities

p. 430

18

Congenital Adrenal Hyperplasia

Rickets

Cholesterol

Disorder of children
Soft and weak bones
fractures
Vitamin D, calcium, and phosphate
Children are particularly susceptible
Rapidly growing bones
Breast
B
t milk
ilk deficiency
d fi i
in
i vitamin
it i D
Prophylaxis with vitamin D supplements

Pregnenolone

17-OHg
Pregnenolone

DHEA
Androstenediol

17-OHProgesterone

Deoxycortisol

Androstenedione

Estrone

Testosterone

Estradiol

17 -hydroxylase deficiency

Cortisol

11 -hydroxylase deficiency

21-hydroxylase deficiency

Rickets

Bone Metabolism/Simplified

Bone Metabolism/Simplified

Vitamin D

PTH

Calcium

Phosphate

Rickets

Vitamin D

PTH

Calcium

Phosphate

Source: Paul M Michaud

MTB S2CK

p. 432

19

Rickets
Disorder of children
Soft and weak bones
fractures
Vitamin D, calcium, and phosphate
Children are particularly susceptible
Rapidly growing bones
Breast
B
t milk
ilk deficiency
d fi i
in
i vitamin
it i D
Prophylaxis with vitamin D supplements
Treat with vitamin D and calcium supplements

MTB S2CK

Pulmonary
Croup
Epiglottitis
pg
Whooping Cough
Asthma

p. 431

Croup

Anatomy of upper airway disease

2-year-old brought in by daycare provider for severe cough,


fever, and runny nose. The childs cough sounds like a bark
and shes in obvious respiratory distress. Upon physical
examination, the child refuses to lie flat. A chest X-ray shows a
positive steeple sign.
What is the most appropriate next step in management?

a.
b.
c.
d.
e.

Intubate
Racemic epinephrine
Empiric antibiotics
Acetaminophen
CT-scan of neck

Not necessary in croup


Not first step in improving airway
Not first step in improving airway
Not first step in improving airway

Croup
MTB S2CK

commons.wikimedia.org. Used with permission

p.435

Croup

Epiglottitis

Laryngotracheitis or
Laryngotracheobronchitis
Infection of upper airway
Subglottic space
Viral
Parainfluenza or RSV
Presentation
Triad: barking cough,
coryza, and stridor
Respiratory distress:
accessory muscle use
CXR: Steeple sign
MTB S2CK

p. 435

Treatment
Moderate severity:
Steroids
Severe: Racemic
epinephrine and steroids
Think bacterial causes:
(1) older kids or
(2) those unresponsive to
racemic epinephrine

The MOST common


cause of stridor in
children

4-year-old child brought in by daycare provider because hes


extremely irritable and refuses to eat. He refuses to lean back,
speaks in muffled words, looks extremely ill, and is drooling.
Chest radiograph shows a positive thumb-print sign.
What is the most appropriate next step in management?

a.
b.
c.
d.
e.

Intubate
Racemic epinephrine
Empiric antibiotics
Acetaminophen
CT-scan of the neck

MTB S2CK

Only for croup


Good not most immeidate
Fever is not a concern
Further imaging not warranted

p. 436

20

Anatomy of upper airway disease

Epiglottitis
Upper-airway infection and emergency
MCC: Bacterial
Non-vaccinated: H. influenzae type-B
Vaccinated: Streptococcus species and nontypeable
H. influenzae
Presentation
Fever,
F
drooling,
d li
respiratory
i t
distress
di t
NO coryza, NO prodrome, NO cough
Management
Transfer to O.R.
INTUBATE
Start empiric antibiotics: ceftriaxone or cefuroxime
No imaging required

Epiglottitis

Croup
commons.wikimedia.org. Used with permission

Epiglottitis

MTB S2CK

p. 436

Pertussis
Bordetella pertussis

Whooping Cough

Gram-negative, non-invasive
Causes ciliary paralysis
Three Stages:

Source: MS-4 USU

MTB S2CK

Pertussis/Stages

p. 437

Pertussis
Bordetella pertussis

~ 14 days

14 30 days

~ 14 days

Catarrhal

Paroxysmal

Convalescent

Rhinorrhea,
congestion,
cold sxs
sx s
cold
Most
contagious
time period
Only time abx
helpful to
patient
MTB S2CK

p. 437

Severe coughing
Post-tussive emesis
Usually no fever
Low requirement for
admission
Abx prescribed to
reduce transmissibility

Prolonged
resolution of
symptoms
Coughing fits
remain less
respiratory
distress

Whooping Cough

Gram-negative, noninvasive
Causes ciliary paralysis
Three Stages:
Catarrhal Stage: 14 days
Runny
R
nose, congestion,
ti
URI Symptoms
S
t

Most contagious time period


Paroxysmal Stage: 14-30 days
Severe coughing, posttussive emesis
Convalescent Stage: 14 days
Resolution of symptoms
MTB S2CK

p. 437

21

Pertussis

Viral Infections of Childhood

Diagnosis

Virus

Presentation

Diagnostic tests Treatment

Varicella

Initial: Tzanck smear


Most accurate:
culture

Rubeola
(Measles)

Multiple highly pruritic


vesicular rash; starts on face;
fever/malaise
Cough, Coryza, and
Conjunctivitis; Koplik spots

Fifths
d
disease

Fever and URI symptoms;


l
slapped
d cheeks
h k rash
h

Clinical diagnosis

Supportive

Roseola

Fever and URI progressing Clinical diagnosis


to diffuse rash

Supportive

Mumps

Fever precedes classic


parotid gland swelling with
possible orchitis

Supportive

This is a clinical diagnosis


CXR is helpful
PCR useful where available
Management
Patient:
P ti t Primarily
Pi
il supportive
ti

Supportive

Initial: Clinical
Supportive
Most accurate: IgM

Catarrhal stage azithromycin/erythromycin


Everybody gets treated
Close contacts: Macrolides
The community: Vaccination campaigns
MTB S2CK

p. 437

MTB S2CK

p. 434

Asthma

Asthma

The most common chronic disease in children


Pathophysiology
Reversible obstruction
Hyperresponsiveness
Inflammation
Remodeling
Diagnosis
Decreased FEV1 and
No single available test
FEV1/FVC ratio!
Clinical
Pulmonary Function Tests (PFTs)
Bronchoprovocation
CXR

Treatment

MTB S2CK

p. 129 130

Clinical diagnosis

Avoidance of triggers
Short-acting 2-agonists (SABA)
Inhaled corticosteroids (ICS)
Long-acting 2-agonists (LABA)
Leukotriene
L k ti
antagonists
t
i t ((modifiers)
difi ) (LTRA)

MTB S2CK

p. 131 132

Asthma/Treatment

SABA as
needed

ADD
Low-dose
ICS

ADD
LABA,
LTRA or
LTRA,
move to
mediumdose ICS

High-dose
ICS
and
LABA

High-dose
ICS
and
LABA
and
Oral
steroids

Severity of Symptoms
MTB S2CK

p. 131 132

22

Childhood Disorders
Mental Retardation
Pervasive Developmental Disorders
Attention Deficit Hyperactivity Disorder
Tourette Disorder

Psychiatry
Sam Asgarian, MD/MBA
Class of 2012
Tulane University

Mental Retardation

Types of Mental Retardation

To determine the level of retardation patients must


exhibit deficits in both
Intellectual functioning

Mild

Moderate

IQ range

IQ range

Level of functioning

Cognitive abilities

Social adaptive functioning


Perform daily activities

More frequent in boys


Highest incidence being in school-age children

MTB S2CK

p. 503

Profound

IQ range

IQ range

Little or no speech,
veryy limited abilities to
manage self care

MTB S2CK

p. 503

2nd grade level


May work with
supervision and support
Needs help in mildly
stressful situations

p. 503

Mental Retardation/Treatment

Severe
2025 to 3540

3040 to 5055

Level of functioning

6th grade level


Can work and live
independently
Needs help in difficult or
stressful situations

MTB S2CK

Types of Mental Retardation

Level of functioning

5055 to 70

< 20

Level of functioning
Needs continuous
p
care and supervision

Treatment
Genetic counseling, prenatal care, and safe
environments for expectant mothers
If due to medical condition (e.g., PKU) treat
disorder
Special education to improve level of functioning
Behavioral therapy to reduce negative behaviors

MTB S2CK

p. 503 504

Pervasive Developmental Disorders/Definition

Autistic Disorder

Characterized by
Social, behavioral, and language problems

Incidence: boys > girls


Lacks peer relationships,
poor eye contact, and
social smile
Absent or bizarre speech
Repetitive behaviors

Occurs before age 3

Childhood Developmental Disorders


Autistic disorder
Rett disorder
Childhood disintegrative disorder
Asperger disorder

MTB S2CK

p. 504

Stacking
Injurious behavior to self
or others

MTB S2CK

~18 mo boy with autism, obsessively stacking cans


Source: Andwhatsnext , commons.wikimedia.org

p. 504

Autistic Treatment

Rett disorder

Improve ability to develop relationships, attend


school, and achieve independent living
May benefit from behavioral modification programs
If aggressive, use antipsychotic medications

Greater incidence in girls


Progressive encephalopathy
Microcephaly
Hand-wringing
Loss of speech
Ataxia
At i
Psychomotor retardation
Treatment
Symptomatic
Behavior therapy for self-injurious behavior
Physiotherapy for muscular dysfunction

MTB S2CK

p. 504

MTB S2CK

p. 504

Childhood Disintegrative Disorder

Childhood Disintegrative Disorder/Treatment

Greater incidence in boys


Normal development for 2 years, followed
by marked functional regression in

Improve ability to develop relationships,


attend school, and achieve independent
living
May benefit from behavioral
modification programs
If aggressive, use antipsychotic
medications

Loss of language
Social interaction
Motor function
Bladder function

Repetitive/stereotyped behaviors

MTB S2CK

p. 504

MTB S2CK

p. 504

Asperger Disorder
Greater incidence in
boys
Social and behavioral
problems
No language or
intellectual deficits
Preoccupied with rules

Gabriel is a healthy 2-year-old boy whose parents


have taken him to the pediatrician. His problems
started at 18 months of age, when he did not speak
much. He does not have much attachment to his
parents and seems aggressive toward other children.
What is the most likely diagnosis?
a. Deafness

Ruled out

Impaired judgment,

b. Schizophrenia, childhood onset behavior, and reality


Often display intense interests. Source: Poindexter
Propellerhead at the English language, commons.wikimedia.org

Treatment
Improve relationships with others

Progressive encephalopathy, loss of

c. Rett disorder speech, ataxia, and psychomotor


d. Autism

retardation

e. Learning deficit No evidence of learning deficit


MTB S2CK

p. 504

MTB S2CK

p. 504 505

Attention Deficit Hyperactivity Disorder

ADHD

Characterized by
Inattention
Short attention span
Or
Hyperactivity that
interferes with daily
functioning in school,
home, or work

Symptoms must be present in at least 2 areas, such as:


School
Home

Source: cdc.gov

Must be present for > 6 months and usually


appears before age 7
The symptoms may persist into adulthood
MTB S2CK

p. 505

Interrupt others
Fidget in chairs
Run or climb
excessivelyy
Unable to engage in
leisure activities
Talk excessively

MTB S2CK

Unable to pay
attention
Make careless
mistakes in
schoolwork
Do not follow through
with instructions
Difficulties organizing
tasks
Easily distracted

p. 505

Treatment

Disruptive Behavioral Disorders

First-line: methylphenidate and dextroamphetamine

Oppositional Defiant Disorder


Epidemiology
Usually noted by age 8
Boys > girls before puberty

Side effects: insomnia, decreased appetite, and


headache

Second-line: atomoxetine (norepinephrine reuptake


inhibitor)

But equal incidence after puberty


On the USMLE Step 2 CK, atomoxetine is usually
chosen over the first-line treatment, given the side
effect profiles of those treatments

MTB S2CK

p. 505

MTB S2CK

p. 506

Disruptive Behavioral Disorders/


Oppositional Defiant Disorder
Features

Argues often
Loses temper
Easily annoyed
Blames others for
their mistakes
Tends to have
problems with
authority figures

Disruptive Behavioral Disorders/


Oppositional Defiant Disorder

Justifies behavior as
response to others actions

Treatment
Teach parents appropriate child
management skills and how to lessen
oppositional
iti
lb
behavior
h i

Source: Chris Willia, commons.wikimedia.org

MTB S2CK

p. 506

Disruptive Behavioral Disorders

Conduct Disorder
Epidemiology
Seen more frequently in
Boys whose parents have antisocial
personality disorder and alcohol
dependence

MTB S2CK

p. 506

Disruptive Behavioral Disorders/


Conduct Disorder
Features
Persistent behavior where
rules are broken
Behaviors include
aggression toward others:
Bullying
Cruelty to animals
Fighting
Or
Using weapons

Source: Diego Grez, commons.wikimedia.org

Vandalize and destroy property; set fires


Steal items from others or lie to obtain goods from others
Violate rules (truancy, running away from home, breaking
curfew)
MTB S2CK

p. 506

Disruptive Behavioral Disorders/


Conduct Disorder

Treatment
Behavioral intervention using rewards
for prosocial and nonaggressive
behavior
If aggressive
Antipsychotic medications have been
used

MTB S2CK

p. 506

A 10-year-old boy is seen by the school counselor after


teachers complained of his behavior. He frequently becomes
angry toward others and loses his temper. His parents report
that he refuses to comply with house rules, stays up past his
bedtime, and frequently talks back to them.
What is the most likely diagnosis?
a. Conduct disorder Breaks rules of society or commits
crimes
b. Tourette disorder Multiple tics
c. Adjustment disorder Maladaptive reaction to identifiable
stressor
d. Oppositional Defiant Disorder
e. Learning disorder, not otherwise specified
Needs evidence of a learning problem

MTB S2CK

p. 506

MTB S2CK

p. 506

Tourette Disorder

Tourette Disorder/Treatment

Characterized by onset of multiple tics

Seen more frequently in

Lasting > 1 yr
Occurs before age 18

Boys
Begins at age 7

Motor tics most commonly involve


Facial and neck muscles (e.g., head shaking and
blinking)

The vocal tics include

Dopamine
p
antagonists
g
Antipsychotic medications (e.g., risperidone)

Grunting, coughing, and throat clearing

MTB S2CK

p. 506

MTB S2CK

p. 507

Mood Disorders
Major Depression
Bipolar Disorder
Dysthymia
Cyclothymia
Atypical Depression
Seasonal Affective Disorder
Bereavement (Grief)

Psychiatry
Sam Asgarian, MD/MBA
Class of 2012
Tulane University

Major Depression

Major Depression

Mood disorders present with at least a 2-week


course of symptoms thats a change from the
previous level of functioning

Symptoms
Depressed mood or anhedonia (absence of
pleasure)
And
4 others including

Vincent van Gogh's 1890 painting Sorrowing old man ('At Eternity's Gate') Source: The
Yorck Project, commons.wikimedia.org

MTB S2CK

p. 507

Depressed mood most


of day
Weight changes
Sleep changes
Psychomotor
disturbances
MTB S2CK

Fatigue
Poor concentration
Thoughts of death and
worthlessness

p. 507

Major Depression/Diagnosis and Treatment

Major Depression/Treatment

Diagnosis
Rule out medical causes

SSRIs
Effective and relatively mild side effects
Less toxic in overdose than other antidepressants
If some improvement, but not full response
Increase dose of SSRI
Psychotherapy (e.g., cognitive therapy) proven to be
effective
Goal of cognitive therapy is
Reduce depression by teaching patients to identify
negative cognitions and develop positive ways of
thinking

Most common (hypothyroidism)

Most common neurological associations are


Parkinson disease and dementia

First-line treatment: SSRIs


Fluoxetine
Paroxetine
Sertraline
Citalopram
Escitalopram
MTB S2CK

p. 507

Major Depression

SSRIs should NOT be taken with MAO


inhibitors as they will cause a dramatic
increase in serotonin.

USMLE Step 2 CK wont give you 2 SSRIs


from which to choose

MTB S2CK

p. 507 508

MTB S2CK

p. 507

Exceptions to SSRI Use


Variety of depression

Specific alternative to SSRIs

Patient with depression and


neuropathic pain

Use desvenlafaxine
Approved for both
depression & neuropathy

Patient with depression who


iis ffearful
f l off weight
i h gain
i or
sexual side effects

Bupropion has fewer


sexuall side
id effects
ff
and
d less
l
weight gain than SSRIs
Also used as adjunct
treatment for SSRI
induced sexual side effects

MTB S2CK

p. 507

Major Depression
45-year-old woman was seen by her PCP due to complaints of
depressed mood, lack of pleasure, sleep problems, decreased
appetite and weight, decreased energy, and problems with
concentration. She states that these symptoms started when
she was fired from her job about 4 weeks ago, and that since
then, she has been unable to function.
What is the most indicated treatment at this time?
a. Alprazolam Anxiolytic
b. Paroxetine
c. Bupropion Not 1st line
d. Venlafaxine
When initial therapy doesnt
e. Trazodone
work, or depression more
f. Electroconvulsive therapy severe and associated with
psychotic features
MTB S2CK

p. 508

Choices on USMLE may include an SSRI


and another antidepressant medication
Pick the cleanest: SSRI

MTB S2CK

p. 508

Bipolar Disorder

Bipolar Disorder

Mood disorder
Patient experiences

Manic symptoms

Manic symptoms
that last at least 1
week
Cause significant
distress in level of
functioning

Elevated mood
Increased self-esteem
Distractibility
Pressured speech
Decreased need for
sleep

Increase in goaldirected activity


Racing thoughts
Excessive involvement
pleasurable
in p
activities

Source: commons.wikimedia.org

Source: commons.wikimedia.org

MTB S2CK

p. 508

MTB S2CK

Source: commons.wikimedia.org

p. 508

Bipolar Disorder

Bipolar Disorder

Typically starts with depression

The difference between mania and hypomania


Severity of symptoms
Level of functioning
Duration

Diagnosis
Exclude drug use
Cocaine/amphetamine
Obtain history and urine drug screen

MTB S2CK

p. 508

Mania
M i

Hypomania
H
i

> 1 week
Affect functioning
Warrant
hospitalization

MTB S2CK

< 1week
Dont severely affect
functioning
Dont warrant
hospitalization

p. 508

Types of Bipolar Disorders


Bipolar disorder type I

Mania and depression

Bipolar disorder type II

Hypomania and depression

21-year-old college student was taken to the university clinic


after she was noted to be acting bizarrely in class. She is
talking fast and reported that she has not slept for over 4 days.
She appears to be giggling and not paying attention in class.
Her roommate reported that she has been drinking alcohol
excessively over the last few days and has had many sexual
contacts with unknown men.
Wh t iis th
What
the mostt lik
likely
l di
diagnosis?
i ?
a. Alcohol-induced mood disorder No history of alcoholism
b.
c.
d.
e.

MTB S2CK

p. 509

Bipolar disorder type I


Bipolar disorder type II Hypomania
Major depression with psychosis Thoughts of death,
preoccupation with
Cyclothymia Hypomanic
worthlessness,
episodes and
psychomotor
mild
depression
MTB S2CK p. 509
retardation, psychosis

Bipolar Disorder/Treatment

Bipolar Disorder

You must distinguish whether


Acute mania
1st line
Lithium
Valproic acid
Atypical
antipsychotics

Severe
symptoms,
consider

If kidneys are compromised, dont use


lithium

Bipolar depression
Lithium
Lamotrigine

Lithi
Lithium iis th
the correctt answer tto mostt
bipolar questions

Atypical antipsychotics
Shorter onset of action
MTB S2CK

p. 509

MTB S2CK

p. 509

Dysthymia
33-year-old man was taken to emergency room by police after
neighbors complained about his behavior. His family informed
the doctor hes been diagnosed with bipolar disorder and was
recently started on lithium. While in the emergency room, he
became combative and punched a nurse on the mouth.
What is the next step in the management of this patient?
a. Obtain lithium level
Symptoms are acute
b. Admit to psychiatric unit > important to treat
c. Refer to psychiatry
Never refer
1st line is an antipsychotic
d. Add valproic acid
e. Olanzapine

MTB S2CK

p. 509 510

Characterized by
Depressed mood that lasts most of the day
and is present almost continuously
Symptoms must be present for: > 2 years
Treatment
Antidepressant medications and
psychotherapy
MTB S2CK

p. 510

Cyclothymia

Atypical Depression

Characterized by
Hypomanic episodes and mild depression

Characterized by

Symptoms must be present for: > 2 years


Treatment
Lithium, valproic acid, or carbamazepine
Psychotherapy

MTB S2CK

p. 510

Reverse vegetative changes


Increased sleep
Increased weight
Increased appetite
Mood tends to be worse in evenings and
patients may complain of extremities
feeling heavy
MTB S2CK

p. 510

Atypical Depression/Treatment

Seasonal Affective Disorder

SSRIs or MAOIs
SSRIs have better side-effect profile
If MAOIs and SSRIs are in the same
question, choose SSRIs because of sideeffect profile

Characterized by
Seasonal changes in mood during fall and winter

Usually MAOIs are answer on Step 2 for


atypical depression

Symptoms
Weight gain
Increased sleep
Lethargy

Treat with
Phototherapy and bupropion

MTB S2CK

p. 510

MTB S2CK

Postpartum Disorders

p. 510

Postpartum Disorders

Disorder

Postpartum blues or baby blues

Disorder

Postpartum depression

Onset
Symptoms
Mothers feelings
toward baby
Treatment

Immediately after birth up to 2 weeks


Sadness, labile mood, tearfulness
No negative feelings

Onset
Symptoms

Within 13 months after birth


Depressed mood, weight changes, sleep
disturbances, and excessive anxiety
May have negative feelings toward baby

Supportive, usually self limited

Mothers feelings
toward baby
Treatment

Antidepressant medications

Source: commons.wikimedia.org

MTB S2CK

p. 511

MTB S2CK

Postpartum Disorders
Disorder

Within 23 weeks after birth


Depression, delusions, and thoughts
of harm
Mothers feelings toward baby May have thoughts of harming baby
Treatment
Antipsychotic medication, lithium,
and possibly antidepressants

p. 511

Source: commons.wikimedia.org

Bereavement (Grief)
Postpartum psychosis

Onset
Symptoms

MTB S2CK

p. 511

Source: commons.wikimedia.org

Bereavement

Major depression
(greater severity than bereavement)

Begins after death of


loved one
Feelings of
Sadness
Worrying
y g
Irritability
Sleep difficulties
Poor concentration
Tearfulness
< 6 months, but can go
on longer

Thoughts of death
Morbid preoccupation with
worthlessness
Marked psychomotor
retardation
Psychosis
Prolonged functional
impairment
> 2 months and adversely
affect functioning

MTB S2CK

p. 511

Bereavement (Grief)
Treatment
Supportive
psychotherapy

Medical therapy is
wrong answer

Source: Bundesarchiv, Bild 146-1996-036-01 /


Unknown / CC-BY-SA, commons.wikimedia.org

A 65-year-old man brought to office by daughter. He has been


hopeless and helpless since his wife died 3 months ago.
Daughter is worried about his isolative behavior and lack of
appetite. He lost over 30 pounds. He doesnt seem interested
in getting better and believes he should have died with his
wife.
What is the most likely diagnosis?
a. Bereavement Severe symptoms just to be bereavement
b. Dysthymia Depressed mood, lasts most of the day and
continuously
c. Major depression
Anxiety, depression or
d. Adjustment disorder disturbances of conduct
e. Bipolar disorder

MTB S2CK

p. 511

MTB S2CK

p. 511

Mood disorder with manic symptoms,


at least 1 week

Antidepressants, Mood Stabilizers,


Electroconvulsive Therapy

Antidepressants, Mood Stabilizers,


Electroconvulsive Therapy

Type of medication

Type of medication

Adverse effects

Tricyclic
antidepressants

Amitriptyline
Nortriptyline
Imipramine

MTB S2CK

Hypotension
Dry mouth
Constipation
Confusion
Arrhythmias
Sexual side effects
Weight gain
GI disturbances

p. 512

Monoamine
oxidase inhibitors
- Phenelzine
- Isocarboxazid
- Tranylcypromine

MTB S2CK

p. 512

Adverse effects

Monitor diet, given that


food rich in tyramine will
produce hypertension
-

Safe foods include white wine


and processed cheese
Unsafe foods include red wine,
aged cheese, and chocolate

Source: Vincent van Gogh, Still life with bottle, two


glasses, cheese and bread. commons.wikimedia.org

Antidepressants, Mood Stabilizers,


Electroconvulsive Therapy

Antidepressants, Mood Stabilizers,


Electroconvulsive Therapy

Type of medication

Type of medication

Serotonin selective
reuptake inhibitors
-

Fluoxetine
Paroxetine
Sertraline
Citalopram
Escitalopram
Fluvoxamine

MTB S2CK

p. 512

Adverse effects

Headaches
Weight changes
Sexual side effects
GI disturbances

Adverse effects

Serotonin norepinephrine Hypertension


reuptake inhibitors
Blurry vision
- Venlafaxine
Weight changes
- Duloxetine
Sexual side effects
- Desvenlafaxine
GI disturbances

MTB S2CK

p. 512

10

Antidepressants, Mood Stabilizers,


Electroconvulsive Therapy

Antidepressants, Mood Stabilizers,


Electroconvulsive Therapy

Type of medication

Type of medication

Others
-

Bupropion
Trazodone
Mirtazapine

Adverse effects

Bupropion: seizures
Trazodone: priapism
Mirtazapine: weight gain
and sedation

Lithium

Adverse effects

Tremors, weight gain, GI


disturbance, nephrotoxic,
teratogenic, leukocytosis,
diabetes insipidus
Severe toxicity gives
confusion, ataxia, lethargy,
and abnormal reflexes

MTB S2CK

p. 512

MTB S2CK

p. 512

Antidepressants, Mood Stabilizers,


Electroconvulsive Therapy

Antidepressants, Mood Stabilizers,


Electroconvulsive Therapy

Type of medication

Type of medication

Valproic acid

Adverse effects

Tremors, weight gain, GI


disturbances, alopecia,
teratogenic, hepatotoxic

Adverse effects

Lamotrigine

Stevens Johnson syndrome

Electroconvulsive
therapy

Headaches, transient memory loss

Must monitor levels; toxicity


causes
-

MTB S2CK

Hyponatremia
Coma
Death

p. 512

MTB S2CK

p. 512

Source: commons.wikimedia.org

Serotonin Syndrome
What is the single most effective treatment for depression?
a. Electroconvulsive therapy
b. Fluoxetine
Equally efficacious, but the SSRIs
c. Venlafaxine are used more frequently due to
side-effect profiles
d. Imipramine
e. Phenelzine

Potentially life-threatening
From use of SSRIs, often with interactions between
drugs, overdose, or recreational use of drugs that are
serotonergic in origin
Symptoms
Cognitive effects
Agitation, confusion, hallucinations, hypomania

MTB S2CK

p. 513

MTB S2CK

p. 513

11

Serotonin Syndrome
Autonomic effects
Sweating, hyperthermia, tachycardia, nausea,
diarrhea, shivering

Psychotic Disorders

Somatic effects

Schizophrenia
Delusional Disorder

Tremors, myoclonus

Treatment
Stop SSRI medication
Symptomatic treatment of fever, diarrhea,
hypertension
Cyproheptadine (serotonin antagonist)
MTB S2CK

p. 513

Classification of Psychotic Disorders

Classification of Psychotic Disorders

Brief Psychotic Disorder

Schizophreniform Disorders

Duration of symptoms
> 1 day, but < 1 month
Symptoms
Delusions
Hallucinations
Disorganized speech
Grossly disorganized
Catatonic behavior

Treatment
Antipsychotic
medication

MTB S2CK p. 513

Duration of symptoms
> 1 month, but < 6
months
Symptoms
Delusions
Hallucinations
Disorganized speech
Grossly disorganized
Catatonic behavior

Negative symptoms
Flat affect
Poor grooming
Social withdrawal

Treatment
Antipsychotic
medication

MTB S2CK p. 513

Classification of Psychotic Disorders

Psychotic Disorders

Schizophrenia
Duration of symptoms
> 6 months
Symptoms
Delusions
Hallucinations
Disorganized speech
Grossly disorganized
Catatonic behavior
Negative symptoms
MTB S2CK p. 513

Severely affects level


of functioning
Treatment
Antipsychotic
medication

Duration of symptoms distinguishes brief


psychosis, schizophreniform, and schizophrenia
If no time is mentioned, always choose
schizophrenia as the correct answer to the What is
the most likelyy diagnosis?
g
question
q

MTB S2CK p. 514

12

Schizophrenia/Definition

Schizophrenia/Definition

Thought disorder: impairs judgment,


behavior, and ability to interpret reality
At least 6 months and must affect
functioning
Incidence: men = women
Affects men at young age

Urine drug screen


Cocaine
Amphetamine
A schizophrenic patient at the Glore Psychiatric Museum made this piece of cloth and it
gives us a peek into her mind. Source: Cometstarmoon, commons.wikimedia.org

MTB S2CK p. 514

Types of Schizophrenia/Diagnostic criteria

Types of Schizophrenia/Diagnostic criteria

Paranoid
Delusions or hallucinations, mostly persecutory
or grandiose type
Most common type of schizophrenia
Later age of onset
Catatonic
Psychomotor disturbances from retardation to
excitation

Disorganized
Marked regression to disinhibited
behavior with little contact with reality
Typically appear disheveled and have
bizarre emotional responses
Worst prognosis and earliest age of
onset

Stupor, rigidity, excitement, or posturing

Mutism is common

MTB S2CK p. 514

MTB S2CK p. 514

Types of Schizophrenia/Diagnostic criteria

Schizophrenia/Treatment

Residual
Lack of positive symptoms

Hospitalize: acutely psychotic


Ensure patient safety, administer atypical
antipsychotic such as

Hallucinations
Delusion

Presence of negative symptoms


Flat affect
Poor grooming
Social withdrawal

Undifferentiated
Characterized by not meeting criteria for other
types
MTB S2CK p. 514

Risperidone, olanzapine, quetiapine,


ziprasidone, aripiprazole, or paliperidone

In emergency situation where intramuscular


medication is needed consider
Olanzapine or ziprazidone
Haloperidol may be used, but has more side
effects
MTB S2CK p. 514 515

13

Schizophrenia/Treatment

Schizophrenia/Treatment

If noncompliant with medication


Long-acting antipsychotic
Risperidone: first-line

Haloperidol may be used, more side effects

Clozapine
Use when NO response to a trial of typical
or atypical antipsychotics

Know the differences in the side effect


profiles of the atypical antipsychotics
Its common to have 2 appear on the
test; you need to pick the best based on
side effects

Never used as first-line

MTB S2CK p. 515

MTB S2CK p. 515

Adverse Effects of
Atypical Antipsychotic Medications
Olanzapine
Greater incidence of
diabetes and weight gain;
avoid in diabetic & obese

Risperidone

Quetiapine

Greater
incidence of
movement
disorders

Less incidence
of movement
disorders

Ziprasidone

Clozapine

Increased risk of
prolongation of QT
interval; avoid in
conduction defects

High risk of agranulocytosis;


monitor CBC on regular
basis; never used as firstline treatment given sideeffect profile

MTB S2CK p. 515

Acute
dystonia

Hours
to days

Akathisia

Weeks

MTB S2CK p. 516

a.
b.
c.
d.
e.

Aripiprazole
Olanzapine Highest risk of metabolic abnormalities
Quetiapine
Have medium risk
Clozapine
Risperidone

MTB S2CK p. 515

Management of Adverse Effects of


Atypical Antipsychotic Medications
Onset of
symptoms

22-year-old woman with schizophrenia. Shes 30


pounds overweight and suffers from type 2 DM. She
is concerned about her medications and asks for
advice.
Which of the following would be most indicated in this
patient?

Acute dystonia

Symptoms
Muscle spasms
Torticollis
Laryngeal spasms
Occulogyric crisis

Generalized
restlessness
Pacing
Rocking
Inability to relax

Treatment

Muscle
spasms

Benztropine
Trihexyphenidyl
Diphenhydramine

Reduce dose
Beta blockers
Switch to
atypical
Source: James Heilman, MD, commons.wikimedia.org

14

Management of Adverse Effects of


Atypical Antipsychotic Medications
Onset of
symptoms

Tardive
dyskinesia

Rare
before 6
months

Neuroleptic
malignant
syndrome

Not
time
limited

Symptoms

Treatment

Abnormal involuntary
movements of

Head
Limb
Trunk
Perioral, most
common

Muscular rigidity
Fever
Autonomic changes
Agitation
Obtundation

Switch to
atypical
antipsychotic
Clozapine
has least risk

Dantrolene or
bromocriptine

MTB S2CK p. 516

23-year-old man recently diagnosed with


schizophrenia is started on haloperidol. Within a
few hours he develops muscle stiffness. His eyes
roll upward and he cannot move them down.
What is the most likely diagnosis?
a Tardive dyskinesia 6 mo. involuntary, perioral movements
a.
No time, muscular
b. Neuroleptic malignant syndrome rigidity, fever
Weeks. Generalized restlessness,
c. Akathisia pacing, rocking, inability to relax
d. Serotonin syndrome SSRIs
e. Acute dystonic reaction
MTB S2CK p. 516

Delusional Disorder

Delusional Disorder

Characterized by
Non-bizarre delusions
for > 1 month and NO
impairment in level of
functioning
The p
patient may
y believe
the country is about to
be invaded, but he or
she still obeys the law,
goes to work, and pays
bills

Hallucinations arent present


Treatment
Atypical antipsychotic: first-line therapy
Psychotherapy to help promote reality
testing

Source: commons.wikimedia.org

MTB S2CK p. 516

MTB S2CK p. 516

Panic Disorder/Definition

Anxiety Disorders

Is the experience of intense anxiety along with


feelings of dread and doom

Panic Disorder
Phobias

Source: commons.wikimedia.org

MTB S2CK p. 517

15

Panic Disorder/Definition

Panic Disorder/Definition

This is accompanied by at least 4 symptoms


of autonomic hyperactivity such as

Last < 30 minutes and may be accompanied


by agoraphobia

Diaphoresis
Trembling
Chest pain
Fear of dying
Chills
Palpitations

SOB
Nausea
Dizziness
Dissociative
symptoms
Paresthesias

MTB S2CK p. 517

Fear of places where escape is felt to be difficult

Typically in women, can occur at any time,


and usually has no specific stressor
R/O thyroid disease, hypoglycemia, and
cardiac disease

MTB S2CK p. 517

Panic Disorder/Treatment
SSRIs, typically
Fluoxetine, paroxetine, and sertraline are indicated
for this disorder

Along with SSRIs, patients may benefit from


benzodiazepines (e.g., alprazolam)
Begin
g with both
Then taper the benzodiazepine (potential abuse)

Behavioral and individual therapy


Helpful
But
Not sufficient as only treatment
MTB S2CK p. 517

a.
b.
c
c.
d.
e.

Alprazolam Panic attack


Buspirone Generalized anxiety
Sertraline
Imipramine Major depression, enuresis
Fluvoxamine Not 1st-line

MTB S2CK p. 517

Phobias Definition

Fear of an object or
situation and the
need to avoid it

Which is the first-line treatment for panic


disorder?

Types of Phobias

Specific phobia
Fear of an object
(e.g., animals,
heights, or cars)

Social phobia
Fear of a situation
Public restrooms
Eating in public
Public speaking

Situations where
something potentially
embarrassing may
happen

May be learned,
involves 2 main
types

Source: Alex.Ramek, commons.wikimedia.org

Source: Kilarin, commons.wikimedia.org

MTB S2CK p. 517 518

MTB S2CK p. 518

16

Phobias/Diagnosis and Treatment

Phobias/Treatment

Diagnosis
History of

Relaxation techniques such as

Anxiety symptoms in specific situations


Or
When in contact with feared objects

Treatment
Behavioral modification techniques such as
Systematic desensitization:

Breathing
Or
Guided imagery
Beta blockers (e
(e.g.,
g atenolol or propanolol) are
used for performance anxiety such as stage
fright. They are given 30 minutes to 1 hour
before performance.

Expose individuals to their feared objects moving from


least
most anxiety-provoking
MTB S2CK p. 518

MTB S2CK p. 518

40-year-old man referred to psychiatrist because he is


too shy. He has problems going to parties, feels anxious
about getting close to others, and stays at home in fear
that others would laugh at him. When confronted by
others, he develops severe anxiety as well as
hyperventilation and increased sweating.
Which is the most likely diagnosis?

Obsessive Compulsive
Disorder

Intense anxiety, dread and doom, 4

a. Panic disorder
autonomic hyperactivity symptoms
b. Social anxiety
Chronic worrying about things
c. Generalized anxiety disorder that dont merit concern
d. Specific phobia Fear of an object
e. Acute stress disorder Stressor, 2 days to 1 month, relive
the event

MTB S2CK p. 518

Obsessive Compulsive Disorder (OCD)/


Definition

Difference between Obsessions and


Compulsions

Obsessions alone
Or, most commonly
Combination of
obsessions and
compulsions
l i
Typically affect the
individuals level of
functioning

Obsessions

MTB S2CK p. 518

Compulsions

Thoughts that are


intrusive, senseless,
and distressing, thus
increasing anxiety
Includes fear of
contamination

Rituals such as counting


and checking done to
neutralize thoughts
Time consuming and
tends to lower anxiety

Source: Lars Klintwall Malmqvist, commons.wikimedia.org

MTB S2CK p. 519

Source: commons.wikimedia.org

17

OCD/Diagnosis and Treatment


Diagnosis
More frequent in young patients
Incidence: men = women
OCD can coexist with Tourette disorder
Treatment
SSRIs: treatment of choice
Most common, first-line: fluoxetine, paroxetine,
sertraline, citalopram, or fluvoxamine
Main behavioral therapy used
Exposure and response prevention

Posttraumatic Stress Disorder


and Acute Stress Disorder

MTB S2CK p. 519

Posttraumatic Stress Disorder (PTSD) and


Acute Stress Disorder/Definition
In both, individuals have Usually overwhelming
been exposed to a
(e.g., war, rape,
stressor to which they
hurricanes, or
react with fear and
earthquakes)
helplessness
Patients continually
relive
li the
h event and
d
avoid anything that
reminds them of event

PTSD and Acute Stress Disorder/Definition

Affect the patients level of functioning


Other symptoms
Increased startle response
Hypervigilance
Sleep disturbances
Anger outbursts
Concentration difficulties

Source: Department of Defense visual information (DVIC)


Author: US Army, Office of War Information (OWI)

MTB S2CK p. 519

MTB S2CK p. 519

Difference between PTSD and


Acute Stress Disorder

PTSD
Symptoms last for
> 1 month

MTB S2CK p. 519

PTSD and Acute Stress Disorder/Diagnosis

Acute stress disorder


Symptoms last for
> 2 days and a
maximum of 1
month
th
They occur within
1 month of
traumatic event

Main feature
Determine the time period when traumatic
events occurred in relationship to
symptoms

Rule out
Depression and substance abuse
Both worsen diagnosis

MTB S2CK p. 519 520

18

PTSD and Acute Stress Disorder/Treatment

First-line treatment
Paroxetine and sertraline

Relaxation techniques and hypnosis,


proven helpful
Psychotherapy after traumatic events
will allow
Development of coping techniques and
acceptance of event

MTB S2CK p. 520

35-year-old woman with palpitations, dizziness, and increased


sweating for 8 months. Has visited numerous physicians, none
have been helpful. Her husband is concerned because she
cannot relax and worries about everything. She worries about
her parents health even though they are healthy. She worries
about her finances, although her husband assures her theyre
financially secure.
Wh t iis th
What
the mostt lik
likely
l di
diagnosis?
i ?
a. Generalized anxiety disorder
b. Phobias Fear of an object or a situation
Intense anxiety, dread and doom, 4 autonomic
c. Panic disorder
hyperactivity symptoms
d. Adjustment disorder
Maladaptive reaction to a stressor
e. Social anxiety Anxiety to social interactions, situations
MTB S2CK p. 520

Generalized Anxiety Disorder/Definition

Generalized Anxiety Disorder

Excessive anxiety and worry about most things,


lasting > 6 months
Typically, anxiety is out of proportion to event
Accompanied by

Fatigue
Concentration difficulties
Sleep problems
Muscle tension
Restlessness

Usually women, who complain of feeling anxious


as long as they can remember
MTB S2CK p. 520

Generalized Anxiety Disorder/Treatment

SSRIs
Fluoxetine, paroxetine, sertraline, or
citalopram

Venlafaxine and buspirone have also


been used
Psychotherapy and behavioral therapy
Beneficial
Not considered first-line

Antianxiety Medications and


Their Adverse Effects

Antianxiety medication
Benzodiazepines
Diazepam
Lorazepam
Clonazepam
Alprazolam
Oxazepam
Chlordiazepoxide
Temazepam
Flurazepam
Buspirone

MTB S2CK p. 520

Adverse effects
Sedation
Confusion
Memory deficits
Respiratory depression
And
Addiction potential

Headaches
Nausea
Dizziness

MTB S2CK p. 521

19

Antianxiety Medications and


Their Specific Indications

Antianxiety Medications and


Their Specific Indications

Lorazepam
Used frequently in emergency situations
because it can be given intramuscularly
Clonazepam
May be used if addiction is a concern
concern, longer
half-life
Chlordiazepoxide, oxazepam
Used frequently in treatment of alcohol
withdrawal

Alprazolam
Used frequently in panic disorder

MTB S2CK p. 521

Flurazepam, temazepam,
Flurazepam
temazepam triazolam
Approved as hypnotics (rarely used)

MTB S2CK p. 521

Generalized Anxiety Disorder


Flumazenil is a benzodiazepine
antagonist used only when
The overdose is acute
And:
Youre certain that theres no
chronic dependence

Substance Related Disorders

Flumazenil can cause seizures in benzodiazepinedependent patients. It causes acute withdrawal: tremor or
seizures similar to delirium tremens (alcohol withdrawal).

MTB S2CK p. 521

Specific Substance Abuse/Definition

Substance Related Disorders/Definition

Intoxication
Reversible experience - substance leads to either
psychological or physiological changes
Withdrawal
Cessation or reduction of a substance leading to
either psychological or physiological changes

Abuse
Maladaptive pattern - substances that leads
to

MTB S2CK

p. 521

Source: commons.wikimedia.org

Engaging in hazardous situations


Legal problems
Inability to fulfill obligations
Continued use despite adverse
consequences

MTB S2CK

p. 521

20

Substance Related Disorders/Definition

Intoxication and Withdrawal/Alcohol


Alcohol

Dependence
Maladaptive pattern, substances leads to

Intoxication

Tolerance

Withdrawal when
trying to cut down
Patients spend a great
deal of time engaging in
drug use
Continued use despite
adverse consequences

Talkative
Sullen
Gregarious
Moody
Disinhibited

p. 521

Mechanical
ventilation, if
severe
MTB S2CK

Intoxication and Withdrawal/


Amphetamines & Cocaine

Benzodiazepines
Thiamine
Multivitamins
Folic acid

p. 522

Cannabis

Signs & Symptoms

Withdrawal

Signs & Symptoms

Euphoria
Hypervigilance
Autonomic hyperactivity
Weight loss
Pupillary dilatation
Perceptual disturbances

Anxiety
Tremulousness
Headache
Increased Appetite
Depression
Risk of suicide

Treatment

Treatment

Antipsychotics and/or
benzodiazepines and/or
antihypertensives
MTB S2CK

Bupropion and/or
bromocriptine

Intoxication

Signs &
Symptoms

Impaired motor
coordination
Slowed sense of
time
Social withdrawal
Increased appetite
Conjunctival injection

Treatment

Antipsychotics and/or
benzodiazepines and/or
talking down
MTB S2CK

p. 522

Source: JonRichfield,
commons.wikimedia.org

Treatment

None

p. 522

Intoxication and Withdrawal/Inhalants

Hallucinogens

Ideas of reference
Perceptual
disturbances
Impaired judgment
Tremors
Incoordination
Dissociative symptoms

None

Treatment

MTB S2CK

Intoxication and Withdrawal/Hallucinogens

Signs &
Symptoms

Withdrawal

Signs &
Symptoms

None

p. 522

Intoxication

Source: GeographBot,
commons.wikimedia.org

Intoxication and Withdrawal/Cannabis

Amphetamines & Cocaine

Intoxication

Tremors
Hallucinations
Seizures
Delirium tremens

Treatment

Source: Carlos t . commons.wikimedia.org

MTB S2CK

Withdrawal
Signs &
Symptoms

Hallucinogens

Withdrawal

Intoxication

Signs &
Symptoms

Signs &
Symptoms

None
Treatment
Source: Ksd5
Ksd5, commons
commons.wikimedia.org
wikimedia org

None

Belligerence
Apathy
Aggression
Impaired judgment
Stupor
Coma

Withdrawal
Signs &
Symptoms

None
Treatment

None

Treatment

Antipsychotics
MTB S2CK

p. 522

21

Intoxication and Withdrawal/Opiates

Intoxication and Withdrawal/


Phencyclidine (PCP)

Opiates
Withdrawal

Intoxication
Signs &
Symptoms

Apathy
Dysphoria
Pupillary
constriction
Drowsiness
Slurred speech
Coma
Death

Source: Eleassar,
commons.wikimedia.org

Treatment

Belligerence
Psychomotor agitation
Violence
Nystagmus
HTN
Seizures
Antipsychotics and/or
benzodiazepines and/
or talking down

p. 522

MTB S2CK

Intoxication and Withdrawal/Anabolic Steroids

Withdrawal
Signs &
Symptoms

Source: Jara172,
commons.wikimedia.org

None
Treatment

Treatment

Clonidine
Methadone
Or
Buprenorphine

Naloxone
MTB S2CK

Signs &
Symptoms

Fever
Chills
Lacrimation
Abdominal cramps
Muscle spasms
Diarrhea

Treatment

PCP

Intoxication

Signs &
Symptoms

None

p. 522

Substance Related Disorders/Treatment

Anabolic Steroids
Intoxication

If you suspect someone is an alcoholic, do CAGE test.


Two positive responses to four questions are
considered a positive test and indicate that further
assessment is warranted.

Withdrawal

Signs &
Symptoms

Signs &
Symptoms

Irritability
Aggression
Mania
Psychosis
Treatment

Depression
Headaches
Anxiety
Increased concern
over bodys
physical state

Antipsychotics

Source: Cristian navas roman


roman,
commons.wikimedia.org

Treatment

C - Have you ever felt you should cut down on your drinking?
A - Have people annoyed you by criticizing your drinking?
G - Have you ever felt bad or guilty about your drinking?
E - Eye opener: Have you ever had a drink first thing in the
morning to steady your nerves or to get rid of a hangover?

SSRIs
MTB S2CK

p. 522

MTB S2CK

p. 523

Substance Related Disorders/Treatment


Detoxification
Usually 5-10 days
Mostly in hospital settings to assure safe detoxification

Rehabilitation
Usually 28 days or more with a focus on relapse
prevention techniques

Alcoholics Anonymous
Narcotics Anonymous
Pharmacologic treatments often include:
Disulfram (acetaldehyde dehydrogenase inhibitor)
Naltrexone (opioid receptor antagonist)
Acamprosate
MTB S2CK

p. 523

65-year-old engineer suffered femur fracture and some


cuts and bruises after being involved in an MVA. Hes
admitted to the medicine floor and started on oxycodone.
The day after admission, he appears confused with
observable tremors in both extremities. He becomes
concerned about bugs on the walls in his room and
asks for your help.
What is the most likely explanation for his symptoms?
a. Brain concussion Typically after injury
b. Alcohol withdrawal
c. Oxycodone intoxication Constipation, miosis

>1 day but <1 month, delusions,

d. Brief psychotic disorder hallucinations, disorganized speech &


behavior

e. Schizophrenia >6 months, delusions, hallucinations, disorganized


MTB S2CK

p. 523

speech & behavior

22

Somatoform Disorders
Characterized by

Somatoform Disorder,
Factitious Disorder, and
Malingering

Physical symptoms with no medical explanation

Symptoms severe enough to adversely affect


level of functioning
More frequent in young women
Usually has psychological component,
component which the
patient is unaware
Treatment of choice: psychotherapy
Source of symptoms is psychological

MTB S2CK

Types of Somatoform Disorders/


Diagnostic criteria
Somatization disorder
Hypochondriasis
Patients must have at
Patients believe that
least
they have some
specific disease
4 Pain
despite constant
2 GI
reassurance
1 Se
Sexual
al
And
1 Pseudoneurological
symptom

MTB S2CK

MTB S2CK

p. 524

Types of Somatoform Disorders/


Diagnostic criteria

Conversion
Affects voluntary motor or sensory
functions, indicative of a medical
condition
Usually caused by psychological factors
Associated with la belle indifference
Unconcerned about impairment

p. 524

MTB S2CK

Types of Somatoform Disorders/


Diagnostic criteria
Body Dysmorphic Disorder
Patients believe that some
part of the body is
abnormal, defective, or
misshapen

Source: elovedfreak,
commons.wikimedia.org

p. 523

Pain Disorder
Main complaint:
presence of pain
And must have
Psychological
f t
factors
associated
i t d
with pain

p. 524

35-year-old married mother of 3 has frequent complaints


of dizziness, nausea, diarrhea, vomiting, pain during
intercourse, paresthesias, leg pain, stomach pain, food
intolerance, and headaches. She has tried numerous
medications, but none have been beneficial. Neurological
examination: normal.
What is the next step in the management of this patient?
a. Lorazepam Anxiety disorder
b. Sertraline Fibromyalgia & depression
c. Individual psychotherapy
d. Lithium Bipolar disorder
e. Risperidone Psychosis

MTB S2CK

p. 524

23

Factitious Disorder

Factitious Disorder

Individual fakes an illness to get attention


and emotional support in patient role
Either psychological or physical illness
Psychological symptoms

Hallucinations, delusions, depression, and


bizarre behavior

Physical symptoms

Inflict life-threatening injuries on themselves


Behavior may be compulsive at times
Formerly known as Mnchausen syndrome
Factitious disorder by proxy, caretaker fakes signs
and symptoms in another person
Usually a child
child, in order to assume the sick role by
proxy as the caregiver

Abdominal pain, fever, nausea, vomiting, or


hematomas

MTB S2CK

p. 524

MTB S2CK

p. 525

Factitious Disorder

Factitious Disorder/Treatment

Typically, women
with a history of
being employed in
healthcare
Men more often have
physical symptoms
Patients ultimate
goal: admission to
hospital
Always exclude any
medical disorder with
similar symptoms

No specific therapy has been


proven to be effective
When a child is involved in factitious
disorder by proxy

MTB S2CK

Contact child protective services to


ensure childs safety
Factitious disorder cannot
be diagnosed without first
ruling out medical illness

p. 525

MTB S2CK

p. 525

Malingering

Malingering/Diagnosis

Characterized by
Conscious production of signs and symptoms
for an obvious gain, such as

Diagnosis
More frequently in prisoners and military
personnel
Typically diagnosed when theres a
discrepancy between

Avoiding work
Evading criminal prosecution
Or
Achieving financial gain

Not a mental illness

MTB S2CK

p. 525

Patients complaints
And
Actual physical or laboratory findings

MTB S2CK

p. 525

24

Malingering

If medical evaluation reveals malingering


Confront patient with the outcome

Adjustment Disorder

A lack of cooperation from


patients is characteristic of
malingering

MTB S2CK

p. 525

Adjustment Disorder
Characterized by
Maladaptive reaction to
an identifiable stressor
Loss of job
Divorce
Or
Failure in school

Adjustment Disorder/Treatment
Symptoms
Anxiety
Depression
Disturbances of conduct

Severe enough to cause


impairment in functioning

Treatment of choice
Psychotherapy
Both individual and group therapy have
been used effectively

Usually occur within 3


months of stressor and
must remit within 6
months of removal of
stressor
MTB S2CK

p. 525

MTB S2CK

p. 525

Personality Disorders

Personality Disorders

Characterized by
Personality patterns that are:
Pervasive
Inflexible
Maladaptive
Cl t A
Cluster

Cl t B
Cluster

Paranoid
Schizoid
Schizotypal

Histrionic
Antisocial
Borderline
Narcissistic

MTB S2CK

Cl t C
Cluster

Avoidant
Dependent
Obsessive
compulsive

p. 525 526

25

Types of Personality Disorders

Types of Personality Disorders

Paranoid

Schizoid

Schizotypal

Histrionic

Suspicious
Mistrustful
Secretive
Isolated
A d
And
Questioning
loyalty of
family &
friends

Choice of
solitary activities
Lack of close
friends
Emotional
E
i
l
coldness
No desire for or
enjoyment of close
relationships

Ideas of
reference
Magical
thinking
Odd thinking
thi ki
Eccentric
behavior
Increased social
anxiety
Brief psychotic
episodes

Must be center
of attention
Inappropriate sexual
behavior
Self-dramatization
Use physical
appearance to draw
attention to self

MTB S2CK

p. 526

MTB S2CK

Types of Personality Disorders


Antisocial
Failure to
conform to
social rules
Deceitful
Lack of remorse
Impulsive
I
l i
Aggressive
towards others
Irresponsible
Must be 18+

MTB S2CK

p. 526

p. 526

Types of Personality Disorders

Borderline

Narcissistic

Unstable
relationships
Impulsive
Recurrent suicidal
behaviors
Chronic
Ch
i ffeelings
li
of emptiness
Inappropriate
anger
Dissociative
symptoms when
severely stressed
Brief psychotic
episodes

Grandiose
sense of self
Belief that they
are special
Lack empathy
Sense
S
off
entitlement
Require
excessive
admiration

Avoidant

Dependent

Unwilling
to get
involved
with people
Views self
as socially
inept
Reluctant to
take risks
Feelings of
inadequacy

Difficulty making
day-to-day
decisions
Unable to
assume
responsibility
Unable to express
disagreement
Fear of being
alone
Seeks relationship
as source of care

MTB S2CK

Obsessive
compulsive
Preoccupied
with details
Rigid
y
Orderly
Perfectionists
Excessively
devoted to
work
Inflexible

p. 526

Personality Disorders/Treatment

Individual psychotherapy
Medications if mood or anxiety
symptoms are present

MTB S2CK

p. 526

Which of the following personality


disorders has been associated with
positive psychotic symptoms?
a. Borderline & Schizotypal
b Histrionic
b.
Not
c. Schizoid
associated
d. Paranoid
e. Antisocial
MTB S2CK

p. 527

26

Eating Disorders
Anorexia Nervosa
Bulimia Nervosa
Eating Disorder Not
Otherwise Specified

15-year-old girl brought to clinic by her mother,


who found her vomiting in the bathroom. She
vomits daily after each meal and exercises
excessively. She has numerous calluses on her
hands as well as cavities. She is 55 and weighs
90 pounds.
What is her most likely diagnosis?
a. Bulimia nervosa Frequent binge eating, normal
weight, obesity history
Neither
b. Anorexia nervosa
anorexia
c. Eating disorder not otherwise specified nor
bulimia
d. Obesity Increased weight
e. Atypical depression Increased apetite & weight
MTB S2CK

p. 527

Anorexia Nervosa

Anorexia Nervosa/Diagnosis

Characterized by

Lose weight by

Failure to maintain a
normal body weight
Fear and
preoccupation of
gaining weight
Body image
disturbance

Strict caloric control


Excessive exercise
Purging
Laxative & diuretic

Source: http://www.womenshealth.gov/bodyimage/eating-disorders/

Unrealistic self-evaluation as overweight


Amenorrhea is common from low body weight
Deny their emaciated condition
Great concern with appearance and frequently examine
and weigh themselves
MTB S2CK

p. 527

And
Fasting
F ti

Bulimia Nervosa

Hospitalization to prevent

Characterized by

Frequent binge eating in a


discrete amount of time
Lack of control of
overeating episodes

Psychotherapy
Behavioral therapy
SSRIs
Source: girlshealth.gov

p. 528

Accompanied by
compensatory
behavior to prevent
weight gain
Purging
Laxatives &
diuretics

Fasting
And
Excessive exercise

To promote weight gain


MTB S2CK

Arrhythmia MCC of
death

p. 527

Anorexia Nervosa/Treatment
Dehydration
Starvation
Electrolyte imbalances
Death

Potassium deficiency
y

More frequently, teenage


girls 14 -18 years of age
Severe weight loss
Hypotension

MTB S2CK

Bradycardia
Lanugo hair
And
Edema
EKG changes

MTB S2CK

Self-evaluation
influenced by body
shape and weight

p. 528

27

Bulimia Nervosa

Eating Disorder Not Otherwise Specified

Diagnosis
More frequently women and later in adolescence
than anorexia nervosa
Most normal weight, but history of obesity

Doesnt meet criteria for either anorexia


nervosa or bulimia nervosa
Examples
Criteria for anorexia present in girls, but
menstruation is normal
Anorexic patient with normal weight
Use of compensatory behavior after eating
normal amounts of food

Treatment
Doesnt require hospitalization, unless
Severe electrolyte abnormality

Psychotherapy
SSRIs
MTB S2CK

p. 528

MTB S2CK

p. 528

Narcolepsy
Characterized by

Sleep Disorders
Narcolepsy
Insomnia

Excessive daytime
sleepiness
And
Abnormalities of REM
sleep

Most frequently begins


in young adulthood
Sleep studies are
usually indicated in
diagnosis
MTB S2CK

Narcolepsy

No curative therapy
Forced daytime naps
Modafinil
To maintain alertness

MTB S2CK

p. 528 529

Source:http://www.cdc.gov/sleep/

p. 528

Psychiatric & Physical Symptoms


of Narcolepsy (Sleep Disorder)
Specific feature of narcolepsy

Sleep
attacks

Cataplexy
Episodic
irresistible
sleepiness
And
Feeling
refreshed
upon
awakening

MTB S2CK

p. 529

Sudden
muscle tone
loss:
pathognomonic
And
May be
precipitated by
loud noise or
emotions

Hypnogogic
and
hypnopompic
hallucinations
Hallucinations
occur as
patient is
going to sleep
and waking
up

Sleep
paralysis

Awake
but unable
to move
Typically
upon
awakening

28

Insomnia

Insomnia

Characterized by

Sleep hygiene techniques

Inability to initiate or
maintain sleep

Going to bed and waking up at the same time


Avoiding caffeinated beverages
And
Avoiding daytime naps

May be due to anxiety


and depression
g to affect
Severe enough
level of functioning
Typically women who
complain of feeling tired
or have increased
appetite and yawning
MTB S2CK

p. 529

Behavioral modification techniques include


Using the bed only for sleeping (not for reading,
watching TV, or eating)

Medical therapy
Zolpidem, eszopiclone, or zaleplon

Source: nih.gov

MTB S2CK

p. 529

Terminology of Human Sexuality

Human Sexuality

Sexual identity
Based on a persons
secondary sexual
characteristics
Gender identity
Based on a persons
sense of maleness or
femaleness,
established by age 3

MTB S2CK

Human Sexuality
Masturbation
Normal precursor of object-related sexual behavior
All men and women masturbate
Problematic if it interferes with daily functioning
Homosexuality
H
lit
Not considered a mental illness unless it is egodystonic (person not happy with sexual orientation)
May be considered normal experimentation in
teenagers

MTB S2CK

p. 530

Gender role
Based on external
patterns of behavior
that reflect inner sense
of gender identity
Sexual orientation
Based on persons
choice of love object;
may be heterosexual,
homosexual, bisexual,
or asexual

p. 529

Types of Sexual Dysfunction


Impotence
Persistent or recurrent
inability to attain or
maintain an erection
until completion of
sexual
se
ua act

Premature ejaculation
Ejaculation before
penetration or just after
penetration, usually
due to anxiety
Treatment

Treatment

R/O medical causes or


medication
Psychotherapy, couples
sexual therapy

MTB S2CK

Psychotherapy, behavioral
modification techniques
(stop and go, squeeze),
SSRI medication

p. 530

29

Types of Sexual Dysfunction

Paraphilias

Dyspareunia

Vaginismus

Pain associated with


sexual intercourse,
not diagnosed if due
to medical condition

Prolonged and painful


contraction or spasm of
the vagina
Usually severe enough
to prevent intercourse

Treatment

Treatment

Psychotherapy

MTB S2CK

Psychotherapy
Dilator therapy

p. 530

Group of disorders that are recurrent,


sexually arousing, and seen more frequently
in men
Usually focus on humiliation, nonconsenting
partners, or use of nonliving objects
Must occur
> 6 months and cause distress
Affect level of functioning

Dont diagnose if experimentation

MTB S2CK

Types of Paraphilias

p. 530

Types of Paraphilias

Exhibitionism
Recurrent urge to
expose oneself to
strangers

Pedophilia
Recurrent urges or
arousal toward
prepubescent
children

Fetishism
Recurrent use of
nonliving objects to
achieve sexual
pleasure

Masochism
Recurrent urge or
behavior involving the
act of humiliation

Sadism
Recurrent urge or behavior involving acts in
which physical or psychological suffering
of victim is exciting

Transvestic fetishism

MTB S2CK

p. 531

Recurrent urge or behavior involving cross


dressing for sexual gratification; usually in
heterosexual males

MTB S2CK

p. 531

Types of Paraphilias

Paraphilias/Treatment

Frotteurism
Rubbing ones pelvis or erect penis against a
nonconsenting person for sexual gratification

Individual psychotherapy
Behavioral modification techniques
Aversive conditioning

Antiandrogens or SSRIs to reduce


sexual drive

MTB S2CK

p. 531

Source: Dschwen, commons.wikimedia.org

MTB S2CK

p. 531

30

Gender Identity Disorder

Gender Identity Disorder

Characterized by

Patients will take hormones when older to deepen


voice, if female, or soften voice, if male
Women may bind their breasts and men may hide
their penis and testicles
Its seen more frequently in young men

Persistent discomfort and sense of


inappropriateness regarding the patients assigned
sex

Diagnosis
Wearing
g opposite
pp
g
genders clothes
Using toys assigned to opposite sex
Play with opposite-sex children when young
And
Feeling unhappy about ones own sexual
assignment
MTB S2CK

p. 531

Treatment
Sexual reassignment surgery if approved
Individual psychotherapy

MTB S2CK

p. 531

Suicide

Suicide

Recent suicide attempt


Complaints of suicidal
thoughts
Admission of suicidal
thoughts
Demonstration of
suicidal behaviors

Source: samhsa.gov

Buying weapons
Giving away
possessions
Writing a will
MTB S2CK

p. 532

Source: samhsa.gov

Suicide
Risk Factors
Men
Older adults
Social isolation
Presence of psychiatric
illness/drug abuse
Perceived
hopelessness
Previous attempts

MTB S2CK

Treatment
Hospitalize patient
Take all threats
seriously

p. 532

31

Asthma/Definition
Asthma or reactive airway disease
Abnormal bronchoconstriction of airways

Asthma
Stephen Bagley, MD
Resident Physician
Internal Medicine
Hospital of the University of Pennsylvania

Characterized by
Reversible airway obstruction secondary to
bronchial smooth muscle hyperactivity
Airway inflammation, mucus plugging, and smooth
muscle hypertrophy
Can lead to chronic, irreversible airway obstruction

MTB S2CK

p. 129

Asthma/Etiology

Asthma/Etiology

Extremely common
Etiology unknown
Associated with atopic disorders and
obesity

Causes acute exacerbations of symptoms


Allergens such as pollen, dust mites, cockroaches,
and cat dander
Infection
Changes in weather (especially cold air)
Exercise
Catamenial (related to menstrual cycle)
Aspirin, NSAIDs, beta blockers, tobacco smoke
Gastroesophageal reflux disease (GERD)

Asthma prevalence,
incidence, and hospitalization
rates are increasing
MTB S2CK

p. 129

MTB S2CK

p. 129

Asthma/Presentation

Asthma/Presentation

Most often presents with

Which of the following is most likely to be found


in an asthmatic patient?

Wheezing
Acute onset of SOB
Cough
And
Chest tightness

Increased sputum production common


Fever is often not present
Remember all that wheezes
is not asthma!
MTB S2CK

p. 129

Symptoms worse at night


Nasal polyps
Eczema or atopic
physical
y
dermatitis on p
examination
Increased length of
expiratory phase of
respiration

Increased use of
accessory
respiratory muscles
(e.g., intercostals)
Hyperresonance to
percussion, pulsus
paradoxus

I/E ratio decreases


(normal is 1:2)
MTB S2CK

p. 129 130

Asthma/Presentation

The answer to the best initial test question


in asthma is based on the severity of
presentation

Asthma/Diagnostic Tests

The best initial test in an acute


exacerbation
Arterial blood gas (ABG) or peak
expiratory flow (PEF)
ABG if mild,
mild early
earl e
exacerbation:
acerbation mild
hypoxia, respiratory alkalosis
ABG if severe, late exacerbation: severe
hypoxia, respiratory acidosis

MTB S2CK

p. 130

MTB S2CK

p. 130

Asthma/Diagnostic Tests

Asthma/Diagnostic Tests

Chest X-ray

Most accurate diagnostic test is


pulmonary function testing (PFTs)

Most often normal in asthma, but may


show hyperinflation
Useful for excluding other disease
processes

You must understand lung volumes!

Pneumonia, CHF, pneumothorax

MTB S2CK

p. 130

Asthma/Diagnostic Tests

MTB S2CK

p. 130

Asthma/Diagnostic Tests
PFTs in asthma show
OBSTRUCTION: FEV1 and FVC with a
FEV1/FVC ratio
HYPERINFLATION: total lung capacity (TLC)
AIR TRAPPING: in residual volume
REVERSIBILITY: in FEV1 > 12% with use of
albuterol
BRONCHIAL HYPERRESONIVENESS: FEV1 >
20% with use of methacholine

MTB S2CK

p. 131

Asthma/Diagnostic Tests
15-year-old boy with occasional SOB every few weeks.
Currently feels well. No medications and denies any other
medical problems. Pulse is 70 and RR is 12. Chest
examination is normal.
Which is the single most accurate diagnostic test at this time?
Acute exacerbation
a. Peak expiratory flow
Less likely in asymptomatic
b. Increase in FEV1 with albuterol
Asymptomatic
c. Diffusion capacity of carbon monoxide
d. > 20% decrease in FEV1 with use of methacholine
e. Increased alveolar-arterial oxygen difference
Asymptomatic
(A-a gradient)
f. Increase in FVC with albuterol Less likely in asymptomatic
g. Flow-volume loop on spirometry Best for fixed obstructions
h. Chest CT scan Shows nothing or hyperinflation
i. Increased pCO2 on ABG Acute exacerbation
MTB S2CK

p. 130

Acetylcholine and histamine provoke


bronchoconstriction and increase in
bronchial secretions
Methacholine is artificial form of
acetylcholine used in diagnostic testing

MTB S2CK

p. 131

Asthma/Diagnostic Tests

Asthma/Treatment

Additional testing options include


CBC may show an increased eosinophil
count
Skin testing to identify specific allergens
that provoke bronchoconstriction
Increased IgE levels suggest allergic
etiology

Chronic management
Stepwise fashion of progressively
adding more types of treatment if no
response

Also seen in allergic bronchopulmonary


aspergillosis

MTB S2CK

p. 131

MTB S2CK

Asthma/Chronic Management
Step 1

Asthma/Treatment

Step 2

Step 3

Step 4

Mild
intermittent:
< 2 days/week
< 2 nights/ month

Mild persistent:
> 2 days/week
> 2 nights/month

Moderate
persistent: daily,
or > 1 night/week

Severe
persistent:
continual,
frequent

Inhaled shortacting
ti beta
b t
agonist SABA
Albuterol
Levalbuterol

Add a long-term
control agent:
Low-dose inhaled
corticosteroids
(ICS) for daily use
Beclomethasone
Budesonide
Flunisolide
Fluticasone
Mometasone
Triamcinolone

MTB S2CK

p. 131 132

p. 131

Add a longg beta


acting
agonist
(LABA) to a
SABA & ICS

Or
Increase
dose of ICS
LABA
Salmeterol
Or
Formoterol

Increase
d
dose
off
ICS to
maximum
in
addition
to LABA &
SABA

Adverse effects of inhaled


steroids are dysphonia and oral
candidiasis
Alternate long-term control agents include
C
Cromolyn
l and
d nedocromil:
d
il inhibit
i hibit mastt cellll mediator
di t
release useful for exercise
Theophylline: phosphodiesterase inhibitor
increasing cAMP levels (cardio- and neurotoxicity)
Leukotriene modifiers: montelukast, zafirlukast, or
zileuton best with atopic patients
MTB S2CK

p. 131

Asthma/Treatment

Zafirlukast is hepatotoxic
and has been associated
with Churg-Strauss
syndrome

MTB S2CK

p. 131

Asthma/Diagnostic Tests

Management of Acute Exacerbation

Step#1: determine severity of asthma


exacerbation, quantified by
Decreased peak expiratory flow (PEF)
ABG with evidence of increased A-a
Aa
gradient or CO2 retention

MTB S2CK

p. 133

Asthma/Diagnostic Tests

Asthma/Treatment

PEF is an approximation of FVC


No precise normal value
Based predominantly on height and age,
not weight
PEF is used in acute assessment byy seeing
g
how much difference theres from patients
usual PEF when stable

Management of Acute Exacerbation


Oxygen
Albuterol (often nebulized to ensure adequate
delivery to lungs) +/- inhaled anticholinergic (e.g.,
ipratropium)
Corticosteroids: need 4-6
4 6 hours to begin to work
work, so
give right away
Epinephrine injections are no more effective
than albuterol and have more adverse systemic
effects

MTB S2CK

p. 133

MTB S2CK

p. 133

Asthma/Treatment

Asthma/Treatment

Anticholinergics
Role of ipratropium and tiotropium in asthma
management unclear

Magnesium
Some modest effect in bronchodilation
Not as effective as albuterol, ipratropium, or
steroids, but it does help

In general, ipratropium should be used, but doesnt


work as rapidly as albuterol
Epinephrine
Rarely used and only as a drug of last resort

MTB S2CK

p. 132 133

Magnesium
g
helps
p bronchospasm.
p
Magnesium is used only in acute,
severe asthma exacerbation not responsive
to several rounds of albuterol while waiting
for steroids to take effect.

MTB S2CK

p. 133

Asthma/Treatment

Asthma/Treatment

Adverse Effects of Chronic Systemic


Corticosteroids
Should be a last resort because of very harsh
adverse effects such as
Osteoporosis
Cataracts
Adrenal suppression and fat redistribution
Hyperlipidemia, hyperglycemia, acne, and
hirsutism (particularly in women)
Thinning of skin, striae, and easy bruising

MTB S2CK

p. 132

High-dose inhaled steroids


rarely lead to adverse effects
associated with prednisone

MTB S2CK

p. 132

Asthma/Treatment
47-year-old man with history of asthma comes to ED
with several days of increasing SOB, cough, and
sputum production. RR is 34. He has diffuse
expiratory wheezing and prolonged expiratory phase.
Which would you use as the best indication of the
severity of his asthma?
a. Respiratory rate
b. Use of accessory muscles Subjective
c. Pulse oximetry Hypoxia, until imminent respiratory failure
d. Pulmonary function testing Short of breath
e. Pulse rate Adds nothing
MTB S2CK

p. 132.4

The following are not effective in


acute exacerbations
Theophylline
Cromolyn and nedocromil (best
with
ith extrinsic
t i i allergies
ll i like
lik h
hay
fever)
Leukotriene modifiers
Omalizumab
Salmeterol
MTB S2CK

p. 133

Asthma/Treatment

If no response to oxygen, albuterol, and


steroids or develops respiratory acidosis
(increased pCO2)
Consider endotracheal intubation and
mechanical ventilation

MTB S2CK

Chronic Obstructive
Pulmonary Disease

p. 133

COPD/Definition

COPD/Definition

Characterized by airflow obstruction


secondary to emphysema and/or
chronic bronchitis
Emphysema

Chronic bronchitis

Terminal airway destruction resulting


in decreased elastic recoil of lungs
Normally, elastin fibers allow passive
exhalation
MTB S2CK

p. 134

Productive cough for > 3 months/year


for 2 consecutive years

Both
Part of COPD spectrum
Results in decrease in FEV1 and FVC
with increase in TLC

MTB S2CK

p. 134

COPD/Etiology

COPD/Presentation

Tobacco smoking
Leads to almost all COPD
Destroys elastin fibers

SOB worsened by exertion


Intermittent exacerbations with cough,
sputum, and SOB often brought on by
infection
Barrel chest from air trapping
Muscle wasting and cachexia

Young and
Y
d a nonsmoker,
k
answer alpha-1 antitrypsin
deficiency as most likely cause

MTB S2CK

p. 134

COPD now thought to have a component of


systemic inflammation

MTB S2CK

p. 134

COPD/Diagnostic Tests

COPD/Diagnostic Tests

Best initial test:


chest X-ray
Increased
anteriorposterior (AP)
di
diameter
t
Air trapping and
flattened
diaphragms

Most accurate diagnostic test: PFT


Decreased FEV1, FVC, and FEV1/FVC ratio
(< 70%)
Increased TLC due to RV
Decreased DLCO in emphysema
p y
(destruction of alveolar septae where
capillaries are found)
Incomplete improvement with albuterol
Little or no worsening with methacholine

MTB S2CK

p. 134

Source: James Heilman, MD, commons.wikimedia.org

MTB S2CK

p. 134

COPD/Diagnostic Tests

COPD/Diagnostic Tests

Reversibility with Inhaled Bronchodilators


COPD generally associated with IRREVERSIBLE
airway obstruction
But
COPD has a broad response to inhaled
bronchodilators (e
(e.g.,
g albuterol)
Ranges from no reversibility to complete
reversibility

Arterial blood gas (ABG)


Acute exacerbations of COPD are
associated with pCO2 and hypoxia

Reversibility in response to
bronchodilators is: > 12% increase and
200 mL increase in FEV1
MTB S2CK

p. 134

Respiratory acidosis may be present if


theres
there
s insufficient metabolic compensation
and bicarbonate level will be elevated to
compensate
In between exacerbations, not all those
with COPD will retain CO2
MTB S2CK

p. 134

COPD/Diagnostic Tests

COPD/Treatment

EKG
Right atrial hypertrophy and right ventricular
hypertrophy
A-fib or multifocal atrial tachycardia (MAT)

Improves Mortality and Delays Progression of


Disease: only 2 interventions!

Echocardiography
Right atrial and right ventricular hypertrophy
Pulmonary hypertension

MTB S2CK

p. 135

COPD/Treatment
Definitely Improves Symptoms (But Does Not
Decrease Disease Progression)

Short-acting beta agonists (e.g., albuterol)


Anticholinergic agents: tiotropium, ipratropium
Inhaled steroids
Long-acting beta agonists (e.g., salmeterol)
Pulmonary rehabilitation

Smoking cessation
Oxygen therapy for hypoxia
PaO2 <55 or SaO2 <90%
PaO2 <60 or SaO2 <90%
If patient also has pulmonary hypertension,
cor pulmonale or polycythemia

Mortality benefit from oxygen is directly proportional


to the number of hours that oxygen is used

MTB S2CK

p. 135

COPD/Treatment
Asthmatics not
controlled with albuterol
Inhaled steroid

COPD not controlled


with albuterol
Anticholinergic
(e.g., tiotropium)
Inhaled steroid

Inhaled anticholinergic agents are most


effective in COPD
MTB S2CK

p. 135

MTB S2CK

p. 135

COPD/Treatment

COPD Acute Exacerbations/Treatment

Possibly Improves Symptoms


Theophylline
Lung volume reduction surgery

Combination of bronchodilators and


corticosteroid therapy

No Benefit
Cromolyn
C
l
Leukotriene modifiers (e.g., montelukast)

COPD exacerbation treatment


is identical to asthma
exacerbation treatment, but
with less proven benefit

When all medical therapy is insufficient,


the answer is refer for transplantation
MTB S2CK

p. 135

COPD Acute Exacerbations/Treatment

Antibiotics
For exacerbation of moderate to severe
COPD
Defined as requiring hospitalization or having at
least two of the three cardinal symptoms of a
COPD flare
Dyspnea
Sputum production
Sputum purulence

MTB S2CK

p. 136

MTB S2CK

p. 136

Chronic Bronchitis Acute Exacerbations/


Treatment
Antibiotics
Although viruses cause 20-50% of episodes,
coverage should be provided against
Streptococcus pneumoniae, H. influenzae, and
Moraxella catarrhalis

Macrolides: azithromycin, clarithromycin


Cephalosporins: cefuroxime, cefixime
Amoxicillin/clavulanic acid
Quinolones: levofloxacin, moxifloxacin

Second-line Agents
Doxycycline, TMP/SMX
MTB S2CK

p. 136

COPD Acute Exacerbations/Treatment


Criteria for Oxygen Use in COPD
Although the hypoxic drive elimination concept is
incorrect
Avoid reflexively placing a patient with COPD on
very high-flow 100% nonrebreather mask
Use only as much oxygen necessary to raise pO2
above 90% saturation

Bronchiectasis &
Associated Conditions

The idea of eliminating hypoxic drive is


not accurate. Dyspneic, hypoxic patients
with COPD must get oxygen
MTB S2CK

p. 136

Bronchiectasis/Definition

Bronchiectasis/Etiology

Uncommon disease
from chronic
destruction, remodeling,
and dilation of the
large bronchi

Single MCC
Cystic fibrosis: 50% of cases
Other causes are
Infections
TB ((and non-tuberculous mycobacterium)
y
)
Pneumonia (staph or repeated aspiration)

Permanent anatomic
abnormality that
cannot be reversed or
cured

MTB S2CK

p. 136

Panhypogammaglobulinemia and immune


deficiency

Source: Yale Rosen, commons.wikimedia.org

MTB S2CK

p. 137

Bronchiectasis/Etiology

Bronchiectasis/Diagnostic Tests

Foreign body or tumors


Allergic bronchopulmonary aspergillosis
(ABPA)
Collagen vascular such as RA
Immotile cilia syndrome
y
((Kartagener
g
syndrome)

Best initial test


Chest X-ray shows

These conditions result in repeated and


persistent lung infections
MTB S2CK

p. 137

Dilated, thickened bronchi, sometimes with


tram-tracks, which is thickening of bronchi

Most accurate test


High-resolution CT, general rule of thumb
Sizable airways are larger in diameter than
corresponding adjacent bronchial arteries

MTB S2CK

Bronchiectasis/Diagnostic Tests

p. 137

Bronchiectasis/Diagnostic Tests

Impossible to diagnose bronchiectasis


without imaging study such as CT

Sputum culture
Only way to determine specific bacterial
etiology

MTB S2CK

p. 137

Source: Thomas P. Eberle

MTB S2CK

p. 137

Bronchiectasis/Treatment

1. Chest physiotherapy (cupping and


clapping) and postural drainage
Essential for dislodging plugged-up bronchi

2. Treat each episode of infection as it


arises
Same antibiotics as for COPD exacerbations
Only difference is inhaled antibiotics seem to
have some efficacy

3. Surgical resection

Hypersensitivity to fungal antigens that


colonize bronchial tree
Almost exclusively with asthma and atopic
disorders
Look for
Asthmatic with recurrent episodes of
brown-flecked sputum and transient
infiltrates on X-ray

Cough, wheezing, hemoptysis, bronchiectasis

Focal lesions
MTB S2CK

Allergic Bronchopulmonary Aspergillosis


(ABPA)

p. 138

MTB S2CK

p. 138

Allergic Bronchopulmonary Aspergillosis


(ABPA)/Diagnostic Tests

Allergic Bronchopulmonary Aspergillosis


(ABPA)/Treatment

Peripheral eosinophilia
Skin test reactivity to aspergillus
antigens
Precipitating
p
g antibodies to aspergillus
p g
on blood test
Elevated serum IgE levels
Pulmonary infiltrates on chest X-ray or
CT

1. Oral steroids (prednisone) for severe cases

MTB S2CK

p. 138

Autosomal recessive from mutation that


codes for chloride transport
Known as CF transmembrane conductance
regulator (CFTR)

Mutations in CFTR g
gene damage
g chloride
and water transport across apical surface of
epithelial cells in exocrine glands throughout
the body

p. 138 139

2. Itraconazole or voriconazole orally for


recurrent episodes
i d
Inhaler cant deliver high enough dose
steroids to be effective in ABPA

MTB S2CK

Cystic Fibrosis (CF)/Etiology

MTB S2CK

Inhaled steroids arent effective for ABPA

Source: Mucoviscidose.PNG:
Mirmillon, commons.wikimedia.org

p. 138

Cystic Fibrosis/Etiology
Leads to abnormally thick mucus in
General
Growth failure (malabsorption)
Vitamin deficiency states
Vitamins A,D,E,K
Nose & Sinuses
Nasal polyps
Sinusitis
Liver
Hepatic steatosis
Portal hypertension
Gallbladder

Biliary cirrhosis

Neonatal
obstructive jaundice

Cholelithiasis

MTB S2CK

p. 139

Lungs
Bronchiectasis
Bronchitis
Broncholitis
Pneumonia
Atelectasis
Hemoptysis
Pneumothorax
Reactive airway
disease

Cor pulmonale
Respiratory failure
Mucoid impaction
of the bronchi
Allergic
bronchopulmonary
aspergillosis

Heart
Right ventricular
hypertrophy
Pulmonary artery
dilation
Source: Maen K Abu Househ, commons .wikimedia.org

10

Cystic Fibrosis/Etiology

Cystic Fibrosis/Etiology

Bone
Hypertrophic
osteoarthropathy
Clubbing

Arthritis

Osteoporosis

Intestines
Meconium ileus
Meconium peritonitis
Rectal prolapse
Intussusception
Volvulus
Fibrosing colonopathy
(strictures)
Appendicitis
Intestinal atresia
Distal intestinal obstruction
syndrome
Inguinal hernia
MTB S2CK p. 139

Spleen

Hypersplenism

Damaged mucus clearance decreases ability to get


rid of inhaled bacteria

Stomach

GERD

Neutrophils in CF dump tons of


DNA into airway secretions
clogging them up

Pancreas
Pancreatitis
Insulin
I
li d
deficiency
fi i
Symptomatic
hyperglycemia
Diabetes
Reproductive
Infertility
Aspermia, absence
of vas deferens
Amenorrhea
Delayed puberty
Source: Maen K Abu Househ, commons .wikimedia.org

Cystic Fibrosis/Etiology

MTB S2CK

p. 139

Cystic Fibrosis/Presentation
Over 1/3 CF patients are adults
Look for
Young adult with chronic lung disease

Cough
Sputum
py
Hemoptysis
Bronchiectasis
Wheezing
Dyspnea

And
Recurrent episodes of infection

Sinus pain and polyps are common


Source: nih.gov

Cystic Fibrosis/Presentation

MTB S2CK

p. 139

p. 139

Cystic Fibrosis/Presentation

Gastrointestinal Involvement
Meconium ileus:
infants with abdominal
distention
Pancreatic
insufficiency (in 90%)
with steatorrhea and
vitamin A, D, E, and K
malabsorption

MTB S2CK

Recurrent
pancreatitis
Distal intestinal
obstruction
Biliaryy cirrhosis

Islets spared,
beta cell function is
normal until much
later in life

Genitourinary Involvement
Men are often infertile
95% have azoospermia, vas deferens
missing in 20%

Women are infertile because


Chronic lung disease alters menstrual
cycle
And
Thick cervical mucus blocks sperm entry
MTB S2CK

p. 139

11

Cystic Fibrosis/Diagnostic Tests

Cystic Fibrosis/Diagnostic Tests

Most accurate test: increased sweat chloride test


Pilocarpine increases acetylcholine levels
increases sweat production

Sputum Culture

Chloride levels in sweat > 60 mEq/L in CF on


repeated testing establishes diagnosis

Genotyping, not as accurate as increased sweat


chloride level

Nontypable Haemophilus influenzae


Pseudomonas aeruginosa
Staphylococcus aureus
Burkholderia cepacia

Because there are so many different types of


mutations leading to CF

MTB S2CK

p. 139

MTB S2CK

Cystic Fibrosis/Treatment

p. 140

Cystic Fibrosis/Treatment

1. Antibiotics are routine

Gross hemoptysis

Eliminating colonization: difficult


Sputum culture: essential to guide therapy
Inhaled aminoglycosides: almost exclusively limited to
CF

Rigid bronchoscopy
Unsuccessful

2. Inhaled recombinant human deoxyribonuclease


( hDN
(rhDNase)
)

Embolization
Interventional radiology

Breaks down massive amounts of DNA in respiratory


mucus that clogs up airways

3. Inhaled bronchodilators
Albuterol

4. Lung transplantation
MTB S2CK

Place patient bad lung-side down


Helps prevent bleeding into healthier lung, which is
providing most of oxygen exchange

p. 140

Community Acquired Pneumonia/Definition

Pneumonia Part 1

Community-acquired pneumonia
(CAP)
Pneumonia occurring before
hospitalization
Or
Within 48 hours of hospital admission

MTB S2CK

p. 140

12

CAP/Definition

Most common infectious cause of death


in U.S.

CAP/Etiology
Common Pathogens in CAP & Their Associations
Haemophilus
influenzae

Klebsiella
pneumoniae

COPD
Alcoholism
Diabetes

MCC: Streptococcus pneumoniae

Staphylococcus
aureus
Recent viral
infection
(influenza)

Mycoplasma
pneumoniae
Young
Healthy
patients

Anaerobes

Animals
at time of
giving
birth
Veterinarians
Farmers

Contaminated water
sources
Air
conditioning
Ventilation
systems

Chlamydophila
pneumoniae

Poor
dentition
Aspiration

Coxiella
burnetii

Legionella

Chlamydia
psittaci
Birds

Hoarseness
MTB S2CK

p. 140

MTB S2CK

p. 141

CAP/Presentation

CAP/Presentation

All forms of pneumonia present with


Fever and cough
Severe infection
Associated with dyspnea
Distinguished by abnormalities of

Abdominal pain or diarrhea can occur with infection


in lower lobes irritating intestines through
diaphragm

Vital signs (tachycardia, hypotension, tachypnea)


Or
Mental status

Dyspnea, high fever, and abnormal


chest X-ray are main ways to
distinguish pneumonia from bronchitis
MTB S2CK

p. 141

CAP/Presentation

Legionella is particularly known for causing diarrhea

Chills or rigors are sometimes a sign of


bacteremia
S. pneumo would be most common

Chest pain (often pleuritic) occurs from


inflammation of pleura
Hypothermia is as bad as fever in terms of
pathologic significance
MTB S2CK

p. 141

CAP/Presentation
Organism-specific Associations on Presentation

USMLE S2 may play abnormal breath


sounds as part of multimedia and ask you
to recognize them
Dullness to percussion if theres
effusion
Bronchial breath sounds and
egophony consolidation of air spaces
MTB S2CK

p. 141

Klebsiella
pneumoniae

Mycoplasma
pneumoniae

Pneumocystis

Hemoptysis from
necrotizing disease
disease,
currant jelly
sputum

Dry cough, rarely


severe bullous
severe,
myringitis

AIDS with
<200
CD4 cells

Legionella

Anaerobes
Foul-smelling sputum,
rotten eggs
MTB S2CK

p. 141

Gastrointestinal symptoms
Abdominal pain, diarrhea,
or CNS symptoms
Headache & confusion

13

CAP/Presentation

CAP/Presentation

Dry or non-productive cough


Mycoplasma
Viruses
Coxiella
Pneumocystis
Chlamydia

Preferentially involve interstitial


space
Air spaces of alveoli empty

MTB S2CK

p. 142

Thats why theres less sputum


production!
Specific sputum colors are
useless in determining etiology
MTB S2CK

p. 142

CAP/Diagnostic Tests

CAP/Diagnostic Tests

Best initial test for all respiratory


infections

Atypical pneumonia
Organism not visible on Gram stain and
not culturable on standard blood agar

Chest X-ray: but cant determine specific


etiology

Sputum Gram stain and sputum


culture
Best ways to first determine specific
etiology
Many organisms wont be detected
MTB S2CK

p. 142

Mycoplasma
Chlamydophila
Legionella
Coxiella
And
Viruses
MTB S2CK

CAP/Diagnostic Tests

30-50% of cases
of CAP

p. 142

CAP/Diagnostic Tests

Chest X-ray
Bilateral interstitial Nonproductive
infiltrates with
cough
Mycoplasma
X-rays lag behind
Viruses
clinical findings
Coxiella
Pneumocystis
Chlamydia

1st chest X-ray can


be falsely negative
in 10-20%

Right middle lobe infiltrate characteristic of bacterial pneumonia. Source: Nirav Thakar, MD

MTB S2CK

p. 142

MTB S2CK

p. 143

14

CAP/Diagnostic Tests

CAP/Diagnostic Tests

Blood Cultures are positive


5-15% of CAP
Particularly with S. pneumoniae!
Sputum Gram stain is adequate if:
> 25 WBCs and < 10 epithelial cells

MTB S2CK

p. 143

Interstitial infiltrates leave the air space empty. This chest x-ray can be consistent
with PCP, Mycoplasma, viruses, and Chlamydia. Source: Craig Thurm, MD

MTB S2CK

p. 143

CAP/Diagnostic Tests

CAP/Diagnostic Tests

Chest CT and MRI show


Greater definition of abnormalities found
on chest X-ray
But
Wont
W t determine
d t
i specific
ifi microbiologic
i bi l i
etiology

Tests Done in Severe Disease with an Unclear


Etiology, or Those Not Responding to Treatment
Thoracentesis

In infectious diseases radiologic test is


never the most accurate test
MTB S2CK

p. 143

Any new large


effusion should be
analyzed
Empyema
Infected pleural
effusion
Acts like abscess
Only improves if
drained with chest
tube
MTB S2CK

CAP/Diagnostic Tests

CAP/Diagnostic Tests

Etiology of pleural effusions


Transudative

Secondary to
Increased PCWP
Or
Decreased
intravascular oncotic
pressure

Source: nih.gov

p. 143

Exudative

Secondar to
Secondary
Increased
vascular
permeability

Empyema
LDH > 60% of serum
OR
Protein > 50% of serum
pH < 7.2,
7 2 +gram stain
stain, +culture
+culture, or frank
pus

New, large effusions


secondary to pneumonia
should be tapped
MTB S2CK

p. 143

15

CAP/Diagnostic Tests

CAP/Diagnostic Tests

Bronchoscopy

S. pneumo can be tested via urine antigen

Only needed if sputum stain and culture


and blood cultures dont yield an organism
and patients condition is worsening
despite empiric therapy
Exception is pneumocystis pneumonia

MTB S2CK

p. 143 144

Specific Diagnostic Tests by Organism


Mycoplasma
pneumoniae

PCR
Cold
agglutinins
Serology
Special
culture
media
MTB S2CK

CAP/Treatment

p. 144

Rising serologic
titers

Urine antigen,
Culture on charcoalyeast extract

Severity of disease,
not the etiology drives
initial therapy

Mycoplasma & Chlamydophila,


rarely confirmed because they
are simply treated empirically

MTB S2CK

CAP/Treatment

p. 144

CAP/Treatment
Outpatient Treatment

Previously healthy or no
antibiotics in past 3 months
and mild symptoms

Comorbidities or
antibiotics in past 3
months

Macrolide
- Azithromycin
or
clarithromycin
Or
Doxycycline

Respiratory
fluoroquinolone
Levofloxacin
Or
Moxifloxacin

p. 144

Bronchoalveolar
lavage (BAL)
Legionella

p. 144

Determines location in which to place


patient

MTB S2CK

Pneumocystis
jiroveci (PCP)

CAP/Treatment

Rare to have specific organism


identified at time treatment is initiated
If case describes organism on Gram
stain treatment is directed towards that
organism
Most important step is determining
severity of disease.

MTB S2CK

Chlamydophila
pneumoniae,
Chlamydia
psittaci &
Coxiella burnetii

Inpatient Treatment
Respiratory fluoroquinolone: levofloxacin or
moxifloxacin
Or
Ceftriaxone and azithromycin

Almost all infectious diseases are initially treated


empiricallythat is, without a specific etiology

MTB S2CK

p. 144

16

CAP/Treatment

CAP/Treatment

Reasons to Hospitalize
80% safely treated outpatient with oral antibiotics
Severe disease is defined as a combination of
Hypotension (systolic <
90 mmHg)
Respiratory rate >
30/min
pO2 < 60 mmHg, pH <
7.35
Elevated BUN > 30
mg/dL,
Sodium <130 mmol/L
MTB S2CK

Glucose >250 mg/dL


Pulse > 125/min
Confusion
Temperature > 104 F
65 or comorbidities
such as cancer, COPD,
CHF, renal failure, or
liver disease

p. 145

Hypoxia and hypotension as


single factors are a reason to
hospitalize a patient

Chest X-ray does not guide


admission cannot tell severity
of hypoxia

MTB S2CK

p. 145

CAP

CURB 65
Confusion
Uremia
Respiratory Distress
BP low

65-year-old woman is hospitalized with CAP. Sputum Gram


stain: Gram-positive diplococci. Sputum culture doesnt grow a
specific organism. Chest X-ray: lobar infiltrate and large
effusion. Shes placed on ceftriaxone and azithromycin.
Thoracentesis: marked elevated LDH and protein level with
17,000 WBC/ L. Blood cultures grow Streptococcus
pneumoniae with minimal inhibitory concentration (MIC) to
penicillin
i illi < 0
0.1
1 g/mL.
/ L Oxygen
O
saturation
t ti 96% on room air.
i BP
110/70, T 102 F, Pulse 112.
What is the next step in management?
a. Repeated thoracentesis Will add nothing
b. Placement of chest tube for suction
c. Add ampicillin to treatment No benefit
d. Place patient in ICU
No need for chest tube
e. Consult pulmonary Will add nothing
MTB S2CK

CAP/Treatment

MTB S2CK

p. 146

p. 145

CAP/Treatment

Pleural effusion with a large meniscus sign. Only a fluid sample from
thoracentesis can determine the specific cause. Source: Craig Thurm, MD

MTB S2CK

p. 146

Effusion should be freely mobile and from a layer when the patient lies
on her side. Source: Nishith Patel.

17

CAP/Treatment

CAP/Treatment
Pneumococcal Vaccination
Everyone > 65 should receive vaccination with 23
polyvalent vaccine
Chronic heart
Liver
Vaccinated as soon as
Kidney
their underlying disease
is apparent, regardless
Or
of age
Lung disease
Including asthma

MTB S2CK

p. 146

Hydropneumothorax is both abnormal air and fluid (effusion) in the pleural


space. Chest tube drainage is the most effective way to remove this
condition. Source: Albert Takem, MD.

MTB S2CK

p. 146

CAP/Treatment

CAP/Treatment

Pneumococcal Vaccination
Other reasons to vaccinate

Pneumococcal Vaccination
Generally healthy: single dose at 65
If first vaccination was given before 65
or with other conditions previously
described a second dose should also
described,
be given 5 years after first dose

Functional or anatomic asplenia (e.g., sickle cell


disease)
Hematologic malignancy (leukemia, lymphoma)
Immunosuppression: diabetes mellitus,
alcoholics, corticosteroid users, AIDS or HIV
positive
CSF leak and cochlear implantation recipients

MTB S2CK

p. 146 147

Healthcare workers do not need


pneumococcal vaccine
MTB S2CK

p. 147

Healthcare Associated Pneumonia or


Hospital Acquired Pneumonia (HAP)/Definition

Pneumonia Part 2

Pneumonia developing > 48 hours after


admission
g
incidence of Gram Much higher
negative bacilli such as E. coli or
Pseudomonas

MTB S2CK

p. 147

18

HAP/Treatment

HAP/Treatment

Main difference in management is


Macrolides (azithromycin or clarithromycin) are
unacceptable as empiric therapy

Piperacillin and ticarcillin


always used in combination
with beta-lactamase inhibitor
((e.g.,
g tazobactam or
clavulanic acid)

Centered around therapy for Gram-negative bacilli


Antipseudomonal cephalosporins: cefepime or
ceftazidime
Or
Antipseudomonal penicillin: piperacillin/tazobactam
Or
Carbapenems: imipenem, meropenem, or
doripenem
MTB S2CK

p. 147

MTB S2CK

p. 147

Ventilator Associated Pneumonia/Definition

VAP/Diagnosis

Mechanical ventilation interferes with normal


mucociliary clearance of respiratory tract
(e.g., ability to cough)
Positive pressure is tremendously damaging
to normal ability to clear colonization
Ventilator-associated pneumonia (VAP) has
an incidence as high as 5% per day in first
few days on ventilator

Because of multiple countercurrent


illnesses (e.g., CHF) even a diagnosis
of VAP can be hard to establish
Look for

MTB S2CK

p. 147

VAP/Diagnostic Tests

Sputum culture: nearly worthless


Due to colonization of endotracheal tube
(ET)

Diagnosis of specific etiology is


extremely difficult on ventilator

MTB S2CK

p. 148

Fever and/or rising WBC count


New infiltrate on chest X-ray
Purulent secretions coming from
endotracheal tube

MTB S2CK

p. 147

VAP/Diagnostic Tests
Least accurate but
easiest to do
Bronchoalveolar
lavage (BAL)

Tracheal
aspirate

Suction catheter
placed
l
d into
i t ET and
d
aspirates contents
below trachea when
catheter is past end
of tube

MTB S2CK

Most accurate but


dangerous

Bronchoscope placed
deeper in lungs
where there arent
supposed to be
organisms
Can be contaminated
when passed through
the nasopharynx

Protected brush
specimen
Tip of
p
bronchoscope
covered when
passed through
nasopharynx,
then uncovered
only inside lungs
Much more
specific,
decreased
contamination

p. 148

19

VAP/Diagnostic Tests

VAP/Treatment
Combine 3 different drugs

1. Antipseudomonal
beta-lactam
PLUS

Cephalosporin
(ceftazidime or cefepime)
Or
Penicillin
(piperacillin/tazobactam)
Or
Carbapenem (imipenem,
meropenem, or doripenem)
d i
)

2. Second antipseudomonal agent


PLUS

MTB S2CK

p. 148

Subcutaneous emphysema is air abnormally leaking into soft tissue of


the chest wall. Chest tube placement may cause air to leak into soft
tissues of the chest wall. Source: Birju Shah, MD.

3. Methicillin-resistant
antistaphylococcal agent
MTB S2CK

p. 148 149

Aminoglycoside
(gentamicin or
tobramycin or amikacin)
Vancomycin
Or
Linezolid

VAP/Treatment

Change initial therapy for VAP if


specific etiology is identified

MTB S2CK

p. 149

Patient hospitalized for head trauma and subdural hematoma


is intubated for hyperventilation and subsequent craniotomy.
Several days after admission hematemesis ensues and
stomach stress ulcers are found. Lansoprazole is started. VAP
develops, placed on imipenem, linezolid, and gentamicin.
Phenytoin, started prophylactically. Three days later creatinine
rises followed by seizures . Repeat head CT shows no
changes.
h
What is the most appropriate next step in management?
a. Switch phenytoin to carbamazepine Less likely to cause
renal failure (RF)
b. Stop lansoprazole Not likely to
cause RF
c. Stop imipenem
d. Stop linezolid Not likely to cause RF
e. Perform an electroencephalogram Will add nothing
MTB S2CK

p. 149

Lung Abscess/Etiology

Lung Abscess/Etiology

Rare because of prompt treatment of


aspiration pneumonia

Large-volume aspiration occurs from

Occurs only with


Large-volume
Large ol me aspiration of oral/phar
oral/pharyngeal
ngeal
contents, usually with poor dentition, who
isnt adequately treated

MTB S2CK

p. 149

Stroke with loss of gag reflex


Seizures
Intoxication
Endotracheal intubation

Aspiration pneumonia happens in


the right upper lobe when lying flat

MTB S2CK

p. 149

20

Lung Abscess/Presentation

Lung Abscess/Diagnostic Tests

Look for

Best initial test: chest X-ray

Person with one of these risk factors


presenting a chronic infection developing
over several weeks with large-volume
sputum that is foul smelling because of
anaerobes

Weight loss is common

MTB S2CK

p. 150

Lung Abscess/Diagnostic Tests

Cavity, possibly with air-fluid level

Chest CT is more accurate


Only lung biopsy can establish specific
microbiologic etiology
Sputum culture is the wrong answer for
diagnosing a lung abscess. Everyones
sputum has anaerobes from mouth flora.
MTB S2CK

p. 150

Lung Abscess/Treatment

Clindamycin is best to cover lung


abscess

Cavity consistent with an abscess with a thick wall and an air-fluid level. Source: Alejandro de la Cruz, MD.

MTB S2CK

p. 150

MTB S2CK

p. 150

Pneumocystis Pneumonia (PCP)/Etiology

PCP/Presentation

Agent causing PCP has been renamed


P. jiroveci instead of P. carinii
Almost exclusively in patients with AIDS
whose CD4 cell count is < 200/ L and
who aren
arentt on prophylactic therapy

Look for

Also occurs in chronically


immunosuppressed patients, especially
those on long term high dose steroids

MTB S2CK

p. 150

Patient with AIDS presenting with dyspnea


on exertion, dry cough, and fever

Question will often suggest or directly


state that CD4 count is low (< 200/ L)
and that patient isnt on prophylaxis

MTB S2CK

p. 150

21

PCP/Diagnostic Tests

PCP/Diagnostic Tests

Best initial test

Sputum stain: quite specific if its positive

Chest X-ray showing bilateral interstitial


infiltrates
Or
ABG looking for hypoxia or an increased
A-a gradient

LDH levels are always elevated


Most accurate test

If positive, there is no need to do further testing


If negative, bronchoscopy as the best
diagnostic test

Normal LDH means DONT answer PCP as


the most likely diagnosis

Bronchoalveolar lavage
MTB S2CK

p. 150

MTB S2CK

PCP/Diagnostic Tests

p. 150 151

PCP/Treatment
Best initial therapy for both treatment and prophylaxis

S2CK asks what is the most likely diagnosis,


not what is the for sure diagnosis

Cannot distinguish PCP from


Mycoplasma, Chlamydophila, or viruses
by X-ray alone. However, in HIV, PCP is
most likely with interstitial infiltrates

MTB S2CK

p. 151

Trimethoprim/sulfamethoxazole (TMP/SMX)

Add steroids to decrease mortality if PCP is severe


Severe PCP: pO2 < 70 or A-a gradient > 35
Atovoquone
Alternative to TMP/SMX if mild PCP, meaning
g there is
only mild hypoxia

If toxicity from TMP/SMX switch to either


Clindamycin and primaquine
Or
Pentamidine

MTB S2CK

p. 151

PCP/Treatment
HIV-positive African American man is admitted with
dyspnea, dry cough, high LDH, and pO2 of 63 mmHg.
He is started on TMP/SMX and prednisone. On 3rd
hospital day he develops severe neutropenia and
rash. He has anemia and smear shows bite cells.
What is the most appropriate next step in
management?
a. Stop TMP/SMX Need to treat
Will not help acute
b. Begin antiretroviral medications opportunistic infection
c. Switch TMP/SMX to intravenous pentamidine
IV for active
d. Switch TMP/SMX to aerosol pentamidine
disease
e. Switch TMP/SMX to clindamycin and primaquine
MTB S2CK

p. 151

Contraindicated due to G6PD

Often students will see 2 correct


treatments and think theres a mistake in
the question.
question If there are 2 correct
treatments, look for a contraindication to
one of them.

MTB S2CK

p. 152

22

PCP/Prophylaxis

PCP/Treatment

AIDS with CD4 count < 200/ L

1. TMP/SMX

Rash
Or
Neutropenia

2. Atovoquone
or Dapsone

Always choose therapy based first on


efficacy, not adverse effects

Dapsone is contraindicated in
those with G6PD deficiency

Aerosol pentamidine: not used as second-line


therapy for prophylaxis
Due to less efficacy than either atovoquone or
dapsone

MTB S2CK

p. 152

MTB S2CK

p. 152

HIV-positive woman with 22 CD4 cells/ L is admitted with PCP


and is treated successfully with TMP/SMX. Prophylactic
TMP/SMX and azithromycin are started. She is then started on
antiretroviral medication and her CD4 rises to 420 cells for last
6 months.
What is the most appropriate next step in management?
p TMP/SMX And azithromycin
a. Stop
y
because CD4
b. Stop both TMP/SMX and azithromycin
Cannot stop antiretrovirals;
c. Stop all medications and observe
CD4 will
d. Stop all medications if the PCR-RNA viral load is
undetectable CD4 will ; antiretrovirals maintain CD4
e. Continue all the medications No prophylaxis needed
f. Stop the azithromycin
And TMP/SMX because CD4
MTB S2CK

Tuberculosis

p. 152

Tuberculosis (TB)/Etiology

Tuberculosis/Etiology

Due to Mycobacterium tuberculosis


Initial infection most commonly leads to
latent TB
Most symptomatic cases due to

Almost all patients with TB have 1+ established risk


factors

Reactivation
R
ti ti off latent
l t t infection
i f ti rather
th
than primary exposure

Pulmonary TB most common, but can


affect any organ system
MTB S2CK

p. 152

Recent immigrants (in past 5 years)


Prisoners
HIV positive
Healthcare workers
Close contact of someone with TB
Steroid use
Hematologic malignancy
Alcoholics
Diabetes mellitus

MTB S2CK

p. 153

23

Tuberculosis/Presentation

Tuberculosis/Diagnostic Tests

Look for

Best initial test: chest X-ray as with all


respiratory infections
Sputum stain and culture specifically for acid-fast
bacilli (mycobacteria) must be done 3 times to
fully exclude TB
g
acid-fast stains,, but clinical suspicion
p
If 3 negative
high

Previously listed risk factors presenting with fever,


cough, sputum, weight loss, hemoptysis, and night
sweats

Symptoms are almost always > 3 weeks in


duration
You cannot answer TB as diagnosis
without a clear risk factor, a cavity on
chest X-ray or a positive smear

MTB S2CK

p. 153

Bronchoscopy with BAL or pleural biopsy

PPD skin testing is never best test for TB in


symptomatic patient
MTB S2CK

p. 153 154

Tuberculosis/Diagnostic Tests

Tuberculosis/Treatment

Most common finding: cavitary upper lobe infiltrate

Smear positive: begin therapy with 4 drugs


Rifampin, isoniazid, pyrazinamide, and ethambutol
(RIPE)

Ethambutol not needed if known from start of


therapy that organism is sensitive to all TB
medications
Ethambutol: given as part of 4-drug empiric therapy prior
to knowing sensitivity of organism

After using RIPE for first 2 months


Stop ethambutol and pyrazinamide
And
Continue rifampin and isoniazid for next 4 months
MTB S2CK

p. 153

Chest x-ray with upper lobe disease consistent with TB. Source: Craig Thurm, MD.

MTB S2CK

p. 154

Tuberculosis/Treatment

Tuberculosis/Treatment

Standard of care: 6 months total of


therapy
Treatment is extended to 9 months for

Toxicity of Therapy
All TB medications cause hepatoxicity

Osteomyelitis
Miliary tuberculosis
Meningitis
Pregnancy or any other time pyrazinamide
isnt used

MTB S2CK

p. 154

Dont stop them unless transaminases


rise
i 3
3-5
5 titimes upper limit
li it off normall

MTB S2CK

p. 154

24

Adverse Effects of Antituberculosis Therapy


Rifampin

Tuberculosis/Treatment

Isoniazid

Pyrazinamide

Ethambutol

Peripheral
neuropathy

Hyperuricemia

Optic
neuritis/
color
vision
i i

Toxicity

Red color
to body
secretions

Use of Steroids
Glucocorticoids decrease risk of constrictive
pericarditis in those with pericardial involvement
Decrease neurologic complication in TB meningitis

Treatment

Pregnant patients shouldnt


receive pyrazinamide

None,
benign
finding
MTB S2CK

Use
pyridoxine
to prevent

No treatment
unless
symptomatic

Decrease
dose in
renal
failure

p. 154

MTB S2CK

p. 154

Latent Tuberculosis/PPD Testing

Latent Tuberculosis/PPD Testing

Indications for PPD Testing


Not a general screening test for whole
population
Only those in risk groups previously
described should be screened
PPD testing isnt useful in those who
are symptomatic or those with abnormal
chest X-rays

What Is Considered a Positive Test?


Only induration is counted towards a positive test
Erythema is irrelevant
Induration > 5 mm

MTB S2CK

p. 154

HIV-positive patients
Glucocorticoid users
Close contact with active TB patients
Abnormal calcifications on chest X-ray
Organ transplant recipients

MTB S2CK

p. 155

Latent Tuberculosis/PPD Testing

Latent Tuberculosis/PPD Testing

Induration > 10 mm

Two-Stage Testing
If patient never had a PPD skin test before, a
second test is indicated within 1-2 weeks if first test
is negative

Recent immigrants (past 5 years)


Prisoners
Healthcare workers
Close contacts with TB patients
Hematologic malignancy
malignancy, alcoholics
alcoholics, DM

Induration > 15 mm

1st test maybe falsely negative

If 2nd test is negative: truly negative


If 2nd test is positive: first test was false negative

Those with no risk factors

If the first test is positive, a


second test isnt necessary

Everyone with a reactive PPD test should have


a chest X-ray to exclude active disease
MTB S2CK

p. 155

MTB S2CK

p. 155

25

Latent Tuberculosis/PPD Treatment

Latent Tuberculosis/PPD Treatment

Treatment for a Positive PPD


After active TB has been excluded with a
chest X-ray, patients should receive 9 months
of isoniazid
Positive PPD confers a 10% lifetime risk of TB
Isoniazid results in 90% reduction in risk; after
isoniazid lifetime risk of TB goes from 10% to
1%

Those at high risk (e.g., healthcare workers),


should have a PPD annually to screen for
conversion
Majority of risk for developing active TB lies
within first 2 years after conversion

MTB S2CK

p. 155

Once PPD is positive, itll


always be positive in future

MTB S2CK

p. 155

Latent Tuberculosis/PPD Treatment


PPD testing is one of the hardest and most
misunderstood tests on USMLE S2 CK. Reread
preceding section and forget what youve learned in
the past.

Solitary Pulmonary Nodule &


Interstitial Lung Disease

Previous BCG has no effect on these


recommendations. If PPD is positive, the
patient must take isoniazid for 9 months
even if he or she had BCG.
MTB S2CK

p. 155

Solitary Pulmonary Nodule

Solitary Pulmonary Nodule

For Step 2, be sure to know when biopsy is


appropriate!
Qualities of Benign and Malignant Pulmonary Nodules
Benign
< 30 years old
No change in size
Nonsmoker
Smooth border
Small, < 1 cm
Normal lung
Adenopathy ( )
Dense, central calcification
Normal PET scan
MTB S2CK

p. 156

Malignant
> 40 years old
Enlarging
Smoker
Spiculated (spikes)
Large, > 2 cm
Atelectasis
Adenopathy (+)
Sparse, eccentric calcification
Abnormal PET scan

Best initial step in all lung lesions is to compare size


with old X-rays
Biopsy all enlarging lung lesions,
particularly
p
y if they
y are rapidly
p y enlarging
g g
However, doubling in size < 30 days more likely to
be infectious than malignant (think about clinical
scenario to decide)

MTB S2CK

p. 156

26

Solitary Pulmonary Nodule

High Probability Lesions/Management

When many features are described under


malignant
Resect (remove) lesion
Sputum cytology, needle biopsy, and PET
scanning shouldnt be done because a
negative
ti ttestt iis lik
likely
l false
f l negative
ti

If resection is one of the choices, then its


the answer

Source: Lange123 at the German language Wikipedia, commons.wikimedia.org

MTB S2CK

p. 156

Intermediate Probability Lesions/


Management

Intermediate Probability Lesions/


Management

There are some gray or inconclusive


aspects of solitary pulmonary nodule such as

Sputum cytology

Age range gap (between 30 and 40)


Or
Size ((between 1 cm and 2 cm))

Definition of intermediate probability

MTB S2CK

p. 156

If positive, this is highly specific, the most


appropriate next step in management is resection
of lesion
If negative cytology doesnt exclude malignancy

Bronchoscopy or transthoracic needle biopsy


Each is the most appropriate next step in most
patients with intermediate probability of malignancy
Bronchoscopy for central lesions
Transthoracic biopsy for peripheral lesions
MTB S2CK

p. 156

Intermediate Probability Lesions/


Management

Intermediate Probability Lesions/


Management

Diagnostic test question in intermediate


lesions; clear answer must be present.
Choice of test may not be clear, but
adverse effects are always clear.
clear Most
common adverse effect of a transthoracic
biopsy is pneumothorax.

Positron emission tomography (PET scan)


Tells whether content of lesion is malignant
without biopsy
Malignancy has increased uptake of tagged
glucose
Sensitivity of PET scan is 85 - 95%
Negative scan points away from malignancy

MTB S2CK

p. 157

MTB S2CK

p. 157

27

Intermediate Probability Lesions/


Management

Solitary Pulmonary Nodule

Video-assisted thoracic surgery (VATS)


VATS is both more sensitive and more
specific than all other forms of testing
Frozen section in operating room allows for
immediate conversion to an open
thoracoscopy and lobectomy if malignancy is
found
MTB S2CK

Benign pulmonary nodules often


represent scar from previous infection
Immigrant, think TB
SW United States, think
coccidioidomycosis
y
Ohio River Valley, think histoplasmosis

p. 157

Interstitial Lung Disease/Definition

Interstitial Lung Disease/Etiology

Diseases characterized
by inflammation and/or
fibrosis of interalveolar
septum

Specific Causes of Interstitial Lung Disease


Idiopathic pulmonary fibrosis (IPF) and other
idiopathic interstitial pneumonias
Collagen vascular disease
Granulomatous disorders
Hypersensitivity disorders
Pneumoconiosis
Radiation
Drugs: bleomycin, busulfan, amiodarone,
methylsergide, nitrofurantoin, cyclophosphamide

Fibrosis causes
Impaired gas exchange
Increased lung stiffness
And
Decreased lung
compliance & expansion
MTB S2CK

p. 157

Source: Ed Uthman. commons.wikimedia.org

Interstitial Lung Disease/Etiology


Types of Pneumoconioses
Exposure
Coal
Sandblasting, rock
mining, tunneling
Shipyard
py
workers,, p
pipe
p
fitting, insulators
Cotton
Electronic manufacture
Moldy sugar cane

Disease
Coal workers pneumoconiosis
Silicosis
Asbestosis

MTB S2CK

p. 157

Interstitial Lung Disease/Presentation


All forms of pulmonary fibrosis, regardless of
etiology, present with

Dyspnea worsening on exertion


Hypoxia worsening on exertion
Fine rales or crackles on examination
Loud P2 heart sound (if pulmonary HTN present)
Clubbed fingers

Byssinosis
Berylliosis
Bagassosis

Inflammatory infiltration with white


cells is reversible (treatable),
whereas fibrosis is irreversible

Biopsy shows granulomas in berylliosis


MTB S2CK

p. 157

MTB S2CK

p. 158

28

Interstitial Lung Disease/Diagnostic Tests

Interstitial Lung Disease/Diagnostic tests

Best initial test: always chest X-ray


High resolution CT scan is more
accurate than chest X-ray
Most accurate test: lung biopsy
Echocardiography
E h
di
h will
ill often
ft show
h
pulmonary HTN and possibly right
ventricular hypertrophy

MTB S2CK

PCP. Source: commons.wikimedia.org

p. 158

Interstitial Lung Disease/Diagnostic tests

Interstitial Lung Disease/Diagnostic Tests


PFTs
Restrictive lung disease with proportional decrease
FEV1, FVC, TLC, and RV all , but since
everything is decreased, the FEV1/FVC ratio will
be normal
DLCO
Decreased in proportion to severity of alveolar
septal thickening

MTB S2CK

p. 158

Severe, long-standing interstitial fibrosis produces thick walls between alveoli


that give the appearance of honeycombing. Source: Craig Thurm, MD.

Interstitial Lung Disease/Treatment


Most types of interstitial lung diseases are
untreatable
Biopsy shows
White cell or inflammatory infiltrate; prednisone
should be used
Mostlyy fibrosis ((e.g.,
g IPF),
) steroids typically
yp
y NOT
effective

Of all causes of pneumoconioses, berylliosis is


most likely to respond to steroids
Due to granulomas (sign of inflammation)

MTB S2CK

p. 158

MTB S2CK

p. 158

Interstitial Lung Disease/


Hypersensitivity Pneumonitis

Specific type of interstitial lung disease


caused by pulmonary hypersensitivity
reaction to certain environmental
antigens
Inflammation from hypersensitivity
reaction results in alveolar thickening
and granuloma formation
MTB S2CK

p. 158

29

Interstitial Lung Disease/


Hypersensitivity Pneumonitis

Interstitial Lung Disease/


Hypersensitivity Pneumonitis

Presentation
Acutely

Most common antigens causing hypersensitivity


pneumonitis include

Fever, dyspnea, severe cough


Within 4-6 hours of antigen exposure

Or
Chronically
Progressive dyspnea on exertion, fine
rales, pulmonary fibrosis

Antigens from feathers (bird fanciers lung)


MAI antigen (hot tub lung)
Spores of actinomyces from moldy hay (farmers
l
lung)
)
Spores of actinomyces from compost (mushroom
workers lung)
Spores of actinomyces from air conditioners (air
conditioner lung)

Interstitial Lung Disease/


Hypersensitivity Pneumonitis

Treatment
Corticosteroids help to reduce
inflammation
AVOID
O ONGOING
O GO G EXPOSURE
OSU
to
inciting agent

Sarcoidosis &
Thromboembolic Disease

Sarcoidosis/Definition/Etiology

Sarcoidosis/Presentation

More common in African American


women

Look for

Idiopathic inflammatory disorder


predominantly of lungs,
lungs but can affect
most of the body

Young African American woman with SOB on


exertion and occasional fine rales on lung exam, but
without wheezing of asthma

Erythema
y
nodosum and lymphadenopathy
y p
p
y on
chest X-ray hands you diagnosis question

Characterized by noncaseating
granulomas
MTB S2CK

p. 159

MTB S2CK

p. 159

30

Sarcoidosis/Presentation

Sarcoidosis/Presentation
Brain complications

Also presents with


Constitutional symptoms: fever, malaise, weight
loss
Arthritis
Parotid gland enlargement
Facial pals
palsy
Heart block and restrictive cardiomyopathy
CNS involvement
Iritis and uveitis

Eye problems (burning,


itching, tearing or pain)
Salivary glands
Enlarged lymph nodes
in neck & chest
Heart complications

Lupus pernio (painful


skin sores on face)
Skin lesions on back, arms,
neck, face, and scalp

Granulomas (inflamed
lumps in lungs)
Liver enlargement
Spleen enlargement

Enlarged lymph
nodes in chest near
windpipe and lungs
Scarring and
granulomas in lung

Erythema nodosum (itchy


and painful rashes) on the
lower legs and ankles

MTB S2CK

p. 159

Source: National Heart, Lung, and Blood Institute, commons.wikimedia.org

Sarcoidosis/Presentation

Sarcoidosis/Diagnostic Tests

Answer sarcoidosis when chest X-ray or CT shows


hilar adenopathy in generally healthy African
American woman
Although liver and kidney
granulomas are very common
on autopsy, they are rarely
symptomatic

MTB S2CK

p. 159

Chest X-ray: best initial test


Hilar lymphadenopathy > 95% with sarcoidosis
Parenchymal involvement can also be present in
combination with lymphadenopathy
CXR can reveal bilateral hilar lymphadenopathy only
Hilar lymphadenopathy with parenchymal disease
Or
Parenchymal disease alone (depending on stage of
disease)

MTB S2CK

Sarcoidosis/Diagnostic Tests

p. 159

Sarcoidosis/Diagnostic Tests

Lymph node biopsy: most accurate test


Granulomas are noncaseating

Elevated ACE level: 60%


Hypercalciuria: 20%
Hypercalcemia:
yp
5%
Granulomas in sarcoidosis make vitamin D

PFTs: restrictive lung disease


Decreased FEV1, FVC, and TLC with a normal
FEV1/FVC ratio
Source: Jmh649, commons.wikimedia.org

MTB S2CK

p. 159

31

Sarcoidosis/Treatment

Thromboembolic Disease/Definition

Prednisone: drug of choice

Pulmonary emboli (PE) and deep venous


thrombosis (DVT) are treated as same
disease

Few patients fail to respond

Asymptomatic hilar adenopathy doesnt


need to be treated

MTB S2CK

p. 159

PE
From DVT vessels of legs in 70% of cases
and pelvic veins in 30%, but since risks and
treatment are the same they can be
discussed simultaneously

MTB S2CK

p. 160

Thromboembolic Disease/Etiology

Thromboembolic Disease/Presentation

Virchows triad outlines conditions that predispose


to venous TE

Look for
Sudden onset SOB with clear lungs on examination
and normal chest X-ray
Other findings in PE

Immobility
CHF
Recent surgery

Trauma
T
Surgery
Recent
fracture

MTB S2CK

p. 160

Factor V Leiden mutation


Any malignancy leads to
DVT

Thromboembolic Disease/Diagnostic Tests


Most questions about PE concern diagnostic testing and
treatment

There is no single, uncomplicated diagnostic test


for PE
y, EKG,, and ABG are best initial tests
Chest X-ray,
In PE, the main issue is to know Whats the most
common finding?
And
Whats the most common abnormality when theres
an abnormality?
MTB S2CK

p. 160

Tachypnea, tachycardia, cough, and hemoptysis


Leg pain from DVT
Pleuritic chest pain from lung infarction
Fever can arise from any cause of clot or hematoma
Extremely severe emboli will produce hypotension

MTB S2CK

p. 160

Thromboembolic Disease/Diagnostic Tests


Chest X-ray
Usually normal in PE
Most common
abnormality is
atelectasis
Wedge
Wedge-shaped
shaped
infarction (Hampton
hump), and oligemia of
one lobe (Westermark
sign) are much less
common than simple
atelectasis
MTB S2CK

EKG
Usually shows sinus
tachycardia
Most common
abnormality is
nonspecific
p
ST-T wave
changes
Only 5% will show right
axis deviation, RV
hypertrophy or RBBB

p. 160 161

32

Thromboembolic Disease/Diagnostic Tests


ABG
Hypoxia and respiratory alkalosis ( pH and
CO2) with normal chest X-ray is extremely
suggestive of PE

Frequent wrong answer is to


choose S1, Q3, T3 as the most
common abnormality on EKG

MTB S2CK

p. 161

Thromboembolic Disease/Diagnostic Tests

65-year-old woman with recent hip replacement has acute


onset of SOB and tachycardia. Chest X-ray normal. Hypoxia
on ABG, increased A-a gradient, EKG with sinus tachycardia.
What is the most appropriate next step in management?
a. Intravenous unfractionated heparin
b. Thrombolytics Hemodynamically unstable & acute RV failure
Anticoagulants contraindicated,
c. Inferior
I f i vena cava filter
filt
recurrent emboli & RV dysfunction
d. Embolectomy If heparin ineffective & persistent
e. Spiral CT scan hypotension, hypoxia, tachycardia
f. Ventilation/perfusion (V/Q) scan If initial labs suggestive, treat!
g. Lower-extremity Doppler studies Dont wait to confirm PE
h. D-dimer Poor specificity
MTB S2CK

p. 161

Thromboembolic Disease/Diagnostic Tests

Spiral CT scan
Also called CT angiogram
Standard of care in diagnostic testing to
confirm presence of PE after X-ray,
EKG and ABG
EKG,
Specificity is excellent (> 95%)
However, sensitivity is 85%, it can miss
15% of clots
MTB S2CK

Chest spiral CT scan with radiocontrast agent showing multiple filling defects both at the bifurcation
and in the pulmonary arteries. Source: James Heilman, MD, commons.wikimedia.org

p. 161

Thromboembolic Disease/Diagnostic Tests

Thromboembolic Disease/Diagnostic Tests

Ventilation/Perfusion (V/Q) scan


May reveal segmental areas of mismatch
High probability scans have no clot (false positive)
in 15%
Low probability scans have a clot (false negative) in
15%
Interpret results in combination with clinical
suspicion
Chest X-ray must be normal for V/Q scan to
have any degree of accuracy. Do a spiral
CT if chest X-ray is abnormal.
MTB S2CK

p. 161 162

V/Q or ventilation perfusion scanning is still very useful in evaluating pulmonary emboli. A
positive test is an area that is ventilated with decreased perfusion. Source: Nishith Patel.

MTB S2CK

p. 161

33

Thromboembolic Disease/Diagnostic Tests

D-dimer
Test is very sensitive (better than 97%
negative predictive value)
Specificity poor. Any clot or bleeding
elevates D-dimer level
Negative test excludes clot
Positive test doesnt mean anything

MTB S2CK

p. 162

Thromboembolic Disease/Diagnostic Tests

D-dimer is answer when pretest probability of PE is


low and you need a simple, noninvasive test to
exclude thromboembolic disease

MTB S2CK

p. 162

Thromboembolic Disease/Diagnostic Tests

Thromboembolic Disease/Diagnostic Tests

Lower extremity (LE) Doppler study


If LE Doppler positive, no further testing needed
Only 70% of PEs originate in legs, so itll miss 30%
of cases
You dont need a spiral CT or V/Q scan to confirm a
PE if there
theress a clot in legs because they wont
won t
change therapy
Patient still needs heparin and 6 months of warfarin

Spiral CT negative
V/Q or LE
Doppler
negative
withhold therapy
with heparin

MTB S2CK

p. 162

Thromboembolic Disease/Diagnostic Tests

LE Dopplers
D
l
are a good
d test if
V/Q and spiral CT do not give
clear diagnosis

MTB S2CK

p. 162

Thromboembolic Disease/Diagnostic Tests

Angiography
Most accurate test
Nearly 100% specificity and a false negative rate
< 1%
Unfortunately, theres 0.5% mortality, which is
high if you consider the tens of thousands of tests
a year that are needed to exclude PE in all cases

When testing for PE, angiography is rarely done

MTB S2CK

p. 162

Thrombus (labeled A) causing a central obstruction in the left main


pulmonary artery. ECG tracing shown at bottom. Source: Aung Myat
and Arif Ahsan, commons.wikimedia.org

34

Thromboembolic Disease/Treatment

Thromboembolic Disease/Treatment

Heparin
Best initial therapy
Warfarin should be started
simultaneously with heparin to

When is an inferior vena cava (IVC) filter the


right answer?
Contraindication to the use of anticoagulants
(e.g., melena, CNS bleeding)
Recurrent emboli while on heparin or fully
therapeutic warfarin (INR of 23)
Right ventricular (RV) dysfunction with an
enlarged RV on echo. In this case, the next
embolus, even if seemingly small, could be
potentially fatal

Achieve
Achie e therapeutic
therape tic INR of 2 to 3 times
normal as quickly as possible

MTB S2CK

p. 162

Thromboembolic Disease/Treatment

When are thrombolytics the right answer?


Hemodynamically unstable patients (e.g.,
hypotension and tachycardia)
Acute RV dysfunction
Contraindicated in p
patients with recent
surgery or bleeding

MTB S2CK

p. 162

Thromboembolic Disease/Treatment

When are direct-acting thrombin


inhibitors (argatroban, lepirudin) the
answer?
Heparin-induced thrombocytopenia
When is aspirin the answer?
Never

No specific time limit in which to


use thrombolytics as there is in
stroke or MI
MTB S2CK

p. 162

MTB S2CK

p. 162 163

Pulmonary Hypertension/Definition

Pulmonary Hypertension,
Obstructive Sleep Apnea, &
Acute Respiratory
Distress Syndrome

Lung or pulmonary circulation normally


extremely low pressure
Systolic 25 mmHg
Diastolic 8 mmHg
Mean 15 mmHg

MTB S2CK

p. 163

35

Pulmonary Hypertension/Definition

Pulmonary Hypertension/Etiology

Classified as either
Primary
Idiopathic dysfunction of pulmonary arteries

Secondary causes include

Chronic hypoxemia leads to pulmonary


HTN, which results in more hypoxemia

L sided-heart failure (MCC)


(
)
Intracardiac L
R shunting
Hypoxic vasoconstriction from chronic lung
disease (e.g., COPD)
Thromboembolic disease
MTB S2CK

p. 163

MTB S2CK

p. 163

Pulmonary Hypertension/Presentation

Pulmonary Hypertension/Diagnostic Tests

Chest X ray and CT

Dyspnea and fatigue


Syncope (exertional!)
Chest pain (exertional!)
Wide splitting of S2 with loud P2 or tricuspid and
pulmonary valve insufficiency
Sign of right
right-sided
sided heart failure
Its impossible to know that pulmonary
HTN is causing dyspnea without tests

Best initial tests


Showing dilation
of proximal
pulmonary
arteries with
narrowing or
pruning of
distal vessels

Source: tmcr.usuhs.mil

MTB S2CK

p. 163

MTB S2CK

p. 163

Pulmonary Hypertension/Diagnostic Tests

Pulmonary Hypertension/Diagnostic Tests

Right heart or Swan Ganz catheter


Most accurate
test
And
Most precise
method to
measure
pressures

EKG
Right axis deviation,
right atrial and
ventricular
hypertrophy
Echocardiography
RA and RV
hypertrophy; Doppler
estimates pulmonary
artery (PA) pressure

MTB S2CK

p. 163

Source: commons.wikimedia.org

MTB S2CK

V/Q scanning
Identifies chronic
PE as cause of
pulmonary HTN
CBC
Shows
polycythemia from
chronic hypoxia

p. 163

36

Pulmonary Hypertension/Treatment

Pulmonary Hypertension/Treatment

1. Correct underlying cause when one is clear


2. Idiopathic disease is treated with

Oxygen slows progression, particularly


with COPD

Inhaled or intravenous prostacyclin analogues


(pulmonary arterial vasodilators): epoprostenol,
treprostinil, iloprost, beraprost
Endothelin antagonists: bosentan
Phosphodiesterase inhibitors: sildenafil
Calcium-channel blockers

Only lung transplantation is


curative for idiopathic
pulmonary HTN

Only if demonstrated to be responsive to them during


right heart catheterization

MTB S2CK

p. 164

MTB S2CK

p. 164

Obstructive Sleep Apnea (OSA)/Definition

Obstructive Sleep Apnea (OSA)/Presentation

Cessation of
airflow due to
upper airway
obstruction during
sleep
Obesity
Ob it is
i mostt
commonly
identified cause

Patients present with daytime


somnolence and history of loud snoring
Other symptoms include
Headache
Impaired memory and judgment
Depression
HTN
Erectile dysfunction
Source: Habib M'henni, commons.wikimedia.org

MTB S2CK

p. 164

MTB S2CK

p. 164

Obstructive Sleep Apnea

Obstructive Sleep Apnea/Treatment

Risk factors include

1. Weight loss and avoidance of alcohol


2. Nasal continuous positive airway
pressure (nasal CPAP)
3. Surgical widening of airway
(uvuloplatopharyngoplasty) if this fails
4. Avoid use of sedatives

Male gender
Obesity
Large uvula/tongue
And
Retrognathia (recession of mandible)

Arrhythmias and erythrocytosis are common


Can lead to pulmonary HTN and RV failure
Most accurate test is polysomnography (sleep
study)
Shows multiple episodes of apnea
MTB S2CK

p. 164

MTB S2CK

p. 164

37

Acute Respiratory Distress Syndrome (ARDS)/


Definition
Acute respiratory failure from overwhelming lung
injury or systemic disease
Characterized by
Severe hypoxia
Poor lung compliance
And
Noncardiogenic pulmonary edema
ARDS is caused by endothelial injury at level of
alveolus, making lung cells leaky so that alveoli fill
up with fluid
MTB S2CK

p. 164 165

ARDS/Diagnostic Tests

ARDS/Etiology

ARDS is idiopathic
Large number of illnesses and injuries are
associated with alveolar epithelial cell and
capillary endothelial cell damage
Illnesses and injuries associated with
developing ARDS include
Sepsis or aspiration
Lung contusion/trauma
Near-drowning
Burns or pancreatitis
MTB S2CK

p. 165

ARDS/Diagnostic Tests

Chest X-ray shows bilateral


infiltrates quickly become confluent
(white out)
Air bronchograms are common

MTB S2CK

p. 165

ARDS/Diagnostic Tests

Source: Samir , commons.wikimedia.org

ARDS/Diagnostic Tests

Defined as having PaO2/FiO2 ratio < 200


The FIO2 is expressed as a decimal (e.g., room
air with 21% oxygen = 0.21)
If pO2 is 105 on room air (21% oxygen or 0.21),
then the ratio of pO2/FIO2 is 500 (i.e., 105/.21)
If pO2 (as measured on ABG) is 70 while
breathing 50% oxygen, the ratio is 70/0.5 or 140

ARDS is associated with normal findings on


right heart catheterization
Wedge pressure is normal (< 18 mmHg)
Air bronchograms are a sign of dense consolidation of the lung air space. This is a case of pneumococcal pneumonia
that left only the air space in the larger bronchi open or air bronchograms. Source: Omid Edrission, MD.

MTB S2CK

p. 165

MTB S2CK

p. 165

38

ARDS/Treatment

ARDS/Treatment

Low tidal volume mechanical


ventilation is the best support while
waiting to see if lungs will recover

Positive end-expiratory pressure (PEEP) is used


when patient is undergoing mechanical ventilation
to FIO2
Levels of FIO2 > 50% are toxic to lungs
Maintain plateau pressure < 30 cm of water
measured on ventilator

Use 6 mL/kg of tidal volume

Steroids aren
arentt clearly beneficial in
most cases
They may help in late-stage disease in
which pulmonary fibrosis develops

MTB S2CK

p. 166

No treatment is proven to reverse ARDS.


Dont forget to treat the underlying cause.

MTB S2CK

p. 166

39

Cancer Screening
Breast Cancer
Cervical Cancer
Colon Cancer
Prostate Cancer
Lung Cancer

Preventative Medicine
Conrad Fischer, MD
Associate Professor of Medicine
Touro College of Medicine
New York City

Breast Cancer Screening

Mammography screening tool of choice


Start at 50 years old
When reward is highest
Starting at 40 is controversial
MRI,
MRI CT
CT, and US are adjuvant screening
Self-examination not recommended.
Never the correct answer on S2CK.

MTB S2CK p. 356

TAMOXIFEN
Greater risk:
DVT/PE and endometrial cancer
Less risk: Fractures
Remember!
Better to be ALIVE with a DVT
than DEAD with normal legs

Which of the following is most likely to benefit an


asymptomatic patient with multiple first-degree relatives
with breast cancer?
a.
b.
c
c.
d.
e.
f.

Tamoxifen
Most familial breast ca. NOT BRCA
BRCA testing
Aromatase inhibitors Unproven in prevention
Effective but to lesser degree
Dietary modification
Performed in breast cancer patients
HER-2/neu testing
Estrogen/progesterone Performed in breast cancer patients
receptor testing

MTB S2CK p. 356

Can STOP mammography at age 75

HER-2/neu positive patients treated


with trastuzumab if you HAVE
cancer! Not a prophylactic!

Cervical Cancer Screening

Colon Cancer Screening

Papanicolaou test =
Pap Smear
Start at age 21
Done every 3 years,
with cytology, until 30
years old
After age 30, if HPV
testing added, then
every 5 years
Stop at age 65 with
adequate screening
history and low risk
MTB S2CK p. 357

Colonoscopy

Katsumi M. Miyai, M.D., Ph.D.,


Regents of the University of California.
Used with permission.

The most accepted screening test


Start at 50 years
All other testing
Repeat every 10 years
modalities are
inferior to
Exceptions
colonoscopy
Family history
1st-degree relative 40 or ten years earlier
Whichever is EARLIER!
HNPCC:
3 relatives, 2 generations, 1 premature
Start at 25 and do colonoscopy every 1-2 years
MTB S2CK p. 357

Colon Cancer Screening

Prostate Cancer Screening

Familial Adenomatous Polyposis

Theres no test that reduces mortality


Prostate Specific Antigen (PSA)
Doesnt reduce mortality
Useful in tracking current disease
Digital Rectal Exam (DRE)
Doesnt reduce mortality

Sigmoidoscopy at age 12, then every year

Personal History of Polyps


Repeat
p
every
y 3-5 y
years

PSAs require a conversation with the


patient regarding pros and cons
MTB S2CK p. 357 358

Lung Cancer Screening

Summary of Preventive Medicine Key Points

Not recommended at this time


Chest radiograph, CT scan, MRI, PET, etc.
All have their purpose but NOT in
screening

Any question asking who and when to


screen for lung cancer is a trick

MTB S2CK p. 358

Mammography >50 to age 75


Tamoxifen for multiple 1st degree relatives
BRCA is NOT clear!
Pap smear interval to 5 years after age 30,
if combined with HPV testing
No lung or prostate screening clearly
effective

MTB S2CK p. 358

Lipid Screening
Different by age and gender

Non Cancer Screening and


Prevention
Lipid Screening
Hypertension: Diagnosis and Control
Diabetes Mellitus
Other Screenings

Men start at 35 yo
Women start at 45 yo
Frequency depends on CVD status

What is the test?


Cholesterol panel
Total CHL, HDL, LDL
LDL guides therapy usually
We do NOT know what to do with Triglycerides!

MTB S2CK p. 358

Lipid Screening

Lipid Screening

Normal Values?

Goals of Therapy

Determined by whats tolerable given risk

Interventions

HDL Cholesterol > 40

0-1 Risk:

Diet, Exercise, Pharmacology

Goals differ based on comorbidities


Diabetes
HTN
Coronary disease or equivalent
Coronary Artery Disease (CAD) equivalents:
Carotid disease, peripheral arterial disease (PAD),
diabetes mellitus, and aortic disease
MTB S2CK p. 358

Diet at LDL >160


Drugs at LDL >190

2 or more risks
Diet at LDL >130
Drugs at LDL >160

For CAD and Equivalents


LDL Cholesterol < 100 Both diet and drugs
Possibly < 70 for CAD combined with Diabetes!
MTB S2CK p. 358

Hypertension

Diabetes Screening

All adults (>18) should be screened at every visit


At least every two years
HTN

Who is screened?

Elevated BP at 2 separate occasions


Not clear what to do about Pre-hypertension

Those with CHD


Hypertension is the clearest risk to screen for DM
Fasting Blood Glucose (FBG)

Test

Diabetes
126 mg/dL

Oral Glucose
Tolerance Test (OGTT)

200 mg/dL

Hemoglobin A1C

6.5%

MTB S2CK p. 358

MTB S2CK p. 359

Osteoporosis

Other Screenings

Bone density 2.5 SDs below average


Recommendation

Abdominal Aortic Aneurysms

Every woman screened at 65 yo


Dual-Energy X-ray Absorptiometry (DEXA) scan

Osteoporosis

Fractures

Increased Mortality

All men
65 -75
ANY smoking history
Receive abdominal ultrasounds at least once

Tobacco (traditional or smokeless)


Employ the Five-As
Ask, Advise, Attempt, Assist, Arrange

Intimate Partner Violence


Not to be overlooked

MTB S2CK p. 361

MTB S2CK p. 361

Alcohol Dependence
Alcohol
Know difference between dependence and abuse
CAGE Questions

Vaccinations

Do they feel the need Cut down?


Do they feel Annoyed with criticism of drinking?

Influenza and Pneumococcal


Varicella
Hepatitis A & B
Tetanus
Meningococcal

Have they ever felt Guilty by their drinking?


Do they ever need an Eye-opener?

MTB S2CK p. 361 362

Influenza Vaccination

Pneumococcal Vaccine

Influenza A and B

23-valent Pneumococcal Polysaccharide Vaccine


Protects against Streptococcal pneumoniae
Who gets it?

Strains circulating in previous year


Inactivated (killed) injection

Who gets it?

Chronically ill

Everybody Every year


Healthcare workers,
workers pregnant women
women, elders

Respiratory, heart, kidney, and liver diseases

Immunocompromised
Asplenia

INFLUENZA VACCINE
Every person
Every year

MTB S2CK p. 359

Elderly:

65 yo

Revaccination after 5 years

MTB S2CK p. 359

Varicella & Herpes Zoster Vaccination

Varicella Vaccination

Live attenuated vaccine against varicella virus


Who gets Chicken Pox vaccination?
ALL adults seronegative for varicella infection

Who gets Herpes Zoster vaccination?


ALL adults

60 yo

Reduces incidence and severity

Herpes Zoster vaccine

Chicken Pox vaccine

Different strengths and indications


Vaccines given to persons 60 to prevent
zoster have 18 times viral antigens of
pediatric vaccines

Richard Usatine, M.D.


Used with permission.

MTB S2CK p. 360

Hepatitis Vaccination
Hepatitis A
Fecal oral transmission
Infection related to poor
hygiene and crowding
No carrier state
Low mortality/morbidity
Hepatitis A Vaccine
Chronic liver disease
MSM or IV drug users
Infected close contacts
Travelers

Hepatitis Vaccination
Hepatitis B
Percutaneous, perinatal,
and sexual routes
(+) Carrier state
10% develop chronic
disease
Hepatitis B Vaccine

Hepatitis B Vaccine
DIABETES is an indication!

Chronic Liver disease


MSM or IV drug users
Infected close contacts
Healthcare workers
Dialysis patients

MTB S2CK p. 360

MTB S2CK p. 360

Tetanus Vaccine

Meningococcal Vaccination

Tetanus Toxoid booster every 10 years


One Tdap (Tetanus Acellular Pertussus) should be
one of them
Five years after a dirty wound

Quadrivalent conjugate polysaccharide vaccine


A,C,Y,W

Very efficacious
Who gets it?
ALL children age 11
Adults:
Asplenia or equivalent immunodeficiency
Epidemic settings (military, college dormitories)

MTB S2CK p. 360

MTB S2CK p. 360

Important Definitions

Epidemiology

Prevalence:
Incidence:
Sensitivity (SN):

Sensitivity & Specificity


Positive and Negative Predictive Values
Changing the Cutoff?
Important Formulas

The 2x2 Table

Specificity (SP):
Positive Predictive
Value (PPV):

Total number of diseased patients in the


population.
Number of NEW cases during a specified
period of time.
Probability that person with disease has
a positive test.
Probability that person without disease
has a negative test.
Of those with a positive test, what
proportion have disease?

Negative Predictive Of those with a negative test, what


proportion do not have disease?
Value (NPV):

The 2x2 Table: Sensitivity and Specificity


(+) D

(-) D

(+) Test

((-)) Test
est

(+) D

(-) D

(+) Test

((-)) Test
est

Sensitivity (SN):

Probability that person with disease has


a positive test.
= a / (a+c)

The 2x2 Table: Sensitivity and Specificity


(+) D

(-) D

(+) Test

((-)) Test
est

Specificity (SP):

Probability that person without disease


has a negative test.
= d / (b+d)

The 2x2 Table: PPV and NPV


(+) D

(-) D

(+) Test

((-)) Test
est

Positive Predictive
Value (PPV):

Of those with a positive test, what


proportion have disease?
= a / (a+b)

The 2x2 Table: PPV and NPV

The 2x2 Table: SN, SP, PPV, NPV

(+) D

(-) D

(+) Test

((-)) Test

Negative Predictive Of those with a negative test, what


proportion do not have disease?
Value (NPV):

(+) D

(-) D

(+) Test

PPV
a / (a+b)

((-)) Test

NPV
d / (c+d)

SN
a / (a+c)

SP
d / (b+d)

= d / (c+d)

Formulas List

Epidemiology in Practice

Sensitivity (SN):

a / (a + c)

Specificity (SP):

d / (b + d)

Positive Predictive
Value (PPV):

a / (a + b)

Disease

No disease

Negative
g
Predictive
d / (c + d)
Value (NPV):
False Negative
Ratio:

c / (a + c) or (1 SN)

False Positive
Ratio:

b / (b + d) or (1 SP)

Epidemiology in Practice

Test
Cutoff

Epidemiology in Practice

Disease

No disease

Test
Cutoff

Positive Test

Negative Test

Disease

No disease

Test
Cutoff

Epidemiology in Practice

Moving the Cutoff

Positive Test

Negative Test

Disease

No disease

Disease

Test
Cutoff

Test
Cutoff

Moving the Cutoff

Moving the Cutoff

Positive Test

Negative Test

Disease

No disease

Positive Test

Negative Test

Disease

No disease

Test
Cutoff

Test
Cutoff

How does this affect: SP, SN, PPV, NPV ?


Positive Test

Negative Test

(+)T
Disease

Important Epidemiology Concepts


(+) D

(-) D

Sensitivity and specificity are independent of


disease prevalence
Characteristics of test

No disease

(-)T
Positive Test

No disease

PPV and NPV are dependent on disease


prevalence

Negative Test

SN:
Disease

No disease

SP:
PPV:
NPV:

Important Epidemiology Concepts

Formulas List

When using a particular test:

Sensitivity (SN):

a / (a + c)

Specificity (SP):

d / (b + d)

Positive Predictive Value (PPV):

a / (a + b)

Negative Predictive Value (NPV):

d / (c + d)

False Negative Ratio:

c / (a + c) or (1 SN)

False Positive Ratio:

b / (b + d) or (1 SP)

Positive Likelihood Ratio:

(SN) / (1 SP)

Negative Likelihood Ratio:

(1 SN) / SP

Raising the Cutoff


Increases Sensitivity and NPV
Decreases Specificity and PPV

Lowering the Cutoff


Increases Specificity and PPV
Decreases Sensitivity and NPV

Epidemiology
Sensitivity & Specificity
Positive and Negative Predictive Values
Changing the cutoff?
Important Formulas

X rays
RADIOLOGY
Matthew Kinney, MD
Orthopedic Resident
University of California San Diego

Chest X ray Indications

Chest X ray Indications

Subjective pulmonary complaints

Objective pulmonary
findings

Cough
SOB/dyspnea
Pleuritic chest pain
Hemoptysis

Rales (Crackles)
Rhonchi
Wheezing
Hyperresonance/
yp
dullness to percussion
Chest wall tenderness
Tracheal deviation
SVC syndrome
JVD
Plethora

MTB S2CK

p. 491

MTB S2CK

p. 491

Chest X ray Views

Chest X ray Views

Posterior/anterior (PA) films

Anterior/posterior (AP) films

Standard CXR positioning


Patient must be capable of standing

Necessary for immobile patients


ICU
Paralysis

Source: Mikael Hggstrm

Source: commons.wikimedia.org

MTB S2CK

p. 491

Source: James Heilman, MD

MTB S2CK

p. 491

Chest X ray Views

Chest X ray Views

Lateral

Lateral decubitus

Useful to localize lesion found on PA films


Most sensitive test for pleural effusion

Patient lies on side


Useful to evaluate
pleural effusions

Source: Nicholas Lange

MTB S2CK

Source: Zachary M Alexander

p. 492

MTB S2CK

p. 491

Chest X ray Findings

Chest X ray Findings

Lobar pneumonia

Solitary pulmonary nodule

Source: Zachary M Alexander

Source: Nicholas Lange

MTB S2CK

p. 491

MTB S2CK

p. 491

Chest X ray Findings

Chest X ray Findings

Pleural effusion

Pneumothorax

Source: James Heilman, MD

Source: Magnus Manske


Source: John Yasmer

MTB S2CK

p. 491

MTB S2CK

p. 491

Chest X ray Findings

Abdominal X ray Indications

Free air under diaphragm

Small bowel obstruction


Mechanical obstruction
Ileus

Source: Brian Johnston

MTB S2CK

p. 491

MTB S2CK

p. 492

Abdominal X ray Views

Abdominal X ray Views

Abdominal plain film

Kidneys, ureters, and bladder (KUB)

Erect view of abdomen

Supine view of abdomen

Source: Nevit Dilmen

Source: Nevit Dilmen

MTB S2CK

p. 492

MTB S2CK

p. 492

Abdominal X ray Findings

Bone X ray Indications

Small Bowel Obstruction

Mechanical obstruction

Fracture
Bone tumor evaluation
Osteomyelitis
Skeletal Survey
Trauma
Child abuse

Air fluid levels


Source: James Heilman, MD

Source: Stephanie A Bernard

MTB S2CK

p. 492

MTB S2CK

p. 492

Bone X ray Findings


Osteomyelitis
Periosteal elevation
Osteolysis

Computed Tomography (CT)

Surrounded by ring
of sclerosis

Note: XR findings are


delayed (2 weeks)
MRI is positive
earlier

Source: commons.wikimedia.org

MTB S2CK

p. 492

CT Basics

Non contrast Head CT Indications

Definition Computed Tomography


Large series of 2D X-rays around a single axis of
rotation to create 3D image

Severe head trauma

Increased detail clarity


Windowing capability

Loss of consciousness
Altered mental status

Stroke
Hemorrhagic
Ischemic

Drawbacks
High doses of radiation
Often requires IV contrast

Intracranial bleeding
Subdural hematoma
Epidural hematoma

Anaphylaxis risk
Renal damage
Consider 1-2 liters of fluids, NaHCO3, and/or

NAC if mild renal insufficiency

Medication contraindications (metformin)


MTB S2CK

p. 493

Non contrast Head CT

Non contrast Head CT Findings

Bone - bright white


Blood - bright white
CSF - dark black

Ischemic Stroke
Darkening of brain
parenchyma
Blurred Gray-White
Junction
Mass effect

Source: Andrew Ciscel

MTB S2CK

p. 493

Source: Lucien Monfils

MTB S2CK

p. 493

Non contrast Head CT Findings

Non contrast Head CT Findings

Hemorrhagic Stroke

Subdural Hematoma
Crescent-shaped, bright
white region

Bright, localized area


corresponding to bleed
Mass effect
Gray-white junction intact

Not confined by skull


sutures
Adjacent
j
to skull

Mass effect

Source: commons.wikimedia.org
Source: James M. Grimson

MTB S2CK

p. 493

MTB S2CK

p. 493

Non contrast Head CT Findings

Contrast Head CT Indications

Epidural Hematoma
Biconvex, bright white
region

Infection
Meningitis
Abscess

Adjacent to skull
Confined by skull
sutures

Tumor
Primary Cancer
Brain Metastasis

Mass effect

Source: Hellerhoff, commons.wikimedia.org

MTB S2CK

p. 493

Source: commons.wikimedia.org

MTB S2CK

Contrast Head CT Findings

p. 493

Contrast Head CT Findings

Tumor

Tumor

Primary tumor:
Single lesion

Metastasis:
Often multiple lesions

Bright white
Mass effect
Identical to abscess

Bright white
Found mostly at
gray-white junction
Mass effect

Source: James Heilman, MD

Source: James Heilman, MD

MTB S2CK

p. 493

MTB S2CK

p. 493

Abdominal CT Indications

Abdominal CT Views

Retroperitoneal structure pathology


Pancreatitis
Adrenal gland tumors

Appendicitis
Diverticulitis
Nephrolithiasis
Abdominal viscera masses
Liver tumors

Splenic laceration
NOTE: Use both oral and IV contrast
MTB S2CK

p. 493

Source: commons.wikimedia.org

MTB S2CK

p. 493

Abdominal CT Findings

Abdominal CT Findings

Pancreatitis

Appendicitis

Irregular pancreatic outline


Pancreatic enlargement
Retroperitoneal fluid

Appendix enlargement (> 6mm)


Wall thickening
Fat stranding

Appendix
dilated to 15
mm that
doesnt fill with
contrast.
Source: James M Grimsom

MTB S2CK

p. 493

MTB S2CK

Abdominal CT Findings
Diverticulitis
Localized bowel wall
thickening
Fat stranding

p. 493

Source: Andrew Sellers

Abdominal CT Findings
Evidence of diverticulae
Note: If advanced,
abscess may be present

Nephrolithiasis
Evidence of hydronephrosis
Visible calcification in (if obstructing)
urinary tract
Note: No oral or IV contrast

Source: Phil Kang

MTB S2CK

p. 493

Source: Hellerhoff, commons.wikimedia.org

MTB S2CK

p. 493 494

Chest CT Indications

Normal Chest CT

Mass lesions
Lung
Mediastinum

Lung pathology

Interstitial lung disease


Sarcoidosis
Bronchiectasis
Cavitary lesions

Pulmonary embolism

Source: Brendan T Doherty

MTB S2CK

p. 494

MTB S2CK

p. 494

MRI Basics
Definition Magnetic Resonance Imaging

Magnetic Resonance Imaging,


Ultrasonography, & Nuclear
Medicine

1) Magnetic field aligns H20 molecular dipoles


2) RF waves are applied, which alters proton spin
3) When waves are stopped, the protons relax to their
natural spin -- producing a measurable signal
Upshot: Allows visualization of tissue based on water
content
Best test for evaluating soft tissue

Drawbacks
Long scans requiring complete immobility
Certain metal implants are contraindicated
Small bore tubes claustrophobia + body mass issues
MTB S2CK

p. 494

MRI Indications

MRI Findings

CNS pathology

Primary Brain Tumor


Contrast enhancing
Solitary lesion
Mass effect

Brain/spinal cord tumors


MS
Specific regions of the head
Sinuses
Orbits

Musculoskeletal disease
Osteomyelitis
Soft tissue injury
Nerve compression
Herniated disc disease
Brachial plexus injury
MTB S2CK

p. 494

Source: Steven J. Goldstein

MTB S2CK

p. 494

MRI Findings

MRI Findings

Multiple Sclerosis
Contrast-enhanced
Demyelinated plaques
appear white

Spinal Disc Herniation


Disc extruding from
IV space
Compression of
spinal cord

MTB S2CK

Source: Jeffery Hirsch

p. 494

MTB S2CK

p. 494

US Basics
Definition Ultrasonography

Ultrasonography

Sound waves of a certain frequency emitted


Echoes return based on specific tissue properties
Dense objects (e.g., stones, tissue) reflect waves
better than fluid/air
Scanner measures echoes and creates image based on
differences in object density

Drawbacks
Poor at visualizing structures beyond bone
Poor at visualizing structures beyond air
Body habitus affects image quality

MTB S2CK

p. 494 495

US Indications

US Findings

Gallstone disease

Cholecystitis

Renal disease

Presence of gallstone
Stone in lumen
Silhouette
Gall bladder wall
thickening
Pericholestatic fluid
Sonographic
Murphys sign

PCKD
Hydronephrosis
Note: CT preferred for kidney stones

Gynecologic evaluation

PCOS
Uterine evaluation
Pregnancy evaluation
Ectopic pregnancy

Source: Joseph Lagrew

Pancreatic disease (via endoscopic US)


MTB S2CK

p. 494 495

MTB S2CK

p. 494 495

Nuclear Scans Basics


Definition

Nuclear Medicine

Radiolabeled molecules
localize to specific organs
Emission detected and
allow for visualization of
organ function
PET scan = Glucose
Cholescintigraphy =
HIDA
V/Q scan
Source: Jeffrey Hirsch

MTB S2CK

p. 495

Nuclear Scan Types

Nuclear Scan Types

HIDA (Hepatobiliary) scan

Ventilation/perfusion (V/Q) scanning

Source: Kieran Maher

Source: Jonathan Sexton

MTB S2CK

p. 495

MTB S2CK

p. 495

Nuclear Scan Types


Bone scan
Uptake into osteoblasts
Useful for detecting area of high bone turnover
Occult cancer metastasis

Indium scan
Uptake into WBCs
Useful in detecting fever of unknown origin (FUO)

Gallium scan
Uptake with iron metabolism
Useful for detecting FUO and some cancers

Nuclear Ventriculography
Used to measure cardiac ejection fraction
MTB S2CK

p. 495

Osteoarthritis, Gout, &


Pseudogout

RHEUMATOLOGY
Niket Sonpal, MD
Chief Resident
Lenox Hill Hospital NSLIJ
Assistant Clinical Professor Touro College of Medicine

Osteoarthritis/Definition

Osteoarthritis/Etiology

Osteoarthritis or degenerative joint


disease (DJD)
Chronic, slowly progressive, erosive
damage to joint surfaces
Loss of articular cartilage

Incidence directly proportional to


increasing age and trauma to joint
Contact sports with trauma

Obesity increases DJD


DJD is, by far, the MCC of joint
disease.

MTB S2CK

p. 167

MTB S2CK

p. 167

Osteoarthritis/Presentation

Osteoarthritis/Presentation

Most commonly symptomatic in weightbearing joints


Hand is affected, but isnt as great a
cause of disability
Distal interphalangeal (DIP) joints are >
proximal interphalangeal joints (PIP)
and metacarpophalangeal joints (MCP)

Crepitations of involved joints are


common
Effusion is rare
Stiffness is of short duration (<15 min)
DIP enlargement:
l
t Heberdens
H b d nodes
d
PIP enlargement: Bouchards nodes

MTB S2CK

p. 167

MTB S2CK

p. 167

Osteoarthritis

Osteoarthritis/Diagnostic Tests

Heberdens nodes (DIP joint)

Erythrocyte
sedimentation rate
Bouchards nodes
(PIP joint)

Complete blood
countt

Normal Lab Tests

Antinuclear
antibody
tib d

Rheumatoid
factor
Richard Usatine, M.D. Used with permission

MTB S2CK

Osteoarthritis/Diagnostic Tests

p. 167

Osteoarthritis

Dense subchondral
bone

Joint space
narrowing

X-rays
y Show:

Osteophytes

Bone cysts

MTB S2CK

Osteoarthritis/Diagnostic Tests

Absence of inflammation,
normal lab tests,
and short duration of stiffness
distinguishes DJD from RA

MTB S2CK

Source: Russell A. Patterson

p. 168

p. 168

Osteoarthritis/Treatment

1. Weight loss and moderate exercise


2. Acetaminophen: best initial analgesic
3. NSAIDs: used if symptoms arent controlled
with acetaminophen; toxicity - GI bleeding
4. Capsaicin
p
cream
5. Hyaluronic acid injections
6. Intra-articular steroids
7. Joint replacement
Glucosamine and

MTB S2CK

p. 168

chondroitin sulfate are


no more effective than
placebo.

Gout/Definition/Etiology

Defect in urate metabolism


90% of cases in men
Can be from:
Overproduction
or
Underexcretion

Gout/Etiology
Idiopathic

Enzyme deficiency
Overproduction

Increased cell turnover


Acidosis

Renal Insufficiency
Underexcretion

Thiazides or ASA
MTB S2CK

p. 168

MTB S2CK

Gout/Presentation

p. 168

Gout

Man who develops sudden, excruciating


pain, redness, and tenderness of big toe at
night after binge drinking with beer
Fever is common
Hard to distinguish initial gouty attack from
infection without arthrocentesis
Metatarsal phalangeal (MTP) joint of great
toe most frequently affected
Also in ankles, feet, and knees
Richard Usatine, M.D. Used with permission.

MTB S2CK

p. 168

Gout/Presentation

Gout/Diagnostic Tests

Chronic Gout
Tophi: tissue deposits of urate crystals with foreign
body reaction
Most often tophi occur in cartilage, subcutaneous
tissues, bone, and kidney
Often take years to develop
Uric acid kidney stones occur in 5% to 10% of
patients
Long asymptomatic periods between attacks are
common

Most accurate test:


Aspiration of joint

Needle-shaped
crystals with
negative
birefringence on
polarized light
microscopy
Commons.Wikimedia.org. used with permission

Tophi can occur anywhere


in the body.

MTB S2CK

p. 169

MTB S2CK

p. 169

Gout/Diagnostic Tests

Gout/Diagnostic Tests

WBC joint fluid elevated 2,000 to 50,000/ L


Predominantly neutrophils
Infected joint has redness, warmth, and
tenderness
Its essential to tap joint to exclude infection

Uric acid levels: elevated at some point in


95% of patients
Single level during an acute attack
normal in 25%
Acute attacks = ESR and leukocytosis

Protein and glucose levels in synovial fluid dont


help answer the most likely diagnosis question

MTB S2CK

p. 169

MTB S2CK

p. 169

Gout/Diagnostic Tests

Gout/Treatment

X-rays:

Acute Attack
NSAIDs superior to colchicine as best initial
therapy
Corticosteroids injection: single joint , oral: multiple
joints
Steroids (e
(e.g.,
g triamcinolone) is answer when:

Normal in early
disease
Erosions of cortical
bone happen later

No response to NSAIDs
Contraindication to NSAIDs such as renal
insufficiency

Contraindication questions are always clear


Source: Wikimedia.org

MTB S2CK

p. 169

MTB S2CK

Chronic Management

p. 169

Gout/Treatment
Management

1) Diet

Colchicine gives diarrhea and


bone marrow suppression
(neutropenia).

Decrease consumption of alcohol, particularly


beer; lose weight
Decrease high-purine foods

2) Stop thiazides, aspirin, and niacin


3) Colchicine is effective at preventing
second attack of gout

Probenecid, NSAIDs, and


sulfinpyrazone are
contraindicated in renal
insufficiency. Allopurinol is safe
with renal injury.

4) Probenecid and sulfinpyrazone increase


the excretion of uric acid in kidney
(uricosuric)
5) Allopurinol decreases production of uric
acid
MTB S2CK

p. 169 170

MTB S2CK

p. 170

Gout/Treatment
Adverse Effects of Chronic Treatment
Hypersensitivity (rash, hemolysis, allergic interstitial
nephritis) occurs with uricosuric agents and
allopurinol
Colchicine can suppress white cell production
Toxic epidermal necrolysis or Stevens-Johnson
Stevens Johnson
syndrome may occur from allopurinol
Dont start uricosuric agents or allopurinol during
acute attacks of gout. If the patient is already on
allopurinol you can safely continue it.
MTB S2CK

p. 170

Calcium Pyrophosphate Deposition Disease


(Pseudogout)/Definition/Etiology

Calcium-containing salts depositing in


articular cartilage
Most common risk
Hemochromatosis and hyperparathyroidism

CPDD can occur:


Diabetes
Hypothyroidism
Wilson disease

MTB S2CK

p. 170

Calcium Pyrophosphate Deposition Disease


(Pseudogout)/Presentation

Calcium Pyrophosphate Deposition Disease


(Pseudogout)/Diagnostic Tests

CPDD differs from gout in that large


joints such as the knee and wrist are
affected, but not particularly the first
MCP of the foot
It differs from DJD in that DIP and PIP
arent affected

Uric acid levels normal


X-ray: calcification of cartilaginous structures
Most accurate test is arthrocentesis, which
reveals p
positively
y birefringent
g
rhomboidshaped crystals
Synovial fluid: elevated level of WBCs 2,000
to 50,000/ L - nonspecific

MTB S2CK

p. 170

MTB S2CK

Pseudogout

p. 170

Pseudogout

Source Wikimedia
Source: Boma O. Afiesimama

Calcium Pyrophosphate Deposition Disease


(Pseudogout)/Diagnostic Tests

Disease

Characteristic
History

Physical
Findings

Synovial Fluid
Analysis

DJD

Older, slow,
worse with use

DIP, PIP, hip,


and knees

< 200 WBCs,


osteophytes and
joint space
narrowing

Gout

Men, acute,
binge drinking

1st big toe

2,000 50,000
WBCs, negatively
birefringent
needles

You cannot confirm a diagnosis of


CPDD without aspiration of the joint.

MTB S2CK

p. 170

MTB S2CK

Disease

Characteristic
History

CPDD

Hemochromatosis,
Wrists and
hyperparathyroidism knees

Rheumatoid Young, female,


arthritis
morning stiffness
better with use

MTB S2CK

Physical
Findings

p. 171

Synovial Fluid
Analysis

Disease

Characteristic
History

Physical
Findings

Synovial Fluid
Analysis

2,000 50,000
WBCs, positively
birefringent
rhomboids

Septic
arthritis

High fever,
very acute

Single hot
joint

> 50,000
neutrophils,
culture of fluid

Multiple joints Anti cyclic


of hands and citrulinated
feet
peptide (anti
CCP)

p. 171

MTB S2CK

p. 171

Calcium Pyrophosphate Deposition Disease


(Pseudogout)/Treatment

Best initial therapy: NSAIDs


Severe disease not responsive to
NSAIDs give intra-articular steroids
(e g triamcinolone)
(e.g.,
Colchicine helps prevent subsequent
attacks as prophylaxis between attacks

MTB S2CK

Low Back Pain & Lumbar


Spinal Stenosis

p. 171

Low Back Pain/Etiology

Low Back Pain

Low back pain is No. 1 complaint in US


DJD on X-ray or MRI of the spine is
nearly universal in those > 50 years
totally nonspecific

What Is the Most Likely Diagnosis?


If all diseases described in the following
are excluded, patient has simple low
back pain from lumbosacral strain
(idiopathic)
No imaging studies and no treatment
beyond NSAIDs

Most frequently tested issue is who


shouldnt get an imaging study
MTB S2CK

p. 171

MTB S2CK

p. 171

Compression of the Spinal Cord

Compression of the Spinal Cord

Malignancy or infection compressing


spinal cord is a neurological emergency
that needs urgent identification and
treatment
Look for a history of cancer with sudden
onset of focal neurological deficits (e.g.,
sensory level)

Compression at 10th thoracic vertebra


leads to sensory loss below the
umbilicus
Point tenderness at spine with
percussion of vertebra is highly
suggestive of cord compression
Hyperreflexia is found below level of
compression

MTB S2CK

p. 171

MTB S2CK

p. 172

Compression of the Spinal Cord

Disk Herniation (Sciatica)

Epidural abscess is most often from


Staphylococcus aureus
Epidural abscess presents in same way
as cord compression from cancer, but
theress high fever and markedly
there
elevated ESR

Herniations at L4/5 and L5/S1 level


account for 95% of all disk herniations
The straight leg raise (SLR) test is pain
going into the buttock and below the knee
when the leg is raised > 60 degrees

MTB S2CK

p. 172

Although only 50% of those with a positive


SLR actually have a herniated disk;
sensitivity is 90%
A negative SLR excludes herniation with
95% sensitivity
MTB S2CK

p. 172

Nerve Root Innervation

Low Back Pain/Diagnostic Tests

Nerve root

Motor deficit

Reflex
affected (lost)

Sensory area
affected

L4

Dorsiflexion
of foot

Knee jerk

Inner calf

L5

Dorsiflexion
of toe

None

Inner
forefoot

S1

Eversion of
foot

Ankle jerk

Outer foot

MTB S2CK

p. 172

Imaging required for cord compression, epidural


abscess, ankylosing spondylitis, and cauda
equina syndrome
Best initial test for cancer with compression,
infection, and fractures is plain X-ray
Most accurate test is MRI
CT scan is used as most accurate test if theres
a contraindication to MRI (e.g., pacemaker)
Intrathecal contrast must be given to increase
accuracy (CT myelogram)

MTB S2CK

p. 172

Low Back Pain/Diagnostic Tests

Classification of Back Pain

Imaging in disk
herniation is
controversial
We recommend you
answer no MRI for
just low back pain and
a positive SLR alone
Neurological deficits =
MRI

Diagnosis

MTB S2CK

Source: Nirav Thakur, MD.

p. 172 173

Classification of Back Pain


Diagnosis

History to answer
Most Likely Diagnosis

Ankylosing
spondylitis

Disk herniation

MTB S2CK

p. 173

Under age 40, pain


worsens with rest
and improves with
activity
Pain/numbness of
medial calf or foot

History to answer
Most Likely Diagnosis

Physical Findings

Cord
Compression

History of cancer

Vertebral tenderness,
sensory level,
hyperreflexia

Epidural abscess

Fever high ESR


Fever,

Same as cord
compression

Cauda equina

Bowel and bladder


incontinence,
erectile dysfunction

Bilateral leg weakness,


saddle area anesthesia

MTB S2CK

p. 173

Low Back Pain/Treatment


Physical Findings
Chemotherapy for
Lymphoma

Decreased chest
mobility
Systemic
Glucocorticoids
Gl
ti id
Loss of knee and ankle
reflexes, positive
straight leg raise

Cord Compression
p

Radiation for
Solid Tumors

MTB S2CK

p. 174

Low Back Pain/Treatment

Low Back Pain/Treatment

MRSA

Vancomycin
Linezolid

Acute Neurologic
Deficits

Epidural
p
Abscess

Systemic
Glucocorticoids

MTB S2CK

MSSA

Oxacillin
Nafcillin
Cefazolin

p. 174

Low Back Pain/Treatment

Cauda equina syndrome: surgical


decompression
Disk herniation (sciatica): NSAIDs with
continuation of ordinary activities
Steroid injection into epidural space achieves
rapid
id and
dd
dramatic
ti b
benefit
fit ffor th
those with
ith
sciatica
Surgery rarely needed
The most common wrong
answer for sciatica is bed rest.
MTB S2CK

p. 174

Low Back Pain/Treatment

Most commonly tested point: no


imaging studies in patients without focal
neurological abnormalities or with
simple lumbosacral strain

Beta-lactam antibiotics when organism is


sensitive
Gentamicin = synergy with staph
Surgical drainage for larger collections

Thi k off epidural


Think
id l abscess
b
lik
like endocarditis
d
diti
Use vancomycin as initial empiric therapy
Switch to oxacillin if its sensitive
Drain it if the infection is large enough to
produce neurological deficits or it doesnt
respond to antibiotics alone

MTB S2CK

p. 174

Man with a history of prostate cancer comes to the emergency


department with severe back pain and leg weakness. He has
tenderness of the spine, hyperreflexia, and decreased sensation
below his umbilicus.
What is the most appropriate next step?
a. Dexamethasone
b. MRI Save neurons before seeing them
c. X-ray Low specificity
d. Radiation Not fast enough
e. Flutamide Not fast enough to save neurons
f. Ketoconazole
Not fast enough to stop androgens
g. Finasteride
Not fast enough to stop androgens
h. Leuprolide
Dangerous with peripheral blockade
i. Biopsy
Only if cause unclear
j. Orchiectomy Best long-term treatment
MTB S2CK

p. 174 175

Lumbar Spinal Stenosis/Presentation

Look for a person > 60 with back pain


while walking, radiating into buttocks and
thighs bilaterally
Pain described as worse when walking
downhill, and better when sitting, but the
pedal pulses and ankle/brachial index are
normal
Spinal stenosis can
simulate peripheral arterial disease,
but vascular studies are normal.

MTB S2CK

p. 175

MTB S2CK

p. 175

Lumbar Spinal Stenosis/Presentation

Lumbar Spinal Stenosis/Treatment

Unsteady gait and leg weakness when


walking also occur
About have diminished lower
extremity reflexes
Pain is less with activities in which
patient is leaning forward (e.g., cycling)

The only test is MRI


Weight loss and steroid injections into
the lumbar epidural space improve 25%
to 50% of cases
Surgical correction to dilate the spinal
canal is needed in 75% of patients

MTB S2CK

p. 175

MTB S2CK

p. 175

Fibromyalgia

Fibromyalgia, Carpal Tunnel


Syndrome, & Dupuytren
Contracture

What Is the Most Likely Diagnosis?


Look for young woman with chronic
musculoskeletal pain and tenderness with
trigger points of focal tenderness at trapezius,
medial fat pad of knee, and lateral epicondyle
Cause is unknown
Pain occurs at many sites (neck, shoulders,
back, and hips) with:
Stiffness, numbness, and fatigue
Headaches
Sleep disorder
MTB S2CK

p. 176

Fibromyalgia/Diagnostic Tests

Fibromyalgia/Treatment

No test to confirm fibromyalgia


Based on complex of symptoms, trigger
points at predictable points
All lab tests are normal (e.g., ESR, Creactive protein
protein, rheumatoid factor (RF),
(RF)
and CPK levels)

Best initial therapy is amitriptyline


Other treatments are milnacipran and
pregabalin
Milnacipran is an inhibitor of serotonin and
norepinephrine reuptake and is approved
specifically for the management of
fibromyalgia
Trigger point injections with local anesthetic
are also sometimes used

MTB S2CK

p. 176

MTB S2CK

p. 176

Steroids are the wrong


answer for fibromyalgia.

10

Carpal Tunnel Syndrome/Definition

Peripheral neuropathy from


compression of median nerve as it
passes under the flexor retinaculum
Pressure on nerve interferes with its
sensory and motor function

Carpal Tunnel Syndrome/Etiology


Amyloidosis

p. 176

Pregnancy

Median Nerve
Compression

Rheumatoid
Arthritis

MTB S2CK

Hypothyroidism

Acromegaly

MTB S2CK

Diabetes

p. 176

Carpal Tunnel Syndrome

Carpal Tunnel Syndrome/Diagnostic Tests

What Is the Most Likely Diagnosis?


Look for pain in hand affecting the palm,
thumb, index finger, and radial half of ring
finger, muscle atrophy of thenar eminence
Pain is worse at night and more frequent in
those whose work involves prolonged use of
the hands (e.g., typing)

Tinel sign:
reproduction of
pain and tingling
with tapping or
percussion of
the median
nerve
IMC 2010 DxR Development Group, Inc. All Rights Reserved.

MTB S2CK

MTB S2CK

p. 177

p. 177

Carpal Tunnel Syndrome/Diagnostic Tests

Carpal Tunnel Syndrome/Diagnostic Tests

Phalen sign:
reproduction of
symptoms with
flexion of wrists
to 90 degrees

Most accurate diagnostic tests are


electromyography and nerve conduction
testing

Sensory symptoms happen


before motor symptoms.
IMC 2010 DxR Development Group, Inc. All Rights Reserved.

MTB S2CK

p. 177

MTB S2CK

p. 177

11

Carpal Tunnel Syndrome/Treatment


Management
Best Initial
Wrist Splints
Immobilization to relieve pressure
NSAIDS

Avoid Manual Activity


Steroid
St
id
Injection is used if splints and NSAIDs dont
control
Surgery
Can be curative by mechanically
decompressing the tunnel such as with
cutting open the flexor retinaculum
MTB S2CK

p. 177

Dupuytren Contracture
Hyperplasia of palmar fascia leading to nodule
formation and contracture of fourth and fifth fingers
Genetic predisposition and association with
alcoholism and cirrhosis
Patients lose ability to extend fingers, which is more
often cosmetic embarrassment than functional
impairment
Triamcinolone injection
Surgical release when function is impaired

MTB S2CK

p. 177

Dupuytren Contracture

Sports Medicine &


Osteoporosis

Commons.Wikimedia.org. used with permission

Rotator Cuff Injury

Rotator Cuff Injury

Damage to rotator cuff of muscles,


tendons, and bursae around shoulder
leads to inability to flex or abduct the
shoulder
It presents with pain in shoulder thats
that s
worse at night when lying on affected
shoulder
There can be severe tenderness at the
insertion of the supraspinatus

MRI is the most


accurate test
Treat with
NSAIDs, rest, and
physical therapy
Steroids
Surgery

MTB S2CK

p. 177

Commons.Wikimedia.org. used with permission

MTB S2CK

p. 177

12

Patellofemoral Syndrome

Patellofemoral Syndrome

Cause of anterior knee pain secondary to


trauma, imbalance of quadriceps strength, or
meniscal tear
Pain in front of knee or underneath patella
Particularly bad when walking up or down
stairs
Worse just after starting to walk after having
been seated for a prolonged period
It improves after walking

Crepitus, joint locking, and instability


X-rays: normal
Most cases respond to physical therapy
and strength training with cycling
Knee
K
b
braces d
dontt help
h l
Theres nothing to fix surgically

MTB S2CK

p. 178

Plantar Fasciitis

MTB S2CK

p. 178

Plantar Fasciitis

Very severe pain in bottom of foot near


calcaneus where fascia inserts
Pain worst in the morning and improves
with walking a few steps
Point tenderness @ the fascia inserts at
the calcaneus

Commons.Wikimedia.org. used with permission

MTB S2CK

p. 178

Plantar Fasciitis

Osteoporosis

Treatment consists of stretching exercises,


arch supports, and NSAIDs
Steroid injection is performed if these dont
solve problem
Surgical release of plantar fascia is rarely
necessary

Look for an older person, more often a


woman, with vertebral fractures leading
to loss of height or wrist fracture
Asymptomatic, fractures are found on
routine screening with bone
densitometry, which is recommended
for all women > 65

X-ray of the foot is not useful in plantar


fasciitis. There is no correlation with the
presence of heel spurs.
MTB S2CK

p. 178

Osteoporosis gives
spontaneous fractures of
weight-bearing bones.
MTB S2CK

p. 198

13

Osteoporosis/Diagnostic Tests

Osteoporosis/Treatment

Most accurate test is bone


densitometry (DEXA) scanning
Osteopenia: bone density (T-score) is
between 1 and 2.5 standard deviations
below normal
Osteoporosis: T-score > 2.5 standard
deviations < normal

1. Vitamin D and calcium are the best


initial therapy
2. Bisphosphonates (alendronate,
risendronate, ibandronate)
3 Estrogen replacement
3.
4. Raloxifene

MTB S2CK

p. 198

MTB S2CK

Osteoporosis/Treatment

Osteoporosis/Treatment

1. Teriparatide is an analogue of PTH that


stimulates new bone matrix formation
2. Used as a nasal spray, calcitonin
decreases vertebral fractures risk
Bisphosphonates are
very rarely associated
with osteonecrosis of
the jaw.

MTB S2CK

p. 198

When multiple treatment options are


presented, choose vitamin D, calcium,
and bisphosphonates.

Bisphosphonates that
have prolonged contact
with the esophagus can
cause esophagitis (pill
esophagitis).

p. 198

Teriparatide has caused


osteosarcoma in rats. It
has also caused
hypercalcemia.
MTB S2CK

p. 198

Rheumatoid Arthritis/Definition/Etiology

Rheumatoid Arthritis, Systemic


Lupus Erythematosus, &
p p
p Syndrome
y
Antiphospholipid

RA is an autoimmune disorder
predominantly of joints
More common in women

MTB S2CK

p. 178

14

Rheumatoid Arthritis/Definition/Etiology

Chronic synovitis leads to overgrowth,


or pannus formation, which damages all
the structures surrounding the joint
(bone, ligaments, tendons, and
g )
cartilage)
Morning stiffness of
multiple small, inflamed
joints is key to
diagnosis.
MTB S2CK

p. 178

Morning
Stiffness > 30 min

Rheumatoid
Nodules

C1 and C2 laxity - subluxation


Baker cyst
Pericarditis and pleural disease
Carpal tunnel syndrome

PIP Hands
MCP Hands

Bilateral
Symmetrical

Rheumatoid Arthritis

Vasculitis

Episcleritis

MTB S2CK

Rheumatoid Arthritis/Presentation

Rheumatoid Arthritis/Presentation

Lung nodules
and effusions

p. 178

Rheumatoid Arthritis/Presentation
Boutonniere and
swan neck are
classic deformities of the
hands

DIP is spared in RA. DIP


involvement happens in
DJD.

MTB S2CK

MTB S2CK

p. 179

Rheumatoid Arthritis

p. 179

Source: Nirav Thakur, MD.

Rheumatoid Arthritis/Diagnostic Tests

Rheumatoid factor (RF) in 70% to 80%


RF is rather nonspecific
Anti-cyclic citrulinated peptide (antiCCP) is > 80% sensitive and > 95%
specific

Richard Usatine, M.D. Used with permission.

MTB S2CK

p. 179

15

Rheumatoid Arthritis/Diagnostic Tests

Rheumatoid Arthritis/Diagnostic Tests

Elevated ESR or C-reactive protein


Anemia: normocytic
Arthrocentesis on initial presentation
excludes crystal disease or infection if
diagnosis isnt clear
Modest elevation in lymphocytes

Felty syndrome:
RA
Splenomegaly
Neutropenia
Caplan syndrome:
RA
Pneumoconiosis
Lung nodules

Sicca syndrome: dry eyes,


mouth, and other mucous
membranes
MTB S2CK

p. 179

MTB S2CK

p. 180

Rheumatoid Arthritis/Diagnostic Tests

Rheumatoid Arthritis/Treatment

The most important issue in RA is stopping


the progression of the disease. Any patient
with erosive disease or X-ray abnormalities
needs at least methotrexate to slow disease
progression.

Disease-Modifying Antirheumatic Drugs


(DMARD)
Neither NSAIDs nor steroids stop RA
from progressing
Any patient with erosive RA needs
DMARD as part of initial therapy

The MCC of death in RA is


coronary artery disease.

MTB S2CK

p. 180

MTB S2CK

p. 180

Rheumatoid Arthritis/Treatment
Patient with long-standing RA is to have coronary
bypass surgery.
Which is most important prior to surgery?

Erosive disease means:


Joint space narrowing
Physical deformity of joints
X-ray
y abnormalities

a. Cervical spine X-ray


Already diagnosed to have RA
b Rheumatoid factor
b.
Doesnt change outcomes
c. Extra dose of methotrexate
Nonspecific test
d. ESR
e. Pneumococcal vaccination Has nothing to do with surgery

MTB S2CK

p. 180

MTB S2CK

p. 180

16

Rheumatoid Arthritis/Treatment

Rheumatoid Arthritis/Treatment

Methotrexate
Best initial DMARD
Adverse effects are:

Tumor Necrosis Factor (TNF)


Inhibitors (infliximab, adalimumab,
etanercept)
TNF inhibitors are first line as DMARDS
Toxicity of anti
anti-TNF
TNF drugs:

Liver toxicity
Bone marrow suppression
Pulmonary toxicity

MTB S2CK

p. 181

Reactivation of TB
Infection

MTB S2CK

p. 181

Rheumatoid Arthritis/Treatment

Rheumatoid Arthritis/Treatment

Rituximab
RA as a DMARD by removing CD20+
lymphocytes from circulation
Excellent long-term
+/- methotrexate

Hydroxychloroquine
Rare as monotherapy as a DMARD
More often used in combination with
methotrexate as a DMARD
Toxic to retina

Hydroxychloroquine
leads to retinal toxicity.
Do a dilated eye exam.
MTB S2CK

p. 181

Rheumatoid Arthritis/Treatment

Symptomatic Control of RA
NSAIDs are the best initial therapy for
the pain of RA
Steroids also work in a matter of hours
to control the pain of RA secondary to
inflammation
Steroids for 2 purposes

MTB S2CK

Rheumatoid Arthritis/Treatment

Use TNF inhibitors as a DMARD with


methotrexate after methotrexate fails
Adverse effects are mandatory for you
to know
Steroids dont prevent
the progression of RA.

NSAIDS arent working


Bridge
MTB S2CK

p. 181

p. 181

MTB S2CK

p. 181 182

17

Adverse Effects of RA Medications


Drug

Adverse effect

Anti TNF

Reactivation of tuberculosis

Hydroxychloroquine

Ocular

Sulfasalazine

Rash, hemolysis
y

Rituximab

Infection

Gold salts

Nephrotic syndrome

Methotrexate

Liver, lung, marrow

MTB S2CK

p. 182

Juvenile Rheumatoid Arthritis or


Adult Still Disease

Juvenile Rheumatoid Arthritis or


Adult Still Disease

Definition/Etiology
Juvenile rheumatoid arthritis (JRA) is
very difficult to define and theres no
known etiology

MTB S2CK

Juvenile Rheumatoid Arthritis or


Adult Still Disease/Presentation

Presentation
The most important feature of JRA is
the presence of high, spiking fever
(often > 104
104F)
F) in a young person that
has no clearly identified etiology, but is
associated with a rash

MTB S2CK

p. 182

Juvenile Rheumatoid Arthritis or


Adult Still Disease

Laboratory Abnormalities
No clear diagnostic test; anemia and
leukocytosis often present
ANA is normal
Ferritin level markedly elevated

p. 182

Features of JRA rash:


Often only with fever spikes
Salmon colored
On chest and abdomen
Other features of JRA:
Splenomegaly
Pericardial effusion
Mild joint symptoms

MTB S2CK

p. 182

Systemic Lupus Erythematosus

Definition/Etiology
Autoimmune disorder
Inflammation diffusely through body

Treatment
Half of cases improve with aspirin or NSAIDs
If theres no response then use steroids
MTB S2CK

p. 183

MTB S2CK

p. 183

18

Systemic Lupus Erythematosus

Systemic Lupus Erythematosus

Presentation
Diagnosis of SLE is based on 4 of 11 known
manifestations of disease
Four skin-related manifestations:
1. Malar rash
2. Discoid rash
3. Photosensitivity
4. Oral ulcers

MTB S2CK

Alopecia is common in
SLE, but isnt one of
the official diagnostic
criteria.
Richard Usatine, M.D. Used with permission.

p. 183

Systemic Lupus Erythematosus

Systemic Lupus Erythematosus

Presentation
Joint: arthritis is present in 90%
X-ray is normal
Serositis: inflammation of pleura and
pericardium
i di
chest
h t pain
i

Presentation
Renal: any degree of abnormality can
occur from mild proteinuria to end-stage
renal disease requiring dialysis
Most common g
glomerulonephritis
p
is
membranous
Red cell casts and hematuria occur

Neurologic: symptoms include


psychosis, seizures, or stroke from
vasculitis
MTB S2CK

p. 183

Systemic Lupus Erythematosus

MTB S2CK

Systemic Lupus Erythematosus

Ocular findings arent part of


formal diagnostic criteria:
Photophobia
Retinal lesions ((cotton wool
spots)
Blindness

Pneumonia, alveolar hemorrhage,


and restrictive lung disease happen
in SLE, but arent criteria for
diagnosis
g
of the disease.

MTB S2CK

p. 183

p. 183

MTB S2CK

p. 184

19

Systemic Lupus Erythematosus

Systemic Lupus Erythematosus

Presentation
Hematologic: hemolytic anemia is part
of diagnostic criteria, but anemia of
chronic disease is more commonly
found
Lymphopenia, leukopenia, and
thrombocytopenia are also seen

Presentation
Immunologic (laboratory)
abnormalities - criteria include positive
ANA, or any one of the following:

MTB S2CK

p. 184

MTB S2CK

Systemic Lupus Erythematosus

p. 184

Systemic Lupus Erythematosus

Additional findings:
Mesenteric vasculitis
Raynaud phenomenon
Antiphospholipid syndromes

MTB S2CK

Anti-double-stranded DNA
Anti-Sm
False positive test for syphilis
Positive LE cell preparation

p. 184

Diagnostic tests
ANA: found in 95% to 99% of cases
Anti-double-stranded DNA (60%) and
anti-Sm (30%):
Found
Fo nd onl
only in SLE
Extremely specific for SLE

MTB S2CK

p. 184

Systemic Lupus Erythematosus

Diagnostic tests
Decreased complement levels:
Correlate with disease activity
Drop further with acute disease exacerbations

Anti-SSA and anti-SSB: found in 10% to 20% of


cases
Add little to diagnosis
Tests most often found in Sjgren syndrome
(65% of cases)
Dont treat
asymptomatic ANA
MTB S2CK

p. 184

34-year-old woman with history of SLE is admitted with


pneumonia and confusion. As youre wrestling with the
decision over a bolus of high-dose steroids in a person
with an infection, you need to determine if this is a flare of
lupus or simply an infection with sepsis causing
confusion.
Which of the following will help you the most?
a. Rise in anti-Sm Level doesnt change in acute disease
b. Rise in ANA Level doesnt change in acute disease
c. Decrease in complement and rise in anti-DS DNA
d. MRI of the brain MRI doesnt diagnose regardless
e. Response to steroids Not diagnostic
MTB S2CK

p. 184 185

20

Systemic Lupus Erythematosus

Treatment
Acute lupus flare treated with high-dose
boluses of steroids
Hydroxychloroquine can control mildly
chronic disease
Lupus nephritis may need steroids either
alone or in combination with
cyclophosphamide or mycophenolate
Only way to determine the severity of lupus
nephritis is kidney biopsy
Belimumab decreases symptoms
MTB S2CK

p. 185

Systemic Lupus Erythematosus

Treatment
Urinalysis is insufficient to determine severity
of lupus nephritis
Biopsy is the only way to diagnose simple
glomerulosclerosis or scarring of the kidney,
which will not respond to therapy
Young patients most commonly die of
infection. In older patients, accelerated
atherosclerosis makes MI the MCC of
death.
MTB S2CK

p. 185

Antiphospholipid Syndrome/Definition

Antiphospholipid Syndrome

Idiopathic disorder with IgG or IgM


antibodies made against negatively
charged phospholipids
The 2 main types are:

Presentation/Diagnostic Tests
Thromboses of both arteries and veins as
well as recurrent spontaneous abortions
Elevation of aPTT with a normal prothrombin
time (PT) and normal INR

Lupus anticoagulant
Anticardiolipin antibodies

APL = clotting + elevated aPTT and


normal PT

MTB S2CK

p. 185

MTB S2CK

p. 185

Antiphospholipid Syndrome

Antiphospholipid Syndrome

Presentation/Diagnostic Tests
False positive VDRL or RPR with a
normal FTA
Anticardiolipin antibodies - spontaneous
abortion
Lupus anticoagulant - elevated aPTT
Best initial test is mixing study

Diagnostic Tests
If the elevation in aPTT is from a clotting factor
deficiency then aPTT will come down to normal
If the APL syndrome antibody is present in
plasma then aPTT remains elevated
Most
M t specific
ifi ttestt for
f lupus
l
anticoagulant
ti
l t iis
Russell viper venom test (RVVT)
RVVT is prolonged with APL antibodies and
doesnt correct on mixing with normal plasma

MTB S2CK

p. 185 186

MTB S2CK

p. 186

21

Antiphospholipid Syndrome/Treatment

Antiphospholipid Syndrome/Treatment

Thromboses (DVT or PE) treated with


heparin and warfarin as you would any
other form of thrombosis with an INR of
2 to 3
Lifelong vs. 6 months primary
occurrence
Recurrent thrombotic episodes are
treated lifelong

USMLE S2 CK questions have to be


unequivocally clear. If an area is
controversial, USMLE will avoid it and
ask only what is clear
clear. The exam will not
trick you.

MTB S2CK

p. 186

MTB S2CK

p. 186

Antiphospholipid Syndrome/Treatment

Warfarin or steroids are wrong answers


for preventing spontaneous abortion.
Steroids arent effective.

Scleroderma, Polymyositis, &


Dermatomyositis

Warfarin is contraindicated in pregnancy


secondary to teratogenicity.

MTB S2CK

p. 186

Scleroderma (Systemic Sclerosis)

Scleroderma (Systemic Sclerosis)

Limited scleroderma is also known as


CREST syndrome:
Calcinosis
Raynaud
Esophageal
E
h
ld
dysmotility
tilit
Sclerodactyly
Telangiectasia

What is the most likely diagnosis?


Look for a young (20s to 40s) woman (3
times more likely than men) with fibrosis
of the skin and internal organs (e.g.,
lung kidney
lung,
kidney, and GI tract)

MTB S2CK

p. 186

MTB S2CK

p. 187

22

Scleroderma/Presentation

Raynaud Syndrome

Raynaud syndrome: increased vascular


reactivity of fingers beginning with pain and
pallor (white) or cyanosis (blue) followed by
reactive hyperemia (red)
Skin manifestations: fibrosis of hands, face,
neck,
k and
d extremities;
t
iti
telangiectasia
t l
i t i and
d
abnormalities of pigmentation occur

MTB S2CK

p. 187

Scleroderma

Scleroderma (Systemic Sclerosis)

MTB S2CK

p. 187

Scleroderma/Presentation

Scleroderma/Diagnostic Tests

GI: esophageal dysmotility with GERD,


large-mouthed diverticuli of small and large
bowel
Renal: sudden hypertensive crisis
Lung: fibrosis leading to restrictive lung
disease and pulmonary hypertension
Cardiac: myocardial fibrosis, pericarditis,
and heart block; lung disease gives right
ventricular hypertrophy

ANA: positive in 85% to 90%, but


nonspecific
ESR: usually normal
SCL-70: most specific test (antitopoisomerase)
Anticentromere: present in half of those
with CREST syndrome

MTB S2CK

p. 187

Anticentromere antibodies are extremely


specific for CREST syndrome.
MTB S2CK

p. 187

23

Scleroderma/Treatment

Polymyositis and Dermatomyositis

Presentation
Proximal muscle weakness
They dont affect facial or ocular
muscles as occurs in myasthenia gravis
Dysphagia

Penicillamine is ineffective
Renal crisis: ACE inhibitors
Esophageal dysmotility: PPIs for GERD
Raynaud: calcium-channel blockers
Pulmonary fibrosis: cyclophosphamide
improves dyspnea and PFTs
Pulmonary hypertension is treated like
primary pulmonary hypertension with
bosentan or ambrisentan (endothelin
antagonist) or Sildenafil
MTB S2CK

p. 187

MTB S2CK

Polymyositis and Dermatomyositis

p. 188

Polymyositis and Dermatomyositis

Presentation
Dermatomyositis presents with:
Malar involvement
Shawl sign: erythema of face, neck,
shoulders,, upper
pp chest,, and back
Heliotrope rash: edema and purplish
discoloration of eyelids
Gottron papules: scaly patches over the
back of hands, particularly PIP and MCP
joints
MTB S2CK

Commons.Wikimedia.org. used with permission

p. 188

Polymyositis and Dermatomyositis

Polymyositis and Dermatomyositis

Commons.Wikimedia.org. used with permission


Commons.Wikimedia.org. used with permission

24

Polymyositis and Dermatomyositis/


Presentation

Dermatomyositis is associated with


cancer in 25% of cases.
Common sites are:
Ovary
y
Lung
GI
Lymphoma

MTB S2CK

p. 188

Polymyositis and Dermatomyositis

Diagnostic Tests
Best initial test is CPK and aldolase
Most accurate test is muscle biopsy
ANA is frequently positive
MRI
Electromyography

MTB S2CK

p. 188

Polymyositis and Dermatomyositis

Treatment
Steroids are usually sufficient
When patient is unresponsive or intolerant of
steroids, use:
Methotrexate
Azathioprine
IVIG
Mycophenolate
Hydroxychloroquine helps skin lesions
MTB S2CK

Sjgren Syndrome,
Vasculitis, & Seronegative
Spondyloarthropathies

p. 188 189

Sjgren Syndrome/Definition/Etiology

Sjgren Syndrome/Presentation

Idiopathic autoimmune disorder secondary to


antibodies predominantly against lacrimal
and salivary glands
90% of those affected are women
Sjgren syndrome is associated with:

Sjgren presents with dryness of


mouth and eyes
Keratoconjunctivitis sicca
Need to constantly drink water
Dysphagia
D
h i
Dental caries

RA
SLE
Primary biliary cirrhosis
Polymyositis
Hashimoto thyroiditis
MTB S2CK

p. 189

MTB S2CK

p. 189

25

Sjgren Syndrome/Presentation

Sjgren Syndrome/Diagnostic Tests

Less common manifestations are:


Vasculitis
Loss of vaginal secretions
Lung disease leads to dyspareunia.
Pancreatitis
Renal tubular acidosis (20%)

Best initial test is called a Schirmer test


Most accurate test is a lip or parotid
gland biopsy
Reveal lymphoid infiltration in salivary
glands

Lymphoma is the most dangerous


complication of Sjgren
MTB S2CK

p. 189

MTB S2CK

p. 189 190

Sjgren Syndrome/Diagnostic Tests

Sjgren Syndrome/Treatment

Best initial test on blood: SS-A and


SS-B
These are also called Ro and La and
each are present in about 65% of
patients

Best initial therapy is to water the mouth


Use frequent sips of water, sugar-free gum, and
fluoride treatments
Artificial tears to avoid corneal ulcers
Pilocarpine and cevimeline increase
acetylcholine,
t l h li
th main
the
i stimulant
ti l t to
t produce
d
saliva
Cevimeline increases rate of saliva production
No cure
Evaluate for lymphoma

SLE is associated with SS-A and SS-B in


10% to 20% of cases
Other abnormalities that are present, but
are nonspecific: ANA, RF, anemia,
leukopenia, and eosinophilia
MTB S2CK

p. 190

MTB S2CK

p. 190

Vasculitis

Polyarteritis Nodosa/Definition

Etiology unknown
Symptoms develop over weeks to
months
All vasculitides give:

Polyarteritis nodosa (PAN) is a disease


of small- and medium-sized arteries
leading to a diffuse vasculitis that
inexplicably spares the lungs
Chronic hepatitis B and C are
associated with PAN

Fever
Fe er
Malaise/fatigue
Weight loss
Arthralgia/myalgia
MTB S2CK

p. 190

MTB S2CK

p. 190

26

Polyarteritis Nodosa/Presentation

Polyarteritis Nodosa/Presentation

Common Features of PAN


Renal: glomerulonephritis without a
biopsy cant be diagnosed

Gastrointestinal: abdominal pain is


worsened by eating from vasculitis of
mesenteric vessels

UA isnt enough to confirm its PAN

Neurological: any large peripheral


nerve can be involved
Peroneal neuropathy leading to foot drop
Look for a stroke in a young person
MTB S2CK

p. 190 191

Bleeding also occurs


Nausea and vomiting are common

Skin: lower extremity ulcers are most


common; livedo reticularis, purpura,
nodules, and rarely gangrene occur
Lung is spared in PAN
MTB S2CK

p. 191

Polyarteritis Nodosa/Presentation

Polyarteritis Nodosa/Diagnostic Tests

Mononeuritis Multiplex
Mononeuritis multiplex is multiple
peripheral neuropathies of nerves
large enough to have a name

Most accurate test is a biopsy of a


symptomatic site
Angiography of renal, mesenteric, or
hepatic artery shows abnormal dilation or
beading.
P-ANCA is present in < 20%
Test all PAN patients for
hepatitis B and C.

MTB S2CK

p. 191

MTB S2CK

p. 191

Polyarteritis Nodosa/Treatment

Polymyalgia Rheumatica

Prednisone and cyclophosphamide


Treat hepatitis when found

Polymyalgia rheumatica (PMR) occurs in


those over age 50 with:
Pain and stiffness in shoulder and pelvic girdle
muscles
Difficulty combing hair and rising from chair
Elevated ESR
Normochromic, normocytic anemia

No Lab Findings
CPK and aldolase are normal
Steroids even at low doses great
response
MTB S2CK

p. 191

MTB S2CK

p. 191

27

Giant Cell (Temporal) Arteritis

Giant Cell (Temporal) Arteritis

The difference is the presence of:


Visual symptoms
Jaw claudication (pain in jaw when
chewing)
Scalp tenderness
Headache
Symptoms in other arteries such as
decreased arm pulses, bruits near the
clavicles, or aortic regurgitation

ESR and C-reactive protein are elevated


Most accurate test is a biopsy of affected
artery (e.g., temporal artery)
Treat with prednisone
Starting high-dose prednisone quickly is
more important than waiting for biopsy

MTB S2CK

p. 191 192

Blindness is irreversible.

MTB S2CK

p. 192

Wegener Granulomatosis

Wegener Granulomatosis/Diagnostic Tests

Presents with:

Best initial test is antineutrophil cytoplasmic


antibody (ANCA)
Most accurate test is a biopsy
Cytoplasmic antibodies are also called CANCA.

Sinusitis
Otitis media
Mastoiditis
Oral and gingival involvement

Wegener is also associated with skin, joint,


and eye lesions
Look for combination of upper and lower
respiratory tract findings in association
with renal insufficiency
MTB S2CK

p. 192

Wegener Granulomatosis/Diagnostic Tests

When asked about the best test for


Wegener, lung biopsy is better than renal
biopsy with sinus biopsy being the least
accurate
When all 3 are in the choices choose lung
biopsy

MTB S2CK

p. 192

C-ANCA = anti-proteinase-3 antibodies


P-ANCA = anti-myeloperoxidase antibodies
Wegener: C-ANCA
Churg-Strauss and
microscopic polyangiitis:
P-ANCA
MTB S2CK

p. 192

Wegener Granulomatosis/Treatment

Treat with prednisone and


cyclophosphamide
The clue to answering the most likely
diagnosis question is unresolving
diagnosis
pneumonia not better with antibiotics.
You will not first think of Wegener when
presented with the case.

MTB S2CK

p. 192

28

Churg Strauss Syndrome

Henoch Schnlein Purpura

Pulmonary-renal syndrome, ChurgStrauss also has:


Asthma
Eosinophilia
Biopsy
Bi
iis the
th mostt accurate
t test
t t
Treat with prednisone and
cyclophosphamide

Vasculitis more frequently seen in


children, Henoch-Schnlein purpura
(HSP) is characterized by involvement
of:

MTB S2CK

p. 193

Henoch Schnlein Purpura

HSP is most often a clinical


diagnosis; however, biopsy is the
most accurate test
Serum IgA levels are the
wrong answer. They
Th are
unreliable when testing
for Henoch-Schnlein
purpura.

MTB S2CK

p. 193

GI tract: p
pain,, bleeding
g
Skin: purpura
Joint: arthralgia
Renal: hematuria

MTB S2CK

p. 193

Henoch Schnlein Purpura


When the case describes
leukocytoclastic vasculitis
on biopsy, the answer is
Henoch-Schnlein
purpura.
Leukoplastic reactions
are painless, palpable
purpura of buttocks and
legs
MTB S2CK

p. 193

Source: Shreya Patel and Nishith Patel

Henoch Schnlein Purpura/Treatment

Cryoglobulinemia

Most cases resolve spontaneously


Steroids if there are severe extrarenal
mainfestations associated with renal
failure

Most commonly associated with chronic


hepatitis C
+/- endocarditis and/or Sjgren syndrome
Dont confuse cryoglobulins with cold
agglutinins

Both are IgM antibodies


Neither respond to steroids

MTB S2CK

p. 193

MTB S2CK

p. 194

29

Differences between Cryoglobulins and Cold Agglutinins


Cryoglobulins

Cold agglutinin

Associated with

Hepatitis C

EBV, Mycoplasma,
Lymphoma

Manifestations

Joint pain
Glomerulonephritis
Purpuric skin lesions
Neuropathy

Hemolysis

Interferon,
Ribavirin, and
boceprevir (or
telaprevir)

Stay warm
Rituximab,
cyclophosphamide,
cyclosporine

Treatment

MTB S2CK

p. 194

Cryoglobulinemia
Lab tests show a positive rheumatoid factor and
cold precipitable immune complexes
Steroids NOT effective
Treat the underlying cause, especially hepatitis C,
with interferon and ribavirin
Despite the rarity of the condition, the USMLE loves
cryoglobulinemia questions.
SLE
decreased C3 or
3 letters (SLE) = C3
Hep C
decreased C4 or
4 letters (Hep C) = C4
.
MTB S2CK

p. 194

Behet Syndrome

Behet Syndrome

The most common Behet questions


are:

Asian or Middle Eastern person with


painful oral and genital ulcers +/erythema
Also with:

What is the most likely diagnosis?


What is pathergy?
pathergy ?
Pathergy: sterile skin
pustules from minor
trauma (e.g., needle stick)
MTB S2CK

p. 194

Ocular lesions leading to uveitis and


blindness
Arthritis
CNS lesions mimicking multiple
sclerosis
MTB S2CK

p. 194

Behet Syndrome/Treatment

Seronegative Spondyloarthropathies

Corticosteroids
To wean patients off of steroids, use:

The 3 types of seronegative


spondyloarthropathies are:

Azathioprine
Cyclophosphamide
Colchicine
C l hi i
Thalidomide

MTB S2CK

p. 195

Ankylosing spondylitis
Psoriatic arthritis
Reactive arthritis (Reiter syndrome)

MTB S2CK

p. 195

30

Seronegative Spondyloarthropathies

Seronegative Spondyloarthropathies

Men < 40 years:


Involvement of spine and large joints
Negative rheumatoid factor (hence the
name seronegative)
Enthesopathy (inflammation where
tendons and ligaments attach to bones)
Uveitis
HLA-B27

Corticosteroids arent a good treatment


for seronegative spondyloarthropathy

MTB S2CK

p. 195

D
Despite
it th
the association
i ti with
ith HLA
HLA-B27,
B27
this is never the best initial or most
accurate test for seronegative
spondyloarthropathies.
MTB S2CK

p. 195

Ankylosing Spondylitis/Diagnosis

Ankylosing Spondylitis/Diagnosis

Young man with low backache and


stiffness of his back has pain that radiates
to buttocks with flattening of the normal
lumbar curvature and decreased chest
expansion
Eventually
E
t ll th
the spine
i will
ill nott expand
d iin any
direction
Enthesopathy occurs at the Achilles tendon

Other Findings of Ankylosing


Spondylitis
Transient peripheral arthritis of knees,
hips, and shoulders (50%)
Cardiac: atrioventricular block in 3%
to 5%; aortic insufficiency
Uveitis
Bamboo spine is a late
finding with fusion of
vertebral joints.

Look for back pain


worsened by rest and
relieved by activity
MTB S2CK

p. 195

Ankylosing Spondylitis/Diagnostic Tests

MTB S2CK

p. 195

Ankylosing Spondylitis/Diagnostic Tests

Best initial test is an X-ray of sacroiliac


(SI) joint
Most accurate test is an MRI
MRI detects abnormalities years before
the X-ray
X ray becomes abnormal
ESR is elevated in 85%

MTB S2CK

p. 196

MTB S2CK

p. 196

31

Ankylosing Spondylitis/Diagnostic Tests

Ankylosing Spondylitis

HLA B27 is not a confirmatory diagnostic test


since 8% of general population is positive
Treatment
Exercise p
program
g
and NSAIDs are best
initial treatment
If NSAIDs are insufficient, use anti-TNF
drugs (e.g., etanercept, adalimumab, or
infliximab)
MTB S2CK

p. 196

Source: Conrad Fischer, MD.

Psoriatic Arthritis

MTB S2CK

p. 196 197

Psoriatic Arthritis

Psoriatic arthritis 80% will have


preceding psoriasis
Besides SI joint involvement,
characteristic findings are:
Sausage
Sausage digits from enthesopathy
Nail pitting

MTB S2CK

p. 197

MTB S2CK

p. 197

Psoriatic Arthritis/Diagnostic Tests

Psoriatic Arthritis/Treatment

ESR is elevated nonspecific


Best initial test is an X-ray of the joint
showing a pencil in a cup
deformity

NSAIDs are best initial therapy


Methotrexate used when question
describes severe disease or no
response to NSAIDs
Anti-TNF
Anti TNF agents are the answer when
methotrexate doesnt control disease
Steroids are a wrong choice

MTB S2CK

p. 197

MTB S2CK

p. 197

32

Reactive Arthritis (Reiter Syndrome)

Reactive Arthritis/Diagnosis

Reactive arthritis occurs secondary to:


Inflammatory bowel disease (equal
sex incidence)
Sexually transmitted infection (far
greater in men)
GI infection (Yersinia, Salmonella,
Campylobacter)

Look for triad of:


Joint pain
Ocular findings (uveitis,
conjunctivitis)
Genital
G it l abnormalities
b
liti ((urethritis,
th iti
balanitis)
Keratoderma

MTB S2CK

p. 197

MTB S2CK

Reactive Arthritis/Diagnosis

p. 198

blennorhagicum is a skin
lesion unique to reactive
arthritis that looks like
pustular psoriasis.

Reactive Arthritis/Diagnostic Tests/Treatment

No specific test for reactive arthritis


Rule out septic joint
Treat underlying cause/use NSAIDs
Sulfasalazine > NSAIDS

Antibiotics dont reverse


reactive arthritis once joint
pain has started.
Source: commons.wikimedia.org. Used with permission

MTB S2CK

p. 198

Septic Arthritis

Septic Arthritis, Gonococcal


Arthritis, & Osteomyelitis

Definition
Septic arthritis is an infection of joint space
Etiology
Septic arthritis is relatively rare in an
undamaged joint
Risk of infection is directly proportional to
degree of joint damage

MTB S2CK

p. 199

33

Septic Arthritis

Etiology of Septic Arthritis

Etiology (contd)
Osteoarthritis (DJD) provides slight risk
RA has greater risk
Greatest risk is with prosthetic joint
Bacteremia can spread into joint space,
space
which is why endocarditis and injection
drug use causes septic arthritis

MTB S2CK

p. 199

MTB S2CK

Etiology

Frequency

Staphylococcus

40%

Streptococcus

30%

Gram negative
rods

20%

p. 199

Septic Arthritis/Presentation

Septic Arthritis/Diagnostic Tests

Joint is warm, red, and immobile often


with palpable effusion
Chills and fever happen because of
bacteremia

Best initial and most accurate test is


aspiration of the joint with a needle
(arthrocentesis); X-ray, CT, and MRI arent
useful and are the wrong answers
Joint fluid shows:
Leukocytosis: more than 50,000 to 100,000
cells, predominantly neutrophils
Gram stain: positive (50%) Gram-negative
bacilli; (75%) with Staphylococcus
Synovial fluid culture: 70% to 90% sensitive
Blood cultures: 50% sensitive

MTB S2CK

p. 199

MTB S2CK

p. 199

Septic Arthritis

Other Options for Treatment of Septic Arthritis

Treatment
Ceftriaxone and vancomycin are best
initial empiric therapy

Gram negative
bacilli

Gram positive cocci


(sensitive)

Gram positive
cocci (resistant)

Quinolones

Oxacillin

Linezolid

Aztreonam

Cefazolin

Daptomycin

Cefotaxime

Piperacillin with
Tazobactam

Ceftaraline

Piperacillin

Tigecycline

Aminoglycosides
MTB S2CK

p. 199

MTB S2CK

p. 200

34

Septic Arthritis/Treatment

Septic Arthritis

Adjust antibiotics according to culture


results.

Prosthetic Joint Infection


Infected prosthetic joint gives a warm, red,
immobile, and tender joint
Must do imaging

If Staphylococcus
p y
is sensitive,,
vancomycin is associated with a worse
outcome than betalactam antibiotic
(e.g., oxacillin or cefazolin). Switch
drugs if organism is sensitive.

MTB S2CK

p. 200

MTB S2CK

p. 200

Septic Arthritis

Septic Arthritis

Prosthetic Joint Infection (contd)


MRI difficult to perform with prosthetic
joints because they are made of metal
If there is lucency around implantation
of the joint on radiologic imaging or if
joint is physically loose, infection is
likely present at implantation site

Treatment of Infected Prosthetic Joint


Remove joint, treat with antibiotics for 6
to 8 weeks, and then replace joint

MTB S2CK

p. 200

The most common


organism for recently
placed artificial joints
is Staphylococcus
epidermidis.

MTB S2CK

p. 200

Gonococcal Arthritis (Gonorrhea)

Gonococcal Arthritis/Diagnostic Tests

The difference in presentation from septic


arthritis is:

Detecting gonorrhea is much more


difficult than detecting Staphylococcus,
Streptococcus, and Gram-negative
bacilli of septic arthritis

Polyarticular involvement
Tenosynovitis (inflammation of tendon
sheaths, making finger movement painful)
Petechial rash

Gonococcal arthritis is
more frequent during
menses.
MTB S2CK

p. 200

MTB S2CK

p. 200

35

Synovial Fluid Analysis for Infectious Arthritis


Test sensitivity

Septic arthritis

Gonococcal Arthritis/Diagnostic Tests


Gonococcal arthritis

Leukocytosis

> 50,000 100,000 cells/ L

30,000 50,000 cells/ L

Gram stain

50 70% sensitive

25% sensitive

C l
Culture

90% sensitive
ii

< 50% sensitive


ii

Blood cultures

50% sensitive

< 10% sensitive

In order to reach maximum sensitivity,


multiple diffuse sites must be cultured
for gonorrhea such as:
Pharynx
What tells you to
Rectum
Rectum
culture everywhere?
Urethra
Rash
Cervix
Tenosynovitis
Polyarticular
involvement

MTB S2CK

p. 201

MTB S2CK

p. 201

Gonococcal Arthritis/Treatment

Osteomyelitis/Definition/Etiology

Ceftriaxone, cefotaxime, or ceftizoxime is the


best empiric therapy for disseminated
gonorrhea
Fluoroquinolones are not the best initial
therapy

Osteomyelitis is an infection of bone


Staphylococcus aureus is MCC
Children get osteomyelitis through
hematogenous spread, but adults get it
from a contiguous (nearby) infection
Salmonella is the most
commonly identified
organism in patients with
sickle cell disease.

If recurrent gonorrhea infection is described,


test for terminal complement deficiencya
favorite subject of USMLE Step 2 CK.

MTB S2CK

p. 201

Osteomyelitis/Diagnostic Tests

MTB S2CK

p. 201

Osteomyelitis/Diagnostic Tests

Best initial test is an X-ray


Most accurate test is a biopsy
If X-ray is normal, the most appropriate next
step in management is MRI
CT scan isnt veryy useful

When is ESR the answer?


To follow the response to therapy
MTB S2CK

p. 202

Source: Eva M. Smietana

36

Osteomyelitis/Diagnostic Tests

When is culturing the drainage the answer?


Never. You cannot reliably distinguish
superficial colonization from whatever
organism is inside the bone causing the
bone infection.
Bone scan is the answer
only if you want to get an
MRI and its contraindicated
(pacemaker).
MTB S2CK

p. 202

Osteomyelitis/Treatment

Osteomyelitis takes weeks to progress


Must biopsy
MSSA
Oxacillin, cefazolin, nafcillin, or
ceftriaxone
MRSA
Vancomycin or linezolid

MTB S2CK

p. 202

Osteomyelitis/Treatment

Osteomyelitis/Treatment

Gram-negative bacilli such as E. coli are


treated with fluoroquinolones (e.g.,
ciprofloxacin)

Toxicity of Quinolones
Fluoroquinolones can cause Achilles
tendon rupture

Its essential to confirm the


sensitivity of the organism prior to
treating with ciprofloxacin

They are also contraindicated in


pregnancy and children because they
interfere with bone growth

Ciprofloxacin is the only oral therapy


for osteomyelitis, but should be used
only if the organism is confirmed as a
sensitive Gram-negative bacillus.
MTB S2CK

p. 202

MTB S2CK

p. 202

37

Surgery

Preoperative Evaluation,
Postoperative Evaluation, &
Vascular Surgery

Matthew Kinney, MD
Orthopedic Resident
University of California San Diego

Preoperative Evaluation

Preoperative Evaluation
Cardiovascular
Pulmonaryy
Renal

Objective: Identify factors that increase risk of


complications in perioperative period
Age
Cardiac history
#1 predictor of perioperative complications

Diabetes status
Risk equivalent to Coronary disease

History of
Pulmonary disease
Renal disease
Stroke
MTB S2CK

p. 379

Preoperative Evaluation/Cardiac

Preoperative Evaluation/Cardiac

Must obtain detailed cardiac history

Management
If age < 35 and no history of cardiac disease

Look for indicators of previous MI or CHF


Recent MI
Defer surgery for 6 months
Follow-up stress test to ensure adequate perfusion

Evidence of CHF (JVD, edema)


Patients with EF < 35% at increased risk
ACE-Is, B-blockers, spironolactone must be
optimized
Proven to decrease overall mortality!!!

MTB S2CK

p. 379

EKG

If history of cardiac disease (MI, CHF)


EKG
Stress testing
Ensure appropriate cardiac perfusion

Echocardiography
Monitor EF
Evaluate structural damage

MTB S2CK

p. 380

Preoperative Evaluation/Pulmonary

71-year old man undergoing femoro-popliteal


bypass for severe claudication of left leg which
causes unbearable pain with exercise. Past
medical/surgical history is significant for remote
appendectomy and insulin-dependent type 2 DM.
What preoperative testing is recommended?
a.Basic Metabolic Panel (BMP) only High cardiac
risk
b.BMP + EKG
c. BMP + EKG + PFTs
Must get stress test/ECHO
d.BMP + EKG + Exercise Stress Test Cannot exercise
e.BMP + EKG + Thallium Stress Test

Must evaluate for history of lung disease (including


smoking)
PFTs required for all patients with known lung
disease
Vital capacity - most important predictor of
perioperative complications

MTB S2CK

Preoperative Evaluation/Pulmonary

p. 380

Preoperative Evaluation/Pulmonary
Must evaluate for history of lung disease (including
smoking)
PFTs required for all patients with known lung
disease
Vital capacity - most important predictor of
perioperative complications

Smokers
PFTs
Smoking cessation for 6-8 weeks preoperatively
Nicotine patch acceptable

MTB S2CK

p. 380

Preoperative Evaluation/Renal
Renal disease increases surgical risk
Intravascular fluid losses occur during surgery
Results in hypoperfusion of kidneys
Physiologic response to decreased intravascular
volume is activation of Renin-Angiotensin system
Constricts renal vasculature

Postoperative Evaluation
Fever Assessment
Complications

Renal hypoperfusion correlates with increased


mortality
Management
Aggressive IV hydration beginning 24 hours pre-op
Dialysis patients must be dialyzed 24 hours pre-op
MTB S2CK

p. 380

Post Operative Fever


WIND

Post Op Complications/Confusion

WATER WALKING
POD

1-2

3-5

Atelectasis #1
Pneumonia
CXR
-Sputum Culture
(if pneumonia
suspected)
-Incentive spirometry
-Antibiotics
Vanc. + Pip/Tazo

5-6

WEIRD

8-15

DVT
Thrombophlebitis
(IV site infection)
Doppler US
Anti-coagulation
Heparin
p
Warfarin
Replace IVs

UTI
Urinalysis
Nitrite +
Leukocyte Esterase +
Antibiotics
MTB S2CK

WOUND

Drug Reaction
Deep Abscess
D/C likely
medication
CT Scan
Drainage
of Abscess

Incision-site
Infection Cellulitis
Physical Exam
Erythema
Pus
Swelling
Abscess
Incision/Drainage
Antibiotics

p. 400

Adult Respiratory Distress Syndrome

Confused Patient
Obtain ABG, CXR, CBC
Evidence of Infection
Abnormal CBC

Evidence of Hypoxemia
Abnormal ABG

Changes on CXR?
Yes

Atelectasis vs.
Pneumonia
Incentive
spirometry
Antibiotics
MTB S2CK

No

Culture Likely Sources


Blood (Bacteremia)
Urine (UTI)

Consider PE
Spiral CT

Treat with
empiric
Antibiotics

p. 401

Adult Respiratory Distress Syndrome

Etiology
Endothelial damage allows fluid to fill alveoli
Prevents O2/CO2 exchange, acts as shunt

Signs/Symptoms
HR, RR
Labored breathing (accessory muscle use)
Fever may be present

Diagnosis
ABG: pO2, pCO2
CXR: Bilateral pulmonary infiltrates
MS-4 USU Teaching File, Uniformed Services University

MTB S2CK

p. 401

Adult Respiratory Distress Syndrome

Pulmonary Embolism

Treatment
Mechanical ventilation

Etiology

Maximize Positive End-Expiratory


Pressure (PEEP)

Passage of a
venous blood clot to
lungs
Origin: Deep leg
vein > 90%

Clot lodges in lung


vasculature
Prevents O2/CO2
exchange

MTB S2CK

Source: nlm.nih.gov

p. 401

Pulmonary Embolism

Risk Factors
Stasis: Immobility (post-surgical, travel, etc.),
obesity
Endothelial damage: Surgery, trauma
Hypercoagulability
yp
g
y: Oral contraceptive
p
pills (OCP), malignancy, genetic disorder
Signs/Symptoms
HR, RR, Temp
Pleuritic chest pain
MTB S2CK

62-year-old woman with no significant PMH


undergoes right total hip replacement 3 days ago.
Recovery is uncomplicated until 30 minutes ago, she
reported moderate SOB and chest pain with deep
inspiration.
Whats the next step in evaluating this patient?
a.
b.
c.
d.
e.

EKG only Insensitive for pulmonary embolism


EKG + V/Q Scan No reported contrast allergy
EKG + Spiral CT scan
EKG + D-Dimer Sensitive, but not specific
EKG + Heparin Injection Must diagnose PE first

p. 401

Pulmonary Embolism

Pulmonary Embolism

Diagnosis
ABG
pO2, pCO2

EKG

S1

Nonspecific ST-segment and T wave changes


Most common

S1-Q3-T3

Q3

MTB S2CK

T3

p. 401

Pulmonary Embolism

Pulmonary Embolism

Diagnosis
ABG
pO2, pCO2

EKG
Nonspecific ST-segment and T wave changes
Most common

S1-Q3-T3
Infrequent during acute PE
Can be found in massive acute PE and cor pulmonale

Spiral CT
Source: James Heilman, MD, commons.wikimedia.org

MTB S2CK

p. 401

Pulmonary Embolism

Pulmonary Embolism

Diagnosis
ABG

Treatment
Respiratory support
Anticoagulation

pO2, pCO2

EKG
Nonspecific ST-segment and T wave changes
Most common

S1-Q3-T3
Infrequent during acute PE
Can be found in massive acute PE and cor pulmonale

Heparin acutely, bridge to warfarin longterm


IVC filter (if anticoagulation
contraindicated)

Spiral CT
Consider V/Q scan (if IV contrast allergy)
MTB S2CK

p. 401

MTB S2CK

Vascular
Abdominal Aortic Aneurysm
Aortic Dissection
Claudication

p. 401

69-year-old male with 50 pack-year smoking history is


brought to the ER by his wife who reports he seems
confused. He feels weak and has pain in middle of
abdomen. He is a pale, elderly male in moderate
distress. BP of 84/55, pulse 120. Palpable, pulsatile
mass in patients abdomen.
Whats the most likely diagnosis?
a.
b.
c.
d.

Ruptured peptic ulcer Would expect peritoneal signs


Hemorrhagic gastritis Would expect hematemesis
Hemorrhagic pancreatitis Would expect flank bruising
Ruptured abdominal aortic aneurysm

MTB S2CK - p. 399

Abdominal Aortic Aneurysm

Abdominal Aortic Aneurysm

Etiology
Weakening of aortic
wall secondary to
atherosclerosis

Risk Factors
Male > Female
Age
Hypertension, Hyperlipidemia
Smoking
Si
Signs/Symptoms
/S
t
Frequently asymptomatic
May report pulsatile abdominal mass

Aortic diameter
expands > 1.5x normal
Involves all layers of
vessel wall (True
Aneurysm)
90% arise from infrarenal aorta

MTB S2CK

p. 399

Source: csm.ornl.gov

MTB S2CK

p. 399

Abdominal Aortic Aneurysm


Diagnosis
CT/MRI
Provides determination
of level and
surrounding structures

Ultrasound for size


measurements

Aortic Dissection
Treatment
3.0-4.0 cm
US every 2-3 years

4.0-5.4 cm
US or CT every 6-12
months

5.5 cm,
asymptomatic
Surgical repair

Etiology
Tearing of aortic
intima forms
false lumen
Blood flows
into false
f
lumen,
extends tear
Sopurce: J. Heuser commons.wikimedia.org

MTB S2CK

p. 399

Aortic Dissection/Etiology

Aortic Dissection

Can occur in ascending or descending aorta

Risk Factors
Male > Female
Age > 40
Hypertension (#1
risk factor))
Marfans disease,
Ehlers-Danlos
syndrome

Signs/Symptoms
Sudden onset,
tearing chest pain
Radiates to the
back

Elevated BP
May be
asymmetric (R > L)

Sopurce: J. Heuser commons.wikimedia.org

Aortic Dissection/Diagnosis

Aortic Dissection

Demonstration of dissection on imaging


CXR
Widening of mediastinum

Source: NNMC

Aortic Dissection/Diagnosis

Aortic Dissection

Demonstration of dissection on imaging


CXR

Treatment
Ascending dissection = Emergent
surgery
Descending dissection = Medical
therapy

Widening of mediastinum

Transesophageal ECHO (TEE)


Acute chest pain and/or clinically unstable

MRA
Chronic chest pain and hemodynamically stable

CT angiogram

Beta-Blockers (#1)
Anti-hypertensive meds

TEE and MRI contraindicated

Claudication

Claudication/Management

Etiology
Atherosclerotic plaques prevent sufficient perfusion
to extremities (lower > upper)

Medical

Associated with smoking, DM, hyperlipidemia

Symptoms
Calf/leg pain with exercise
Relieved by rest

Diagnosis
Ankle-Brachial Index

Risk Modification
Smoking cessation (#1)
Graded exercise
Pharmacologic therapy
Cilostazol
Cil t
l
Antiplatelet agents: Aspirin, Clopidogrel

Percutaneous
Stenting, angioplasty

< 0.9 = pathologic


< 0.4 = symptomatic

Surgical

Trauma/ABC Assessment
Initial assessment rely on ABC algorithm
Airway

Shock
Trauma Assessment (ABCs)
yp of Shock
Types
Hypovolemic
Cardiogenic
Neurogenic
Septic
Anaphylactic

Breathing
Circulation
Disability (CNS)
Exposure
ABCs are a roadmap, but you must know what to do
at each step
MTB S2CK

p. 380 381

Trauma/ABC Assessment

Trauma/ABC Assessment
Concern for airway compromise?
AMS
Facial trauma
Apnea

A = Airway
Must assess for airway compromise
If patient can talk airway is clear
Look for traumatic obstruction, evidence of smoke
inhalation

Evaluate need for intubation

No

Yes

Assess
Breathing/
Oxygenation

Yes

Yes

Facial Trauma
Altered Mental Status
Apnea

MTB S2CK

Careful Orotracheal Intubation


Flexible bronchoscopy

p. 381

MTB S2CK

Facial Trauma?

Cricothyroidotomy

Indications

Intubate

No

Cervical Spine Injury?


No

Orotracheal Intubation

p. 381

Trauma/ABC Assessment

Trauma/ABC Assessment

B = Breathing
Assess breath sounds
Monitor oxygenation status with pulse oximetry

C = Circulation
Evaluate pulses (distal first, proximal if
absent)
Manage hemorrhage sites

Goal is O2 Saturation
> 90%
If O2 Sat < 90%
consider

Source:UusiAjaja, commons.wikimedia.org

MTB S2CK

a. Supplemental O2
via nasal cannula
b. O2 face mask
c. Intubation

p. 381

Direct pressure slows blood loss

Monitor blood pressure


If the patient is hypotensive, place 2 largebore IVs and begin aggressive fluid
resuscitation
Start with normal saline (only use blood
products as secondary measure)
MTB S2CK

p. 381

Trauma/ABC Assessment
D = Disability (Altered Mental Status)
Assessed with Glasgow Coma Scale
Eye Response (1-4)
Verbal Response (1-5)
Motor Response (1-6)

Score < 8 requires intubation


E = Exposure
Remove all clothing on patient
Assess for hidden injuries

Thorough physical examination

52-year-old woman ejected from her car during a


high-speed motor vehicle accident . Upon arrival to
ED, she complains of severe, left-sided chest pain.
Pale, cool patient in severe distress. Heart rate 130
bpm, BP 86/44 mmHg. JVD along angle of jaw. Chest
X-ray shows anterior rib fractures on the left.
Which is the most likely diagnosis?
a. Hypovolemic shock Would not see JVD
b. Neurogenic shock
c. Anaphylactic shock
d. Cardiogenic shock
Warm, flush patients
e. Septic shock
MTB S2CK

p. 381

Shock

Types of Shock

Definition Inadequate perfusion/oxygenation that


impairs organ function
Signs/Symptoms
Vitals Signs

Hypovolemic
Causes: Hemorrhage (#1), Dehydration, Burns

Decreased BP
Increased HR

CNS: Confusion, altered mental status


Kidney: Decreased urine output, increased BUN/Cr
ratio
Liver: Massively elevated AST/ALT (Shock Liver)
Heart: Chest pain, shortness of breath
MTB S2CK

p. 381

MTB S2CK

p. 382

Types of Shock/Hypovolemic

Types of Shock

Signs/Symptoms

Cardiogenic
Causes: MI (#1), CHF,
arrhythmia
Signs/Symptoms

Pale, cold
Trauma

Lab Findings

Pale, cold
Symptoms associated
with MI (Chest pain,
SOB)
JVD

SVR =
CVP =
PCWP =
CO =

Treatment

Source: U.S. Navy photo by Mass Communication


Specialist 2nd Class Michael Russell, public.navy.mil

Lab Findings

CO =
SVR =
CVP =
PCWP =

Treatment
Treat cardiac problem
Do NOT give fluid!!!

Aggressive IV fluid replacement


Pressors
MTB S2CK

p. 382

MTB S2CK

Types of Shock

Types of Shock

Neurogenic
Causes: CNS damage
(cervical/thoracic spinal
cord - #1)
Signs/Symptoms:
Warm,, flush
Evidence of CNS
damage (trauma)

p. 382

Lab Findings

SVR =
CVP =
PCWP =
CO =

Treatment
Aggressive IV fluid
delivery
Pressors

Septic
Causes: Infection
E. coli and S. aureus
(most common
organisms)

Signs/Symptoms
Warm, flush
Possible nidus of
infection (UTI,
pneumonia, wound)
Source: cdc.gov

MTB S2CK

p. 382

MTB S2CK

p. 382

Types of Shock/Septic

Lab Findings
SVR =
CVP =
CO =
PCWP =

MTB S2CK

Types of Shock

Treatment
Broad-spectrum
antibiotics
Fluid and
pressors
Dopamine
Norepinephrine

p. 382

Anaphylactic
Causes: Allergy
(insects, food,
medication)
Signs/Symptoms
Warm, flush
Wheezing,
hives,
associated
evidence of
allergic reaction

MTB S2CK

Types of Shock/Anaphylactic

p. 382

Shock Algorithm
Pale/Cold

Lab Findings
CVP =
SVR =
PCWP =
CO =

Source: James Heilman, MD, commons.wikimedia.org

Is patient pale/cold or
warm/flush?

PCWP change?
Elevated

Cardiogenic
Treat
cardiac
problem

Treatment
Epinephrine

CO change?

Decreased

Decreased

Hypovolemic
Fluid and
pressors

Elevated

Neurogenic
Fluid and
pressors

PCWP change?
Decreased
No Change

Anaphylactic
Epinephrine

MTB S2CK

Warm/Flush

Septic
Antibiotics
Fluids and
pressors

p. 382

Abdominal Trauma

Trauma
Abdominal Trauma
Thoracic Trauma
Pelvic Trauma

Penetrating
Gunshot wounds
Must do exploratory laparotomy in ALL
patients
Stab wounds
If hemodynamically stable, do a FAST
ultrasound scan
If hemodynamically unstable, perform an
exploratory laparotomy

10

Abdominal Trauma

Abdominal Trauma

Diaphragmatic Rupture

Cause
Penetrating or blunt trauma
Left > Right

Symptoms

Loops
of Bowel

Respiratory distress
Kehrs sign = Left shoulder pain

Diagnosis
CXR: Bowel loops in thorax
Absent Hemi-Diaphragm
MTB S2CK

p. 383

Source: Hariharan D, Singhal R, Kinra S, Chilton A, commons.wikimedia.org

Abdominal Trauma/Blunt

Abdominal Trauma/Blunt

Splenic/Liver injury
Causes
Most commonly injured in blunt abdominal
trauma

Pancreatic Injury
Causes
Blunt trauma to epigastrum

Spleen #1
Liver #2

Associated with lower rib fractures


Signs/Symptoms
Hypotension (due to hemorrhage)
Kehrs sign
MTB S2CK

p. 383

Bike handlebars
Car dashboard

Signs/Symptoms
Cullens sign = Bruising around umbilicus

MTB S2CK

Abdominal Trauma/Blunt

p. 383

Abdominal Trauma/Blunt
Diagnosis
FAST scan (ultrasound)
To evaluate for intraabdominal bleeding

CT scan
If negative FAST
FAST, but
suspect splenic
rupture
To evaluate
retroperitoneal bleed

Fluid = Blood

Source: commons.wikimedia.org

Source: Herbert L. Fred, MD and Hendrik A. van Dijk, commons.wikimedia.org

MTB S2CK

p. 383

MTB S2CK

p. 383

11

Abdominal Trauma/Blunt

Management
If hemodynamically stable
Close monitoring
Serial abdominal exams
IV fluids

33-year-old female jogging at night was struck by a


drunk driver. She complains of severe abdominal pain
that radiates to the back. Vital signs are stable. After
ER work-up, patient is admitted for evaluation. Three
days later, the following picture is taken.

If hemodynamically unstable
Exploratory laparotomy

MTB S2CK

p. 383

33-year-old female jogging at night was struck by a


drunk driver. She complains of severe abdominal pain
that radiates to the back. Vital signs are stable. After
ER work-up, patient is admitted for evaluation. Three
days later, the following picture is taken.
What is the most likely diagnosis?
a.
b.
c.
d.
e.

Pancreatic pseudocyst Delayed appearance


Hemorrhagic pancreatitis
Ruptured AAA No pulsatile mass
Aortic dissection Stable BP, no chest pain
Splenic rupture Wrong side, no L shoulder pain

Source: Herbert L. Fred, MD and Hendrik A. van Dijk, commons.wikimedia.org

Thoracic Trauma

Thoracic Trauma/Pneumothorax

Pneumothorax
Etiology
Air in pleural space
pulmonary collapse
Signs/Symptoms
Chest pain
Decreased breath sounds
Hyperresonance to percussion
Tracheal deviation toward affected side

Diagnosis
CXR

Collapsed
Lung
Border

Treatment
Chest tube

Source: John Yasmer

MTB S2CK

p. 384

MTB S2CK

p. 384

12

Thoracic Trauma

Thoracic Trauma/Tension Pneumothorax

Tension Pneumothorax A MEDICAL EMERGENCY


Etiology

Resulting from laceration of cervical


trachea and mediastinal pleura

Resulting from sucking wound of valvular


type

Chest wall defect acts


as one-way valve
Air can enter pleural
space, but cannot exit
Resulting increase in
pressure can
compress vital
structures
Trachea, vena
cava

MTB S2CK

a. Laceration of cervical trachea


b. Laceration of mediastinal pleura
c. Tension pneumothorax
d. Collapsed lung
e. Shift of heart
f. Subcutaneous emphysema
g. Partial collapse of contralateral lung
Source: army.mil

p. 384

Thoracic Trauma/Tension Pneumothorax

Thoracic Trauma

Signs/Symptoms

Hemothorax
Etiology
Blood in pleural
space
Signs/Symptoms
g
y p
Chest pain
Absent breath
sounds
Dullness to
percussion

Chest pain
Hyperresonance
Decreased breath
sounds
Tracheal deviation
(away from affected
side)

Diagnosis

Source: James G. Smirniotopoulos, M.D.

CXR

Treatment
Immediate needle decompression
Chest tube placement
MTB S2CK

a. Valvular sucking wound of chest wall


b. Tension pneumothorax
c. Collapsed lung
d. Cardiac shift to intact side
e. Partial collapse of contralateral lung

p. 384

Thoracic Trauma/Hemothorax

Diagnosis
CXR

MTB S2CK

Source: army.mil

p. 384

Thoracic Trauma/Hemothorax
White-Out

Blunted costophrenic angle

CT scan
Treatment
Chest tube drainage
Thoracotomy
Posteroanterior CXR made shortly after
wounding and showing hemothorax

10 days later, after 6 thoracenteses,


three 150 cc of blood and other fluid
was removed
Source: army.mil

MTB S2CK

p. 384

13

Hemo/Pneumothorax Diagnosis

Thoracic Trauma

Symptoms

Pericardial Tamponade

Chest Pain
Decreased/Absent Breath Sounds

Etiology
Trauma to pericardium
Broken ribs
Penetrating trauma

Response to Percussion?
Dullness

Hyperresonance

Hemothorax

Tracheal Deviation?

Chest tube drainage

Away from lung

Toward lung

Pneumothorax

Tension pneumothorax

Chest tube

Immediate needle
thoracotomy
Chest tube

Signs/Symptoms
JVD
Hypotension
Decreased heart
sounds

MTB S2CK

p. 384

Source: James Heilman, MD, commons.wikimedia.org

Thoracic Trauma/Pericardial Tamponade

Thoracic Trauma/Pericardial Tamponade

Diagnosis

Diagnosis

EKG

EKG

Electrical Alternans

Electrical Alternans

ECHO
Diagnostic test of choice

Treatment
Pericardiocentesis

Source: army.mil

MTB S2CK

p. 384

26-year-old man suffers blow to chest with a baseball bat.


He is brought to the ED with severe right-sided chest pain
and difficulty breathing. He is tachypneic at 26
breaths/minute, with a heart rate of 130 beats/minute,
diminished breath sounds on the right, and left tracheal
deviation.
What is the next step in management?
a.
b.
c.
d.
e.

Pericardiocentesis Not pericardial tamponade


Chest X-ray
Too slow. This is an emergency
Needle thoracotomy
Chest tube placement
EKG
Not indicated in this patient

MTB S2CK

p. 384

14-year-old boy hits a pothole while riding his bike and


falls directly onto the central bar. He presents to the ED
with severe groin pain and swelling. Physical examination
reveals blood at the urethral meatus and a high-riding
prostate.
What is the next appropriate step in management?
a.
a
b.
c.
d.
e.

Place a Foley Can


C ffurther
th damage
d
urethra
th
Get a retrograde urethrogram
Empiric antibiotics Must assess urethral patency
CBC and electrolytes
Discharge the patient with reassurance

MTB S2CK

p. 385

14

Abdominal Pain Differential Diagnosis


RUQ

The Abdomen Part 1


Abdominal Pain
Mesenteric Ischemia
Esophageal Pathology
Gastric Perforation

LUQ

Cholecystitis
Splenic Rupture
- Radiates to R shoulder
- Radiates to L shoulder
Cholangitis
Ischemic Bowel Disease
Perforated Ulcer Mid-Epigastrum
Pancreatitis
Peptic Ulcer Disease
Aortic Dissection
- Radiates to back

RLQ

LLQ

Appendicitis
Ovarian Torsion ( )
Ectopic Pregnancy ( )
Diverticulitis (Cecal)

Diverticulitis (Sigmoid)
Sigmoid Volvulus
Ovarian Torsion ( )
Ectopic Pregnancy ( )

MTB S2CK

p. 387

Chronic Mesenteric Ischemia


66-year-old man in the ED with acute onset, severe
abdominal pain. Abdominal exam benign, without
guarding or rebound tenderness. White count
18,000/mm3 (4,500-11,000/mm3), lactic acid 4.2 mg/dL
(4.5 to 19.8 mg/dL). Only medication is coumadin.
Whats the most appropriate next step in management?
a. MRI scan off th
the abdomen
bd
T k too
Takes
t long
l
b. Exploratory Laparotomy
c. Colonoscopy
Risk of bowel perforation
d. Oral antibiotics Doesnt treat primary concern
e. EKG Symptoms not consistent with cardiac etiology

Etiology
Mesenteric artery
atherosclerosis
insufficient
blood flow to
bowel
Patients often
have other
atherosclerotic
diseases
Angina
Claudication
MTB S2CK

p. 385 386

Chronic Mesenteric Ischemia

Chronic Mesenteric Ischemia

Signs/Symptoms
Diffuse, postprandial abdominal pain

Diagnosis
CT abdomen (initial test)

Severe, crampy, associated nausea


Due to O2 requirement by gut after meals
May lead to weight loss due to fear of eating

Bloody diarrhea
Progressive disease begins with mild
ischemia, progresses to full occlusion of
blood flow
MTB S2CK

p. 386

A lack of blood flow causes ischemia to the bowel wall


and sloughing of the mucosa. Source: Niket Sonopal, MD.

Fast, non-invasive

CT angiography is most accurate


Treatment
Surgical revascularization via arterial bypass
NPO
Hydration with IV fluids
Bowel prep 24 hours preceding surgery

Nitrates may provide short-term symptom


improvement
MTB S2CK

p. 386

15

Acute Mesenteric Ischemia

Acute Mesenteric Ischemia

Etiology
Acute occlusion of mesenteric arteries
Most commonly the superior mesenteric
Causes:
Embolism secondary to A
A-fib
fib (#1)

MTB S2CK

Source: Mikael Hggstrm from original image


created and uploaded by Dr. I-Chen Tsai,
commons.wikimedia.org

p. 386

Acute Mesenteric Ischemia

Acute Mesenteric Ischemia

Etiology
Acute occlusion of mesenteric arteries

Signs/Symptoms
Sudden onset, severe
abdominal pain

Most commonly the superior mesenteric


Causes:
Embolism secondary to A
A-fib
fib (#1)
Acute thrombus formation on
atherosclerotic plaque

Pain out of
proportion to exam

Nausea,, vomiting
g
Bloody diarrhea

Results in bowel infarction

Air in bowel wall


((Pneumatosis
Intestinalis)
Edema
Ileus

CT angiography
(Most accurate)

Splenic flexure (#1) and hepatic flexure


(#2) are most common sites
MTB S2CK

Diagnosis
Labs: WBC, pH,
Lactate
Abdominal X-ray/CT

p. 386

Acute Mesenteric Ischemia

Acute Mesenteric Ischemia

Treatment
Emergent Laparotomy
Resection of necrotic bowel

Mesenteric artery thromboembolism. Source: Joel McFarland, Medpix 28818

MTB S2CK

p. 386

16

Acute Mesenteric Ischemia


52-year-old woman, alcoholic, with severe chest pain
after an episode of persistent vomiting. No blood in
vomitus, only stomach contents. Exam reveals
crepitus over upper anterior chest wall.
What is the next step?
a.
b.
c.
d.
e.

Source: haitham alfalah, commons.wikimedia.org

Ethanol level Not useful in this case


Chest X-Ray Will not show esophageal rupture
Upper Endoscopy Risk further perforation
Barium Esophagogram Not water-soluble
Gastrografin Esophagogram

MTB S2CK

p. 387

Esophageal Injuries
Mucosal Tear
Mallory Weiss Syndrome

Cause

Vomiting/Retching
Alcoholics

Symptoms

Hematemesis
Odynophagia

Location

Gastroesophageal junction

Diagnosis

Gastrografin esophagogram
No leakage
Treatment
Supportive
Cauterization if necessary
Complications Rare
MTB S2CK

Esophageal Perforation
Boerhaave Syndrome

Iatrogenic is #1 (Endoscopy)
Vomiting/Retching
Alcoholics
Retrosternal chest pain
Severe, acute onset
Radiates to L shoulder
Subcutaneous Emphysema
Distal esophagus
Left Posterolateral Aspect
Gastrografin esophagogram
Leakage
Emergent Surgery
High mortality (25%)
Acute mediastinitis
Very high mortality

57-year-old male with BMI of 37 (18.5-24.9 normal)


presents to ED with weakness. Vomiting large amounts of
blood twice in last hour and passing one bright red stool.
He has persistent pain in his mid-abdomen. Takes
omeprazole for heartburn. BP 100/60, HR 120.
What is the diagnosis?
a Gastric Perforation No
a.
N bl
bleed.
d P
Peritoneal
it
l signs.
i
b. Hemorrhagic ulcer
c. Boerhaaves syndrome No hematemesis
d. Diverticulosis No hematemesis
e. Acute mesenteric ischemia No hematochezia

p. 387 388

Gastric Perforation

Gastric Perforation

Etiology
Secondary to
peptic ulcer
disease

Risk Factors
H. Pylori
infection
NSAIDs
Burns
Trauma
Head trauma
Cancer
Ethanol
Tobacco

Increased
production of
gastric acid,
combined with
compromise of
stomach lining
results in ulcer
formation

Source: User:KGH, commons.wikimedia.org


Source: commons.wikimedia.org

MTB S2CK

p. 388

MTB S2CK

p. 388

17

Gastric Perforation

Gastric Perforation/Signs and Symptoms

Pathophysiology
Perforation

Hemorrhage

Ulcer completely
erodes through
visceral wall

Ulcer erodes into


gastroduodenal artery

Gastric contents are


released into abdominal
cavity
Result is damage to
peritoneal structures
Peritonitis (anterior
+ posterior ulcers)
Pancreatitis
(posterior ulcers)
MTB S2CK

More common than


gastric perforation
Seen with posterior
ulcers
Requires upper
endoscopy to
evaluate/treat bleed

p. 388

Acute onset mid-epigastric abdominal pain


May radiate to right shoulder (phrenic nerve
involvement)
Worsening over time

Peritoneal signs (if any delay in seeking


t t
treatment)
t)
Rigidity
Guarding
Rebound tenderness

MTB S2CK

Gastric Perforation

p. 389

Gastric Perforation
Management

Diagnosis
Upright
chest X-ray

1) Make Patient NPO


Prevents further extrusion of gastric contents
into peritoneal cavity

2) Place NG Tube

Free air
under
diaphragm

Suction gastric contents


Mitigates risk from newly-formed
newly formed acid

3) Medical Management

Abdominal
CT

Broad-spectrum antibiotics to combat infection


IV fluids in preparation for surgery

4) Emergent Surgery
Exploratory laparotomy
- Repair perforation

Source: commons.wikimedia.org

MTB S2CK

p. 389

MTB S2CK

The Abdomen Part 2


Abdominal Abscess
Inflammatory GI Conditions
Abdominal Obstruction

p. 389

16-year-old boy comes with new onset RLQ pain. He


reports vague, mid-abdominal pain for last day, but
when he awoke this morning it was much sharper and
had changed location. He displays guarding and
rebound tenderness, and palpation of LLQ hurts on
right side. WBC count of 16,000/mm3 (4,5003)
11 000/
11,000/mm
What is the next step in management?
a.
b.
c.
d.

Abdominal X-ray Non-specific findings


Colonoscopy Risk of perforation
RLQ Ultrasound Non-specific findings
Laparoscopic Appendectomy

MTB S2CK

p. 389

18

Right Lower Quadrant Pain

Abdominal Abscess

Age > 60 yo?


Yes

Cause
Surgical complication (#1)
Inflammatory disease
Trauma

No

Cecal Diverticulitis

Male or Female?

Make NPO, Place NG Tube


Broad-Spectrum Abx

Female
Male

Order -HCG, Pelvic US


Appendicitis
Emergent
Laparoscopic
Appendectomy

Normal

Elevated or
Abnormal

Ovarian Torsion

Ectopic Pregnancy

Doppler US to
diagnosis
Laparoscopic
surgery

Emergent Surgery

Signs/Symptoms
Abdominal pain/distension
Non-specific symptoms
Fever/chills
GI symptoms nausea/vomiting, diarrhea

Rectal fullness
MTB S2CK

Abdominal Abscess

p. 390

Abdominal Abscess

Diagnosis
Abdominal CT
Visual evidence of abscess
Information about surrounding
structures

Source: Joel McFarland, Medpix 28716

MTB S2CK

p. 390

Abdominal Abscess
Diagnosis
Abdominal CT
Visual evidence of abscess
Information about surrounding
structures

CBC
May show elevated WBC count

Treament
Broad-Spectrum Antibiotics
Incision + Drainage

78-year-old woman with severe LLQ pain, fever, and


anorexia of one day. Shes had dark, tarry stools in the
past. The patients husband brings in a medication list,
which includes a stool softener and iron supplements.
She displays guarding and rigidity.
What is the diagnosis?
a Ectopic pregnancy Patient
a.
P ti t is
i postmenopausal
t
l
b. Acute appendicitis Rare in elderly, RLQ in location
c. Cholecystitis RUQ pain
d. Acute diverticulitis
e. Acute pancreatitis Central abd pain, radiates to back

Percutaneous (CT-guided)
versus Open
MTB S2CK

p. 390

19

Inflammatory GI Conditions
Appendicitis

Pancreatitis

Diverticulitis

Cholecystitis

Fecolith
obstructing
appendiceal
orifice
Anorexia
Fever
Periumbilical
Pain RLQ

Fecal
obstruction of
bowel wall out
pouchings
N/V
Fever
LLQ Pain

Gallstone
obstructing cystic
duct

Antibiotics (x1)
Resection (if
recurrent)
Abscess

Diagnosis

Phys. Exam
CT Scan

Alcohol
Gallstone
obstructing
pancreatic duct
N/V
Fever
Abdominal Pain
Radiates to
back
Amylase/Lipase
CT Scan (#1)

Treatment

Laparoscopic
Removal

IV Fluids
NPO

Cause

Symptoms

Complication Abscess

MTB S2CK

Pseudocyst

CT Scan

N/V
Fever
RUQ pain
Worse with
inspiration
Ultrasound
Fluid, Stones,
Thick Wall
HIDA scan (#1)
Laparoscopic
Removal
Perforation

p. 391

Bowel Obstruction

Bowel Obstruction

Pathophysiology
A mechanical or
functional obstruction of
intestines
Leads to fluid/gas
accumulation proximal
to site off obstruction
Resulting pressure
increase leads to
1. Pain
2. Decreased perfusion
(and risk of necrosis)

MTB S2CK

Causes/Risk Factors
Obstruction can be
partial or complete
Partial = GI contents
are able to pass
obstruction site
Complete = No
avenue for passage
Represents a much
more severe condition

p. 392

Signs/Symptoms
Severe, crampy
abdominal pain
Colicky in nature

Nausea/vomiting
Fever
High-pitched,
Tinkling bowel
sounds
p. 392 393

Adhesions from
previous abdominal
surgery
#1 in developed
countries

Hernia
#1 in undeveloped
countries

Volvulus = Twisting of
bowel on its mesentery
Crohns disease
MTB S2CK Only

Bowel Obstruction

MTB S2CK

55 year-old man presents to the ED with severe, intermittent


abdominal pain and nausea/vomiting. He cannot recall when
he last passed flatus, but is certain he didnt have a bowel
movement this morning. He has type 1 diabetes and had an
open appendectomy in his 30s. Temperature of 101.8. Loud,
high-pitched bowel sounds noted on auscultation. An
abdominal plain film shows a partial small bowel obstruction.
What is the best next step in management?
a. Prescribe a laxative Will not relieve obstruction (may worsen)
b. Place an NG-tube
c. Emergent exploratory laparotomy Medical management first
d. Prescribe antibiotics No evidence of bowel perforation
e. Perform a colonoscopy Will not relieve obstruction

Neoplasms
Intussusception =
Telescoping bowel
Most frequently seen
in pediatric population

Foreign bodies
Intestinal atresia =
Blind Pouch
Only seen in
neonates

p. 393

Bowel Obstruction

Diagnosis
Labs
WBC,
pH

Lactate,

Abdominal X-ray
Air-fluid levels,
dilated loops of
bowel

Source: James Heilman, MD, commons.wikimedia.org

20

Bowel Obstruction

Bowel Obstruction

Signs/Symptoms
Severe, crampy
abdominal pain

Diagnosis
Labs

Management
1) Make Patient NPO

WBC, Lactate, pH

Colicky in nature

Prevents further increase in bowel pressure

Abdominal X-ray

Nausea/vomiting
Fever
High-pitched, Tinkling
bowel sounds

2) Place an NG Tube (w/suction)

Air-fluid levels, dilated


loops of bowel

Lowers bowel pressure proximal to obstruction

Abdominal CT w/ oral
contrast (#1)

3) Medical Management
IV Fluids
Volume is lost due to third-spacing

Clear transition zone


correlates with
obstruction
MTB S2CK

p. 392 393

4) Surgical Decompression (if indicated)


Complete obstruction (Emergent)
Lack of improvement with medical management
MTB S2CK

p. 393

Fractures Algorithm

Diagnosis

Pain
Swelling
Bony deformity

X-rays

Timing:
Acute onset
with trauma

Neurovascular
exam

Closed
(skin intact)

Closed reduction

Open
(skin puncture)

Emergency surgery
(I&D)

Management

MTB S2CK

Fracture Types
Upper
pp Extremityy Injuries
j
Back Pain
Fat Embolism
Compartment Syndrome
Knee Injuries

Symptoms:

Orthopedics

Presentation

Surgery (ORIF)

p. 393 394

Fracture Types

Fractures Types

Stress Fractures
Cause

Compression Fractures
Cause
Vertebral fracture associated with poor
bone quality
Osteoporosis
p
((classical example)
p )
Presentation Clue
Elderly patient with back pain
33% thoracic spine, 33% thoracolumbar,
33% lumbar

Overuse injury secondary to repetitive insult to bone

Presentation Clue
High-performance athlete
Common sites
1. Metatarsals
2. Tibia

Diagnosis
CT/MRI

Treatment
Rest and rehabilitation
MTB S2CK

p. 394

MTB S2CK

p. 394

21

Fractures Types/Compression Fractures

Fractures Types

Diagnosis
X-ray
CT, if
inconclusive

Pathologic Fractures
Cause

Diagnosis

Fracture in bone
weakened by disease
Metastatic cancer
Multiple myeloma
Pagets disease

Presentation Clue

Treatment
Controversial
MTB S2CK

p. 394

X-ray

Treatment
Treat fracture
ID & treat primary
disease

Patient with fracture


after minimal trauma

Source:commons.wikimedia.org

MTB S2CK

p. 394

Upper Extremity Injuries

Upper Extremity Injuries/Clavicle Fracture

Clavicle Fracture
Cause
History of fall
Blunt shoulder trauma
Presentation Clue
Pain over anterior shoulder
Clavicle step-off

Workup
X-ray
Careful distal
neurovascular
exam
Must rule out
subclavian
artery/brachial
plexus injury

Treatment
Arm sling
Source: Mark D. Travis

MTB S2CK

p. 395

Upper Extremity Injuries

Anterior Shoulder Dislocation


Cause
Fall on outstretched hand
Sports, blunt trauma
Signs/Symptoms
Severe shoulder pain, swelling
Arm is held in external rotation

MTB S2CK

p. 395

MTB S2CK

p. 395

Upper Extremity Injuries/


Anterior Shoulder Dislocation
Diagnosis
X-ray, MRI (if
necessary)
Careful distal
neurovascular exam
Must rule out
axillary artery or
nerve injury
Treatment
Reduction w/sling
immobilization
MTB S2CK

Source: Daniel Patrick Moloney

p. 395

22

Upper Extremity Injuries

Posterior Shoulder Dislocation


Cause
History of seizure or electric shock

Signs/Symptoms
g
y p
Severe shoulder pain, swelling
Arm is held in internal rotation

MTB S2CK

p. 395

Upper Extremity Injuries/


Posterior Shoulder Dislocation

Diagnosis
XR, MRI (if
necessary)

Treatment
Reduction
w/sling
immobilization

MTB S2CK

p. 395

Source: Medpix 20030

19-year old with persistent wrist pain of 3 days


duration. Pain began when he braced himself from a
fall. Right wrist is notable for point tenderness and
mild swelling at base of dorsal aspect of thumb, but
no deformity. An X-ray is shown on the next slide.

Source: Medpix 6094

Upper Extremity Injuries


19-year old with persistent wrist pain of 3 days
duration. Pain began when he braced himself from a
fall. Right wrist is notable for point tenderness and
mild swelling at base of dorsal aspect of thumb, but
no deformity. An X-ray is shown below.
What is the best next step in management?
Scaphoid may be fractured
a. Reassurance
b. MRI Scan Unnecessary, worse than CT
c. Thumb Spica Cast
d. Amputation No indication
e. Open Reduction + Internal Fixation No fracture (yet)

Trigger finger
Cause
Inflammation of finger flexor pulley system
Leads to catching/locking of flexor tendon

Signs/Symptoms
Lone digit caught in flexion
Popping sensation if digit is manually extended

Moderatesevere pain

Diagnosis
Clinical Exam

Treatment
Corticosteroid injection
MTB S2CK

p. 395

23

Upper Extremity Injuries

Upper Extremity Injuries/


Dupuytrens Contracture

Dupuytrens Contracture
Cause

Risk Factors
Male > Female
Age > 40
Northern European descent

Thickening of
palmar fascia,
leading to
flexion
contracture

Diagnosis
Di
i
Clinical Exam

Digits cannot
fully extend

Treatment
Source: commons.wikimedia.org

MTB S2CK

p. 395

Surgery
MTB S2CK

p. 395

Back Pain
A 66-year-old man comes to his PCP with bilateral
leg pain of several months duration. The pain seems
to be worst when he has to walk several blocks, and
improves when he sits down. Leaning forward (on a
bench, shopping cart, etc.) alleviates the pain. He is a
non-smoker.
What is the most appropriate next diagnostic step?
a.
b.
c.
d.
e.

Lower Extremity XR Useful for bony trauma


Doppler US of the calf Useful for DVT evaluation
Ankle-Brachial Indices Diagnoses claudication
Spine MRI
Leg MRI Useful for soft-tissue injury/stress fractures

Spinal Stenosis
Cause
Arthritic changes result in narrowing of spinal
canal
Lumbar #1, Cervical #2
Symptoms
Neck/Back Pain
Bilateral leg/buttock pain + numbness
Pseudo-claudication
Worse with walking, improves with spine
flexion

Back Pain/Spinal Stenosis

Back Pain

Diagnosis
MRI
Treatment
NSAIDs vs.
surgery
g y

Herniated Disk Disease


Cause
Intervertebral disk herniates, compressing
spinal nerve root
Often associated with lifting injury
Most frequently seen in elderly
Symptoms

Electric Pain following a dermatome


distribution

Source: nih.gov, commons.wikimedia.org

24

Back Pain/Herniated Disk Disease

Back Pain/Herniated Disk Disease


Diagnosis
Clinical Exam
(Straight Leg
Raise)
Consider
MRI if red
flags present

Treatment
NSAIDS,
Activity
Modification

Source:Mjorter, commons.wikimedia.org

Source: user:debivort, commons.wikimedia.org

Back Pain

Back Pain

Red Flags Concern for Metastasis or Abscess

Pain not relieved


by rest
Not worse with
activity

Night pain
Constant, dull pain
> 6 weeks

Fever
Neurological
deficits

Management
MRI to evaluate for mass lesion
Emergent glucocorticoids if neurological
findings

Bowel/Bladder
incontinence
Abnormal
reflexes

History of Cancer

Fat Embolism A Medical EMERGENCY


22-year-old woman was struck by a car 3 days ago while
walking her dog. The impact broke her femur, which
required immediate surgical repair. This morning the
patient is confused and has difficulty catching her breath.
ABG shows PaO2 55 mmHg, and exam reveals a
petechial rash.
What is the most likely diagnosis?
a. Myocardial infarction No rash
b. Pancreatitis Expect abdominal pain
c. Rhabdomyolysis No hypoxemia
d. Fat embolism

MTB S2CK

p. 396

Etiology
Traumatic long bone
fracture (#1 = Femur)
Releases marrow fat
into circulation
Fat vesicles are too
large to pass through
capillaries
Result is vascular
occlusion

MTB S2CK

H&E stain, lungs blood vessel with fibrinoid material and an


optical empty space indicative of the presence of lipid. Source:
Boris L Kanen, Ruud JLF Loffeld, commons.wikimedia.org

p. 396

25

Fat Embolism

Fat Embolism

Signs/Symptoms Onset 0-5 days after


fracture
Respiratory Distress (V/Q mismatch)
SOB
Tachypnea
yp
Confusion (Involves brain vasculature)
Petechial Rash (Involves skin capillaries)
Chest wall, upper extremities

Diagnosis
ABG: PO2 < 60 mmHg
CBC: Decreased platelet count
Chest X-ray: Infiltrates
Urinalysis: Fat droplets in urine
T t
Treatment
t
Respiratory Support

MTB S2CK

p. 396

Goal O2 Sat > 90%


Consider intubation/mechanical ventilation if
severely hypoxic

MTB S2CK

Compartment Syndrome

Compartment Syndrome/Signs & Symptoms

Pathophysiology
Injury occurs, resulting in Pressure builds,
swelling
leading to severe
tissue compression
Fracture #1 (tibial,
forearm)
Nerves
Burns
Muscle
Crush injuries
Vessels
(Reperfusion syndrome) Resulting damage can

In closed compartment
(fascial sheath, cast),
theres no escape for
increasing pressure
MTB S2CK

p. 396

lead to Volkmanns
ischemic contracture,
limb loss, or death

p. 396

Early
Findings
Pain

Parathesias

Poikilothermia

Pulselessness

Severe,
worse
with
muscle
stretch

Pins and
needles
nerve
involvement

Cold, due to
decreased
blood flow

Absent distal
pulses
(ominous
finding)

Paralysis
Pallor

Pale skin from


blood flow
MTB S2CK

Inability to move
distal musculature

p. 396 397

Compartment Syndrome

Knee Injuries

Treatment A Medical EMERGENCY

Basic Principles

Surgical
fasciotomy
Releases
compartment
p
pressure

Late
Findings

The 6 Ps

Ligamentous/meniscal knee injuries have similar


symptoms
Knee Pain
Swelling
Instability

The key factor is time course in


which symptoms present

Pay attention to mechanism of injury!


Physical exam provides clues to etiology
Special tests for each type of knee injury

Diagnostic test of choice is always MRI


MTB S2CK

p. 397

Source: Sarte, commons.wikimedia.org

26

Knee Injuries
Mechanism of
Injury

Symptom
Onset

MCL/LCL Trauma to
Immediate
Injury
contralateral
aspect of knee
ACL Tear Twisting or
direct impact

Exam
Maneuver

Management

Medial/Lateral
Instability

Conservative

Immediate and Anterior Drawer Arthroscopic


Repair
Severe
sign
Lachman Test

Immediate but Posterior


PCL Tear Forced
Hyperextension Mild
Drawer sign

Repair vs.
Conservative
management

Meniscal Twisting Injury Delayed (12 24 Joint line


Conservative
hours)
tenderness
Tear
McMurrays
Clicking/Locking
MTB S2CK

p. 398

22-year-old hockey star is checked by an opposing


player, and his leg gets tangled as he falls. He
experiences a popping sensation in his left knee, which
immediately swells. The team physician performs a
physical exam and notes a positive anterior drawer sign
as well as medial knee instability. He recommends an
MRI to evaluate the likely ACL and MCL tears.
What other structure will likely be injured on MRI?
a.
b.
c.
d.

Lateral Collateral Ligament Different injury mechanism


Lateral Meniscus
Medial Meniscus
Posterior Cruciate Ligament

MTB S2CK

p. 398

27

You might also like